*NURSING > EXAM > PATHO TEST BANK. CHAPTER 1 – CHAPTER 30 (GRADE A GUARANTEED) (All)

PATHO TEST BANK. CHAPTER 1 – CHAPTER 30 (GRADE A GUARANTEED)

Document Content and Description Below

PATHO TEST BANK. CHAPTER 1 – CHAPTER 30 (GRADE A GUARANTEED) Chapter 1: Cellular Biology MULTIPLE CHOICE 1. Which statement best describes the cellular function of metabolic absorption... ? a. Cells can produce proteins. c. Cells can take in and use nutrients. b. Cells can secrete digestive enzymes. d. Cells can synthesize fats. In metabolic absorption, all cells take in and use nutrients and other substances from their surroundings. The remaining options are not inclusive in their descriptions of cellular metabolic absorption. PTS: 1 REF: Page 2 2. Most of a cell’s genetic information, including RNA and DNA, is contained in the: a. Mitochondria c. Nucleolus b. Ribosome d. Lysosome The nucleus contains the nucleolus, a small dense structure composed largely of RNA, most of the cellular DNA, and the DNA-binding proteins, such as the histones, which regulate its activity. The other options do not contain most of a cell’s genetic information. PTS: 1 REF: Page 2 3. Which component of the cell produces hydrogen peroxide (H2O2) by using oxygen to remove hydrogen atoms from specific substrates in an oxidative reaction? a. Lysosomes c. Ribosomes b. Peroxisomes d. Oxyhydrosomes Peroxisomes are so named because they usually contain enzymes that use oxygen to remove hydrogen atoms from specific substrates in an oxidative reaction that produces H2O2, which is a powerful oxidant and potentially destructive if it accumulates or escapes from peroxisomes. Ribosomes are RNA-protein complexes (nucleoproteins) that are synthesized in the nucleolus and secreted into the cytoplasm through pores in the nuclear envelope called nuclear pore complexes. Lysosomes are saclike structures that originate from the Golgi complex and contain more than 40 digestive enzymes called hydrolases, which catalyze bonds in proteins, lipids, nucleic acids, and carbohydrates. Oxyhydrosomes are involved in enzyme production. PTS: 1 REF: Page 8 4. Which cell component is capable of cellular autodigestion when it is released during cell injury? a. Ribosome c. Smooth endoplasmic reticulum b. Golgi complex d. Lysosomes The lysosomal membrane acts as a protective shield between the powerful digestive enzymes within the lysosome and the cytoplasm, preventing their leakage into the cytoplasmic matrix. Disruption of the membrane by various treatments or cellular injury leads to a release of the lysosomal enzymes, which can then react with their specific substrates, causing cellular self-digestion. The other options do not correctly describe this process. PTS: 1 REF: Pages 7-8 5. What is the sequence of steps in the development of a digestive enzyme by the pancreas cells from the initial transcription to the release from the cell? a. The enzyme is transcribed from DNA by RNA in the nucleus, proceeds to the ribosome for synthesis, and is transported in a secretory vesicle to the cell membrane. b. The enzyme is transcribed from RNA by DNA in the nucleus, proceeds to the lysosome for synthesis, and is transported in an encapsulated membrane to the cell membrane. c. The enzyme is transcribed by the mitochondria in the nucleus, proceeds to the ribosome for synthesis, and is transported in a cytoskeleton to the cell membrane. d. The enzyme is transcribed from DNA by RNA in the nucleus, proceeds to the Golgi complex for synthesis, and is transported in a cytosol to the cell membrane. The enzyme is transcribed from DNA by RNA in the nucleus, proceeds to the ribosome for synthesis, and is transported in a secretory vesicle to the cell membrane. The other options do not correctly describe this process. PTS: 1 REF: Page 7 | Figure 1-5 6. During which phase of the cell cycle is DNA synthesized? a. G1 c. G2 b. S d. M The four designated phases of the cell cycle are: (1) the G1 phase (G = gap), which is the period between the M phase (M = mitosis) and the start of DNA synthesis; (2) the S phase (S = synthesis), during which DNA is synthesized in the cell nucleus; (3) the G2 phase, during which RNA and protein synthesis occurs, the period between the completion of DNA synthesis and the next phase (M); and (4) the M phase, which includes nuclear and cytoplasmic division. PTS: 1 REF: Page 37 7. What organic compound facilitates transportation across cell membranes by acting as receptors, transport channels for electrolytes, and enzymes to drive active pumps? a. Lipids c. Proteins b. Proteases d. Carbohydrates Proteins act as (1) recognition and binding units (receptors) for substances moving in and out of the cell; (2) pores or transport channels for various electrically charged particles called ions or electrolytes and specific carriers for amino acids and monosaccharides; and (3) specific enzymes that drive active pumps that promote the concentration of certain ions, particularly potassium (K+), within the cell while keeping concentrations of other ions, for example, sodium (Na+), below the concentrations found in the extracellular environment. The other options do not correctly describe this process. PTS: 1 REF: Page 13 | Page 15 8. Understanding the various steps of proteolytic cascades, such as caspase-mediated apoptosis and complement cascades, may be useful in designing drug therapy for which human diseases? a. Cardiac and vascular disorders b. Autoimmune and malignant disorders c. Gastrointestinal and renal disorders d. Endocrine and gastrointestinal disorders Understanding the various steps involved in this process is crucial for designing drug interventions. Dysregulation of proteases features prominently in many human diseases, including cancer, autoimmunity, and neurodegenerative disorders. The other options do not correctly describe this process. PTS: 1 REF: Page 15 9. Which structure prevents water-soluble molecules from entering cells across the plasma membrane? a. Carbohydrate chains c. Membrane channel proteins b. Glycoprotein channels d. Lipid bilayer The bilayer’s structure accounts for one of the essential functions of the plasma membrane. It is impermeable to most water-soluble molecules (molecules that dissolve in water) because the water-soluble molecules are insoluble in the oily core region. The bilayer serves as a barrier to the diffusion of water and hydrophilic substances while allowing lipid-soluble molecules, such as oxygen (O2) and carbon dioxide (CO2), to diffuse through it readily. The other options do not correctly describe this process. PTS: 1 REF: Pages 12-13 10. The fluid mosaic model explains: a. How a cell membrane functions b. Why our bodies appear to be solid c. How tissue is differentiated d. How fluid moves between the intracellular and extracellular compartments The fluid mosaic model accounts for the flexibility of cellular membranes, their self-sealing properties, and their impermeability to many substances. The remaining options do not explain the mosaic model. PTS: 1 REF: Page 12 | What's New box 11. Which form of cell communication is used to communicate within the cell itself and with other cells in direct physical contact? a. Protein channel (gap junction) b. Plasma membrane–bound signaling molecules (involving receptors) c. Hormone secretion such as neurotransmitters d. Extracellular chemical messengers such as ligands Cells communicate by using hundreds of kinds of signal molecules, for example, insulin. Cells communicate in three main ways; they display plasma membrane–bound signaling molecules (receptors) that affect the cell itself and other cells in direct physical contact. The other options do not correctly describe this process. PTS: 1 REF: Page 20 12. Which mode of chemical signaling uses blood to transport communication to cells some distance away? a. Paracrine c. Neurotransmitter b. Autocrine d. Hormonal Chemical signaling can be classified into three categories: (1) local-chemical mediator, (2) hormone, and (3) neurotransmitter. In the local-chemical mediator model, the secreted chemical acts on the cells in the immediate environment. Hormones are used for communication with distant target cells. For example, cells can secrete a chemical and rely on the blood system to deliver the signal to a distant cell. Finally, neurotransmitters are secreted by neurons to stimulate an adjoining cell. For example, a neuron might secrete acetylcholine to stimulate the movement of a muscle cell. PTS: 1 REF: Page 20 13. Which mode of chemical signaling uses local chemical mediators that are quickly taken up, destroyed, or immobilized? a. Paracrine c. Neurotransmitter b. Autocrine d. Hormone In paracrine signaling, cells secrete local chemical mediators that are quickly taken up, destroyed, or immobilized. The other options do not correctly describe this process. PTS: 1 REF: Page 20 14. Neurotransmitters affect the postsynaptic membrane by binding to: a. Lipids c. Amphipathic lipids b. Ribosomes d. Receptors In each type of chemical signaling, the target cell receives the signal by first attaching to its receptors. The other options do not correctly describe this process. PTS: 1 REF: Page 17 15. How do cells receive communication from the extracellular fluid surrounding them? a. Protein channel (gap junction) b. Plasma membrane–bound signaling molecules (involving receptors) c. Hormone secretion such as neurotransmitters d. Chemical messengers such as ligands Cellular communication can occur by the binding of a chemical massager (a ligand) to a specific membrane receptor that is closely associated with the channel (e.g., G proteins). The other options do not correctly describe how cells communicate. PTS: 1 REF: Pages 21-22 16. When a second message is necessary for extracellular communication to be activated, it is provided by which one? a. Guanosine triphosphate (GTP) c. Adenosine triphosphate (ATP) b. Adenosine monophosphate (AMP) d. Guanosine diphosphate (GDP) The two major second messenger pathways are cyclic AMP (cAMP) and calcium (Ca++). PTS: 1 REF: Pages 22-23 17. Under anaerobic conditions, what process provides energy for the cell? a. Oxidative phosphorylation c. Lactolysis b. Glycolysis d. Passive transport Glycolysis produces a net of two molecules of ATP per glucose molecule through the process of oxidation or the removal and transfer of a pair of electrons. The other options do not correctly identify an anaerobic process that provides energy to the cell. PTS: 1 REF: Page 28 18. What is the mechanism by which the energy produced from carbohydrates, proteins, and lipids is transferred to adenosine triphosphate (ATP)? a. Anaerobic glycolysis c. Oxidative phosphorylation b. Oxidative cellular metabolism d. Tricarboxylic acid phosphorylation Oxidative phosphorylation occurs in the mitochondria and is the mechanism by which the energy produced from carbohydrates, fats, and proteins is transferred to ATP. The other options do not correctly identify the mechanism described in the question. PTS: 1 REF: Pages 27-28 19. Passive transport is best described with which statement? a. Being driven by osmosis, hydrostatic pressure, and diffusion b. Involving receptors that can bind with substances being transported c. Being capable of transporting macromolecules d. Requiring energy generated by the cell Water and small electrically uncharged molecules move easily through pores in the plasma membrane’s lipid bilayer. This process, called passive transport, naturally occurs through any semipermeable barrier. It is driven by osmosis, hydrostatic pressure, and diffusion, all of which depend on the laws of physics and do not require life. The other options do not correctly describe passive transport. PTS: 1 REF: Page 28 20. Active transport occurs across which type of membranes? a. Membranes that have a higher concentration of the solute on the outside of the cell b. Membranes that are semipermeable to water and small electrically uncharged molecules c. Membranes that have receptors that are capable of binding with the substances to be transported d. Membranes that have a cell membrane that is hydrophobic rather than hydrophilic Some molecules are moved into the cell by mechanisms of active transport, which require receptors that are capable of recognizing and binding with the substance to be transported. Diffusion is the movement of a solute molecule from an area of greater solute concentration to an area of lesser solute concentration. Hydrostatic pressure is the mechanical force of water pushing against cellular membranes. Osmosis is the movement of water down a concentration gradient; that is, across a semipermeable membrane from a region of higher water concentration to a lower water concentration PTS: 1 REF: Page 28 21. Which method of transport uses transmembrane proteins with receptors with a high degree of specificity for the substance being transported? a. Active c. Transmembranous b. Mediated d. Passive Mediated transport (passive and active) involves integral or transmembrane proteins with receptors having a high degree of specificity for the substance being transported. Inorganic anions and cations (e.g., Na+, K+, Ca++, chloride [Cl–], bicarbonate [HCO3–]) and charged and uncharged organic compounds (e.g., amino acids, sugars) require specific transport systems to facilitate movement through different cellular membranes. The remaining options do not correctly identify the process described. PTS: 1 REF: Page 31 22. The movement of fluid across the arterial end of capillary membranes into the interstitial fluid surrounding the capillary is an example of which fluid movement process? a. Hydrostatic pressure c. Diffusion b. Osmosis d. Active transport Hydrostatic pressure is the mechanical force of water pushing against cellular membranes. In the vascular system, hydrostatic pressure is the blood pressure generated in vessels by the contraction of the heart. Blood reaching the capillary bed has a hydrostatic pressure of 25 to 30 mm Hg, which is sufficient force to push water across the thin capillary membranes into the interstitial space. The remaining options do not correctly identify the process described. PTS: 1 REF: Pages 29-30 23. Why is osmolality preferred over osmolarity as the measurement of osmotic activity in the clinical assessment of individuals? a. Plasma contains sodium and chloride, which influence the volume of solution. b. Volume affects perfusion more than the weight of solutes. c. More of the weight of plasma is influenced by solutes, such as protein and glucose, rather than by water. d. Osmotic activity depends on the concentration of solutes present in plasma, such as proteins and glucose. In plasma, less of the plasma weight is water; therefore the overall concentration of particles is greater. The osmolality will be greater than the osmolarity because of the smaller proportion of water. Osmolality is thus the preferred measure of osmotic activity in clinical assessment of individuals. PTS: 1 REF: Page 30 24. A patient who has diarrhea receives a 3% saline solution intravenously to replace the sodium and chloride lost in the stool. What effect will this fluid replacement have on cells? a. Become hydrated c. Shrink b. Swell or burst d. Divide A hypertonic solution has a concentration of greater than 285 to 294 mOsm/kg. An example of a hypertonic solution is 3% saline solution. Water can be pulled out of the cells by a hypertonic solution; therefore the cells shrink. The remaining options do not correctly describe the effect identified in the stem. PTS: 1 REF: Page 31 25. The transport of glucose from the blood to the cell is accomplished by which process? a. Active-mediated transport (active transport) b. Active diffusion c. Passive osmosis d. Passive-mediated transport (facilitated diffusion) Facilitated diffusion is the means by which glucose is transported from the blood to the cells. The remaining options do not correctly identify this process. PTS: 1 REF: Pages 31-32 26. Potassium and sodium are transported across plasma membranes by: a. Passive electrolyte channels b. Coupled channels c. Adenosine triphosphatase (ATPase) enzyme d. Diffusion The exact mechanism for the transport of Na+ and K+ across the membrane is uncertain. One proposal is that ATPase enzyme induces the transporter protein to undergo several conformational changes, causing Na+ and K+ to move short distances (see Figure 1-29). The remaining options do not correctly describe the means by which K+ and Na+ are transported. PTS: 1 REF: Pages 32-33 27. What occurs during exocytosis? a. Macromolecules can be secreted across eukaryotic cell membranes. b. All substances are secreted into the cellular matrix. c. No repairs in the plasma membrane can take place. d. Solute molecules flow freely into and out of the cell. In eukaryotic cells, secretion of macromolecules almost always occurs by exocytosis. The remaining options do not correctly describe exocytosis. PTS: 1 REF: Pages 35-36 28. Why is it possible for potassium to diffuse easily into and out of cells? a. Potassium has a greater concentration in the intracellular fluid (ICF). b. Sodium has a greater concentration in the extracellular fluid (ECF). c. The resting plasma membrane is more permeable to potassium. d. An excess of anions are inside the cell. Because the resting plasma membrane is more permeable to K+ than to Na+, K+ can easily diffuse from its area of higher concentration in the ICF to its area of lower concentration in the ECF. Because Na+ and K+ are both cations, the net result is an excess of anions inside the cell, resulting in the resting membrane potential. The remaining options do not correctly identify the process that most easily diffuses K+. PTS: 1 REF: Page 36 29. The cellular uptake of the nutrient cholesterol depends on which process? a. Receptor-mediated exocytosis c. Receptor-mediated endocytosis b. Antiport system d. Passive transport The cellular uptake of nutrients, such as cholesterol, for example, depends on receptor-mediated endocytosis. Nutrients are not transported via the other options. PTS: 1 REF: Page 33 30. What causes the rapid change in the resting membrane potential to initiate an action potential? a. Potassium gates open, and potassium rushes into the cell, changing the membrane potential from negative to positive. b. Sodium gates open, and sodium rushes into the cell, changing the membrane potential from negative to positive. c. Sodium gates close, allowing potassium into the cell to change the membrane potential from positive to negative. d. Potassium gates close, allowing sodium into the cell to change the membrane potential from positive to negative. When a resting cell is stimulated through voltage-regulated channels, the cell membranes become more permeable to Na+. A net Na+ moves into the cell, and the membrane potential decreases, or moves forward, from a negative value (in millivolts) to zero. The Na+ gates open, and Na+ rushes into the cell, causing the membrane potential to reduce to zero and then become positive (depolarization). The remaining options do not correctly describe the change that initiates an action potential. PTS: 1 REF: Page 36 31. The action of platelet-derived growth factor is to stimulate the production of which cells? a. Platelets c. Connective tissue cells b. Epidermal cells d. Fibroblast cells Different types of cells require different factors; for example, platelet-derived growth factor stimulates the production of connective tissue cells. The remaining options do not correctly describe the action of platelet-derived growth factor. PTS: 1 REF: Page 39 32. The role of cytokines in cell reproduction is that they: a. Provide growth factor for tissue growth and development. b. Block progress of cell reproduction through the cell cycle. c. Restrain cell growth and development. d. Provide nutrients for cell growth and development. Growth factors, also called cytokines, are peptides that transmit signals within and among cells. They have a major role in the regulation of tissue growth and development (see Table 1-6). The remaining options do not correctly describe the role of cytokines in cell reproduction. PTS: 1 REF: Pages 38-39 33. What is the process of cellular reproduction? a. The process often takes months or years to complete. b. Cellular reproduction typically has a short interphase. c. Two diploid cells, called daughter cells, have been formed. d. The process involves the interaction of male and female cells. During telophase, the final stage, a new nuclear membrane is formed around each group of 46 chromosomes, the spindle fibers disappear, and the chromosomes begin to uncoil. Cytokinesis causes the cytoplasm to divide into roughly equal parts during this phase. At the end of telophase, two identical diploid cells, called daughter cells, have been formed from the original cell. PTS: 1 REF: Page 37 34. Which statement is true about eukaryotic cells? a. They lack distinct nucleus. b. They contain compartments called organelles. c. They lack an encasing nuclear membrane. d. They are smaller than the typical prokaryote cell. Eukaryotic cells have a characteristic set of membrane-bound intracellular compartments called organelles that include a well-defined nucleus and are larger than prokaryotes. The remaining statements are not true regarding eukaryotic cells. PTS: 1 REF: Page 2 35. Which statement is true about phagocytosis? a. Phagocytosis is an example of exocytosis. b. Phagocytosis is dependent on small vesicles. c. Phagocytosis involves the ingestion of bacteria. d. Phagocytosis focuses on solute molecules. In phagocytosis, the large molecular substances are engulfed by the plasma membrane and enter the cell so that they can be isolated and destroyed by lysosomal enzymes. Two types of endocytosis are designated, based on the size of the vesicle formed. Pinocytosis (cell drinking) involves the ingestion of fluids and solute molecules through the formation of small vesicles, and phagocytosis (cell eating) involves the ingestion of large particles, such as bacteria, through formation of large vesicles (also called vacuoles). Phagocytosis in an example of endocytosis, not exocytosis. PTS: 1 REF: Pages 33-34 36. A muscle cell possesses which specialized function? a. Movement c. Secretion b. Conductivity d. Respiration A cell has the potential to differentiation and to gain the ability to perform one of eight specialized functions. Muscle cells can generate forces that produce motion. Nerves cells are capable of conductivity. Cells of the adrenal gland, testis, and ovary can secrete. Respiration is a function that all cells possess. PTS: 1 REF: Page 2 37. When a mucous gland cell creates a new substance from previously absorbed material, this process is known as which specialized cellular function? a. Excretion c. Reproduction b. Metabolic absorption d. Secretion Certain cells, such as mucous gland cells, can synthesize new substances from substances they absorb and then secrete the new substances to serve elsewhere as needed. The other options are not used to describe the function described in the stem. PTS: 1 REF: Page 2 38. All cells are capable of what process? a. Excretion c. Metabolic absorption b. Movement d. Continuous division All cells have the capacity to excrete, thus allowing them to rid themselves of waste products resulting from the metabolic breakdown of nutrients. The remaining options are not functions possessed by all cells. PTS: 1 REF: Page 2 MULTIPLE RESPONSE 39. What are the major chemical components of the cell membranes? (Select all that apply.) a. Lipids b. Sodium ions c. Carbohydrates d. DNA e. Proteins , E The major chemical components of all cell membranes are lipids and proteins, but the percentage of each varies among different membranes. PTS: 1 REF: Page 12 40. Which cells lose their ability to replicate and divide? (Select all that apply.) a. Intestines b. Nerves c. Skin d. Lens of the eye e. Skeletal muscle , D, E All types of cells undergo mitosis during the formation of the embryo, but many adult cells, such as nerve cells, lens cells of the eye, and muscle cells, lose their ability to replicate and divide. Intestines and skin cells retain their ability to replicate and divide. PTS: 1 REF: Page 37 41. Which statements are true concerning the process of facilitated diffusion? (Select all that apply). a. Facilitated diffusion is also referred to as passive mediated transport. b. This process expends no metabolic energy. c. Moving solute molecules through cellular membranes are involved in this process. d. Movement up a concentration gradient is necessary. e. Facilitated diffusion is the primary means for water transport. , B, C In passive mediated transport, also called facilitated diffusion, the protein transporter moves solute molecules through cellular membranes without expending metabolic energy. Downward movement along a concentration gradient is necessary. Osmosis is the movement of water down a concentration gradient. PTS: 1 REF: Pages 31-32 42. Passive transport is dependent on: (Select all that apply.) a. Semipermeable barrier membrane b. The process of osmosis c. Diffusion as a driving force d. A living host e. Hydrostatic pressure , B, C, E Passive transport naturally occurs through any semipermeable barrier. It is driven by osmosis, hydrostatic pressure, and diffusion, all of which depend on the laws of physics and do not require life. PTS: 1 REF: Page 28 43. What is the primary function of proteins? (Select all that apply.) a. Proteins are binding units. b. Proteins are transport channels. c. Proteins are ribonucleoproteins. d. Proteins provide cell surface markers. e. Proteins are chemical reaction catalysts. , B, D, E Protein functions include (a) recognition and binding units (receptors) for substances moving in and out of the cell; (b) pores or transport channels; (c) enzymes that drive active pumps; (d) cell surface markers, such as glycoproteins; (e) cell adhesion molecules; and (f) catalysts of chemical reactions. PTS: 1 REF: Page 15 MATCHING Match the structure with its function. Answers may be used more than once. A. Endoplasmic reticulum B. Ribosome C. Secretory vesicle D. Lysosomes 44. Packages and transports proteins. 45. Fuses with the plasma membrane to release contents from the cell. 46. Synthesizes and transports lipids. 47. Provides energy to digest proteins into amino acids. 44. PTS: 1 REF: Pages 5-6 MSC: The endoplasmic reticulum (endo = within; plasma = cytoplasm; reticulum = network) is a membrane factory that specializes in the synthesis and transport of the protein and lipid components of most of the cell's organelles. 45. PTS: 1 REF: Pages 6-7 MSC: Proteins from the endoplasmic reticulum are processed and packaged into small membrane-bound sacs or vesicles called secretory vesicles that collect at the end of the membranous folds of the Golgi bodies. The secretory vesicles then break off from the Golgi complex and migrate to a variety of intracellular and extracellular destinations, including the plasma membrane. The vesicles fuse with the plasma membrane, and their contents are released from the cell. 46. PTS: 1 REF: Pages 5-6 MSC: The endoplasmic reticulum (endo = within; plasma = cytoplasm; reticulum = network) is a membrane factory that specializes in the synthesis and transport of the protein and lipid components of most of the cell's organelles. 47. PTS: 1 REF: Page 7 MSC: Lysosomes function as the intracellular digestive system (see Figure 1-6). Lysosomal enzymes are capable of digesting most cellular constituents down to their basic forms, such as amino acids, fatty acids, and sugars. Match the structure with its function. Answers may be used more than once. A. Passive-mediated transport B. Active-mediated transport C. Osmosis 48. Movement of water 49. Protein carrier 50. Facilitated diffusion 48. PTS: 1 REF: Page 30 MSC: Osmosis is the movement of water down a concentration gradient; that is, across a semipermeable membrane from a region of higher water concentration to a lower water concentration. 49. PTS: 1 REF: Page 32 MSC: In active-mediated transport, also called active transport, the protein transporter moves molecules against, or up, the concentration gradient. Unlike passive-mediated transport, active-mediated transport requires the expenditure of energy. 50. PTS: 1 REF: Pages 31-32 MSC: In passive-mediated transport, also called facilitated diffusion, the protein transporter moves solute molecules through cellular membranes without expending metabolic energy. Chapter 9 Alterations in Immunity and Inflammation Multiple Choice Identify the choice that best completes the statement or answers the question. 1. Hypersensitivity is best defined as a(an): a. Disturbance in the immunologic tolerance of self-antigens b. Immunologic reaction of one person to the tissue of another person c. Altered immunologic response to an antigen that results in disease d. Undetectable immune response in the presence of antigens 2. A hypersensitivity reaction that produces an allergic response is called: a. Hemolytic shock c. Necrotizing vasculitis b. Anaphylaxis d. Systemic erythematosus 3. The common hay fever allergy is expressed through a reaction that is mediated by which class of immunoglobulins? a. IgE c. IgM b. IgG d. T cells 4. Which type of antibody is involved in type I hypersensitivity reaction? a. IgA c. IgG b. IgE d. IgM 5. Blood transfusion reactions are an example of: a. Autoimmunity c. Homoimmunity b. Alloimmunity d. Hypersensitivity 6. During an IgE-mediated hypersensitivity reaction, which leukocyte is activated? a. Neutrophils c. Eosinophils b. Monocytes d. T lymphocytes 7. During an IgE-mediated hypersensitivity reaction, what causes bronchospasm? a. Bronchial edema caused by the chemotactic factor of anaphylaxis b. Bronchial edema caused by binding of the cytotropic antibody c. Smooth muscle contraction caused by histamine bound to H1 receptors d. Smooth muscle contraction caused by histamine bound to H2 receptors 8. During an IgE-mediated hypersensitivity reaction, the degranulation of mast cells is a result of which receptor action? a. Histamine bound to H2 b. Chemotactic factor binding to the receptor c. Epinephrine bound to mast cells d. Acetylcholine bound to mast cells 9. What characteristic do atopic individuals have that make them genetically predisposed to develop allergies? a. Greater quantities of histamine c. Greater quantities of IgE b. More histamine receptors d. A deficiency in epinephrine 10. What is the mechanism that results in type II hypersensitivity reactions? a. Antibodies coat mast cells by binding to receptors that signal its degranulation, followed by a discharge of preformed mediators. b. Antibodies bind to soluble antigens that were released into body fluids, and the immune complexes are then deposited in the tissues. c. Cytotoxic T lymphocytes or lymphokine-producing helper T 1 cells directly attack and destroy cellular targets. d. Antibodies bind to the antigens on the cell surface. 11. When mismatched blood is administered causing an ABO incompatibility, the erythrocytes are destroyed by: a. Complement-mediated cell lysis c. Phagocytosis in the spleen b. Phagocytosis by macrophages d. Natural killer cells 12. When antibodies are formed against red blood cell antigens of the Rh system, the blood cells are destroyed by: a. Complement-mediated cell lysis b. Phagocytosis by macrophages c. Phagocytosis in the spleen d. Neutrophil granules and toxic oxygen products 13. When soluble antigens from infectious agents enter circulation, tissue damage is a result of: a. Complement-mediated cell lysis b. Phagocytosis by macrophages c. Phagocytosis in the spleen d. Neutrophil granules and toxic oxygen products 14. How are target cells destroyed in a type II hypersensitivity reaction? a. Complement-mediated cell lysis b. Phagocytosis by macrophages c. Neutrophil granules and toxic oxygen products d. Natural killer cells 15. Graves disease (hyperthyroidism) is an example of which type II hypersensitivity reaction? a. Modulation b. Antibody-dependent cell-mediated cytotoxicity c. Neutrophil-mediated damage d. Complement-mediated lysis 16. Type III hypersensitivity reactions are a result of which of the following? a. Antibodies coating mast cells by binding to receptors that signal its degranulation, followed by the discharge of preformed mediators b. Antibodies binding to soluble antigens that were released into body fluids and the immune complexes being deposited in the tissues c. Tc cells or lymphokine-producing Th1 cells directly attacking and destroying cellular targets d. Antibodies binding to the antigen on the cell surface 17. A type IV hypersensitivity reaction causes which result? a. Antibodies coating mast cells by binding to receptors that signal its degranulation, followed by the discharge of preformed mediators b. Antibodies binding to soluble antigens that were released into body fluids and the immune complexes being deposited in the tissues c. Lymphokine-producing Th1 cells directly attacking and destroying cellular targets d. Antibodies binding to the antigen on the cell surface 18. In a type III hypersensitivity reaction, the harmful effects after the immune complexes that are deposited in tissues are a result of: a. Cytotoxic T cells c. Complement activation b. Natural killer cells d. Degranulation of mast cells 19. Raynaud phenomenon is classified as a type III hypersensitivity reaction and is due to: a. Immune complexes that are deposited in capillary beds, blocking circulation b. Mast cells that are bound to specific endothelial receptors, causing them to degranulate and creating a localized inflammatory reaction that occludes capillary circulation c. Cytotoxic T cells that attack and destroy the capillaries so that they are unable to perfuse local tissues d. Antibodies that detect the capillaries as foreign protein and destroy them using lysosomal enzymes and toxic oxygen species 20. Deficiencies in which element can produce depression of both B- and T-cell function? a. Iron c. Iodine b. Zinc d. Magnesium 21. When the maternal immune system becomes sensitized against antigens expressed by the fetus, what reaction occurs? a. T-cell immunity c. Fetal immunity b. Alloimmunity d. Autoimmunity 22. Tissue damage caused by the deposition of circulating immune complexes containing an antibody against the host DNA is the cause of which disease? a. Hemolytic anemia c. Systemic lupus erythematosus b. Pernicious anemia d. Myasthenia gravis 23. Why does tissue damage occurs in acute rejection after organ transplantation? a. Th1 cells release cytokines that activate infiltrating macrophages, and cytotoxic T cells directly attack the endothelial cells of the transplanted tissue. b. Circulating immune complexes are deposited in the endothelial cells of transplanted tissue, where the complement cascade lyses tissue. c. Receptors on natural killer cells recognize antigens on the cell surface of transplanted tissue, which releases lysosomal enzymes that destroy tissue. d. Antibodies coat the surface of transplanted tissue to which mast cells bind and liberate preformed chemical mediators that destroy tissue. 24. Which blood cell carries the carbohydrate antigens for blood type? a. Platelets c. Lymphocytes b. Neutrophils d. Erythrocytes 25. A person with type O blood is likely to have high titers of which anti-antibodies? 26. Which class of immunoglobulins forms isohemagglutinins? 27. Which component of the immune system is deficient in individuals with infections caused by viruses, fungi, or yeast? 28. In which primary immune deficiency is there a partial-to-complete absence of T-cell immunity? 29. How many months does it take for the newborn to be sufficiently protected by antibodies produced by its own B cells? 30. Considering the effects of nutritional deficiencies on the immune system, severe deficits in calories and protein lead to deficiencies in the formation of which immune cells? 31. Urticaria are a manifestation of a which type of hypersensitivity reaction? 32. Gr aves disease is a result of: a. Increased levels of circulating immunoglobulins b. The infiltration of the thyroid with T lymphocytes c. Autoantibodies binding to thyroid-stimulating hormone (TSH)-receptor sites d. Exposure to acetylates in substances such as rubber 33. Ra ynaud phenomenon is an example of which type of hypersensitivity? a. IV c. II b. III d. I 34. Which statement is true concerning an atopic individual? 35. Which statement is true regarding immunodeficiency? a. Immunodeficiency is generally not present in other family members. b. Immunodeficiency is never acquired; rather, it is congenital. c. Immunodeficiency is almost immediately symptomatic. d. Immunodeficiency is a result of a postnatal mutation. 36. A person with type O blood is considered to be the universal blood donor because type O blood contains which of the following? a. No antigens c. Both A and B antigens b. No antibodies d. Both A and B antibodies 37. Immunoglobulin E (IgE) is associated with which type of hypersensitivity reaction? a. I c. III b. II d. IV 38. Graves disease is an autoimmune disease that results in which maternal antibody? a. Binding with receptors for neural transmitters on muscle cells, causing neonatal muscular weakness b. Affecting the receptor for TSH, causing neonatal hyperthyroidism c. Inducing anomalies in the fetus or causing pregnancy loss d. Destroying platelets in the fetus and neonate Multiple Response Identify one or more choices that best complete the statement or answer the question. 39. When a tuberculin skin test is positive, the hard center and erythema surrounding the induration are a result of which of the following? (Select all that apply.) a. Histamine d. Products of complement b. T lymphocytes e. Macrophages c. Immune complexes 40. Exposure to which of the following could result in a type IV hypersensitivity reaction? (Select all that apply.) a. Poison ivy d. Nickel b. Neomycin e. Detergents c. Dairy products 41. Which disorders are considered autoimmune? (Select all that apply.) a. Crohn disease d. Systemic lupus erythematosus b. Addison disease e. Noninsulin-dependent diabetes c. Rheumatoid arthritis 42. Which statements best define acute rejection? (Select all that apply.) a. Acute rejection is a cell-mediated immune response. b. Acute rejection is usually a type III rejection. c. Immunosuppressive drugs delay or lessen the intensity of an acute rejection. d. Acute rejection is associated with the body’s response to an organ transplant. e. Acute rejection is a response against unmatched human leukocyte antigens (HLAs). Thought Question 1. Briefly compare and contrast the four different types of hypersensitivity reactions. Be able to discuss the mediators and responses to the mediators; provide an example of each reaction. 2. Briefly describe the pathophysiologic mechanism of the autoimmune disorders SLE and transfusion reactions. 3. What are the pathophysiologic principles on which gamma-globulin therapies are based? Chapter 9 Alterations in Immunity and Inflammation Multiple Choice Identify the choice that best completes the statement or answers the question. 1. Hypersensitivity is best defined as a(an): a. Disturbance in the immunologic tolerance of self-antigens b. Immunologic reaction of one person to the tissue of another person c. Altered immunologic response to an antigen that results in disease d. Undetectable immune response in the presence of antigens 2. A hypersensitivity reaction that produces an allergic response is called: a. Hemolytic shock c. Necrotizing vasculitis b. Anaphylaxis d. Systemic erythematosus 3. The common hay fever allergy is expressed through a reaction that is mediated by which class of immunoglobulins? a. IgE c. IgM b. IgG d. T cells 4. Which type of antibody is involved in type I hypersensitivity reaction? a. IgA c. IgG b. IgE d. IgM 5. Blood transfusion reactions are an example of: a. Autoimmunity c. Homoimmunity b. Alloimmunity d. Hypersensitivity 6. During an IgE-mediated hypersensitivity reaction, which leukocyte is activated? a. Neutrophils c. Eosinophils b. Monocytes d. T lymphocytes 7. During an IgE-mediated hypersensitivity reaction, what causes bronchospasm? a. Bronchial edema caused by the chemotactic factor of anaphylaxis b. Bronchial edema caused by binding of the cytotropic antibody c. Smooth muscle contraction caused by histamine bound to H1 receptors d. Smooth muscle contraction caused by histamine bound to H2 receptors 8. During an IgE-mediated hypersensitivity reaction, the degranulation of mast cells is a result of which receptor action? a. Histamine bound to H2 b. Chemotactic factor binding to the receptor c. Epinephrine bound to mast cells d. Acetylcholine bound to mast cells 9. What characteristic do atopic individuals have that make them genetically predisposed to develop allergies? a. Greater quantities of histamine c. Greater quantities of IgE b. More histamine receptors d. A deficiency in epinephrine 10. What is the mechanism that results in type II hypersensitivity reactions? a. Antibodies coat mast cells by binding to receptors that signal its degranulation, followed by a discharge of preformed mediators. b. Antibodies bind to soluble antigens that were released into body fluids, and the immune complexes are then deposited in the tissues. c. Cytotoxic T lymphocytes or lymphokine-producing helper T 1 cells directly attack and destroy cellular targets. d. Antibodies bind to the antigens on the cell surface. 11. When mismatched blood is administered causing an ABO incompatibility, the erythrocytes are destroyed by: a. Complement-mediated cell lysis c. Phagocytosis in the spleen b. Phagocytosis by macrophages d. Natural killer cells 12. When antibodies are formed against red blood cell antigens of the Rh system, the blood cells are destroyed by: a. Complement-mediated cell lysis b. Phagocytosis by macrophages c. Phagocytosis in the spleen d. Neutrophil granules and toxic oxygen products 13. When soluble antigens from infectious agents enter circulation, tissue damage is a result of: a. Complement-mediated cell lysis b. Phagocytosis by macrophages c. Phagocytosis in the spleen d. Neutrophil granules and toxic oxygen products 14. How are target cells destroyed in a type II hypersensitivity reaction? a. Complement-mediated cell lysis b. Phagocytosis by macrophages c. Neutrophil granules and toxic oxygen products d. Natural killer cells 15. Graves disease (hyperthyroidism) is an example of which type II hypersensitivity reaction? a. Modulation b. Antibody-dependent cell-mediated cytotoxicity c. Neutrophil-mediated damage d. Complement-mediated lysis 16. Type III hypersensitivity reactions are a result of which of the following? a. Antibodies coating mast cells by binding to receptors that signal its degranulation, followed by the discharge of preformed mediators b. Antibodies binding to soluble antigens that were released into body fluids and the immune complexes being deposited in the tissues c. Tc cells or lymphokine-producing Th1 cells directly attacking and destroying cellular targets d. Antibodies binding to the antigen on the cell surface 17. A type IV hypersensitivity reaction causes which result? a. Antibodies coating mast cells by binding to receptors that signal its degranulation, followed by the discharge of preformed mediators b. Antibodies binding to soluble antigens that were released into body fluids and the immune complexes being deposited in the tissues c. Lymphokine-producing Th1 cells directly attacking and destroying cellular targets d. Antibodies binding to the antigen on the cell surface 18. In a type III hypersensitivity reaction, the harmful effects after the immune complexes that are deposited in tissues are a result of: a. Cytotoxic T cells c. Complement activation b. Natural killer cells d. Degranulation of mast cells 19. Raynaud phenomenon is classified as a type III hypersensitivity reaction and is due to: a. Immune complexes that are deposited in capillary beds, blocking circulation b. Mast cells that are bound to specific endothelial receptors, causing them to degranulate and creating a localized inflammatory reaction that occludes capillary circulation c. Cytotoxic T cells that attack and destroy the capillaries so that they are unable to perfuse local tissues d. Antibodies that detect the capillaries as foreign protein and destroy them using lysosomal enzymes and toxic oxygen species 20. Deficiencies in which element can produce depression of both B- and T-cell function? a. Iron c. Iodine b. Zinc d. Magnesium 21. When the maternal immune system becomes sensitized against antigens expressed by the fetus, what reaction occurs? a. T-cell immunity c. Fetal immunity b. Alloimmunity d. Autoimmunity 22. Tissue damage caused by the deposition of circulating immune complexes containing an antibody against the host DNA is the cause of which disease? a. Hemolytic anemia c. Systemic lupus erythematosus b. Pernicious anemia d. Myasthenia gravis 23. Why does tissue damage occurs in acute rejection after organ transplantation? a. Th1 cells release cytokines that activate infiltrating macrophages, and cytotoxic T cells directly attack the endothelial cells of the transplanted tissue. b. Circulating immune complexes are deposited in the endothelial cells of transplanted tissue, where the complement cascade lyses tissue. c. Receptors on natural killer cells recognize antigens on the cell surface of transplanted tissue, which releases lysosomal enzymes that destroy tissue. d. Antibodies coat the surface of transplanted tissue to which mast cells bind and liberate preformed chemical mediators that destroy tissue. 24. Which blood cell carries the carbohydrate antigens for blood type? a. Platelets c. Lymphocytes b. Neutrophils d. Erythrocytes 25. A person with type O blood is likely to have high titers of which anti-antibodies? 26. Which class of immunoglobulins forms isohemagglutinins? 27. Which component of the immune system is deficient in individuals with infections caused by viruses, fungi, or yeast? 28. In which primary immune deficiency is there a partial-to-complete absence of T-cell immunity? 29. How many months does it take for the newborn to be sufficiently protected by antibodies produced by its own B cells? 30. Considering the effects of nutritional deficiencies on the immune system, severe deficits in calories and protein lead to deficiencies in the formation of which immune cells? 31. Urticaria are a manifestation of a which type of hypersensitivity reaction? 32. Gr aves disease is a result of: a. Increased levels of circulating immunoglobulins b. The infiltration of the thyroid with T lymphocytes c. Autoantibodies binding to thyroid-stimulating hormone (TSH)-receptor sites d. Exposure to acetylates in substances such as rubber 33. Ra ynaud phenomenon is an example of which type of hypersensitivity? a. IV c. II b. III d. I 34. Which statement is true concerning an atopic individual? 35. Which statement is true regarding immunodeficiency? a. Immunodeficiency is generally not present in other family members. b. Immunodeficiency is never acquired; rather, it is congenital. c. Immunodeficiency is almost immediately symptomatic. d. Immunodeficiency is a result of a postnatal mutation. 36. A person with type O blood is considered to be the universal blood donor because type O blood contains which of the following? a. No antigens c. Both A and B antigens b. No antibodies d. Both A and B antibodies 37. Immunoglobulin E (IgE) is associated with which type of hypersensitivity reaction? a. I c. III b. II d. IV 38. Graves disease is an autoimmune disease that results in which maternal antibody? a. Binding with receptors for neural transmitters on muscle cells, causing neonatal muscular weakness b. Affecting the receptor for TSH, causing neonatal hyperthyroidism c. Inducing anomalies in the fetus or causing pregnancy loss d. Destroying platelets in the fetus and neonate Multiple Response Identify one or more choices that best complete the statement or answer the question. 39. When a tuberculin skin test is positive, the hard center and erythema surrounding the induration are a result of which of the following? (Select all that apply.) a. Histamine d. Products of complement b. T lymphocytes e. Macrophages c. Immune complexes 40. Exposure to which of the following could result in a type IV hypersensitivity reaction? (Select all that apply.) a. Poison ivy d. Nickel b. Neomycin e. Detergents c. Dairy products 41. Which disorders are considered autoimmune? (Select all that apply.) a. Crohn disease d. Systemic lupus erythematosus b. Addison disease e. Noninsulin-dependent diabetes c. Rheumatoid arthritis 42. Which statements best define acute rejection? (Select all that apply.) a. Acute rejection is a cell-mediated immune response. b. Acute rejection is usually a type III rejection. c. Immunosuppressive drugs delay or lessen the intensity of an acute rejection. d. Acute rejection is associated with the body’s response to an organ transplant. e. Acute rejection is a response against unmatched human leukocyte antigens (HLAs). Thought Question 1. Briefly compare and contrast the four different types of hypersensitivity reactions. Be able to discuss the mediators and responses to the mediators; provide an example of each reaction. 2. Briefly describe the pathophysiologic mechanism of the autoimmune disorders SLE and transfusion reactions. 3. What are the pathophysiologic principles on which gamma-globulin therapies are based? Chapter 7: Innate Immunity: Inflammation MULTIPLE CHOICE 1. Which action is a purpose of the inflammatory process? a. To provide specific responses toward antigens b. To lyse cell membranes of microorganisms c. To prevent infection of the injured tissue d. To create immunity against subsequent tissue injury If the epithelial barrier is damaged, then a highly efficient local and systemic response (inflammation) is mobilized to limit the extent of damage, to protect against infection, and to initiate the repair of damaged tissue. The other options do not accurately identify a purpose of the inflammatory process. PTS: 1 REF: Page 191 2. How do surfactant proteins A through D provide innate resistance? a. Initiate the complement cascade. c. Secrete mucus. b. Promote phagocytosis. d. Synthesize lysosomes. The lung produces and secretes a family of glycoproteins, collectins, which includes surfactant proteins A through D and mannose-binding lectin. Collectin binding facilitates macrophages to recognize the microorganism, enhancing macrophage attachment, phagocytosis, and killing. The other options do not accurately identify how surfactant proteins provide innate resistance. PTS: 1 REF: Page 194 3. Which secretion is a first line of defense against pathogen invasion that involves antibacterial and antifungal fatty acids, as well as lactic acid? a. Optic tears c. Sweat gland perspiration b. Oral saliva d. Sebaceous gland sebum Sebaceous glands in the skin secrete sebum that is made up of antibacterial and antifungal fatty acids and lactic acid that provide the first-line barrier against pathogen invasion. PTS: 1 REF: Pages 192-193 4. Which bacterium grows in the intestines after prolonged antibiotic therapy? a. Lactobacillus c. Clostridium difficile b. Candida albicans d. Helicobacter pylori Prolonged antibiotic treatment can alter the normal intestinal flora, decreasing its protective activity and leading to the overgrowth of other microorganisms, such as the yeast C. albicans or the bacterium C. difficile. The other options do not accurately identify intestinal bacterium whose growth is a result of prolonged antibiotic therapy. PTS: 1 REF: Page 194 5. What causes the edema that occurs during the inflammatory process? a. Vasodilation of blood vessels c. Endothelial cell contraction b. Increased capillary permeability d. Emigration of neutrophils The increased flow and capillary permeability result in a leakage of plasma from the vessels, causing swelling (edema) in the surrounding tissue and is solely responsible for inflammation-induced edema. PTS: 1 REF: Page 195 6. What process causes heat and redness to occur during the inflammatory process? a. Vasodilation of blood vessels c. Decreased capillary permeability b. Platelet aggregation d. Endothelial cell contraction The increased blood flow as a result of vasodilation and increasing concentration of red cells at the site of inflammation cause locally increased warmth and redness. The other options do not accurately identify the process that results in inflammatory redness and heat. PTS: 1 REF: Page 195 7. Activation of the classical pathway begins with: a. Viruses c. Mast cells b. Antigen-antibody complexes d. Macrophages Activation of the classical pathway begins only with the activation of protein C1 and is preceded by the formation of a complex between an antigen and an antibody to form an antigen-antibody complex (immune complex) (see Chapter 8). PTS: 1 REF: Page 197 8. What plasma protein system forms a fibrinous meshwork at an inflamed site? a. Complement c. Kinin b. Coagulation d. Fibrinolysis The coagulation (clotting) system is a group of plasma proteins that form a fibrinous meshwork at an injured or inflamed site. This protein system (1) prevents the spread of infection to adjacent tissues, (2) traps microorganisms and foreign bodies at the site of inflammation for removal by infiltrating cells (e.g., neutrophils and macrophages), (3) forms a clot that stops the bleeding, and (4) provides a framework for future repair and healing. The other options do not accurately identify such a protein system. PTS: 1 REF: Page 199 | Page 201 9. Which component of the plasma protein system tags pathogenic microorganisms for destruction by neutrophils and macrophages? a. Complement cascade c. Kinin system b. Coagulation system d. Immune system C3b (a component of the complement cascade) adheres to the surface of a pathogenic microorganism and serves as an efficient opsonin. Opsonins are molecules that tag microorganisms for destruction by cells of the inflammatory system, primarily neutrophils and macrophages. The other options do not accurately identify a component capable of tagging pathogenic microorganisms. PTS: 1 REF: Pages 197-199 10. What is the vascular effect of histamine released from mast cells? a. Platelet adhesion c. Vasodilation b. Initiation of the clotting cascade d. Increased endothelial adhesiveness C2b affects smooth muscle, causing vasodilation and increased vascular permeability. C3a, C5a, and, to a limited extent, C4a are anaphylatoxins; that is, they induce rapid mast cell degranulation (i.e., release of granular contents) and the release of histamine, causing vasodilation and increased capillary permeability. The other options do not accurately describe the vascular effect of histamine released from mast cells? PTS: 1 REF: Pages 198-199 11. What is an outcome of the complement cascade? a. Activation of the clotting cascade b. Prevention of the spread of infection to adjacent tissues c. Inactivation of chemical mediators such as histamine d. Lysis of bacterial cell membranes The complement cascade can be activated by at least three different means, and its products have four functions: (1) anaphylatoxic activity, resulting in mast cell degranulation, (2) leukocyte chemotaxis, (3) opsonization, and (4) cell lysis. The other options do not accurately describe an outcome of the complement cascade. PTS: 1 REF: Page 199 12. The function of opsonization related to the complement cascade is to: a. Tag of pathogenic microorganisms for destruction by neutrophils and macrophages. b. Process pathogenic microorganisms so that activated lymphocytes can be created for acquired immunity. c. Destroy glycoprotein cell membranes of pathogenic microorganisms. d. Promote anaphylatoxic activity, resulting in mast cell degranulation. C3b adheres to the surface of a pathogenic microorganism and serves as an efficient opsonin. Opsonins are molecules that tag microorganisms for destruction by cells of the inflammatory system, primarily neutrophils and macrophages. The other options do not accurately describe the function of opsonization related to the complement cascade. PTS: 1 REF: Page 199 13. In the coagulation (clotting) cascade, the intrinsic and the extrinsic pathways converge at which factor? a. XII c. X b. VII d. V The coagulation cascade consists of the extrinsic and intrinsic pathways that converge only at factor X. PTS: 1 REF: Page 201 14. Which chemical interacts among all plasma protein systems by degrading blood clots, activating complement, and activating the Hageman factor? a. Kallikrein c. Bradykinin b. Histamine d. Plasmin Only plasmin regulates clot formation by degrading fibrin and fibrinogen, and it can activate the complement cascade through components C1, C3, and C5. Plasmin can activate the plasma kinin cascade by activating the Hageman factor (factor XII) and producing prekallikrein activator. PTS: 1 REF: Page 201 15. The chemotactic factor affects the inflammatory process by: a. Causing vasodilation around the inflamed area b. Stimulating smooth muscle contraction in the inflamed area c. Directing leukocytes to the inflamed area d. Producing edema around the inflamed area Two chemotactic factors, neutrophil chemotactic factor (NCF) and eosinophil chemotactic factor of anaphylaxis (ECF-A), are released during mast cell degranulation. NCF attracts neutrophils (a type of leukocytes), and ECF-A attracts eosinophils to the site of inflammation. The other options do not accurately describe the affect chemotactic factors have on the inflammatory process. PTS: 1 REF: Page 207 16. What affect does the process of histamine binding to the histamine-2 (H2) receptor have on inflammation? a. Inhibition c. Acceleration b. Activation d. Termination Binding of histamine to the H1 receptor is essentially proinflammatory; that is, it promotes inflammation. On the other hand, binding histamine to the H2 receptor is generally antiinflammatory because it results in the suppression of leukocyte function. The other options do not accurately describe the affect histamine binding to the H2 receptor has on inflammation. PTS: 1 REF: Page 206 17. Frequently when H1 and H2 receptors are located on the same cells, they act in what fashion? a. Synergistically c. Antagonistically b. Additively d. Agonistically Both types of receptors are distributed among many different cells and are often present on the same cells and may act in an antagonistic fashion. For instance, neutrophils express both types of receptors, with stimulation of H1 receptors resulting in the augmentation of neutrophil chemotaxis and H2 stimulation resulting in its inhibition. The other options do not accurately describe the relationship between H1 and H2 receptors. PTS: 1 REF: Page 207 18. Some older adults have impaired inflammation and wound healing because of which problem? a. Circulatory system cannot adequately perfuse tissues. b. Complement and chemotaxis are deficient. c. Underlying chronic illness(es) exists. d. Number of mast cells is insufficient. In some cases, impaired healing is not directly associated with aging, in general, but can instead be linked to a chronic illness such as cardiovascular disease or diabetes mellitus. The other problems are not related to the aging process. PTS: 1 REF: Page 220 19. Which chemical mediator derived from mast cells retracts endothelial cells to increase vascular permeability and to cause leukocyte adhesion to endothelial cells? a. Leukotrienes c. Platelet-activating factor b. Prostaglandin E d. Bradykinin The biologic activity of platelet-activating factor is virtually identical to that of leukotrienes; namely, it causes endothelial cell retraction to increase vascular permeability, leukocyte adhesion to endothelial cells, and platelet activation. The other options do not accurately identify the chemical mediator derived from the process described in the question. PTS: 1 REF: Page 207 20. What is the inflammatory effect of nitric oxide (NO)? a. Increases capillary permeability, and causes pain. b. Increases neutrophil chemotaxis and platelet aggregation. c. Causes smooth muscle contraction and fever. d. Decreases mast cell function, and decreases platelet aggregation. Effects of NO on inflammation include vasodilation by inducing relaxation of vascular smooth muscle, a response that is local and short lived, and by suppressing mast cell function, as well as platelet adhesion and aggregation. The other options do not accurately identify the effect of NO on the process of inflammation. PTS: 1 REF: Page 209 21. What is the correct sequence in phagocytosis? a. Engulfment, recognition, fusion, destruction b. Fusion, engulfment, recognition, destruction c. Recognition, engulfment, fusion, destruction d. Engulfment, fusion, recognition, destruction Once the phagocytic cell enters the inflammatory site, the only correct sequence of phagocytosis involves the following steps: (1) opsonization, or recognition, of the target and adherence of the phagocyte to it; (2) engulfment, or ingestion or endocytosis, and the formation of phagosome; (3) fusion with lysosomal granules within the phagocyte (phagolysosome); and (4) destruction of the target. PTS: 1 REF: Pages 210-211 22. When considering white blood cell differentials, acute inflammatory reactions are related to elevations of which leukocyte? a. Monocytes c. Neutrophils b. Eosinophils d. Basophils Only neutrophils are the predominant phagocytes in the early inflammatory site, arriving within 6 to 12 hours after the initial injury, they ingest (phagocytose) bacteria, dead cells, and cellular debris at the inflammatory site. PTS: 1 REF: Pages 208-209 23. In the later stages of an inflammatory response, which phagocytic cell is predominant? a. Neutrophils c. Chemokines b. Monocytes d. Eosinophils Only monocytes and macrophages perform many of the same functions as neutrophils but for a longer time and in a later stage of the inflammatory response. PTS: 1 REF: Page 209 | Page 212 24. In regulating vascular mediators released from mast cells, the role of eosinophils is to release: a. Arylsulfatase B, which stimulates the formation of B lymphocytes b. Histaminase, which limits the effects of histamine during acute inflammation c. Lysosomal enzymes, which activate mast cell degranulation during acute inflammation d. Immunoglobulin E, which defends the body against parasites Eosinophil lysosomes contain several enzymes that degrade vasoactive molecules, thereby controlling the vascular effects of inflammation. These enzymes include histaminase, which mediates the degradation of histamine, and arylsulfatase B, which mediates the degradation of some of the lipid-derived mediators produced by mast cells. The other options do not accurately describe the role of eosinophils. PTS: 1 REF: Page 209 25. What is the role of a natural killer (NK) cells? a. Initiation of the complement cascade b. Elimination of malignant cells c. Binding tightly to antigens d. Proliferation after immunization with antigen The main function of NK cells is to recognize and eliminate cells infected with viruses, although they are also somewhat effective at eliminating other abnormal host cells, specifically cancer cells. The other options do not accurately identify the role of a NK cell. PTS: 1 REF: Page 213 26. Which cytokine is produced and released from virally infected host cells? a. IL-1 c. TNF- b. IL-10 d. IFN- Only interferons (IFNs) are produced and released by virally infected cells in response to viral double-stranded ribonucleic acid (RNA). IFN- and IFN- induce the production of antiviral proteins, thereby conferring protection on uninfected cells. IFN- or IFN- is released from virally infected cells and attaches to a receptor on a neighboring cell. IFNs also enhance the efficiency of developing an acquired immune response. PTS: 1 REF: Pages 204-205 27. IFN- is secreted from which cells? a. Virally infected cells c. Macrophages b. Bacterial infected cells d. Mast cells Different kinds of interferons (IFNs) are produced by different types of cells—macrophages are the primary producers of both IFN- and IFN-. The other options do not accurately identify cells secreted by IFN-. PTS: 1 REF: Pages 204-205 28. Which manifestation of inflammation is systemic? a. Formation of exudates c. Redness and heat b. Fever and leukocytosis d. Pain and edema The only three primary systemic changes associated with the acute inflammatory response are fever, leukocytosis (a transient increase in circulating leukocytes), and increased levels in circulating plasma proteins. PTS: 1 REF: Page 213 29. The acute inflammatory response is characterized by fever that is produced by the hypothalamus being affected by: a. Endogenous pyrogens c. Antigen-antibody complexes b. Bacterial endotoxin d. Exogenous pyrogens Fever-causing cytokines are known as endogenous pyrogens. These pyrogens act directly on the hypothalamus, which is the portion of the brain that controls the body’s thermostat. The other options do not accurately describe the cause of fever related to the effects on the hypothalamus. PTS: 1 REF: Page 213 30. What occurs during the process of repair after tissue damage? a. Nonfunctioning scar tissue replaces destroyed tissue. b. Regeneration occurs; the original tissue is replaced. c. Resolution occurs; tissue is regenerated. d. Epithelialization replaces destroyed tissue. Repair is the replacement of destroyed tissue with scar tissue. Scar tissue is primarily made up of collagen, which fills in the lesion and restores tensile strength but cannot carry out the physiologic functions of the destroyed tissue. The other options do not accurately describe the process of repair after tissue damage. PTS: 1 REF: Pages 215-216 31. The role of fibroblasts during the reconstructive phase of wound healing is to: a. Generate new capillaries from vascular endothelial cells around the wound. b. Establish connections between neighboring cells and contract their fibers. c. Synthesize and secrete collagen and the connective tissue proteins. d. Provide enzymes that débride the wound bed of dead cells. Fibroblasts are the most important cells during the reconstructive phase of wound healing because they synthesize and secrete collagen and other connective tissue proteins. Macrophage-derived transforming growth factor–beta (TGF-ß) stimulates fibroblasts. The other options do not accurately describe the role of fibroblasts in the reconstructive phase of wound healing. PTS: 1 REF: Pages 216-218 32. A keloid is the result of which dysfunctional wound healing response? a. Epithelialization c. Collagen matrix assembly b. Contraction d. Maturation An imbalance between collagen synthesis and collagen degradation, during which synthesis is increased relative to degradation, causes both keloids and hypertrophic scars. The other options are not involved in keloids production. PTS: 1 REF: Page 219 33. Which solution is best to use when cleaning a wound that is healing by epithelialization? a. Normal saline c. Hydrogen peroxide b. Povidone-iodine d. Dakin solution Normal saline is the most innocuous solution that can be used to cleanse or irrigate a wound that is primarily healing by epithelialization and is the only correct answer for this question. PTS: 1 REF: Page 220 34. Many neonates have a transient depressed inflammatory response as a result of which condition? a. The circulatory system is too immature to perfuse tissues adequately. b. Complement and chemotaxis are deficient. c. Mast cells are lacking. d. The respiratory system is too immature to deliver oxygen to tissues. Neonates commonly have transiently depressed inflammatory and immune function partially as a result of a deficiency in components of the alternative pathway. For example, neutrophils and perhaps monocytes may not be capable of efficient chemotaxis. The other options do not accurately explain the common cause of a transient depressed inflammatory response in neonates. PTS: 1 REF: Page 220 35. During phagocytosis, what is occurring during the step referred to as opsonization? a. Phagocytes recognize and adhere to the bacteria. b. Microorganisms are ingested. c. Microorganisms are killed and digested. d. An intracellular phagocytic vacuole is formed. During phagocytosis, opsonization involves only the recognition and adherence of phagocytes to bacteria. PTS: 1 REF: Page 210 36. Fusion is the step in phagocytosis during which: a. Microorganisms are killed and digested. b. An intracellular phagocytic vacuole is formed. c. Lysosomal granules enter the phagocyte. d. Microorganisms are ingested. Fusion occurs with lysosomal granules entering the phagocyte (phagolysosome). The remaining options do not accurately describe fusion as a step in phagocytosis. PTS: 1 REF: Page 210 37. During the process of endocytosis, the phagosome step results in: a. Microorganisms are ingested. b. Microorganisms are killed and digested. c. Phagocytes recognize and adhere to bacteria. d. An intracellular phagocytic vacuole is formed. Small pseudopods that extend from the plasma membrane and surround the adherent microorganism, forming an intracellular phagocytic vacuole or phagosome, carry out engulfment (endocytosis). The membrane that surrounds the phagosome consists of inverted plasma membrane. After the formation of the phagosome, lysosomes converge, fuse with the phagosome, and discharge their contents, creating a phagolysosome. PTS: 1 REF: Pages 210-211 38. When cellular damage occurs and regeneration is minor with no significant complications, the process of returning the cells to preinjury function is referred to as: a. Restoration c. Regrowth b. Resolution d. Replacement If damage is minor with no complications and destroyed tissues are capable of regeneration, then returning the injured tissues to an approximation of their original structure and physiologic function is possible. This restoration is called resolution. The other terms are not used to describe this process. PTS: 1 REF: Page 215 39. Newborns often have deficiencies in collectin-like proteins, making them more susceptible to what type of infection? a. Cardiac c. Respiratory b. Urinary d. Gastrointestinal Neonates may also be deficient in some of the collectins and collectin-like proteins. This deficiency is especially true of preterm neonates. Some preterm infants with respiratory distress syndrome are deficient in at least one collectin, which negatively affects its innate defense against respiratory infections. The other options are not necessarily related to collectin deficiencies. PTS: 1 REF: Page 220 40. Which cell is the body’s primary defense against parasite invasion? a. Eosinophil c. T lymphocytes b. Neutrophils d. B lymphocytes Eosinophils serve as the body’s primary defense against parasites. T lymphocytes and B lymphocytes are involved in acquired immunity. Neutrophils are the predominant phagocytes in the early inflammatory site. PTS: 1 REF: Page 209 MULTIPLE RESPONSE 41. Which chemical mediators induce pain during an inflammatory response? (Select all that apply.) a. Prostaglandins b. Leukotrienes c. Tryptase d. Phospholipase e. Bradykinin , E The only chemical mediators that induce pain during an inflammatory response are the prostaglandins and bradykinin. PTS: 1 REF: Page 201 42. Sebaceous glands protect the body from infection by secreting: (Select all that apply.) a. Antibacterial fatty acids b. Antifungal fatty acids c. Ascorbic acid d. Lactic acid e. Hydrochloric acid , B, D Sebaceous glands secrete only antibacterial and antifungal fatty acids and lactic acid. PTS: 1 REF: Pages 192-193 43. Which body fluid has the ability to attack the cell walls of gram-positive bacteria? (Select all that apply.) a. Perspiration b. Semen c. Tears d. Saliva e. Urine , C, D Only perspiration, tears, and saliva contain an enzyme (lysozyme) that attacks the cell walls of gram-positive bacteria. PTS: 1 REF: Page 193 44. The main function of NK cells includes: (Select all that apply.) a. Recognizing virus-infected cells b. Eliminating virus-infected cells c. Recognizing bacteria-infected cells d. Eliminating bacteria-infected cells e. Eliminating previously identified cancer cells , B, E The main functions of NK cells are recognizing and eliminating cells infected with viruses, not bacteria. They are also somewhat effective at eliminating other abnormal host cells, specifically cancer cells. PTS: 1 REF: Page 213 45. Normal bacterial flora found in the intestines produce vitamin K to assist in the absorption of which of the following? (Select all that apply.) a. Calcium b. Fatty acids c. Large polysaccharides d. Iron e. Magnesium , D, E The flora’s production of vitamin K is needed to absorb various ions, such as calcium, iron, and magnesium. Normal intestinal flora is responsible for digesting fatty acids, large polysaccharides, and other dietary substance, but such digestion is not reliant on vitamin K. PTS: 1 REF: Page 194 46. An individual’s acquired immunity is dependent on the function of which cells? (Select all that apply.) a. T lymphocytes b. B lymphocytes c. Macrophages d. Opsonins e. Neutrophils , B, C T lymphocytes, B lymphocytes, macrophages, and dendritic cells are involved in acquired immunity. Opsonins are molecules that tag microorganisms for destruction by cells of the inflammatory system; these cells are primarily neutrophils. PTS: 1 REF: Page 192 | Table 7-1 47. An example of a pathogen capable of surviving and even multiplying inside a macrophage is known as: (Select all that apply.) a. Mycobacterium tuberculosis (tuberculosis) b. Mycobacterium leprae (leprosy) c. Salmonella typhi (typhoid fever) d. Clostridium difficile e. Brucella abortus (brucellosis) , B, C, E Several bacteria are resistant to killing by granulocytes and can even survive inside macrophages. Microorganisms such as M. tuberculosis (tuberculosis), M. leprae (leprosy), S. typhi (typhoid fever), and B. abortus (brucellosis) can remain dormant or even multiply inside the phagolysosomes of macrophages. C. difficile is said to be resistant to antibiotics, making it difficult to control. PTS: 1 REF: Pages 212-213 48. An older adult is particularly susceptible to infections of which body parts? (Select all that apply.) a. Lungs b. Skin c. Liver d. Eyes e. Bladder , B, E Older adults have increased susceptibility to bacterial infections of the lungs, urinary tract, and skin. Other infections may occur but on an individualized basis. PTS: 1 REF: Page 220 MATCHING Match each step of phagocytosis with its function. A. Opsonization B. Engulfment C. Phagosome D. Fusion E. Destruction 49. Microorganisms are ingested. 50. Microorganisms are killed and digested. 51. Phagocytes gain enhanced recognition and adherence of bacteria. 52. Lysosomal granules enter the phagocyte. 53. Intracellular phagocytic vacuole is formed. 49. PTS: 1 REF: Pages 210-211 MSC: Engulfment is the ingestion of phagosomes. 50. ANS: E PTS: 1 REF: Pages 210-211 MSC: Destruction is the step during which microorganisms are killed and digested. 51. PTS: 1 REF: Pages 210-211 MSC: Opsonization is the recognition and adherence of phagocytes to bacteria. 52. PTS: 1 REF: Pages 210-211 MSC: Fusion occurs with lysosomal granules within the phagocyte (phagolysosome). 53. PTS: 1 REF: Pages 210-211 MSC: Small pseudopods that extend from the plasma membrane and surround the adherent microorganism, forming an intracellular phagocytic vacuole or phagosome, carry out engulfment (endocytosis). Chapter 8: Adaptive Immunity MULTIPLE CHOICE 1. Which primary characteristic is unique for the immune response? a. The immune response is similar each time it is activated. b. The immune response is specific to the antigen that initiates it. c. The response to a specific pathogen is short term. d. The response is innate, rather than acquired. Unlike inflammation, which is nonspecifically activated by cellular damage and pathogenic microorganisms, the immune response is primarily designed to afford long-term specific protection (i.e., immunity) against particular invading microorganisms; that is, it has a memory function. The other options are not unique characteristics of the immune response. PTS: 1 REF: Page 225 2. In which structure does B lymphocytes mature and undergo changes that commit them to becoming B cells? a. Thymus gland c. Bone marrow b. Regional lymph nodes d. Spleen B lymphocytes mature and become B cells in specialized (primary) lymphoid organs—the thymus gland for T cells and the bone marrow for B cells. Neither regional lymph nodes nor the spleen are involved in changing B lymphocytes into B cells. PTS: 1 REF: Page 225 3. What is the term for the process during which lymphoid stem cells migrate and change into either immunocompetent T cells or immunocompetent B cells? a. Clonal diversity c. Clonal selection b. Clonal differentiation d. Clonal competence The process is called the generation of clonal diversity and occurs in specialized (primary) lymphoid organs—the thymus gland for T cells and the bone marrow for B cells. The other options do not accurately identify the process described in the question. PTS: 1 REF: Page 225 4. Which type of immunity is produced by an individual after either natural exposure to the antigen or after immunization against the antigen? a. Passive-acquired immunity c. Passive-innate immunity b. Active-acquired immunity d. Active-innate immunity An individual produces active-acquired immunity (active immunity) after natural exposure to an antigen or after immunization, whereas passive-acquired immunity (passive immunity) does not involve the host’s immune response at all. The innate immune system, also known as nonspecific immune system and the first line of defense, is composed of the cells and mechanisms that defend the host from infection by other organisms in a nonspecific manner, which means that the cells of the innate system recognize and respond to pathogens in a generic way. PTS: 1 REF: Page 227 5. What type of immunity is produced when an immunoglobulin crosses the placenta? a. Passive-acquired immunity c. Passive-innate immunity b. Active-acquired immunity d. Active-innate immunity Passive-acquired immunity (passive immunity) does not involve the host’s immune response at all. Rather, passive immunity occurs when preformed antibodies or T lymphocytes are transferred from a donor to the recipient. This transfer can occur naturally, as in the passage of maternal antibodies across the placenta to the fetus, or artificially, as in a clinic using immunotherapy for a specific disease. The remaining options do not produce immunity via immunoglobulin transfer across the placenta. PTS: 1 REF: Page 227 6. The portion of the antigen that is configured for recognition and binding is referred to as what type of determinant? a. Immunotope c. Epitope b. Paratope d. Antigenitope The precise portion of the antigen that is configured for recognition and binding is called its antigenic determinant or epitope. The other options are not used to identify this portion of the antigen. PTS: 1 REF: Page 228 7. Which characteristic is the most important determinant of immunogenicity when considering the antigen? a. Size c. Complexity b. Foreignness d. Quantity Foremost among the criteria for immunogenicity is the antigen’s foreignness. A self-antigen that fulfills all of these criteria except foreignness does not normally elicit an immune response. Thus most individuals are tolerant of their own antigens. The immune system has an exquisite ability to distinguish self (self-antigens) from nonself (foreign antigens). The other options are considered when determining immunogenicity. PTS: 1 REF: Page 229 8. When antigens are administered to produce immunity, why are different routes of administration considered? a. Different routes allow the speed of onset of the antigen to be varied, with the intravenous route being the fastest. b. Some individuals appear to be unable to respond to an antigen by a specific route, thus requiring the availability of different routes for the same antigen. c. Antigen-presenting cells are highly specialized and thus require stimulation by different routes. d. Each route stimulates a different lymphocyte-containing tissue, resulting in different types of cellular and humoral immunity. Each route preferentially stimulates a different set of lymphocyte-containing (lymphoid) tissues and therefore results in the induction of different types of cell-mediated or humoral immune responses. The other options do not accurately explain the use of different routes when administering antigens to produce immunity. PTS: 1 REF: Page 229 9. The functions of the major histocompatibility complex (MHC) and CD1 molecules are alike because both: a. Are antigen-presenting molecules. b. Bind antigens to antibodies. c. Secrete interleukins during the immune process. d. Are capable of activating cytotoxic T lymphocytes. MHC and CD1 molecules are both antigen presenting molecules (APCs). The other options do not accurately describe the common function of these cells. PTS: 1 REF: Page 233 | Page 235 10. Where are antibodies produced? a. Helper T lymphocytes c. Plasma cells b. Thymus gland d. Bone marrow An antibody or immunoglobulin is a serum glycoprotein produced only by plasma cells in response to a challenge by an immunogen. PTS: 1 REF: Page 229 11. Which immunoglobulin is present in blood, saliva, breast milk, and respiratory secretions? a. IgA c. IgG b. IgE d. IgM IgA can be divided into two subclasses, IgA1 and IgA2. IgA1 molecules are predominantly found in the blood, whereas IgA2 is the predominant class of antibody found in normal body secretions. The other options are not found in the substances identified in the question. PTS: 1 REF: Page 229 12. Which antibody initially indicates a typical primary immune response? a. IgG c. IgA b. IgM d. IgE Typically, IgM is produced first (primary immune response), followed by IgG against the same antigen. The other options are not involved. PTS: 1 REF: Page 247 13. An individual is more susceptible to infections of mucous membranes when he or she has a seriously low level of which immunoglobulin antibody? a. IgG c. IgA b. IgM d. IgE The IgA molecules found in bodily secretions are dimers anchored together through a J-chain and secretory piece. This secretory piece is attached to the IgA antibodies inside the mucosal epithelial cells and may function to protect these immunoglobulin antibodies against degradation by enzymes also found in the secretions, thus decreasing the risk of infections in the mucous membrane. The other options do not accurately identify the immunoglobulin antibody involved in mucous membrane infections. PTS: 1 REF: Page 229 14. The B-cell receptor (BCR) complex functions uniquely by: a. Communicating information about the antigen to the helper T cell b. Secreting chemical signals to communicate between cells c. Recognizing the antigen on the surface of the B lymphocyte d. Communicating information about the antigen to the cell nucleus The role of the BCR is to recognize the antigen; however, unlike circulating antibodies, the receptor must communicate that information to the cell’s nucleus. The other options are not unique to the function of the BCR complex. PTS: 1 REF: Page 232 15. The generation of clonal diversity occurs primarily during which phase of life? a. Fetal c. Infancy b. Neonatal d. Puberty Generation of clonal diversity primarily occurs in the fetus and probably continues to a low degree throughout most of adult life. PTS: 1 REF: Pages 236-237 16. The generation of clonal diversity includes a process that: a. Involves antigens that select those lymphocytes with compatible receptors. b. Allows the differentiation of cells into antibody-secreting plasma cells or mature T cells. c. Takes place in the primary (central) lymphoid organs. d. Causes antigens to expand and diversify their populations. This process occurs in central lymphoid organs—the thymus gland for T cells and bone marrow for B cells. The other options do not accurately describe the processes included in clonal diversity. PTS: 1 REF: Pages 236-237 17. Which statement is true concerning clonal selection? a. Clonal selection is driven by hormones and does not require foreign antigens. b. This theory involves antigens that select those lymphocytes with compatible receptors. c. Clonal selection takes place in the primary (central) lymphoid organs. d. This process generates immature but immunocompetent T and B cells with receptors. Clonal selection, a process during which antigens select those lymphocytes with compatible receptors, expands their population and causes differentiation into antibody-secreting plasma cells or mature T cells (see Table 8-6). The other statements are not true regarding clonal selection. PTS: 1 REF: Page 236 18. Which is an example of an endogenous antigen? a. Yeast c. Bacteria b. Cancer cells d. Fungus Of the options provided, endogenous antigens include only those uniquely produced by cancerous cells. PTS: 1 REF: Page 244 19. Which cytokine is needed for the maturation of a functional helper T cell? a. IL-1 c. IL-4 b. IL-2 d. IL-12 Of the options provided, IL-2 production is critical for the Th cell to mature efficiently into a functional helper cell. PTS: 1 REF: Page 245 20. Th2 cells produce IL-4 and suppress which cells? a. B lymphocytes c. Th1 cells b. Cytotoxic T lymphocytes d. Memory T lymphocytes Th2 cells produce IL-4, which suppresses only Th1 and Th17 cells through their IL-4 receptors. PTS: 1 REF: Pages 246-247 21. Which statement is believed to be true concerning Th1 cells? a. Th1 cells are induced by antigens derived from allergens. b. They are induced by antigens derived from cancer cells. c. Th1 cells produce IL-4, IL-5, IL-6, and IL-13. d. They assist in the development of humoral immunity. Antigens derived from viral or bacterial pathogens and those derived from cancer cells are hypothesized to induce a greater number of Th1 cells relative to Th2 cells. The other statements are not true regarding Th1 cells. PTS: 1 REF: Pages 246-247 22. Which statement is believed to be true concerning Th2 cells? a. Th2 cells are induced by antigens derived from allergens. b. They are induced by antigens derived from cancer cells. c. Th2 cells produce IL-2, TNF-ß, and IFN- . d. They assist in the development of cell-mediated immunity. Antigens derived from multicellular parasites and allergens are hypothesized to be involved in the production of more Th2 cells. The other statements are not true regarding Th2 cells. PTS: 1 REF: Pages 246-247 23. When a person is exposed to most antigens, antibodies can be usually detected in his or her circulation within: a. 12 hours c. 3 days b. 24 hours d. 6 days After only approximately 5 to 7 days is an IgM antibody specific for that antigen detected in the circulation. PTS: 1 REF: Page 247 24. Vaccinations are able to provide protection against certain microorganisms because of the: a. Strong response from IgM c. Memory cells for IgE b. Level of protection provided by IgG d. Rapid response from IgA IgG production is considerably increased, making it the predominant antibody class of the secondary response. IgG is often present in concentrations several times larger than those of IgM, and levels of circulating IgG specific for that antigen may remain elevated for an extended period. The other options are not relevant to how vaccinations protect against certain microorganisms. PTS: 1 REF: Page 247 25. Why is the herpes virus inaccessible to antibodies after the initial infection? a. The virus does not circulate in the blood. b. It does not have antibody receptors. c. It resists agglutination. d. The virus is a soluble antigen. Many viruses (e.g., measles, herpes) are inaccessible to antibodies after the initial infection only because these viruses do not circulate in the bloodstream; rather, they remain inside infected cells, spreading by direct cell-to-cell contact. PTS: 1 REF: Page 252 26. Increased age may cause which change in lymphocyte function? a. Increased production of antibodies against self-antigens b. Decreased number of circulating T cells c. Decreased production of autoantibodies d. Increased production of helper T cells B-cell function is altered with age as shown by decreases in specific antibody production in response to antigenic challenge, with concomitant increases in circulating immune complexes and in circulating autoantibodies (antibodies against self-antigens). Aging does not play a role in either decreasing T cells circulation or increasing helper T cells production. PTS: 1 REF: Pages 257-258 27. How do antibodies protect the host from bacterial toxins? a. Lysing the cell membrane of the toxins b. Binding to the toxins to neutralize their biologic effects c. Inhibiting the synthesis of DNA proteins needed for growth d. Interfering with the DNA enzyme needed for replication To cause disease, most toxins must bind to surface molecules on the individual’s cells. Protective antibodies can bind to the toxins, prevent their interaction with cells, and neutralize their biologic effects. The other options fail to explain how antibodies protect the host from bacterial toxins. PTS: 1 REF: Page 252 28. Which T cell controls or limits the immune response to protect the host’s own tissues against an autoimmune response? a. Cytotoxic T cells c. Th2 cells b. Th1 cells d. Regulatory T (Treg) cells The regulatory T (Treg) cell is the only option whose role is to control or limit the immune response to protect the host’s own tissues against autoimmune reactions. PTS: 1 REF: Page 257 29. Evaluation of umbilical cord blood can confirm that which immunoglobulin level is near adult levels? a. IgA c. IgM b. IgG d. IgE At birth, the total IgG level in the umbilical cord is the only immunoglobulin that is near adult levels (see Figure 8-30). PTS: 1 REF: Page 257 30. Which statement is true concerning the IgM? a. IgM is the first antibody produced during the initial response to an antigen. b. IgM mediates many common allergic responses. c. IgM is the most abundant class of immunoglobulins. d. IgM is capable of crossing the human placenta. Typically, IgM is produced first (primary immune response), followed by IgG against the same antigen. The other options are not true statements regarding IgM. PTS: 1 REF: Page 247 31. Which cell has the ability to recognize antigens presented by the MHC class I molecules? a. T cytotoxic c. CD 8 b. CD 4 d. T helper CD8 cells recognize antigens presented by the major histocompatibility complex (MHC) class I molecules and become mediators of cell-mediated immunity and directly kill other cells (T-cytotoxic cells). CD4 cells tend to recognize antigen presented by MHC class II molecules and develop into helpers in the later clonal selection process (T-helper cells) PTS: 1 REF: Page 240 32. Which cell has a role in developing cell-mediated immunity? a. Th1 c. CD8 b. CD4 d. Th2 Only Th1 cells help develop cellular immunity. PTS: 1 REF: Pages 245-246 33. How does the aging process of the T-cell activity affect older adults? a. Poor heat regulation abilities b. Increased risk for bone fractures c. Tendency to develop various infections d. Likelihood of experiencing benign skin lesions T-cell activity is deficient in older adults, and a shift in the balance of T-cell subsets is observed. These changes may result in increased susceptibility to infection. The other issues are not related to T-cell activity. PTS: 1 REF: Pages 257-258 34. Which statement is true regarding maternal antibodies provided to the neonate? a. The antibodies enter into the fetal circulation by means of active transport. b. The antibodies are transferred to the fetus via the lymphatic system. c. The antibodies are directly related to the mother’s nutritional intake. d. The antibodies reach protective levels after approximately 6 months of age. To protect the child against infectious agents both in utero and during the first few postnatal months, a system of active transport facilitates the passage of maternal antibodies into the fetal circulation. The antibodies are transmitted via the placenta and are related to the mother’s immune system. The infant’s own IgG-related antibodies reach protective levels by 6 months of age. PTS: 1 REF: Page 257 35. Antibodies that are associated with mucosal immune system, such as immunoglobulins, function to prevent which type of infections? a. Infections that attack the respiratory system b. Infections that tend to be chronic in nature c. Infections likely to be resistant to antibiotics d. Infections that focus on epithelial surfaces of the body Antibodies of the systemic immune system function throughout the body, whereas antibodies of the secretory (mucosal) immune system—primarily immunoglobulins of the IgA class—are associated with bodily secretions and function to prevent pathogenic infection on epithelial surfaces. The other options are not necessarily true when considering the immunoglobulins. PTS: 1 REF: Pages 252-253 36. Cytokines are vital to a cell’s ability to do which function? a. Excrete c. Metabolize b. Reproduce d. Communicate During their interactions, cells must communicate with each other through soluble cytokines. The other options are not so rigidly related to cytokines. PTS: 1 REF: Pages 235-236 MULTIPLE RESPONSE 37. Which is an example of a bacterial toxin that has been inactivated but still retains its immunogenicity to protect the person? (Select all that apply.) a. Poliomyelitis b. Measles c. Tetanus d. Gonorrhea e. Diphtheria , E The symptoms of tetanus or diphtheria are mediated by specific toxins. To prevent harming the recipient of the immunization, bacterial toxins are chemically inactivated so that they have lost most of their harmful properties but still retain their immunogenicity. These agents are referred to as toxoids. Tetanus or diphtheria are the only examples of such inactivated toxins. PTS: 1 REF: Page 252 38. Which statements are true concerning the humoral immune response? (Select all that apply.) a. The humoral immune response is divided into major and minor phases. b. The response has IgG and IgM produced during each of its phrases. c. It has a greater presence of IgG than IgM in one of its phases. d. The humoral immune response is produced in reaction to the presence of an antigen. e. Phases differ in their response time as a result of the effect of memory cells. , C, D, E The humoral immune response is divided into two phases, primary and secondary. These phases differ in the relative amounts of IgG produced—the secondary response having a significantly higher proportion of IgG relative to IgM. The two phases also differ in the speed with which each occurs after the antigen challenge—the secondary phases is significantly more rapid than the primary phase because of the presence of memory cells in the secondary phase. PTS: 1 REF: Page 247 39. CD4 is a characteristic surface marker and a result of which of the following? (Select all that apply.) a. Activity in the primary lymphoid organs b. Process of cellular differentiation c. Alterations to T cells d. Changes to B cells e. Clonal selection , B, C, D Differentiation of B cells and T cells in the primary lymphoid organs results in the expression of several characteristic surface markers, such as CD4 on helper T cells, CD8 on cytotoxic T cells, and CD21 and CD40 on B cells. Clonal selection is the process during which antigens select those lymphocytes with complementary T-cell receptors (TCRs) or BCRs. PTS: 1 REF: Pages 236-241 | Page 258 40. What are the necessary components of an adaptive immune response? (Select all that apply.) a. Antigen b. Gamma IgG c. Lymphocyte surface receptors d. Crystalline fragment e. Antibody , C, E Antigens are the molecules that can react with components of the adaptive immune system, including antibodies and lymphocyte surface receptors. PTS: 1 REF: Page 228 MATCHING Match each immunoglobulin with its characteristic or function. Each immunoglobulin can be used only once. A. IgA B. IgE C. IgG 41. Crosses the placenta. 42. Is predominantly found in the blood and body secretions. 43. Mediates many common allergic responses 41. PTS: 1 REF: Page 229 MSC: As a result of selective transport across the placenta, maternal IgG is the major class of antibody found in the blood of the fetus and newborn. 42. PTS: 1 REF: Page 229 MSC: IgA can be divided into two subclasses, IgA1 and IgA2. IgA1 molecules are found predominantly in the blood, whereas IgA2 is the predominant class of antibody found in normal body secretions. 43. PTS: 1 REF: Page 230 MSC: IgE is the least concentrated of any of the immunoglobulin classes in the circulation. It appears to have very specialized functions as a mediator of many common allergic responses and in the defense against parasitic infections. Chapter 9: Alterations in Immunity and Inflammation MULTIPLE CHOICE 1. Hypersensitivity is best defined as a(an): a. Disturbance in the immunologic tolerance of self-antigens b. Immunologic reaction of one person to the tissue of another person c. Altered immunologic response to an antigen that results in disease d. Undetectable immune response in the presence of antigens Hypersensitivity is an altered immunologic response to an antigen that results in disease or damage to the host. The other options are not accurate definitions of hypersensitivity. PTS: 1 REF: Page 262 2. A hypersensitivity reaction that produces an allergic response is called: a. Hemolytic shock c. Necrotizing vasculitis b. Anaphylaxis d. Systemic erythematosus Examples of systemic anaphylaxis are allergic reactions to beestings, peanuts, and fish. The other options are not accurate examples of hypersensitivity. PTS: 1 REF: Page 263 3. The common hay fever allergy is expressed through a reaction that is mediated by which class of immunoglobulins? a. IgE c. IgM b. IgG d. T cells Type I reactions are mediated by antigen-specific IgE and the products of tissue mast cells (see Figure 9-1). The most common allergies (e.g., pollen allergies) are type I reactions. In addition, most type I reactions occur against environmental antigens and are therefore allergic. The other options do not accurately identify the mediation factor related to hay fever. PTS: 1 REF: Page 263 | Table 9-1 4. Which type of antibody is involved in type I hypersensitivity reaction? a. IgA c. IgG b. IgE d. IgM Type I reactions are only mediated by antigen-specific IgE and the products of tissue mast cells (see Figure 9-1). PTS: 1 REF: Page 263 5. Blood transfusion reactions are an example of: a. Autoimmunity c. Homoimmunity b. Alloimmunity d. Hypersensitivity Only alloimmunity (also termed isoimmunity) occurs when the immune system of one individual produces an immunologic reaction against tissues of another individual. PTS: 1 REF: Page 262 6. During an IgE-mediated hypersensitivity reaction, which leukocyte is activated? a. Neutrophils c. Eosinophils b. Monocytes d. T lymphocytes Of the options provided, only eosinophils are activated during IgE-mediated hypersensitivity reactions. PTS: 1 REF: Page 263 | Page 265 7. During an IgE-mediated hypersensitivity reaction, what causes bronchospasm? a. Bronchial edema caused by the chemotactic factor of anaphylaxis b. Bronchial edema caused by binding of the cytotropic antibody c. Smooth muscle contraction caused by histamine bound to H1 receptors d. Smooth muscle contraction caused by histamine bound to H2 receptors During an IgE-mediated hypersensitivity reaction, only smooth muscle contraction caused by histamine bound to H1 receptors results in bronchospasms. PTS: 1 REF: Page 263 | Page 265 8. During an IgE-mediated hypersensitivity reaction, the degranulation of mast cells is a result of which receptor action? a. Histamine bound to H2 b. Chemotactic factor binding to the receptor c. Epinephrine bound to mast cells d. Acetylcholine bound to mast cells Histamine bound to H2 results in the degranulation of mast cells during an IgE-medicated hypersensitivity reaction. The other options do not cause this reaction. PTS: 1 REF: Page 265 9. What characteristic do atopic individuals have that make them genetically predisposed to develop allergies? a. Greater quantities of histamine c. Greater quantities of IgE b. More histamine receptors d. A deficiency in epinephrine Atopic individuals tend to produce higher quantities of IgE and to have more crystalline fragment (Fc) receptors for IgE on their mast cells. The other options do not cause this reaction. PTS: 1 REF: Page 271 10. What is the mechanism that results in type II hypersensitivity reactions? a. Antibodies coat mast cells by binding to receptors that signal its degranulation, followed by a discharge of preformed mediators. b. Antibodies bind to soluble antigens that were released into body fluids, and the immune complexes are then deposited in the tissues. c. Cytotoxic T lymphocytes or lymphokine-producing helper T 1 cells directly attack and destroy cellular targets. d. Antibodies bind to the antigens on the cell surface. The mechanism that results in a type II hypersensitivity reaction begins with antibody binding to tissue-specific antigens or antigens that have attached to particular tissues. The cell can be destroyed by antibody IgG or IgM and activation of the complement cascade through the classical pathway. PTS: 1 REF: Page 266 11. When mismatched blood is administered causing an ABO incompatibility, the erythrocytes are destroyed by: a. Complement-mediated cell lysis c. Phagocytosis in the spleen b. Phagocytosis by macrophages d. Natural killer cells Erythrocytes are destroyed by complement-mediated lysis in individuals with autoimmune hemolytic anemia or as a result of an alloimmune reaction to ABO-mismatched transfused blood cells. The other options are not involved in an ABO incompatibility reaction. PTS: 1 REF: Page 266 12. When antibodies are formed against red blood cell antigens of the Rh system, the blood cells are destroyed by: a. Complement-mediated cell lysis b. Phagocytosis by macrophages c. Phagocytosis in the spleen d. Neutrophil granules and toxic oxygen products Antibodies against platelet-specific antigens or against red blood cell antigens of the Rh system coat those cells at low density, resulting in their preferential removal by phagocytosis in the spleen, rather than by complement-mediated lysis. The other options do not cause this reaction. PTS: 1 REF: Pages 266-267 13. When soluble antigens from infectious agents enter circulation, tissue damage is a result of: a. Complement-mediated cell lysis b. Phagocytosis by macrophages c. Phagocytosis in the spleen d. Neutrophil granules and toxic oxygen products Of the options available, only the components of neutrophil granules damage the tissue. PTS: 1 REF: Page 267 14. How are target cells destroyed in a type II hypersensitivity reaction? a. Complement-mediated cell lysis b. Phagocytosis by macrophages c. Neutrophil granules and toxic oxygen products d. Natural killer cells The mechanism that results in a type II hypersensitivity reaction involves a subpopulation of cytotoxic cells that are not antigen specific (natural killer [NK] cells). Antibody on the target cell is recognized by Fc receptors on the NK cells, which releases toxic substances that destroy the target cell. The other options do not cause the destruction of target cells related to a type II hypersensitivity reaction. PTS: 1 REF: Page 267 15. Graves disease (hyperthyroidism) is an example of which type II hypersensitivity reaction? a. Modulation b. Antibody-dependent cell-mediated cytotoxicity c. Neutrophil-mediated damage d. Complement-mediated lysis The antibody reacts with the receptors on the target cell surface and modulates the function of the receptor by preventing interactions with their normal ligands, replacing the ligand and inappropriately stimulating the receptor or destroying the receptor. For example, in the hyperthyroidism (excessive thyroid activity) of Graves disease, autoantibody binds to and activates receptors for thyroid-stimulating hormone (TSH) (a pituitary hormone that controls the production of the hormone thyroxine by the thyroid). The other options are not examples of type II hypersensitivity reactions. PTS: 1 REF: Page 267 | Page 269 16. Type III hypersensitivity reactions are a result of which of the following? a. Antibodies coating mast cells by binding to receptors that signal its degranulation, followed by the discharge of preformed mediators b. Antibodies binding to soluble antigens that were released into body fluids and the immune complexes being deposited in the tissues c. Tc cells or lymphokine-producing Th1 cells directly attacking and destroying cellular targets d. Antibodies binding to the antigen on the cell surface Antigen-antibody (immune) complexes that are formed in the circulation and then deposited later in vessel walls or extravascular tissues (see Figure 9-3) cause most type III hypersensitivity diseases. The other options do not cause this type of reaction. PTS: 1 REF: Page 269 17. A type IV hypersensitivity reaction causes which result? a. Antibodies coating mast cells by binding to receptors that signal its degranulation, followed by the discharge of preformed mediators b. Antibodies binding to soluble antigens that were released into body fluids and the immune complexes being deposited in the tissues c. Lymphokine-producing Th1 cells directly attacking and destroying cellular targets d. Antibodies binding to the antigen on the cell surface Types I, II, and III hypersensitivity reactions are mediated by antibody, type IV reactions are mediated by T lymphocytes and do not involve antibody. Type IV mechanisms occur through either Tc cells or lymphokine-producing Th1 cells. Tc cells directly attack and destroy cellular targets. PTS: 1 REF: Page 270 18. In a type III hypersensitivity reaction, the harmful effects after the immune complexes that are deposited in tissues are a result of: a. Cytotoxic T cells c. Complement activation b. Natural killer cells d. Degranulation of mast cells Complement activation, particularly through the generation of chemotactic factors for neutrophils, causes the harmful effects of immune complex deposition. The neutrophils bind to antibody and C3b contained in the complexes and attempt to ingest the immune complexes. Type III hypersensitivity reactions as described are not the result of any of the other options. PTS: 1 REF: Page 269 19. Raynaud phenomenon is classified as a type III hypersensitivity reaction and is due to: a. Immune complexes that are deposited in capillary beds, blocking circulation b. Mast cells that are bound to specific endothelial receptors, causing them to degranulate and creating a localized inflammatory reaction that occludes capillary circulation c. Cytotoxic T cells that attack and destroy the capillaries so that they are unable to perfuse local tissues d. Antibodies that detect the capillaries as foreign protein and destroy them using lysosomal enzymes and toxic oxygen species Raynaud phenomenon is a condition caused by the temperature-dependent deposition of immune complexes in the capillary beds of the peripheral circulation. None of the other options are involved in causing this condition. PTS: 1 REF: Page 270 20. Deficiencies in which element can produce depression of both B- and T-cell function? a. Iron c. Iodine b. Zinc d. Magnesium Of the options available, only deficient zinc intake can profoundly depress T- and B-cell function. PTS: 1 REF: Page 291 21. When the maternal immune system becomes sensitized against antigens expressed by the fetus, what reaction occurs? a. T-cell immunity c. Fetal immunity b. Alloimmunity d. Autoimmunity Alloimmunity occurs when an individual’s immune system reacts against antigens on the tissues of other members of the same species. Sensitization against fetal antigens is not the cause of any other available option. PTS: 1 REF: Page 276 22. Tissue damage caused by the deposition of circulating immune complexes containing an antibody against the host DNA is the cause of which disease? a. Hemolytic anemia c. Systemic lupus erythematosus b. Pernicious anemia d. Myasthenia gravis Only the deposition of circulating immune complexes containing an antibody against the host DNA produce tissue damage in individuals with systemic lupus erythematosus (SLE). PTS: 1 REF: Page 277 23. Why does tissue damage occurs in acute rejection after organ transplantation? a. Th1 cells release cytokines that activate infiltrating macrophages, and cytotoxic T cells directly attack the endothelial cells of the transplanted tissue. b. Circulating immune complexes are deposited in the endothelial cells of transplanted tissue, where the complement cascade lyses tissue. c. Receptors on natural killer cells recognize antigens on the cell surface of transplanted tissue, which releases lysosomal enzymes that destroy tissue. d. Antibodies coat the surface of transplanted tissue to which mast cells bind and liberate preformed chemical mediators that destroy tissue. The recipient’s lymphocytes interacting with the donor’s dendritic cells within the transplanted tissue usually initiate sensitization, resulting in the induction of recipient Th1 and Tc cells against the donor’s antigens. The Th1 cells release cytokines that activate infiltrating macrophages, and the Tc cells directly attack the endothelial cells in the transplanted tissue. The other options do not accurately describe how acute rejection after organ transplantation results in tissue damage. PTS: 1 REF: Page 280 24. Which blood cell carries the carbohydrate antigens for blood type? a. Platelets c. Lymphocytes b. Neutrophils d. Erythrocytes The reaction that causes a blood transfusion recipient’s red blood cells to clump together is related to the ABO antigens located on the surface of only erythrocytes. PTS: 1 REF: Page 278 25. A person with type O blood is likely to have high titers of which anti-antibodies? a. A c. A and B b. B d. O Type O individuals have neither A or B antigen but have both anti-A and anti-B antibodies and therefore cannot accept blood from any of the other three types. PTS: 1 REF: Page 278 26. Which class of immunoglobulins forms isohemagglutinins? a. IgA c. IgG b. IgE d. IgM Naturally occurring antibodies, called isohemagglutinins, are immunoglobulins of only the IgM class. PTS: 1 REF: Page 278 27. Which component of the immune system is deficient in individuals with infections caused by viruses, fungi, or yeast? a. Natural killer cells c. B cells b. Macrophages d. T cells Of the available options, deficiencies in T-cell immune responses are suggested when certain viruses (e.g., varicella, vaccinia, herpes, cytomegalovirus), fungi, and yeasts (e.g., Candida, Histoplasma) or certain atypical microorganisms (e.g., Pneumocystis jiroveci) cause recurrent infections. PTS: 1 REF: Page 281 28. In which primary immune deficiency is there a partial-to-complete absence of T-cell immunity? a. Bruton disease c. Reticular dysgenesis b. DiGeorge syndrome d. Adenosine deaminase deficiency The principal immunologic defect in DiGeorge syndrome is the partial or complete absence of T-cell immunity. The other options are not the result of either a partial or complete absence of T-cell immunity. PTS: 1 REF: Page 281 29. How many months does it take for the newborn to be sufficiently protected by antibodies produced by its own B cells? a. 1 to 2 c. 6 to 8 b. 4 to 5 d. 10 to 12 By 6 to 8 months, the newborn should be efficiently protected by antibodies produced by its own B cells. PTS: 1 REF: Page 290 30. Considering the effects of nutritional deficiencies on the immune system, severe deficits in calories and protein lead to deficiencies in the formation of which immune cells? a. B cells c. Natural killer cells b. T cells d. Neutrophils Severe deficits in calorie or protein intake lead to deficiencies in T-cell function and numbers. The other options are not necessarily affected. PTS: 1 REF: Pages 290-291 31. Urticaria are a manifestation of a which type of hypersensitivity reaction? a. IV c. II b. III d. I Urticaria, or hives, is a dermal (skin) manifestation of only type I allergic reactions. PTS: 1 REF: Page 272 32. Graves disease is a result of: a. Increased levels of circulating immunoglobulins b. The infiltration of the thyroid with T lymphocytes c. Autoantibodies binding to thyroid-stimulating hormone (TSH)-receptor sites d. Exposure to acetylates in substances such as rubber In the hyperthyroidism (excessive thyroid activity) of Graves disease, autoantibody binds to and activates receptors for TSH (a pituitary hormone that controls the production of the hormone thyroxine by the thyroid). The other options do not accurately describe the cause of Graves disease. PTS: 1 REF: Page 267 | Page 269 33. Raynaud phenomenon is an example of which type of hypersensitivity? a. IV c. II b. III d. I The characteristics of serum sickness are observed in only systemic type III autoimmune diseases such as Raynaud phenomenon. PTS: 1 REF: Page 270 34. Which statement is true concerning an atopic individual? a. They tend to produce less IgE. b. They tend to produce more Fc receptors. c. They tend to attract very few mast cells. d. They tend to produce very high levels of IgM. Atopic individuals tend to produce higher quantities of IgE and have more Fc receptors for IgE on their mast cells. The other available options are not true. PTS: 1 REF: Page 271 35. Which statement is true regarding immunodeficiency? a. Immunodeficiency is generally not present in other family members. b. Immunodeficiency is never acquired; rather, it is congenital. c. Immunodeficiency is almost immediately symptomatic. d. Immunodeficiency is a result of a postnatal mutation. Generally, the mutations are sporadic and not inherited; a family history exists in only approximately 25% of individuals. The sporadic mutations occur before birth, but the onset of symptoms may be early or later, depending on the particular syndrome. The immunodeficiency can be either congenital or acquired. PTS: 1 REF: Page 281 36. A person with type O blood is considered to be the universal blood donor because type O blood contains which of the following? a. No antigens c. Both A and B antigens b. No antibodies d. Both A and B antibodies Because individuals with type O blood lack both types of antigens, they are considered universal donors, meaning that anyone can accept their red blood cells. Type O individuals, who have neither A or B antigen but have both anti-A and anti-B antibodies, cannot accept blood from any of the other three types. PTS: 1 REF: Page 278 37. Immunoglobulin E (IgE) is associated with which type of hypersensitivity reaction? a. I c. III b. II d. IV Hypersensitivity reactions have been divided into four distinct types: type I (IgE-mediated) hypersensitivity reactions, type II (tissue-specific) hypersensitivity reactions, type III (immune complex–mediated) hypersensitivity reactions, and type IV (cell-mediated) hypersensitivity reactions. PTS: 1 REF: Page 263 38. Graves disease is an autoimmune disease that results in which maternal antibody? a. Binding with receptors for neural transmitters on muscle cells, causing neonatal muscular weakness b. Affecting the receptor for TSH, causing neonatal hyperthyroidism c. Inducing anomalies in the fetus or causing pregnancy loss d. Destroying platelets in the fetus and neonate Graves disease is an autoimmune disease in which maternal antibody against the receptor for TSH causes neonatal hyperthyroidism. Myasthenia gravis is an autoimmune disease in which maternal antibody binds with receptors for neural transmitters on muscle cells (acetylcholine receptors), causing neonatal muscular weakness. Systemic lupus erythematosus is an autoimmune disease in which diverse maternal autoantibodies induce anomalies (e.g., congenital heart defects) in the fetus or cause pregnancy loss. Immune thrombocytopenic purpura causes both autoimmune and alloimmune variants to occur, during which maternal antiplatelet antibody destroys platelets in the fetus and neonate. PTS: 1 REF: Page 277 MULTIPLE RESPONSE 39. When a tuberculin skin test is positive, the hard center and erythema surrounding the induration are a result of which of the following? (Select all that apply.) a. Histamine d. Products of complement b. T lymphocytes e. Macrophages c. Immune complexes , E The reaction site is infiltrated with only T lymphocytes and macrophages, resulting in a clear hard center (induration) and a reddish surrounding area (erythema). PTS: 1 REF: Page 270 40. Exposure to which of the following could result in a type IV hypersensitivity reaction? (Select all that apply.) a. Poison ivy d. Nickel b. Neomycin e. Detergents c. Dairy products , B, D, E Allergens that primarily elicit type IV allergic hypersensitivities include plant resins (e.g., poison ivy, poison oak); metals (e.g., nickel, chromium); acetylates and chemicals in rubber, cosmetics, detergents; and topical antibiotics (e.g., neomycin). PTS: 1 REF: Page 271 41. Which disorders are considered autoimmune? (Select all that apply.) a. Crohn disease d. Systemic lupus erythematosus b. Addison disease e. Noninsulin-dependent diabetes c. Rheumatoid arthritis , B, C, D Crohn disease, Addison disease, rheumatoid arthritis, and systemic lupus erythematosus are all diseases that result from autoimmune pathologic conditions. Insulin-dependent diabetes is also an autoimmune disorder, but noninsulin-dependent diabetes is not. PTS: 1 REF: Pages 264-265 | Table 9-2 42. Which statements best define acute rejection? (Select all that apply.) a. Acute rejection is a cell-mediated immune response. b. Acute rejection is usually a type III rejection. c. Immunosuppressive drugs delay or lessen the intensity of an acute rejection. d. Acute rejection is associated with the body’s response to an organ transplant. e. Acute rejection is a response against unmatched human leukocyte antigens (HLAs). , C, D, E Acute rejection is primarily a cell-mediated immune response that occurs within days to months after transplantation. This type of rejection occurs when the recipient develops an immune response against unmatched HLAs after transplantation. A biopsy of the rejected organ usually shows an infiltration of lymphocytes and macrophages characteristic of a type IV reaction. Immunosuppressive drugs may delay or lessen the intensity of an acute rejection. PTS: 1 REF: Page 280 Chapter 10: Infection MULTIPLE CHOICE 1. What is a significant cause of morbidity and mortality worldwide? a. Starvation c. Cardiovascular disease b. Traumatic injury d. Infectious disease Despite the wide-scale implementation of progressive public health and immunization policies, infectious disease remains a significant cause of morbidity and mortality. The other options are not significant causes. PTS: 1 REF: Page 299 2. What is the first stage in the infectious process? a. Invasion c. Spread b. Colonization d. Multiplication From the perspective of the microorganisms that cause disease, the infectious process undergoes four separate stages of progression: (1) colonization, (2) invasion, (3) multiplication, and (4) spread. PTS: 1 REF: Pages 300-301 3. Which type of microorganism reproduces on the skin? a. Viruses c. Protozoa and Rickettsiae b. Bacteria and fungi d. Mycoplasma Only bacteria and fungi have the capacity to reproduce on the skin. PTS: 1 REF: Page 303 | Table 10-3 4. Phagocytosis involves neutrophils actively attacking, engulfing, and destroying which microorganisms? a. Bacteria c. Viruses b. Fungi d. Yeasts Invasion is the direct confrontation with an individual’s primary defense mechanisms against only bacteria, which include the complement system, antibodies, and phagocytes, such as neutrophils and macrophages. PTS: 1 REF: Page 306 5. Once they have penetrated the first line of defense, which microorganisms do natural killer (NK) cells actively attack? a. Bacteria c. Viruses b. Fungi d. Mycoplasma NK cells are the principal defenders against only tumor cells or virally infected cells. PTS: 1 REF: Page 320 6. Which statement concerning exotoxins is true? a. Exotoxins are contained in cell walls of gram-negative bacteria. b. Exotoxins are released during the lysis of bacteria. c. Exotoxins are able to initiate the complement and coagulation cascades. d. Exotoxins are released during bacterial growth. Exotoxins are proteins released during bacterial growth. The other options are not true of exotoxins. PTS: 1 REF: Page 306 7. Which statement is true concerning a fungal infection? a. Fungal infections occur only on skin, hair, and nails. b. Phagocytes and T lymphocytes control fungal infections. c. Fungal infections release endotoxins. d. Vaccines prevent fungal infections. The host defense against fungal infection includes the fungistatic properties of neutrophils and macrophages. T lymphocytes are crucial in limiting the extent of infection and producing cytokines to further activate macrophages. The other options are not true of fungal infections. PTS: 1 REF: Page 312 8. Cytokines are thought to cause fevers by stimulating the synthesis of which chemical mediator? a. Leukotriene c. Prostaglandin b. Histamine d. Bradykinin Cytokines seem to raise the thermoregulatory set point through stimulation of prostaglandin synthesis and turnover in thermoregulatory (brain) and nonthermoregulatory (peripheral) tissues. The other options do not accurately identify the appropriate chemical mediator. PTS: 1 REF: Pages 301-302 9. Considering the hypothalamus, a fever is produced by: a. Endogenous pyrogens acting directly on the hypothalamus. b. Exogenous pyrogens acting directly on the hypothalamus. c. Immune complexes acting indirectly on the hypothalamus. d. Cytokines acting indirectly on the hypothalamus. Little evidence suggests that exogenous pyrogens directly cause fever. Such pyrogens indirectly affect the hypothalamus through endogenous pyrogens released by cells of the host. Neither immune complexes nor cytokines are involved in the process. PTS: 1 REF: Page 302 10. Which statement about vaccines is true? a. Most bacterial vaccines contain attenuated organisms. b. Most viral vaccines are made by using dead organisms. c. Vaccines require booster injections to maintain life-long protection. d. Vaccines provide effective protection against most infections. In general, vaccine-induced protection does not persist as long as infection-induced immunity, thus booster injections may be necessary to maintain protection throughout life. The other options are not true of vaccines. PTS: 1 REF: Page 332 11. Vaccines against viruses are created from: a. Killed organisms or extracts of antigens b. Live organisms weakened to produce antigens c. Purified toxins that have been chemically detoxified d. Recombinant pathogenic protein Most vaccines against viral infections (e.g., measles, mumps, rubella, varicella [chickenpox], rotavirus) contain live viruses that are weakened (attenuated) to continue expressing the appropriate antigens but are unable to establish more than a limited and easily controlled infection. The other options are not used in virus-focused vaccines. PTS: 1 REF: Page 332 12. Which statement is a characteristic of HIV? a. HIV only infects T-helper (Th) cells. b. HIV is a retrovirus. c. HIV carries genetic information in its DNA. d. HIV has five identified strains. HIV is a member of the retrovirus family, which carries genetic information in the form of two copies of RNA (see Figure 10-12). The other statements are not true of HIV. PTS: 1 REF: Page 324 13. What is the role of reverse transcriptase in HIV infection? a. Reverse transcriptase converts single-stranded DNA into double-stranded DNA. b. It is needed to produce integrase. c. It transports the RNA into the cell nucleus. d. It converts RNA into double-stranded DNA. One particular family of viruses, retroviruses (e.g., HIV) carries an enzyme, reverse transcriptase, which creates a double-stranded DNA version of the virus. PTS: 1 REF: Page 324 14. After sexual transmission of HIV, a person can be infected yet seronegative for how many months? a. 1 to 2 c. 18 to 20 b. 6 to 14 d. 24 to 36 Antibody appears rather rapidly after infection through blood products, usually within 4 to 7 weeks. After sexual transmission, however, the individual can be infected yet seronegative for 6 to 14 months or, in at least one case, for years. PTS: 1 REF: Page 326 15. Which cells are primary targets for HIV? a. CD4+ Th cells only b. CD4+ Th cells, macrophages, and natural killer cells c. CD8-positive cytotoxic T (Tc) cells and plasma cells d. CD8-positive Tc cells only The primary cellular targets for HIV include CD4+ Th cells, macrophages, and NK cells. The other options are not the primary target cells of HIV. PTS: 1 REF: Page 325 16. What area in the body may act as a reservoir in which HIV can be relatively protected from antiviral drugs? a. Central nervous system c. Thymus gland b. Bone marrow d. Lungs HIV may persist in regions where the antiviral drugs are not as effective, such as the central nervous system (CNS). The other options are not as protected from antiviral drugs. PTS: 1 REF: Page 327 17. AIDS produces a striking decrease in the number of which cells? a. Macrophages c. CD4+ Th cells b. CD8+ T cells d. Memory T cells The major immunologic finding in AIDS is the striking decrease in the number of CD4+ Th cells (see Figure 10-15). This finding is not true of the other options. PTS: 1 REF: Page 325 18. HIV antibodies appear within how many weeks after infection through blood products? a. 1 to 2 c. 10 to 12 b. 4 to 7 d. 20 to 24 Antibody appears rather rapidly after infection through blood products, usually within 4 to 7 weeks. PTS: 1 REF: Page 326 19. What is the final stage of the infectious process? a. Colonization c. Multiplication b. Invasion d. Spread From the perspective of the microorganisms that cause disease, the infectious process undergoes four separate stages of progression: (1) colonization, (2) invasion, (3) multiplication, and (4) spread. PTS: 1 REF: Page 300 20. Toxigenicity is defined as the: a. Ability of the pathogen to invade and multiply in the host b. Pathogen’s ability to produce disease by the production of a soluble toxin c. Ability of an agent to produce disease d. Potency of a pathogen measured in terms of the number of microorganisms required to kill the host Toxigenicity is the ability of a pathogen to produce soluble toxins or endotoxins, which are factors that greatly influence the pathogen’s degree of virulence. The other options do not accurately define toxigenicity. PTS: 1 REF: Page 302 21. The ability of the pathogen to invade and multiply in the host is referred to as: a. Infectivity c. Pathogenicity b. Toxigenicity d. Virulence Infectivity is the ability of the pathogen to invade and multiply in the host. The other options do not accurately denote the pathogen’s ability to invade and multiply in the host. PTS: 1 REF: Page 302 22. Some bacterial surface proteins bind with the crystalline fragment (Fc) portion of an antibody to: a. Hide in cells to avoid triggering an immune response b. Form self-protecting toxins c. Make staining possible for microscopic observation d. Produce a protective “self” protein Some bacterial surface proteins (protein A of Staphylococcus aureus, protein G of Streptococcus pyogenes) bind the Fc portion of the individual’s antibody, thus forming a protective coat of “self” protein. The other options do not accurately define the role of bacterial surface proteins as they bind with the Fc portion on an antibody. PTS: 1 REF: Page 308 23. Which organism is a common sexually transmitted bacterial infection? a. Staphylococcus aureus c. Helicobacter pylori b. Clostridium perfringens d. Treponema pallidum Treponema pallidum (spirochete, syphilis) is a sexually transmitted disease. Staphylococcus aureus is commonly ingested, causing food poisoning; Clostridium perfringens (gas gangrene) is a skin or wound infection; and Helicobacter pylori (gastritis, peptic ulcers) is found in the gastrointestinal tract. PTS: 1 REF: Pages 304-305 | Table 10-4 24. Which disease is an example of a rickettsial infection? a. Cholera c. Sleeping sickness b. Candida d. Rocky Mountain spotted fever Rocky Mountain spotted fever is a result of a rickettsiae. Cholera is a bacterial infection, candida is a fungal infection, and sleeping sickness is a protozoal infection. PTS: 1 REF: Page 302 MULTIPLE RESPONSE 25. Which secretion transmits HIV? (Select all that apply.) a. Semen b. Urine c. Saliva d. Breast milk e. Sweat , D HIV is a blood-borne pathogen present in body fluids (e.g., blood, vaginal fluid, semen, breast milk). PTS: 1 REF: Page 322 26. Which infection is fungal? (Select all that apply.) a. Ringworm b. Candida c. Cholera d. Athlete’s foot e. Aspergillus , B, D, E Infection with a fungus is called mycosis and includes dermatophytes (e.g., tineas, which refers to several skin mycoses including ringworm, athlete’s foot, and others) or yeasts (e.g., Candida, Aspergillus, Cryptococcus). Cholera is a bacterial infection. PTS: 1 REF: Page 311 27. Which statement is true regarding the development of HIV symptoms? (Select all that apply.) a. Symptoms generally appear in the clinical latency stage. b. Symptoms are generally observable within 5 years of the initial infection. c. T cells levels, particularly those of memory T cells, progressively decrease. d. Untreated infected individuals may remain asymptomatic for up to10 years. e. Secondary lymphoid organs experience damage and resulting malfunction. , D, E Individuals during the early stages of HIV (early stage disease or clinical latency) are usually asymptomatic. The early stage may last as long as 10 years in untreated people, during which the viral load increases and the numbers of CD4+ cells progressively decrease. As a result of these processes, the level of T cells decreases (particularly memory T cells, which seem more susceptible to HIV infection); thymic production of new T cells is decreased; and the secondary lymphoid organs (particularly the lymph nodes) are damaged. PTS: 1 REF: Pages 326-327 28. Which statements are true regarding endotoxins? (Select all that apply.) a. Endotoxins are lipopolysaccharides. b. Endotoxins are located in the walls of bacteria. c. Endotoxins are created during the process of lysis. d. Endotoxins are found in gram-negative microorganisms. e. Endotoxins are released during the destruction of its host. , B, D, E Endotoxins are lipopolysaccharides (LPSs) contained in the cell walls of gram-negative bacteria and released during lysis (or destruction) of the bacteria. PTS: 1 REF: Page 306 29. Which statements are true regarding viruses? (Select all that apply.) a. Viruses are very complex microorganisms. b. Viruses are referred to as eukaryotes. c. Viruses are capable of producing messenger RNA (mRNA). d. Viruses penetrate plasma membranes via endocytosis. e. Viruses are capable of uncoating cytoplasmic nucleocapsid. , D, E Viruses are extremely simple microorganisms and do not possess any of the metabolic organelles found in prokaryotes (e.g., bacteria) or eukaryotes (e.g., human cells). Once bound, the virus can penetrate the plasma membrane by receptor-mediated endocytosis. Within the cytoplasm, the virus uncoats the protective nucleocapsid and releases viral genetic information. Most RNA viruses directly produce mRNA, which is translated into viral proteins, and genomic RNA, which is eventually packaged into new viruses. PTS: 1 REF: Pages 317-319 30. Which of the following play a role in the control of fungal infections? (Select all that apply.) a. Cytokines b. Macrophages c. Natural killer cells d. Neutrophils e. T lymphocytes , B, D, E The host defense against fungal infection includes the fungistatic properties of neutrophils and macrophages. T lymphocytes are crucial in limiting the extent of infection and producing cytokines to further activate macrophages. Natural killer cells are a component of innate immune system. PTS: 1 REF: Page 312 31. Complications of AIDS include: (Select all that apply.) a. Kaposi sarcoma b. Helicobacter pylori c. Cytomegalovirus retinitis d. Herpes simplex infection e. Legionella pneumophila , C, D Kaposi sarcoma, cytomegalovirus retinitis, and herpes simplex infection are clinical complications characteristically observed in patients with AIDS. Neither Helicobacter pylori nor Legionella pneumophila are considered classic AIDS opportunistic diseases. PTS: 1 REF: Page 328 | Figure 10-16 MATCHING Match each term with its definition. A. Toxigenicity B. Infectivity C. Pathogenicity D. Virulence 32. Ability of the pathogen to invade and multiply in the host 33. Capacity of a pathogen to cause severe disease 34. An important factor in determining a pathogen’s ability to produce disease by the production of a soluble toxin 35. Ability of an agent to produce disease 32. PTS: 1 REF: Page 302 MSC: Infectivity is the ability of the pathogen to invade and multiply in the host. 33. PTS: 1 REF: Page 302 MSC: Virulence is the capacity of a pathogen to cause severe disease. 34. PTS: 1 REF: Page 302 MSC: Toxigenicity is the ability to produce soluble toxins or endotoxins, factors that greatly influence the pathogen's degree of virulence. 35. PTS: 1 REF: Page 302 MSC: Pathogenicity is the ability of an agent to produce disease. Chapter 11: Stress and Disease MULTIPLE CHOICE 1. Exhaustion occurs if stress continues when which stage of the general adaptation syndrome is not successful? a. Flight or fight c. Adaptation b. Alarm d. Arousal Exhaustion occurs if stress continues and adaptation is not successful, ultimately causing impairment of the immune response, heart failure, and kidney failure, leading to death. The other stages occur before the adaptation stage. PTS: 1 REF: Page 339 2. Which organ is stimulated during the alarm phase of the general adaptation syndrome (GAS)? a. Adrenal cortex c. Anterior pituitary b. Hypothalamus d. Limbic system The alarm phase of the GAS begins when a stressor triggers the actions of the hypothalamus and the sympathetic nervous system (SNS) (see Figure 11-1). The other organs are not stimulated by the alarm phase of GAS. PTS: 1 REF: Page 339 3. During an anticipatory response to stress, the reaction from the limbic system is stimulated by the: a. Retronucleus of the anterior pituitary b. Anterior nucleus of the hippocampus c. Paraventricular nucleus of the hypothalamus d. Prefrontal nucleus of the amygdala The paraventricular nucleus (PVN) of the hypothalamus must be stimulated to cause the limbic system to be stimulated. The other options are not involved in the stimulation of the limbic system. PTS: 1 REF: Page 341 4. Which hormone prompts increased anxiety, vigilance, and arousal during a stress response? a. Norepinephrine b. Epinephrine c. Cortisol d. Adrenocorticotropic hormone (ACTH) Only the release of norepinephrine promotes arousal, increased vigilance, increased anxiety, and other protective emotional responses. PTS: 1 REF: Page 343 5. Perceived stress elicits an emotional, anticipatory response that begins where? a. Prefrontal cortex c. Limbic system b. Anterior pituitary d. Hypothalamus Perceived stressors elicit an anticipatory response that begins in the limbic system of the brain, the only option responsible for emotions and cognition. PTS: 1 REF: Page 343 6. During a stress response, the helper T (Th) 1 response is suppress by which hormone? a. ACTH c. Prolactin b. Cortisol d. Growth hormone Stress can activate an excessive immune response and, through cortisol and catecholamines, suppress the Th1 response, causing a Th2 shift. This response is not active by any of the other options. PTS: 1 REF: Page 349 7. What is the effect that low-serum albumin has on the central stress response? a. Impaired circulation of epinephrine and norepinephrine b. Impaired wound healing c. Lessened circulation of cortisol d. Diminished oncotic pressure Low-serum albumin impairs circulation of both epinephrine and norepinephrine since both bind to plasma protein albumin. The other options do not accurately describe the effect of low-serum albumin. PTS: 1 REF: Page 345 8. Stress-age syndrome directly results in depressed function of which system? a. Respiratory c. Digestive b. Endocrine d. Immune Of the available options, immunodepression is the only characteristic change observed in stress-age syndrome. PTS: 1 REF: Page 358 9. Stress-induced sympathetic stimulation of the adrenal medulla causes the secretion of: a. Epinephrine and aldosterone c. Epinephrine and norepinephrine b. Norepinephrine and cortisol d. Acetylcholine and cortisol The sympathetic nervous system is aroused during the stress response and causes the medulla of the adrenal gland to release catecholamines (80% epinephrine and 20% norepinephrine) into the bloodstream. The stress-induced efforts on the adrenal medulla do not include any of the other options. PTS: 1 REF: Pages 344-345 10. Stress-induced norepinephrine results in: a. Pupil constriction c. Increased sweat gland secretions b. Peripheral vasoconstriction d. Decreased blood pressure During stress, norepinephrine raises blood pressure by constricting peripheral vessels; it dilates the pupils of the eye, causes piloerection, and increases sweat gland action in the armpits and palms. PTS: 1 REF: Page 345 11. Released stress-induced cortisol results in the stimulation of gluconeogenesis by affecting which structure? a. Adrenal cortex c. Liver b. Pancreas d. Anterior pituitary One of the primary effects of cortisol is the stimulation of gluconeogenesis through stimulation of the adrenal cortex. The other options do not produce a stimulation of gluconeogenesis when exposed to cortisol. PTS: 1 REF: Page 346 12. What is the effect of increased secretions of epinephrine, glucagon, and growth hormone? a. Hyperglycemia c. Bronchodilation b. Hypertension d. Pupil dilation Cortisol enhances the elevation of blood glucose promoted by other hormones, such as epinephrine, glucagon, and growth hormone. This effect is not true of the other options. PTS: 1 REF: Page 346 13. Which hormone increases the formation of glucose from amino acids and free fatty acids? a. Epinephrine c. Cortisol b. Norepinephrine d. Growth hormone One of the primary effects of cortisol is the stimulation of gluconeogenesis or the formation of glucose from noncarbohydrate sources, such as amino or free fatty acids in the liver. Neither reaction is a result of the effects of any of the other options. PTS: 1 REF: Page 346 14. What effect do androgens have on lymphocytes? a. Suppression of B-cell responses and enhancement of T-cell responses b. Suppression of T-cell responses and enhancement of B-cell responses c. Suppression of B- and T-cell responses d. Enhancement of B- and T-cell responses Androgens suppress T- and B-cell responses. The other options do not occur in response to androgens. PTS: 1 REF: Page 353 15. Which gland regulates the immune response and mediates the apparent effects of circadian rhythms on immunity? a. Anterior pituitary c. Basal ganglia b. Adrenal d. Pineal Of the options available, only the pineal gland regulates the immune response and mediates the apparent effects of circadian rhythm on immunity. PTS: 1 REF: Page 354 16. Which cytokines initiate the production of corticotropin-releasing hormone (CRH)? a. IL–1 and IL-6 c. IFN and IL-12 b. IL-2 and TNF- d. TNF-ß and IL-4 Although a number of stress factors initiate the production of CRH, of the options available, only high levels of IL-1 and IL-6 initiate such a response. PTS: 1 REF: Pages 348-349 17. The release of which cytokines is triggered by bacterial or viral infections, cancer, and tissue injury that, in turn, initiate a stress response? a. IL-1 and IL-2 b. IL-12, TNF-, and colony-stimulating factor c. IFN, TNF-ß, and IL-6 d. IL-4 and IL-24 Of the options offered, only the release of immune inflammatory mediators IL-6, TNF-ß, and IFN is triggered by bacterial or viral infections, cancer, and tissue injury that, in turn, initiates a stress response through the hypothalamic-pituitary-adrenal (HPA) pathway. PTS: 1 REF: Page 349 18. The action of which hormone helps explain increases in affective anxiety and eating disorders, mood cycles, and vulnerability to autoimmune and inflammatory diseases in women as a result of stimulation of the CRH gene promoter and central norepinephrine system? a. Progesterone c. Estrogen b. Cortisol d. Prolactin Of the options provided, only estrogen directly stimulates the CRH gene promoter and the central noradrenergic (norepinephrine) system, which may help explain adult women’s slight hypercortisolism, increases in affective anxiety and eating disorders, mood cycles, and vulnerability to autoimmune and inflammatory disease, all of which follow estradiol fluctuations. PTS: 1 REF: Page 350 19. What effect does estrogen have on lymphocytes? a. Depression of B-cell functions and enhancement of T-cell functions b. Depression of T-cell functions and enhancement of B-cell functions c. Depression of B- and T-cell functions d. Enhancement of B- and T-cell functions Estrogens generally are associated with only a depression of T-cell–dependent immune functions and an enhancement of B-cell functions. PTS: 1 REF: Page 353 20. Which statement is true concerning the differences between stress-induced hormonal alterations of men and women? a. After injury, women produce more proinflammatory cytokines than men, a profile that is associated with poor outcomes. b. Androgens appear to induce a greater degree of immune cell apoptosis after injury, creating greater immunosuppression in injured men than in injured women. c. Psychologic stress associated with some types of competition decreases both testosterone and cortisol, especially in athletes older than 45 years of age. d. After stressful stimuli, estrogen is increased in women, but testosterone is decreased in men. Androgens appear to induce a greater degree of immune cell apoptosis after injury, a mechanism that may elicit a greater immunosuppression in injured men versus injured women. The other options are not true statements concerning the differences between how the genders are affected by stress-induced hormones. PTS: 1 REF: Page 353 21. Diagnostic blood work on individuals who perceive themselves to be in a chronic stress state will likely demonstrate: a. Decreased Th lymphocytes c. Decreased Tc cells b. Increased erythrocytes d. Increased platelets Illustrating the influence of chronic stress appraisal on the physiologic processes, a meta-analysis of the relationships between stressors and immunity found that a higher perception of stress was associated with reduced T cytotoxic (Tc)-cell cytotoxicity, although not with levels of circulating Th or Tc lymphocytes. Research has substantiates the other options. PTS: 1 REF: Page 355 22. What are the signs that a patient is in the adaptive stage of the general adaptation syndrome? a. He or she begins to experience elevated heart and respiratory rates. b. He or she finds it difficult to concentrate on a solution for the stress. c. The patient perceives his or her only options are to run away or fight back. d. The patient has exceeded his or her ability to cope with the current situation. Fight-or-flight behaviors are characteristic of the more advanced adaptive stage, whereas the remaining options are noted in the initial alarm stage. PTS: 1 REF: Page 339 23. The most influential factor in whether a person will experience a stress reaction is his or her: a. General state of physical health c. Intellectual abilities b. Spiritual belief system d. Ability to cope A person does not have a stress reaction unless the stress exceeds his or her coping abilities. The other options do not have the same degree of influence, as does a person’s ability to cope. PTS: 1 REF: Page 339 24. A reduction is an individual’s number of natural killer (NK) cells appears to correlate with an increased risk for the development of: a. Depression b. Type 1 diabetes c. Obsessive compulsive disorder (OCD) d. Gastroesophageal reflux disorder (GERD) A meta-analysis of studies shows a relationship between depression and the reduction in lymphocyte proliferation and natural killer cell activity. Currently, no research supports the other options. PTS: 1 REF: Page 355 25. A nurse is providing care to a terminally ill adult who has been with his life partner for over 56 years. Research supports the nurse’s assessment of the life partner for signs of: a. Suicidal ideations c. Severe stress reaction b. Cardiac dysrhythmia d. Anorexia induced weight loss The results of a Harvard study showed evidence that a spouse’s illness or death can increase a partner’s mortality by causing severe stress and removing a primary source of emotional, psychologic, practical, and financial support. Although the other options may exist, research does not currently support them as having the stated correlation. PTS: 1 REF: Page 357 | What's New box MULTIPLE RESPONSE 26. The effect epinephrine has on the immune system during the stress response is to increase which cells? (Select all that apply.) a. NK cells b. Immunoglobulins c. Cytokines d. T cells e. Th cells , D The injection of epinephrine into healthy human beings is associated with a transient increase of the number of lymphocytes (e.g., T cells, natural killer (NK) cells) in the peripheral blood. This association is not true of the other options. PTS: 1 REF: Page 346 27. Which immune cells are suppressed by the corticotropin-releasing hormone (CRH)? (Select all that apply.) a. Monocyte-macrophage cells b. Cytokines c. Tc cells d. Th cells e. B cells , D Direct suppressive effects of CRH have been reported on two immune cell types possessing CRH receptors—the monocyte and macrophage and CD4 (T helper) lymphocytes. CRH does not suppress the remaining options. PTS: 1 REF: Pages 353-354 28. The increased production of proinflammatory cytokines is associated with which considerations? (Select all that apply.) a. Chronic respiratory dysfunction b. Elevated anxiety levels c. Immune disorders d. Age and gender e. Dementia , C Increased levels of proinflammatory cytokines has been shown to have a possible link between stress and immune function. The other options are not as directly linked to cytokine levels. PTS: 1 REF: Page 353 29. Which statements are true regarding lymphocytes? (Select all that apply.) a. Lymphocytes are involved in the production of the human growth hormone. b. Elevated catecholamine levels influence lymphocytes. c. Lymphocytes are synthesized in the anterior pituitary gland. d. Lymphocytes have receptors for the hormone prolactin. e. Lymphocytes produce endorphins in large amounts. , B, D GH is synthesized from the anterior pituitary gland and is produced by lymphocytes and mononuclear phagocytic cells. Several classes of lymphocytes have receptors for prolactin, suggesting a direct effect of prolactin on immune function. Although the effects of acute elevation of catecholamines on the alteration of lymphocyte function are real, they are short lived, lasting only approximately 2 hours. The other statements regarding lymphocytes are not true. PTS: 1 REF: Page 350 | Page 352 MATCHING Match the hormone with its effects during a stress response. Hormones may be used more than once. A. Epinephrine B. Norepinephrine C. Cortisol 30. Constricts peripheral vessels to increase blood pressure. 31. Increases cardiac output by increasing heart rate and myocardial contractility. 32. Increases gastric secretions. 30. PTS: 1 REF: Page 345 MSC: Norepinephrine regulates blood pressure by constricting smooth muscle in all blood vessels. 31. PTS: 1 REF: Page 345 MSC: Epinephrine enhances myocardial contractility (inotropic effect), increases the heart rate (chronotropic effect), and increases venous return to the heart, all of which increase cardiac output and blood pressure. 32. PTS: 1 REF: Page 347 MSC: Cortisol promotes gastric secretion in the gastrointestinal tract. Chapter 7: Innate Immunity: Inflammation MULTIPLE CHOICE 1. Which action is a purpose of the inflammatory process? a. To provide specific responses toward antigens b. To lyse cell membranes of microorganisms c. To prevent infection of the injured tissue d. To create immunity against subsequent tissue injury If the epithelial barrier is damaged, then a highly efficient local and systemic response (inflammation) is mobilized to limit the extent of damage, to protect against infection, and to initiate the repair of damaged tissue. The other options do not accurately identify a purpose of the inflammatory process. PTS: 1 REF: Page 191 2. How do surfactant proteins A through D provide innate resistance? a. Initiate the complement cascade. c. Secrete mucus. b. Promote phagocytosis. d. Synthesize lysosomes. The lung produces and secretes a family of glycoproteins, collectins, which includes surfactant proteins A through D and mannose-binding lectin. Collectin binding facilitates macrophages to recognize the microorganism, enhancing macrophage attachment, phagocytosis, and killing. The other options do not accurately identify how surfactant proteins provide innate resistance. PTS: 1 REF: Page 194 3. Which secretion is a first line of defense against pathogen invasion that involves antibacterial and antifungal fatty acids, as well as lactic acid? a. Optic tears c. Sweat gland perspiration b. Oral saliva d. Sebaceous gland sebum Sebaceous glands in the skin secrete sebum that is made up of antibacterial and antifungal fatty acids and lactic acid that provide the first-line barrier against pathogen invasion. PTS: 1 REF: Pages 192-193 4. Which bacterium grows in the intestines after prolonged antibiotic therapy? a. Lactobacillus c. Clostridium difficile b. Candida albicans d. Helicobacter pylori Prolonged antibiotic treatment can alter the normal intestinal flora, decreasing its protective activity and leading to the overgrowth of other microorganisms, such as the yeast C. albicans or the bacterium C. difficile. The other options do not accurately identify intestinal bacterium whose growth is a result of prolonged antibiotic therapy. PTS: 1 REF: Page 194 5. What causes the edema that occurs during the inflammatory process? a. Vasodilation of blood vessels c. Endothelial cell contraction b. Increased capillary permeability d. Emigration of neutrophils The increased flow and capillary permeability result in a leakage of plasma from the vessels, causing swelling (edema) in the surrounding tissue and is solely responsible for inflammation-induced edema. PTS: 1 REF: Page 195 6. What process causes heat and redness to occur during the inflammatory process? a. Vasodilation of blood vessels c. Decreased capillary permeability b. Platelet aggregation d. Endothelial cell contraction The increased blood flow as a result of vasodilation and increasing concentration of red cells at the site of inflammation cause locally increased warmth and redness. The other options do not accurately identify the process that results in inflammatory redness and heat. PTS: 1 REF: Page 195 7. Activation of the classical pathway begins with: a. Viruses c. Mast cells b. Antigen-antibody complexes d. Macrophages Activation of the classical pathway begins only with the activation of protein C1 and is preceded by the formation of a complex between an antigen and an antibody to form an antigen-antibody complex (immune complex) (see Chapter 8). PTS: 1 REF: Page 197 8. What plasma protein system forms a fibrinous meshwork at an inflamed site? a. Complement c. Kinin b. Coagulation d. Fibrinolysis The coagulation (clotting) system is a group of plasma proteins that form a fibrinous meshwork at an injured or inflamed site. This protein system (1) prevents the spread of infection to adjacent tissues, (2) traps microorganisms and foreign bodies at the site of inflammation for removal by infiltrating cells (e.g., neutrophils and macrophages), (3) forms a clot that stops the bleeding, and (4) provides a framework for future repair and healing. The other options do not accurately identify such a protein system. PTS: 1 REF: Page 199 | Page 201 9. Which component of the plasma protein system tags pathogenic microorganisms for destruction by neutrophils and macrophages? a. Complement cascade c. Kinin system b. Coagulation system d. Immune system C3b (a component of the complement cascade) adheres to the surface of a pathogenic microorganism and serves as an efficient opsonin. Opsonins are molecules that tag microorganisms for destruction by cells of the inflammatory system, primarily neutrophils and macrophages. The other options do not accurately identify a component capable of tagging pathogenic microorganisms. PTS: 1 REF: Pages 197-199 10. What is the vascular effect of histamine released from mast cells? a. Platelet adhesion c. Vasodilation b. Initiation of the clotting cascade d. Increased endothelial adhesiveness C2b affects smooth muscle, causing vasodilation and increased vascular permeability. C3a, C5a, and, to a limited extent, C4a are anaphylatoxins; that is, they induce rapid mast cell degranulation (i.e., release of granular contents) and the release of histamine, causing vasodilation and increased capillary permeability. The other options do not accurately describe the vascular effect of histamine released from mast cells? PTS: 1 REF: Pages 198-199 11. What is an outcome of the complement cascade? a. Activation of the clotting cascade b. Prevention of the spread of infection to adjacent tissues c. Inactivation of chemical mediators such as histamine d. Lysis of bacterial cell membranes The complement cascade can be activated by at least three different means, and its products have four functions: (1) anaphylatoxic activity, resulting in mast cell degranulation, (2) leukocyte chemotaxis, (3) opsonization, and (4) cell lysis. The other options do not accurately describe an outcome of the complement cascade. PTS: 1 REF: Page 199 12. The function of opsonization related to the complement cascade is to: a. Tag of pathogenic microorganisms for destruction by neutrophils and macrophages. b. Process pathogenic microorganisms so that activated lymphocytes can be created for acquired immunity. c. Destroy glycoprotein cell membranes of pathogenic microorganisms. d. Promote anaphylatoxic activity, resulting in mast cell degranulation. C3b adheres to the surface of a pathogenic microorganism and serves as an efficient opsonin. Opsonins are molecules that tag microorganisms for destruction by cells of the inflammatory system, primarily neutrophils and macrophages. The other options do not accurately describe the function of opsonization related to the complement cascade. PTS: 1 REF: Page 199 13. In the coagulation (clotting) cascade, the intrinsic and the extrinsic pathways converge at which factor? a. XII c. X b. VII d. V The coagulation cascade consists of the extrinsic and intrinsic pathways that converge only at factor X. PTS: 1 REF: Page 201 14. Which chemical interacts among all plasma protein systems by degrading blood clots, activating complement, and activating the Hageman factor? a. Kallikrein c. Bradykinin b. Histamine d. Plasmin Only plasmin regulates clot formation by degrading fibrin and fibrinogen, and it can activate the complement cascade through components C1, C3, and C5. Plasmin can activate the plasma kinin cascade by activating the Hageman factor (factor XII) and producing prekallikrein activator. PTS: 1 REF: Page 201 15. The chemotactic factor affects the inflammatory process by: a. Causing vasodilation around the inflamed area b. Stimulating smooth muscle contraction in the inflamed area c. Directing leukocytes to the inflamed area d. Producing edema around the inflamed area Two chemotactic factors, neutrophil chemotactic factor (NCF) and eosinophil chemotactic factor of anaphylaxis (ECF-A), are released during mast cell degranulation. NCF attracts neutrophils (a type of leukocytes), and ECF-A attracts eosinophils to the site of inflammation. The other options do not accurately describe the affect chemotactic factors have on the inflammatory process. PTS: 1 REF: Page 207 16. What affect does the process of histamine binding to the histamine-2 (H2) receptor have on inflammation? a. Inhibition c. Acceleration b. Activation d. Termination Binding of histamine to the H1 receptor is essentially proinflammatory; that is, it promotes inflammation. On the other hand, binding histamine to the H2 receptor is generally antiinflammatory because it results in the suppression of leukocyte function. The other options do not accurately describe the affect histamine binding to the H2 receptor has on inflammation. PTS: 1 REF: Page 206 17. Frequently when H1 and H2 receptors are located on the same cells, they act in what fashion? a. Synergistically c. Antagonistically b. Additively d. Agonistically Both types of receptors are distributed among many different cells and are often present on the same cells and may act in an antagonistic fashion. For instance, neutrophils express both types of receptors, with stimulation of H1 receptors resulting in the augmentation of neutrophil chemotaxis and H2 stimulation resulting in its inhibition. The other options do not accurately describe the relationship between H1 and H2 receptors. PTS: 1 REF: Page 207 18. Some older adults have impaired inflammation and wound healing because of which problem? a. Circulatory system cannot adequately perfuse tissues. b. Complement and chemotaxis are deficient. c. Underlying chronic illness(es) exists. d. Number of mast cells is insufficient. In some cases, impaired healing is not directly associated with aging, in general, but can instead be linked to a chronic illness such as cardiovascular disease or diabetes mellitus. The other problems are not related to the aging process. PTS: 1 REF: Page 220 19. Which chemical mediator derived from mast cells retracts endothelial cells to increase vascular permeability and to cause leukocyte adhesion to endothelial cells? a. Leukotrienes c. Platelet-activating factor b. Prostaglandin E d. Bradykinin The biologic activity of platelet-activating factor is virtually identical to that of leukotrienes; namely, it causes endothelial cell retraction to increase vascular permeability, leukocyte adhesion to endothelial cells, and platelet activation. The other options do not accurately identify the chemical mediator derived from the process described in the question. PTS: 1 REF: Page 207 20. What is the inflammatory effect of nitric oxide (NO)? a. Increases capillary permeability, and causes pain. b. Increases neutrophil chemotaxis and platelet aggregation. c. Causes smooth muscle contraction and fever. d. Decreases mast cell function, and decreases platelet aggregation. Effects of NO on inflammation include vasodilation by inducing relaxation of vascular smooth muscle, a response that is local and short lived, and by suppressing mast cell function, as well as platelet adhesion and aggregation. The other options do not accurately identify the effect of NO on the process of inflammation. PTS: 1 REF: Page 209 21. What is the correct sequence in phagocytosis? a. Engulfment, recognition, fusion, destruction b. Fusion, engulfment, recognition, destruction c. Recognition, engulfment, fusion, destruction d. Engulfment, fusion, recognition, destruction Once the phagocytic cell enters the inflammatory site, the only correct sequence of phagocytosis involves the following steps: (1) opsonization, or recognition, of the target and adherence of the phagocyte to it; (2) engulfment, or ingestion or endocytosis, and the formation of phagosome; (3) fusion with lysosomal granules within the phagocyte (phagolysosome); and (4) destruction of the target. PTS: 1 REF: Pages 210-211 22. When considering white blood cell differentials, acute inflammatory reactions are related to elevations of which leukocyte? a. Monocytes c. Neutrophils b. Eosinophils d. Basophils Only neutrophils are the predominant phagocytes in the early inflammatory site, arriving within 6 to 12 hours after the initial injury, they ingest (phagocytose) bacteria, dead cells, and cellular debris at the inflammatory site. PTS: 1 REF: Pages 208-209 23. In the later stages of an inflammatory response, which phagocytic cell is predominant? a. Neutrophils c. Chemokines b. Monocytes d. Eosinophils Only monocytes and macrophages perform many of the same functions as neutrophils but for a longer time and in a later stage of the inflammatory response. PTS: 1 REF: Page 209 | Page 212 24. In regulating vascular mediators released from mast cells, the role of eosinophils is to release: a. Arylsulfatase B, which stimulates the formation of B lymphocytes b. Histaminase, which limits the effects of histamine during acute inflammation c. Lysosomal enzymes, which activate mast cell degranulation during acute inflammation d. Immunoglobulin E, which defends the body against parasites Eosinophil lysosomes contain several enzymes that degrade vasoactive molecules, thereby controlling the vascular effects of inflammation. These enzymes include histaminase, which mediates the degradation of histamine, and arylsulfatase B, which mediates the degradation of some of the lipid-derived mediators produced by mast cells. The other options do not accurately describe the role of eosinophils. PTS: 1 REF: Page 209 25. What is the role of a natural killer (NK) cells? a. Initiation of the complement cascade b. Elimination of malignant cells c. Binding tightly to antigens d. Proliferation after immunization with antigen The main function of NK cells is to recognize and eliminate cells infected with viruses, although they are also somewhat effective at eliminating other abnormal host cells, specifically cancer cells. The other options do not accurately identify the role of a NK cell. PTS: 1 REF: Page 213 26. Which cytokine is produced and released from virally infected host cells? a. IL-1 c. TNF- b. IL-10 d. IFN- Only interferons (IFNs) are produced and released by virally infected cells in response to viral double-stranded ribonucleic acid (RNA). IFN- and IFN- induce the production of antiviral proteins, thereby conferring protection on uninfected cells. IFN- or IFN- is released from virally infected cells and attaches to a receptor on a neighboring cell. IFNs also enhance the efficiency of developing an acquired immune response. PTS: 1 REF: Pages 204-205 27. IFN- is secreted from which cells? a. Virally infected cells c. Macrophages b. Bacterial infected cells d. Mast cells Different kinds of interferons (IFNs) are produced by different types of cells—macrophages are the primary producers of both IFN- and IFN-. The other options do not accurately identify cells secreted by IFN-. PTS: 1 REF: Pages 204-205 28. Which manifestation of inflammation is systemic? a. Formation of exudates c. Redness and heat b. Fever and leukocytosis d. Pain and edema The only three primary systemic changes associated with the acute inflammatory response are fever, leukocytosis (a transient increase in circulating leukocytes), and increased levels in circulating plasma proteins. PTS: 1 REF: Page 213 29. The acute inflammatory response is characterized by fever that is produced by the hypothalamus being affected by: a. Endogenous pyrogens c. Antigen-antibody complexes b. Bacterial endotoxin d. Exogenous pyrogens Fever-causing cytokines are known as endogenous pyrogens. These pyrogens act directly on the hypothalamus, which is the portion of the brain that controls the body’s thermostat. The other options do not accurately describe the cause of fever related to the effects on the hypothalamus. PTS: 1 REF: Page 213 30. What occurs during the process of repair after tissue damage? a. Nonfunctioning scar tissue replaces destroyed tissue. b. Regeneration occurs; the original tissue is replaced. c. Resolution occurs; tissue is regenerated. d. Epithelialization replaces destroyed tissue. Repair is the replacement of destroyed tissue with scar tissue. Scar tissue is primarily made up of collagen, which fills in the lesion and restores tensile strength but cannot carry out the physiologic functions of the destroyed tissue. The other options do not accurately describe the process of repair after tissue damage. PTS: 1 REF: Pages 215-216 31. The role of fibroblasts during the reconstructive phase of wound healing is to: a. Generate new capillaries from vascular endothelial cells around the wound. b. Establish connections between neighboring cells and contract their fibers. c. Synthesize and secrete collagen and the connective tissue proteins. d. Provide enzymes that débride the wound bed of dead cells. Fibroblasts are the most important cells during the reconstructive phase of wound healing because they synthesize and secrete collagen and other connective tissue proteins. Macrophage-derived transforming growth factor–beta (TGF-ß) stimulates fibroblasts. The other options do not accurately describe the role of fibroblasts in the reconstructive phase of wound healing. PTS: 1 REF: Pages 216-218 32. A keloid is the result of which dysfunctional wound healing response? a. Epithelialization c. Collagen matrix assembly b. Contraction d. Maturation An imbalance between collagen synthesis and collagen degradation, during which synthesis is increased relative to degradation, causes both keloids and hypertrophic scars. The other options are not involved in keloids production. PTS: 1 REF: Page 219 33. Which solution is best to use when cleaning a wound that is healing by epithelialization? a. Normal saline c. Hydrogen peroxide b. Povidone-iodine d. Dakin solution Normal saline is the most innocuous solution that can be used to cleanse or irrigate a wound that is primarily healing by epithelialization and is the only correct answer for this question. PTS: 1 REF: Page 220 34. Many neonates have a transient depressed inflammatory response as a result of which condition? a. The circulatory system is too immature to perfuse tissues adequately. b. Complement and chemotaxis are deficient. c. Mast cells are lacking. d. The respiratory system is too immature to deliver oxygen to tissues. Neonates commonly have transiently depressed inflammatory and immune function partially as a result of a deficiency in components of the alternative pathway. For example, neutrophils and perhaps monocytes may not be capable of efficient chemotaxis. The other options do not accurately explain the common cause of a transient depressed inflammatory response in neonates. PTS: 1 REF: Page 220 35. During phagocytosis, what is occurring during the step referred to as opsonization? a. Phagocytes recognize and adhere to the bacteria. b. Microorganisms are ingested. c. Microorganisms are killed and digested. d. An intracellular phagocytic vacuole is formed. During phagocytosis, opsonization involves only the recognition and adherence of phagocytes to bacteria. PTS: 1 REF: Page 210 36. Fusion is the step in phagocytosis during which: a. Microorganisms are killed and digested. b. An intracellular phagocytic vacuole is formed. c. Lysosomal granules enter the phagocyte. d. Microorganisms are ingested. Fusion occurs with lysosomal granules entering the phagocyte (phagolysosome). The remaining options do not accurately describe fusion as a step in phagocytosis. PTS: 1 REF: Page 210 37. During the process of endocytosis, the phagosome step results in: a. Microorganisms are ingested. b. Microorganisms are killed and digested. c. Phagocytes recognize and adhere to bacteria. d. An intracellular phagocytic vacuole is formed. Small pseudopods that extend from the plasma membrane and surround the adherent microorganism, forming an intracellular phagocytic vacuole or phagosome, carry out engulfment (endocytosis). The membrane that surrounds the phagosome consists of inverted plasma membrane. After the formation of the phagosome, lysosomes converge, fuse with the phagosome, and discharge their contents, creating a phagolysosome. PTS: 1 REF: Pages 210-211 38. When cellular damage occurs and regeneration is minor with no significant complications, the process of returning the cells to preinjury function is referred to as: a. Restoration c. Regrowth b. Resolution d. Replacement If damage is minor with no complications and destroyed tissues are capable of regeneration, then returning the injured tissues to an approximation of their original structure and physiologic function is possible. This restoration is called resolution. The other terms are not used to describe this process. PTS: 1 REF: Page 215 39. Newborns often have deficiencies in collectin-like proteins, making them more susceptible to what type of infection? a. Cardiac c. Respiratory b. Urinary d. Gastrointestinal Neonates may also be deficient in some of the collectins and collectin-like proteins. This deficiency is especially true of preterm neonates. Some preterm infants with respiratory distress syndrome are deficient in at least one collectin, which negatively affects its innate defense against respiratory infections. The other options are not necessarily related to collectin deficiencies. PTS: 1 REF: Page 220 40. Which cell is the body’s primary defense against parasite invasion? a. Eosinophil c. T lymphocytes b. Neutrophils d. B lymphocytes Eosinophils serve as the body’s primary defense against parasites. T lymphocytes and B lymphocytes are involved in acquired immunity. Neutrophils are the predominant phagocytes in the early inflammatory site. PTS: 1 REF: Page 209 MULTIPLE RESPONSE 41. Which chemical mediators induce pain during an inflammatory response? (Select all that apply.) a. Prostaglandins b. Leukotrienes c. Tryptase d. Phospholipase e. Bradykinin , E The only chemical mediators that induce pain during an inflammatory response are the prostaglandins and bradykinin. PTS: 1 REF: Page 201 42. Sebaceous glands protect the body from infection by secreting: (Select all that apply.) a. Antibacterial fatty acids b. Antifungal fatty acids c. Ascorbic acid d. Lactic acid e. Hydrochloric acid , B, D Sebaceous glands secrete only antibacterial and antifungal fatty acids and lactic acid. PTS: 1 REF: Pages 192-193 43. Which body fluid has the ability to attack the cell walls of gram-positive bacteria? (Select all that apply.) a. Perspiration b. Semen c. Tears d. Saliva e. Urine , C, D Only perspiration, tears, and saliva contain an enzyme (lysozyme) that attacks the cell walls of gram-positive bacteria. PTS: 1 REF: Page 193 44. The main function of NK cells includes: (Select all that apply.) a. Recognizing virus-infected cells b. Eliminating virus-infected cells c. Recognizing bacteria-infected cells d. Eliminating bacteria-infected cells e. Eliminating previously identified cancer cells , B, E The main functions of NK cells are recognizing and eliminating cells infected with viruses, not bacteria. They are also somewhat effective at eliminating other abnormal host cells, specifically cancer cells. PTS: 1 REF: Page 213 45. Normal bacterial flora found in the intestines produce vitamin K to assist in the absorption of which of the following? (Select all that apply.) a. Calcium b. Fatty acids c. Large polysaccharides d. Iron e. Magnesium , D, E The flora’s production of vitamin K is needed to absorb various ions, such as calcium, iron, and magnesium. Normal intestinal flora is responsible for digesting fatty acids, large polysaccharides, and other dietary substance, but such digestion is not reliant on vitamin K. PTS: 1 REF: Page 194 46. An individual’s acquired immunity is dependent on the function of which cells? (Select all that apply.) a. T lymphocytes b. B lymphocytes c. Macrophages d. Opsonins e. Neutrophils , B, C T lymphocytes, B lymphocytes, macrophages, and dendritic cells are involved in acquired immunity. Opsonins are molecules that tag microorganisms for destruction by cells of the inflammatory system; these cells are primarily neutrophils. PTS: 1 REF: Page 192 | Table 7-1 47. An example of a pathogen capable of surviving and even multiplying inside a macrophage is known as: (Select all that apply.) a. Mycobacterium tuberculosis (tuberculosis) b. Mycobacterium leprae (leprosy) c. Salmonella typhi (typhoid fever) d. Clostridium difficile e. Brucella abortus (brucellosis) , B, C, E Several bacteria are resistant to killing by granulocytes and can even survive inside macrophages. Microorganisms such as M. tuberculosis (tuberculosis), M. leprae (leprosy), S. typhi (typhoid fever), and B. abortus (brucellosis) can remain dormant or even multiply inside the phagolysosomes of macrophages. C. difficile is said to be resistant to antibiotics, making it difficult to control. PTS: 1 REF: Pages 212-213 48. An older adult is particularly susceptible to infections of which body parts? (Select all that apply.) a. Lungs b. Skin c. Liver d. Eyes e. Bladder , B, E Older adults have increased susceptibility to bacterial infections of the lungs, urinary tract, and skin. Other infections may occur but on an individualized basis. PTS: 1 REF: Page 220 MATCHING Match each step of phagocytosis with its function. A. Opsonization B. Engulfment C. Phagosome D. Fusion E. Destruction 49. Microorganisms are ingested. 50. Microorganisms are killed and digested. 51. Phagocytes gain enhanced recognition and adherence of bacteria. 52. Lysosomal granules enter the phagocyte. 53. Intracellular phagocytic vacuole is formed. 49. PTS: 1 REF: Pages 210-211 MSC: Engulfment is the ingestion of phagosomes. 50. ANS: E PTS: 1 REF: Pages 210-211 MSC: Destruction is the step during which microorganisms are killed and digested. 51. PTS: 1 REF: Pages 210-211 MSC: Opsonization is the recognition and adherence of phagocytes to bacteria. 52. PTS: 1 REF: Pages 210-211 MSC: Fusion occurs with lysosomal granules within the phagocyte (phagolysosome). 53. PTS: 1 REF: Pages 210-211 MSC: Small pseudopods that extend from the plasma membrane and surround the adherent microorganism, forming an intracellular phagocytic vacuole or phagosome, carry out engulfment (endocytosis). Chapter 8: Adaptive Immunity MULTIPLE CHOICE 1. Which primary characteristic is unique for the immune response? a. The immune response is similar each time it is activated. b. The immune response is specific to the antigen that initiates it. c. The response to a specific pathogen is short term. d. The response is innate, rather than acquired. Unlike inflammation, which is nonspecifically activated by cellular damage and pathogenic microorganisms, the immune response is primarily designed to afford long-term specific protection (i.e., immunity) against particular invading microorganisms; that is, it has a memory function. The other options are not unique characteristics of the immune response. PTS: 1 REF: Page 225 2. In which structure does B lymphocytes mature and undergo changes that commit them to becoming B cells? a. Thymus gland c. Bone marrow b. Regional lymph nodes d. Spleen B lymphocytes mature and become B cells in specialized (primary) lymphoid organs—the thymus gland for T cells and the bone marrow for B cells. Neither regional lymph nodes nor the spleen are involved in changing B lymphocytes into B cells. PTS: 1 REF: Page 225 3. What is the term for the process during which lymphoid stem cells migrate and change into either immunocompetent T cells or immunocompetent B cells? a. Clonal diversity c. Clonal selection b. Clonal differentiation d. Clonal competence The process is called the generation of clonal diversity and occurs in specialized (primary) lymphoid organs—the thymus gland for T cells and the bone marrow for B cells. The other options do not accurately identify the process described in the question. PTS: 1 REF: Page 225 4. Which type of immunity is produced by an individual after either natural exposure to the antigen or after immunization against the antigen? a. Passive-acquired immunity c. Passive-innate immunity b. Active-acquired immunity d. Active-innate immunity An individual produces active-acquired immunity (active immunity) after natural exposure to an antigen or after immunization, whereas passive-acquired immunity (passive immunity) does not involve the host’s immune response at all. The innate immune system, also known as nonspecific immune system and the first line of defense, is composed of the cells and mechanisms that defend the host from infection by other organisms in a nonspecific manner, which means that the cells of the innate system recognize and respond to pathogens in a generic way. PTS: 1 REF: Page 227 5. What type of immunity is produced when an immunoglobulin crosses the placenta? a. Passive-acquired immunity c. Passive-innate immunity b. Active-acquired immunity d. Active-innate immunity Passive-acquired immunity (passive immunity) does not involve the host’s immune response at all. Rather, passive immunity occurs when preformed antibodies or T lymphocytes are transferred from a donor to the recipient. This transfer can occur naturally, as in the passage of maternal antibodies across the placenta to the fetus, or artificially, as in a clinic using immunotherapy for a specific disease. The remaining options do not produce immunity via immunoglobulin transfer across the placenta. PTS: 1 REF: Page 227 6. The portion of the antigen that is configured for recognition and binding is referred to as what type of determinant? a. Immunotope c. Epitope b. Paratope d. Antigenitope The precise portion of the antigen that is configured for recognition and binding is called its antigenic determinant or epitope. The other options are not used to identify this portion of the antigen. PTS: 1 REF: Page 228 7. Which characteristic is the most important determinant of immunogenicity when considering the antigen? a. Size c. Complexity b. Foreignness d. Quantity Foremost among the criteria for immunogenicity is the antigen’s foreignness. A self-antigen that fulfills all of these criteria except foreignness does not normally elicit an immune response. Thus most individuals are tolerant of their own antigens. The immune system has an exquisite ability to distinguish self (self-antigens) from nonself (foreign antigens). The other options are considered when determining immunogenicity. PTS: 1 REF: Page 229 8. When antigens are administered to produce immunity, why are different routes of administration considered? a. Different routes allow the speed of onset of the antigen to be varied, with the intravenous route being the fastest. b. Some individuals appear to be unable to respond to an antigen by a specific route, thus requiring the availability of different routes for the same antigen. c. Antigen-presenting cells are highly specialized and thus require stimulation by different routes. d. Each route stimulates a different lymphocyte-containing tissue, resulting in different types of cellular and humoral immunity. Each route preferentially stimulates a different set of lymphocyte-containing (lymphoid) tissues and therefore results in the induction of different types of cell-mediated or humoral immune responses. The other options do not accurately explain the use of different routes when administering antigens to produce immunity. PTS: 1 REF: Page 229 9. The functions of the major histocompatibility complex (MHC) and CD1 molecules are alike because both: a. Are antigen-presenting molecules. b. Bind antigens to antibodies. c. Secrete interleukins during the immune process. d. Are capable of activating cytotoxic T lymphocytes. MHC and CD1 molecules are both antigen presenting molecules (APCs). The other options do not accurately describe the common function of these cells. PTS: 1 REF: Page 233 | Page 235 10. Where are antibodies produced? a. Helper T lymphocytes c. Plasma cells b. Thymus gland d. Bone marrow An antibody or immunoglobulin is a serum glycoprotein produced only by plasma cells in response to a challenge by an immunogen. PTS: 1 REF: Page 229 11. Which immunoglobulin is present in blood, saliva, breast milk, and respiratory secretions? a. IgA c. IgG b. IgE d. IgM IgA can be divided into two subclasses, IgA1 and IgA2. IgA1 molecules are predominantly found in the blood, whereas IgA2 is the predominant class of antibody found in normal body secretions. The other options are not found in the substances identified in the question. PTS: 1 REF: Page 229 12. Which antibody initially indicates a typical primary immune response? a. IgG c. IgA b. IgM d. IgE Typically, IgM is produced first (primary immune response), followed by IgG against the same antigen. The other options are not involved. PTS: 1 REF: Page 247 13. An individual is more susceptible to infections of mucous membranes when he or she has a seriously low level of which immunoglobulin antibody? a. IgG c. IgA b. IgM d. IgE The IgA molecules found in bodily secretions are dimers anchored together through a J-chain and secretory piece. This secretory piece is attached to the IgA antibodies inside the mucosal epithelial cells and may function to protect these immunoglobulin antibodies against degradation by enzymes also found in the secretions, thus decreasing the risk of infections in the mucous membrane. The other options do not accurately identify the immunoglobulin antibody involved in mucous membrane infections. PTS: 1 REF: Page 229 14. The B-cell receptor (BCR) complex functions uniquely by: a. Communicating information about the antigen to the helper T cell b. Secreting chemical signals to communicate between cells c. Recognizing the antigen on the surface of the B lymphocyte d. Communicating information about the antigen to the cell nucleus The role of the BCR is to recognize the antigen; however, unlike circulating antibodies, the receptor must communicate that information to the cell’s nucleus. The other options are not unique to the function of the BCR complex. PTS: 1 REF: Page 232 15. The generation of clonal diversity occurs primarily during which phase of life? a. Fetal c. Infancy b. Neonatal d. Puberty Generation of clonal diversity primarily occurs in the fetus and probably continues to a low degree throughout most of adult life. PTS: 1 REF: Pages 236-237 16. The generation of clonal diversity includes a process that: a. Involves antigens that select those lymphocytes with compatible receptors. b. Allows the differentiation of cells into antibody-secreting plasma cells or mature T cells. c. Takes place in the primary (central) lymphoid organs. d. Causes antigens to expand and diversify their populations. This process occurs in central lymphoid organs—the thymus gland for T cells and bone marrow for B cells. The other options do not accurately describe the processes included in clonal diversity. PTS: 1 REF: Pages 236-237 17. Which statement is true concerning clonal selection? a. Clonal selection is driven by hormones and does not require foreign antigens. b. This theory involves antigens that select those lymphocytes with compatible receptors. c. Clonal selection takes place in the primary (central) lymphoid organs. d. This process generates immature but immunocompetent T and B cells with receptors. Clonal selection, a process during which antigens select those lymphocytes with compatible receptors, expands their population and causes differentiation into antibody-secreting plasma cells or mature T cells (see Table 8-6). The other statements are not true regarding clonal selection. PTS: 1 REF: Page 236 18. Which is an example of an endogenous antigen? a. Yeast c. Bacteria b. Cancer cells d. Fungus Of the options provided, endogenous antigens include only those uniquely produced by cancerous cells. PTS: 1 REF: Page 244 19. Which cytokine is needed for the maturation of a functional helper T cell? a. IL-1 c. IL-4 b. IL-2 d. IL-12 Of the options provided, IL-2 production is critical for the Th cell to mature efficiently into a functional helper cell. PTS: 1 REF: Page 245 20. Th2 cells produce IL-4 and suppress which cells? a. B lymphocytes c. Th1 cells b. Cytotoxic T lymphocytes d. Memory T lymphocytes Th2 cells produce IL-4, which suppresses only Th1 and Th17 cells through their IL-4 receptors. PTS: 1 REF: Pages 246-247 21. Which statement is believed to be true concerning Th1 cells? a. Th1 cells are induced by antigens derived from allergens. b. They are induced by antigens derived from cancer cells. c. Th1 cells produce IL-4, IL-5, IL-6, and IL-13. d. They assist in the development of humoral immunity. Antigens derived from viral or bacterial pathogens and those derived from cancer cells are hypothesized to induce a greater number of Th1 cells relative to Th2 cells. The other statements are not true regarding Th1 cells. PTS: 1 REF: Pages 246-247 22. Which statement is believed to be true concerning Th2 cells? a. Th2 cells are induced by antigens derived from allergens. b. They are induced by antigens derived from cancer cells. c. Th2 cells produce IL-2, TNF-ß, and IFN- . d. They assist in the development of cell-mediated immunity. Antigens derived from multicellular parasites and allergens are hypothesized to be involved in the production of more Th2 cells. The other statements are not true regarding Th2 cells. PTS: 1 REF: Pages 246-247 23. When a person is exposed to most antigens, antibodies can be usually detected in his or her circulation within: a. 12 hours c. 3 days b. 24 hours d. 6 days After only approximately 5 to 7 days is an IgM antibody specific for that antigen detected in the circulation. PTS: 1 REF: Page 247 24. Vaccinations are able to provide protection against certain microorganisms because of the: a. Strong response from IgM c. Memory cells for IgE b. Level of protection provided by IgG d. Rapid response from IgA IgG production is considerably increased, making it the predominant antibody class of the secondary response. IgG is often present in concentrations several times larger than those of IgM, and levels of circulating IgG specific for that antigen may remain elevated for an extended period. The other options are not relevant to how vaccinations protect against certain microorganisms. PTS: 1 REF: Page 247 25. Why is the herpes virus inaccessible to antibodies after the initial infection? a. The virus does not circulate in the blood. b. It does not have antibody receptors. c. It resists agglutination. d. The virus is a soluble antigen. Many viruses (e.g., measles, herpes) are inaccessible to antibodies after the initial infection only because these viruses do not circulate in the bloodstream; rather, they remain inside infected cells, spreading by direct cell-to-cell contact. PTS: 1 REF: Page 252 26. Increased age may cause which change in lymphocyte function? a. Increased production of antibodies against self-antigens b. Decreased number of circulating T cells c. Decreased production of autoantibodies d. Increased production of helper T cells B-cell function is altered with age as shown by decreases in specific antibody production in response to antigenic challenge, with concomitant increases in circulating immune complexes and in circulating autoantibodies (antibodies against self-antigens). Aging does not play a role in either decreasing T cells circulation or increasing helper T cells production. PTS: 1 REF: Pages 257-258 27. How do antibodies protect the host from bacterial toxins? a. Lysing the cell membrane of the toxins b. Binding to the toxins to neutralize their biologic effects c. Inhibiting the synthesis of DNA proteins needed for growth d. Interfering with the DNA enzyme needed for replication To cause disease, most toxins must bind to surface molecules on the individual’s cells. Protective antibodies can bind to the toxins, prevent their interaction with cells, and neutralize their biologic effects. The other options fail to explain how antibodies protect the host from bacterial toxins. PTS: 1 REF: Page 252 28. Which T cell controls or limits the immune response to protect the host’s own tissues against an autoimmune response? a. Cytotoxic T cells c. Th2 cells b. Th1 cells d. Regulatory T (Treg) cells The regulatory T (Treg) cell is the only option whose role is to control or limit the immune response to protect the host’s own tissues against autoimmune reactions. PTS: 1 REF: Page 257 29. Evaluation of umbilical cord blood can confirm that which immunoglobulin level is near adult levels? a. IgA c. IgM b. IgG d. IgE At birth, the total IgG level in the umbilical cord is the only immunoglobulin that is near adult levels (see Figure 8-30). PTS: 1 REF: Page 257 30. Which statement is true concerning the IgM? a. IgM is the first antibody produced during the initial response to an antigen. b. IgM mediates many common allergic responses. c. IgM is the most abundant class of immunoglobulins. d. IgM is capable of crossing the human placenta. Typically, IgM is produced first (primary immune response), followed by IgG against the same antigen. The other options are not true statements regarding IgM. PTS: 1 REF: Page 247 31. Which cell has the ability to recognize antigens presented by the MHC class I molecules? a. T cytotoxic c. CD 8 b. CD 4 d. T helper CD8 cells recognize antigens presented by the major histocompatibility complex (MHC) class I molecules and become mediators of cell-mediated immunity and directly kill other cells (T-cytotoxic cells). CD4 cells tend to recognize antigen presented by MHC class II molecules and develop into helpers in the later clonal selection process (T-helper cells) PTS: 1 REF: Page 240 32. Which cell has a role in developing cell-mediated immunity? a. Th1 c. CD8 b. CD4 d. Th2 Only Th1 cells help develop cellular immunity. PTS: 1 REF: Pages 245-246 33. How does the aging process of the T-cell activity affect older adults? a. Poor heat regulation abilities b. Increased risk for bone fractures c. Tendency to develop various infections d. Likelihood of experiencing benign skin lesions T-cell activity is deficient in older adults, and a shift in the balance of T-cell subsets is observed. These changes may result in increased susceptibility to infection. The other issues are not related to T-cell activity. PTS: 1 REF: Pages 257-258 34. Which statement is true regarding maternal antibodies provided to the neonate? a. The antibodies enter into the fetal circulation by means of active transport. b. The antibodies are transferred to the fetus via the lymphatic system. c. The antibodies are directly related to the mother’s nutritional intake. d. The antibodies reach protective levels after approximately 6 months of age. To protect the child against infectious agents both in utero and during the first few postnatal months, a system of active transport facilitates the passage of maternal antibodies into the fetal circulation. The antibodies are transmitted via the placenta and are related to the mother’s immune system. The infant’s own IgG-related antibodies reach protective levels by 6 months of age. PTS: 1 REF: Page 257 35. Antibodies that are associated with mucosal immune system, such as immunoglobulins, function to prevent which type of infections? a. Infections that attack the respiratory system b. Infections that tend to be chronic in nature c. Infections likely to be resistant to antibiotics d. Infections that focus on epithelial surfaces of the body Antibodies of the systemic immune system function throughout the body, whereas antibodies of the secretory (mucosal) immune system—primarily immunoglobulins of the IgA class—are associated with bodily secretions and function to prevent pathogenic infection on epithelial surfaces. The other options are not necessarily true when considering the immunoglobulins. PTS: 1 REF: Pages 252-253 36. Cytokines are vital to a cell’s ability to do which function? a. Excrete c. Metabolize b. Reproduce d. Communicate During their interactions, cells must communicate with each other through soluble cytokines. The other options are not so rigidly related to cytokines. PTS: 1 REF: Pages 235-236 MULTIPLE RESPONSE 37. Which is an example of a bacterial toxin that has been inactivated but still retains its immunogenicity to protect the person? (Select all that apply.) a. Poliomyelitis b. Measles c. Tetanus d. Gonorrhea e. Diphtheria , E The symptoms of tetanus or diphtheria are mediated by specific toxins. To prevent harming the recipient of the immunization, bacterial toxins are chemically inactivated so that they have lost most of their harmful properties but still retain their immunogenicity. These agents are referred to as toxoids. Tetanus or diphtheria are the only examples of such inactivated toxins. PTS: 1 REF: Page 252 38. Which statements are true concerning the humoral immune response? (Select all that apply.) a. The humoral immune response is divided into major and minor phases. b. The response has IgG and IgM produced during each of its phrases. c. It has a greater presence of IgG than IgM in one of its phases. d. The humoral immune response is produced in reaction to the presence of an antigen. e. Phases differ in their response time as a result of the effect of memory cells. , C, D, E The humoral immune response is divided into two phases, primary and secondary. These phases differ in the relative amounts of IgG produced—the secondary response having a significantly higher proportion of IgG relative to IgM. The two phases also differ in the speed with which each occurs after the antigen challenge—the secondary phases is significantly more rapid than the primary phase because of the presence of memory cells in the secondary phase. PTS: 1 REF: Page 247 39. CD4 is a characteristic surface marker and a result of which of the following? (Select all that apply.) a. Activity in the primary lymphoid organs b. Process of cellular differentiation c. Alterations to T cells d. Changes to B cells e. Clonal selection , B, C, D Differentiation of B cells and T cells in the primary lymphoid organs results in the expression of several characteristic surface markers, such as CD4 on helper T cells, CD8 on cytotoxic T cells, and CD21 and CD40 on B cells. Clonal selection is the process during which antigens select those lymphocytes with complementary T-cell receptors (TCRs) or BCRs. PTS: 1 REF: Pages 236-241 | Page 258 40. What are the necessary components of an adaptive immune response? (Select all that apply.) a. Antigen b. Gamma IgG c. Lymphocyte surface receptors d. Crystalline fragment e. Antibody , C, E Antigens are the molecules that can react with components of the adaptive immune system, including antibodies and lymphocyte surface receptors. PTS: 1 REF: Page 228 MATCHING Match each immunoglobulin with its characteristic or function. Each immunoglobulin can be used only once. A. IgA B. IgE C. IgG 41. Crosses the placenta. 42. Is predominantly found in the blood and body secretions. 43. Mediates many common allergic responses 41. PTS: 1 REF: Page 229 MSC: As a result of selective transport across the placenta, maternal IgG is the major class of antibody found in the blood of the fetus and newborn. 42. PTS: 1 REF: Page 229 MSC: IgA can be divided into two subclasses, IgA1 and IgA2. IgA1 molecules are found predominantly in the blood, whereas IgA2 is the predominant class of antibody found in normal body secretions. 43. PTS: 1 REF: Page 230 MSC: IgE is the least concentrated of any of the immunoglobulin classes in the circulation. It appears to have very specialized functions as a mediator of many common allergic responses and in the defense against parasitic infections. Chapter 9: Alterations in Immunity and Inflammation MULTIPLE CHOICE 1. Hypersensitivity is best defined as a(an): a. Disturbance in the immunologic tolerance of self-antigens b. Immunologic reaction of one person to the tissue of another person c. Altered immunologic response to an antigen that results in disease d. Undetectable immune response in the presence of antigens Hypersensitivity is an altered immunologic response to an antigen that results in disease or damage to the host. The other options are not accurate definitions of hypersensitivity. PTS: 1 REF: Page 262 2. A hypersensitivity reaction that produces an allergic response is called: a. Hemolytic shock c. Necrotizing vasculitis b. Anaphylaxis d. Systemic erythematosus Examples of systemic anaphylaxis are allergic reactions to beestings, peanuts, and fish. The other options are not accurate examples of hypersensitivity. PTS: 1 REF: Page 263 3. The common hay fever allergy is expressed through a reaction that is mediated by which class of immunoglobulins? a. IgE c. IgM b. IgG d. T cells Type I reactions are mediated by antigen-specific IgE and the products of tissue mast cells (see Figure 9-1). The most common allergies (e.g., pollen allergies) are type I reactions. In addition, most type I reactions occur against environmental antigens and are therefore allergic. The other options do not accurately identify the mediation factor related to hay fever. PTS: 1 REF: Page 263 | Table 9-1 4. Which type of antibody is involved in type I hypersensitivity reaction? a. IgA c. IgG b. IgE d. IgM Type I reactions are only mediated by antigen-specific IgE and the products of tissue mast cells (see Figure 9-1). PTS: 1 REF: Page 263 5. Blood transfusion reactions are an example of: a. Autoimmunity c. Homoimmunity b. Alloimmunity d. Hypersensitivity Only alloimmunity (also termed isoimmunity) occurs when the immune system of one individual produces an immunologic reaction against tissues of another individual. PTS: 1 REF: Page 262 6. During an IgE-mediated hypersensitivity reaction, which leukocyte is activated? a. Neutrophils c. Eosinophils b. Monocytes d. T lymphocytes Of the options provided, only eosinophils are activated during IgE-mediated hypersensitivity reactions. PTS: 1 REF: Page 263 | Page 265 7. During an IgE-mediated hypersensitivity reaction, what causes bronchospasm? a. Bronchial edema caused by the chemotactic factor of anaphylaxis b. Bronchial edema caused by binding of the cytotropic antibody c. Smooth muscle contraction caused by histamine bound to H1 receptors d. Smooth muscle contraction caused by histamine bound to H2 receptors During an IgE-mediated hypersensitivity reaction, only smooth muscle contraction caused by histamine bound to H1 receptors results in bronchospasms. PTS: 1 REF: Page 263 | Page 265 8. During an IgE-mediated hypersensitivity reaction, the degranulation of mast cells is a result of which receptor action? a. Histamine bound to H2 b. Chemotactic factor binding to the receptor c. Epinephrine bound to mast cells d. Acetylcholine bound to mast cells Histamine bound to H2 results in the degranulation of mast cells during an IgE-medicated hypersensitivity reaction. The other options do not cause this reaction. PTS: 1 REF: Page 265 9. What characteristic do atopic individuals have that make them genetically predisposed to develop allergies? a. Greater quantities of histamine c. Greater quantities of IgE b. More histamine receptors d. A deficiency in epinephrine Atopic individuals tend to produce higher quantities of IgE and to have more crystalline fragment (Fc) receptors for IgE on their mast cells. The other options do not cause this reaction. PTS: 1 REF: Page 271 10. What is the mechanism that results in type II hypersensitivity reactions? a. Antibodies coat mast cells by binding to receptors that signal its degranulation, followed by a discharge of preformed mediators. b. Antibodies bind to soluble antigens that were released into body fluids, and the immune complexes are then deposited in the tissues. c. Cytotoxic T lymphocytes or lymphokine-producing helper T 1 cells directly attack and destroy cellular targets. d. Antibodies bind to the antigens on the cell surface. The mechanism that results in a type II hypersensitivity reaction begins with antibody binding to tissue-specific antigens or antigens that have attached to particular tissues. The cell can be destroyed by antibody IgG or IgM and activation of the complement cascade through the classical pathway. PTS: 1 REF: Page 266 11. When mismatched blood is administered causing an ABO incompatibility, the erythrocytes are destroyed by: a. Complement-mediated cell lysis c. Phagocytosis in the spleen b. Phagocytosis by macrophages d. Natural killer cells Erythrocytes are destroyed by complement-mediated lysis in individuals with autoimmune hemolytic anemia or as a result of an alloimmune reaction to ABO-mismatched transfused blood cells. The other options are not involved in an ABO incompatibility reaction. PTS: 1 REF: Page 266 12. When antibodies are formed against red blood cell antigens of the Rh system, the blood cells are destroyed by: a. Complement-mediated cell lysis b. Phagocytosis by macrophages c. Phagocytosis in the spleen d. Neutrophil granules and toxic oxygen products Antibodies against platelet-specific antigens or against red blood cell antigens of the Rh system coat those cells at low density, resulting in their preferential removal by phagocytosis in the spleen, rather than by complement-mediated lysis. The other options do not cause this reaction. PTS: 1 REF: Pages 266-267 13. When soluble antigens from infectious agents enter circulation, tissue damage is a result of: a. Complement-mediated cell lysis b. Phagocytosis by macrophages c. Phagocytosis in the spleen d. Neutrophil granules and toxic oxygen products Of the options available, only the components of neutrophil granules damage the tissue. PTS: 1 REF: Page 267 14. How are target cells destroyed in a type II hypersensitivity reaction? a. Complement-mediated cell lysis b. Phagocytosis by macrophages c. Neutrophil granules and toxic oxygen products d. Natural killer cells The mechanism that results in a type II hypersensitivity reaction involves a subpopulation of cytotoxic cells that are not antigen specific (natural killer [NK] cells). Antibody on the target cell is recognized by Fc receptors on the NK cells, which releases toxic substances that destroy the target cell. The other options do not cause the destruction of target cells related to a type II hypersensitivity reaction. PTS: 1 REF: Page 267 15. Graves disease (hyperthyroidism) is an example of which type II hypersensitivity reaction? a. Modulation b. Antibody-dependent cell-mediated cytotoxicity c. Neutrophil-mediated damage d. Complement-mediated lysis The antibody reacts with the receptors on the target cell surface and modulates the function of the receptor by preventing interactions with their normal ligands, replacing the ligand and inappropriately stimulating the receptor or destroying the receptor. For example, in the hyperthyroidism (excessive thyroid activity) of Graves disease, autoantibody binds to and activates receptors for thyroid-stimulating hormone (TSH) (a pituitary hormone that controls the production of the hormone thyroxine by the thyroid). The other options are not examples of type II hypersensitivity reactions. PTS: 1 REF: Page 267 | Page 269 16. Type III hypersensitivity reactions are a result of which of the following? a. Antibodies coating mast cells by binding to receptors that signal its degranulation, followed by the discharge of preformed mediators b. Antibodies binding to soluble antigens that were released into body fluids and the immune complexes being deposited in the tissues c. Tc cells or lymphokine-producing Th1 cells directly attacking and destroying cellular targets d. Antibodies binding to the antigen on the cell surface Antigen-antibody (immune) complexes that are formed in the circulation and then deposited later in vessel walls or extravascular tissues (see Figure 9-3) cause most type III hypersensitivity diseases. The other options do not cause this type of reaction. PTS: 1 REF: Page 269 17. A type IV hypersensitivity reaction causes which result? a. Antibodies coating mast cells by binding to receptors that signal its degranulation, followed by the discharge of preformed mediators b. Antibodies binding to soluble antigens that were released into body fluids and the immune complexes being deposited in the tissues c. Lymphokine-producing Th1 cells directly attacking and destroying cellular targets d. Antibodies binding to the antigen on the cell surface Types I, II, and III hypersensitivity reactions are mediated by antibody, type IV reactions are mediated by T lymphocytes and do not involve antibody. Type IV mechanisms occur through either Tc cells or lymphokine-producing Th1 cells. Tc cells directly attack and destroy cellular targets. PTS: 1 REF: Page 270 18. In a type III hypersensitivity reaction, the harmful effects after the immune complexes that are deposited in tissues are a result of: a. Cytotoxic T cells c. Complement activation b. Natural killer cells d. Degranulation of mast cells Complement activation, particularly through the generation of chemotactic factors for neutrophils, causes the harmful effects of immune complex deposition. The neutrophils bind to antibody and C3b contained in the complexes and attempt to ingest the immune complexes. Type III hypersensitivity reactions as described are not the result of any of the other options. PTS: 1 REF: Page 269 19. Raynaud phenomenon is classified as a type III hypersensitivity reaction and is due to: a. Immune complexes that are deposited in capillary beds, blocking circulation b. Mast cells that are bound to specific endothelial receptors, causing them to degranulate and creating a localized inflammatory reaction that occludes capillary circulation c. Cytotoxic T cells that attack and destroy the capillaries so that they are unable to perfuse local tissues d. Antibodies that detect the capillaries as foreign protein and destroy them using lysosomal enzymes and toxic oxygen species Raynaud phenomenon is a condition caused by the temperature-dependent deposition of immune complexes in the capillary beds of the peripheral circulation. None of the other options are involved in causing this condition. PTS: 1 REF: Page 270 20. Deficiencies in which element can produce depression of both B- and T-cell function? a. Iron c. Iodine b. Zinc d. Magnesium Of the options available, only deficient zinc intake can profoundly depress T- and B-cell function. PTS: 1 REF: Page 291 21. When the maternal immune system becomes sensitized against antigens expressed by the fetus, what reaction occurs? a. T-cell immunity c. Fetal immunity b. Alloimmunity d. Autoimmunity Alloimmunity occurs when an individual’s immune system reacts against antigens on the tissues of other members of the same species. Sensitization against fetal antigens is not the cause of any other available option. PTS: 1 REF: Page 276 22. Tissue damage caused by the deposition of circulating immune complexes containing an antibody against the host DNA is the cause of which disease? a. Hemolytic anemia c. Systemic lupus erythematosus b. Pernicious anemia d. Myasthenia gravis Only the deposition of circulating immune complexes containing an antibody against the host DNA produce tissue damage in individuals with systemic lupus erythematosus (SLE). PTS: 1 REF: Page 277 23. Why does tissue damage occurs in acute rejection after organ transplantation? a. Th1 cells release cytokines that activate infiltrating macrophages, and cytotoxic T cells directly attack the endothelial cells of the transplanted tissue. b. Circulating immune complexes are deposited in the endothelial cells of transplanted tissue, where the complement cascade lyses tissue. c. Receptors on natural killer cells recognize antigens on the cell surface of transplanted tissue, which releases lysosomal enzymes that destroy tissue. d. Antibodies coat the surface of transplanted tissue to which mast cells bind and liberate preformed chemical mediators that destroy tissue. The recipient’s lymphocytes interacting with the donor’s dendritic cells within the transplanted tissue usually initiate sensitization, resulting in the induction of recipient Th1 and Tc cells against the donor’s antigens. The Th1 cells release cytokines that activate infiltrating macrophages, and the Tc cells directly attack the endothelial cells in the transplanted tissue. The other options do not accurately describe how acute rejection after organ transplantation results in tissue damage. PTS: 1 REF: Page 280 24. Which blood cell carries the carbohydrate antigens for blood type? a. Platelets c. Lymphocytes b. Neutrophils d. Erythrocytes The reaction that causes a blood transfusion recipient’s red blood cells to clump together is related to the ABO antigens located on the surface of only erythrocytes. PTS: 1 REF: Page 278 25. A person with type O blood is likely to have high titers of which anti-antibodies? a. A c. A and B b. B d. O Type O individuals have neither A or B antigen but have both anti-A and anti-B antibodies and therefore cannot accept blood from any of the other three types. PTS: 1 REF: Page 278 26. Which class of immunoglobulins forms isohemagglutinins? a. IgA c. IgG b. IgE d. IgM Naturally occurring antibodies, called isohemagglutinins, are immunoglobulins of only the IgM class. PTS: 1 REF: Page 278 27. Which component of the immune system is deficient in individuals with infections caused by viruses, fungi, or yeast? a. Natural killer cells c. B cells b. Macrophages d. T cells Of the available options, deficiencies in T-cell immune responses are suggested when certain viruses (e.g., varicella, vaccinia, herpes, cytomegalovirus), fungi, and yeasts (e.g., Candida, Histoplasma) or certain atypical microorganisms (e.g., Pneumocystis jiroveci) cause recurrent infections. PTS: 1 REF: Page 281 28. In which primary immune deficiency is there a partial-to-complete absence of T-cell immunity? a. Bruton disease c. Reticular dysgenesis b. DiGeorge syndrome d. Adenosine deaminase deficiency The principal immunologic defect in DiGeorge syndrome is the partial or complete absence of T-cell immunity. The other options are not the result of either a partial or complete absence of T-cell immunity. PTS: 1 REF: Page 281 29. How many months does it take for the newborn to be sufficiently protected by antibodies produced by its own B cells? a. 1 to 2 c. 6 to 8 b. 4 to 5 d. 10 to 12 By 6 to 8 months, the newborn should be efficiently protected by antibodies produced by its own B cells. PTS: 1 REF: Page 290 30. Considering the effects of nutritional deficiencies on the immune system, severe deficits in calories and protein lead to deficiencies in the formation of which immune cells? a. B cells c. Natural killer cells b. T cells d. Neutrophils Severe deficits in calorie or protein intake lead to deficiencies in T-cell function and numbers. The other options are not necessarily affected. PTS: 1 REF: Pages 290-291 31. Urticaria are a manifestation of a which type of hypersensitivity reaction? a. IV c. II b. III d. I Urticaria, or hives, is a dermal (skin) manifestation of only type I allergic reactions. PTS: 1 REF: Page 272 32. Graves disease is a result of: a. Increased levels of circulating immunoglobulins b. The infiltration of the thyroid with T lymphocytes c. Autoantibodies binding to thyroid-stimulating hormone (TSH)-receptor sites d. Exposure to acetylates in substances such as rubber In the hyperthyroidism (excessive thyroid activity) of Graves disease, autoantibody binds to and activates receptors for TSH (a pituitary hormone that controls the production of the hormone thyroxine by the thyroid). The other options do not accurately describe the cause of Graves disease. PTS: 1 REF: Page 267 | Page 269 33. Raynaud phenomenon is an example of which type of hypersensitivity? a. IV c. II b. III d. I The characteristics of serum sickness are observed in only systemic type III autoimmune diseases such as Raynaud phenomenon. PTS: 1 REF: Page 270 34. Which statement is true concerning an atopic individual? a. They tend to produce less IgE. b. They tend to produce more Fc receptors. c. They tend to attract very few mast cells. d. They tend to produce very high levels of IgM. Atopic individuals tend to produce higher quantities of IgE and have more Fc receptors for IgE on their mast cells. The other available options are not true. PTS: 1 REF: Page 271 35. Which statement is true regarding immunodeficiency? a. Immunodeficiency is generally not present in other family members. b. Immunodeficiency is never acquired; rather, it is congenital. c. Immunodeficiency is almost immediately symptomatic. d. Immunodeficiency is a result of a postnatal mutation. Generally, the mutations are sporadic and not inherited; a family history exists in only approximately 25% of individuals. The sporadic mutations occur before birth, but the onset of symptoms may be early or later, depending on the particular syndrome. The immunodeficiency can be either congenital or acquired. PTS: 1 REF: Page 281 36. A person with type O blood is considered to be the universal blood donor because type O blood contains which of the following? a. No antigens c. Both A and B antigens b. No antibodies d. Both A and B antibodies Because individuals with type O blood lack both types of antigens, they are considered universal donors, meaning that anyone can accept their red blood cells. Type O individuals, who have neither A or B antigen but have both anti-A and anti-B antibodies, cannot accept blood from any of the other three types. PTS: 1 REF: Page 278 37. Immunoglobulin E (IgE) is associated with which type of hypersensitivity reaction? a. I c. III b. II d. IV Hypersensitivity reactions have been divided into four distinct types: type I (IgE-mediated) hypersensitivity reactions, type II (tissue-specific) hypersensitivity reactions, type III (immune complex–mediated) hypersensitivity reactions, and type IV (cell-mediated) hypersensitivity reactions. PTS: 1 REF: Page 263 38. Graves disease is an autoimmune disease that results in which maternal antibody? a. Binding with receptors for neural transmitters on muscle cells, causing neonatal muscular weakness b. Affecting the receptor for TSH, causing neonatal hyperthyroidism c. Inducing anomalies in the fetus or causing pregnancy loss d. Destroying platelets in the fetus and neonate Graves disease is an autoimmune disease in which maternal antibody against the receptor for TSH causes neonatal hyperthyroidism. Myasthenia gravis is an autoimmune disease in which maternal antibody binds with receptors for neural transmitters on muscle cells (acetylcholine receptors), causing neonatal muscular weakness. Systemic lupus erythematosus is an autoimmune disease in which diverse maternal autoantibodies induce anomalies (e.g., congenital heart defects) in the fetus or cause pregnancy loss. Immune thrombocytopenic purpura causes both autoimmune and alloimmune variants to occur, during which maternal antiplatelet antibody destroys platelets in the fetus and neonate. PTS: 1 REF: Page 277 MULTIPLE RESPONSE 39. When a tuberculin skin test is positive, the hard center and erythema surrounding the induration are a result of which of the following? (Select all that apply.) a. Histamine d. Products of complement b. T lymphocytes e. Macrophages c. Immune complexes , E The reaction site is infiltrated with only T lymphocytes and macrophages, resulting in a clear hard center (induration) and a reddish surrounding area (erythema). PTS: 1 REF: Page 270 40. Exposure to which of the following could result in a type IV hypersensitivity reaction? (Select all that apply.) a. Poison ivy d. Nickel b. Neomycin e. Detergents c. Dairy products , B, D, E Allergens that primarily elicit type IV allergic hypersensitivities include plant resins (e.g., poison ivy, poison oak); metals (e.g., nickel, chromium); acetylates and chemicals in rubber, cosmetics, detergents; and topical antibiotics (e.g., neomycin). PTS: 1 REF: Page 271 41. Which disorders are considered autoimmune? (Select all that apply.) a. Crohn disease d. Systemic lupus erythematosus b. Addison disease e. Noninsulin-dependent diabetes c. Rheumatoid arthritis , B, C, D Crohn disease, Addison disease, rheumatoid arthritis, and systemic lupus erythematosus are all diseases that result from autoimmune pathologic conditions. Insulin-dependent diabetes is also an autoimmune disorder, but noninsulin-dependent diabetes is not. PTS: 1 REF: Pages 264-265 | Table 9-2 42. Which statements best define acute rejection? (Select all that apply.) a. Acute rejection is a cell-mediated immune response. b. Acute rejection is usually a type III rejection. c. Immunosuppressive drugs delay or lessen the intensity of an acute rejection. d. Acute rejection is associated with the body’s response to an organ transplant. e. Acute rejection is a response against unmatched human leukocyte antigens (HLAs). , C, D, E Acute rejection is primarily a cell-mediated immune response that occurs within days to months after transplantation. This type of rejection occurs when the recipient develops an immune response against unmatched HLAs after transplantation. A biopsy of the rejected organ usually shows an infiltration of lymphocytes and macrophages characteristic of a type IV reaction. Immunosuppressive drugs may delay or lessen the intensity of an acute rejection. PTS: 1 REF: Page 280 Chapter 10: Infection MULTIPLE CHOICE 1. What is a significant cause of morbidity and mortality worldwide? a. Starvation c. Cardiovascular disease b. Traumatic injury d. Infectious disease Despite the wide-scale implementation of progressive public health and immunization policies, infectious disease remains a significant cause of morbidity and mortality. The other options are not significant causes. PTS: 1 REF: Page 299 2. What is the first stage in the infectious process? a. Invasion c. Spread b. Colonization d. Multiplication From the perspective of the microorganisms that cause disease, the infectious process undergoes four separate stages of progression: (1) colonization, (2) invasion, (3) multiplication, and (4) spread. PTS: 1 REF: Pages 300-301 3. Which type of microorganism reproduces on the skin? a. Viruses c. Protozoa and Rickettsiae b. Bacteria and fungi d. Mycoplasma Only bacteria and fungi have the capacity to reproduce on the skin. PTS: 1 REF: Page 303 | Table 10-3 4. Phagocytosis involves neutrophils actively attacking, engulfing, and destroying which microorganisms? a. Bacteria c. Viruses b. Fungi d. Yeasts Invasion is the direct confrontation with an individual’s primary defense mechanisms against only bacteria, which include the complement system, antibodies, and phagocytes, such as neutrophils and macrophages. PTS: 1 REF: Page 306 5. Once they have penetrated the first line of defense, which microorganisms do natural killer (NK) cells actively attack? a. Bacteria c. Viruses b. Fungi d. Mycoplasma NK cells are the principal defenders against only tumor cells or virally infected cells. PTS: 1 REF: Page 320 6. Which statement concerning exotoxins is true? a. Exotoxins are contained in cell walls of gram-negative bacteria. b. Exotoxins are released during the lysis of bacteria. c. Exotoxins are able to initiate the complement and coagulation cascades. d. Exotoxins are released during bacterial growth. Exotoxins are proteins released during bacterial growth. The other options are not true of exotoxins. PTS: 1 REF: Page 306 7. Which statement is true concerning a fungal infection? a. Fungal infections occur only on skin, hair, and nails. b. Phagocytes and T lymphocytes control fungal infections. c. Fungal infections release endotoxins. d. Vaccines prevent fungal infections. The host defense against fungal infection includes the fungistatic properties of neutrophils and macrophages. T lymphocytes are crucial in limiting the extent of infection and producing cytokines to further activate macrophages. The other options are not true of fungal infections. PTS: 1 REF: Page 312 8. Cytokines are thought to cause fevers by stimulating the synthesis of which chemical mediator? a. Leukotriene c. Prostaglandin b. Histamine d. Bradykinin Cytokines seem to raise the thermoregulatory set point through stimulation of prostaglandin synthesis and turnover in thermoregulatory (brain) and nonthermoregulatory (peripheral) tissues. The other options do not accurately identify the appropriate chemical mediator. PTS: 1 REF: Pages 301-302 9. Considering the hypothalamus, a fever is produced by: a. Endogenous pyrogens acting directly on the hypothalamus. b. Exogenous pyrogens acting directly on the hypothalamus. c. Immune complexes acting indirectly on the hypothalamus. d. Cytokines acting indirectly on the hypothalamus. Little evidence suggests that exogenous pyrogens directly cause fever. Such pyrogens indirectly affect the hypothalamus through endogenous pyrogens released by cells of the host. Neither immune complexes nor cytokines are involved in the process. PTS: 1 REF: Page 302 10. Which statement about vaccines is true? a. Most bacterial vaccines contain attenuated organisms. b. Most viral vaccines are made by using dead organisms. c. Vaccines require booster injections to maintain life-long protection. d. Vaccines provide effective protection against most infections. In general, vaccine-induced protection does not persist as long as infection-induced immunity, thus booster injections may be necessary to maintain protection throughout life. The other options are not true of vaccines. PTS: 1 REF: Page 332 11. Vaccines against viruses are created from: a. Killed organisms or extracts of antigens b. Live organisms weakened to produce antigens c. Purified toxins that have been chemically detoxified d. Recombinant pathogenic protein Most vaccines against viral infections (e.g., measles, mumps, rubella, varicella [chickenpox], rotavirus) contain live viruses that are weakened (attenuated) to continue expressing the appropriate antigens but are unable to establish more than a limited and easily controlled infection. The other options are not used in virus-focused vaccines. PTS: 1 REF: Page 332 12. Which statement is a characteristic of HIV? a. HIV only infects T-helper (Th) cells. b. HIV is a retrovirus. c. HIV carries genetic information in its DNA. d. HIV has five identified strains. HIV is a member of the retrovirus family, which carries genetic information in the form of two copies of RNA (see Figure 10-12). The other statements are not true of HIV. PTS: 1 REF: Page 324 13. What is the role of reverse transcriptase in HIV infection? a. Reverse transcriptase converts single-stranded DNA into double-stranded DNA. b. It is needed to produce integrase. c. It transports the RNA into the cell nucleus. d. It converts RNA into double-stranded DNA. One particular family of viruses, retroviruses (e.g., HIV) carries an enzyme, reverse transcriptase, which creates a double-stranded DNA version of the virus. PTS: 1 REF: Page 324 14. After sexual transmission of HIV, a person can be infected yet seronegative for how many months? a. 1 to 2 c. 18 to 20 b. 6 to 14 d. 24 to 36 Antibody appears rather rapidly after infection through blood products, usually within 4 to 7 weeks. After sexual transmission, however, the individual can be infected yet seronegative for 6 to 14 months or, in at least one case, for years. PTS: 1 REF: Page 326 15. Which cells are primary targets for HIV? a. CD4+ Th cells only b. CD4+ Th cells, macrophages, and natural killer cells c. CD8-positive cytotoxic T (Tc) cells and plasma cells d. CD8-positive Tc cells only The primary cellular targets for HIV include CD4+ Th cells, macrophages, and NK cells. The other options are not the primary target cells of HIV. PTS: 1 REF: Page 325 16. What area in the body may act as a reservoir in which HIV can be relatively protected from antiviral drugs? a. Central nervous system c. Thymus gland b. Bone marrow d. Lungs HIV may persist in regions where the antiviral drugs are not as effective, such as the central nervous system (CNS). The other options are not as protected from antiviral drugs. PTS: 1 REF: Page 327 17. AIDS produces a striking decrease in the number of which cells? a. Macrophages c. CD4+ Th cells b. CD8+ T cells d. Memory T cells The major immunologic finding in AIDS is the striking decrease in the number of CD4+ Th cells (see Figure 10-15). This finding is not true of the other options. PTS: 1 REF: Page 325 18. HIV antibodies appear within how many weeks after infection through blood products? a. 1 to 2 c. 10 to 12 b. 4 to 7 d. 20 to 24 Antibody appears rather rapidly after infection through blood products, usually within 4 to 7 weeks. PTS: 1 REF: Page 326 19. What is the final stage of the infectious process? a. Colonization c. Multiplication b. Invasion d. Spread From the perspective of the microorganisms that cause disease, the infectious process undergoes four separate stages of progression: (1) colonization, (2) invasion, (3) multiplication, and (4) spread. PTS: 1 REF: Page 300 20. Toxigenicity is defined as the: a. Ability of the pathogen to invade and multiply in the host b. Pathogen’s ability to produce disease by the production of a soluble toxin c. Ability of an agent to produce disease d. Potency of a pathogen measured in terms of the number of microorganisms required to kill the host Toxigenicity is the ability of a pathogen to produce soluble toxins or endotoxins, which are factors that greatly influence the pathogen’s degree of virulence. The other options do not accurately define toxigenicity. PTS: 1 REF: Page 302 21. The ability of the pathogen to invade and multiply in the host is referred to as: a. Infectivity c. Pathogenicity b. Toxigenicity d. Virulence Infectivity is the ability of the pathogen to invade and multiply in the host. The other options do not accurately denote the pathogen’s ability to invade and multiply in the host. PTS: 1 REF: Page 302 22. Some bacterial surface proteins bind with the crystalline fragment (Fc) portion of an antibody to: a. Hide in cells to avoid triggering an immune response b. Form self-protecting toxins c. Make staining possible for microscopic observation d. Produce a protective “self” protein Some bacterial surface proteins (protein A of Staphylococcus aureus, protein G of Streptococcus pyogenes) bind the Fc portion of the individual’s antibody, thus forming a protective coat of “self” protein. The other options do not accurately define the role of bacterial surface proteins as they bind with the Fc portion on an antibody. PTS: 1 REF: Page 308 23. Which organism is a common sexually transmitted bacterial infection? a. Staphylococcus aureus c. Helicobacter pylori b. Clostridium perfringens d. Treponema pallidum Treponema pallidum (spirochete, syphilis) is a sexually transmitted disease. Staphylococcus aureus is commonly ingested, causing food poisoning; Clostridium perfringens (gas gangrene) is a skin or wound infection; and Helicobacter pylori (gastritis, peptic ulcers) is found in the gastrointestinal tract. PTS: 1 REF: Pages 304-305 | Table 10-4 24. Which disease is an example of a rickettsial infection? a. Cholera c. Sleeping sickness b. Candida d. Rocky Mountain spotted fever Rocky Mountain spotted fever is a result of a rickettsiae. Cholera is a bacterial infection, candida is a fungal infection, and sleeping sickness is a protozoal infection. PTS: 1 REF: Page 302 MULTIPLE RESPONSE 25. Which secretion transmits HIV? (Select all that apply.) a. Semen b. Urine c. Saliva d. Breast milk e. Sweat , D HIV is a blood-borne pathogen present in body fluids (e.g., blood, vaginal fluid, semen, breast milk). PTS: 1 REF: Page 322 26. Which infection is fungal? (Select all that apply.) a. Ringworm b. Candida c. Cholera d. Athlete’s foot e. Aspergillus , B, D, E Infection with a fungus is called mycosis and includes dermatophytes (e.g., tineas, which refers to several skin mycoses including ringworm, athlete’s foot, and others) or yeasts (e.g., Candida, Aspergillus, Cryptococcus). Cholera is a bacterial infection. PTS: 1 REF: Page 311 27. Which statement is true regarding the development of HIV symptoms? (Select all that apply.) a. Symptoms generally appear in the clinical latency stage. b. Symptoms are generally observable within 5 years of the initial infection. c. T cells levels, particularly those of memory T cells, progressively decrease. d. Untreated infected individuals may remain asymptomatic for up to10 years. e. Secondary lymphoid organs experience damage and resulting malfunction. , D, E Individuals during the early stages of HIV (early stage disease or clinical latency) are usually asymptomatic. The early stage may last as long as 10 years in untreated people, during which the viral load increases and the numbers of CD4+ cells progressively decrease. As a result of these processes, the level of T cells decreases (particularly memory T cells, which seem more susceptible to HIV infection); thymic production of new T cells is decreased; and the secondary lymphoid organs (particularly the lymph nodes) are damaged. PTS: 1 REF: Pages 326-327 28. Which statements are true regarding endotoxins? (Select all that apply.) a. Endotoxins are lipopolysaccharides. b. Endotoxins are located in the walls of bacteria. c. Endotoxins are created during the process of lysis. d. Endotoxins are found in gram-negative microorganisms. e. Endotoxins are released during the destruction of its host. , B, D, E Endotoxins are lipopolysaccharides (LPSs) contained in the cell walls of gram-negative bacteria and released during lysis (or destruction) of the bacteria. PTS: 1 REF: Page 306 29. Which statements are true regarding viruses? (Select all that apply.) a. Viruses are very complex microorganisms. b. Viruses are referred to as eukaryotes. c. Viruses are capable of producing messenger RNA (mRNA). d. Viruses penetrate plasma membranes via endocytosis. e. Viruses are capable of uncoating cytoplasmic nucleocapsid. , D, E Viruses are extremely simple microorganisms and do not possess any of the metabolic organelles found in prokaryotes (e.g., bacteria) or eukaryotes (e.g., human cells). Once bound, the virus can penetrate the plasma membrane by receptor-mediated endocytosis. Within the cytoplasm, the virus uncoats the protective nucleocapsid and releases viral genetic information. Most RNA viruses directly produce mRNA, which is translated into viral proteins, and genomic RNA, which is eventually packaged into new viruses. PTS: 1 REF: Pages 317-319 30. Which of the following play a role in the control of fungal infections? (Select all that apply.) a. Cytokines b. Macrophages c. Natural killer cells d. Neutrophils e. T lymphocytes , B, D, E The host defense against fungal infection includes the fungistatic properties of neutrophils and macrophages. T lymphocytes are crucial in limiting the extent of infection and producing cytokines to further activate macrophages. Natural killer cells are a component of innate immune system. PTS: 1 REF: Page 312 31. Complications of AIDS include: (Select all that apply.) a. Kaposi sarcoma b. Helicobacter pylori c. Cytomegalovirus retinitis d. Herpes simplex infection e. Legionella pneumophila , C, D Kaposi sarcoma, cytomegalovirus retinitis, and herpes simplex infection are clinical complications characteristically observed in patients with AIDS. Neither Helicobacter pylori nor Legionella pneumophila are considered classic AIDS opportunistic diseases. PTS: 1 REF: Page 328 | Figure 10-16 MATCHING Match each term with its definition. A. Toxigenicity B. Infectivity C. Pathogenicity D. Virulence 32. Ability of the pathogen to invade and multiply in the host 33. Capacity of a pathogen to cause severe disease 34. An important factor in determining a pathogen’s ability to produce disease by the production of a soluble toxin 35. Ability of an agent to produce disease 32. PTS: 1 REF: Page 302 MSC: Infectivity is the ability of the pathogen to invade and multiply in the host. 33. PTS: 1 REF: Page 302 MSC: Virulence is the capacity of a pathogen to cause severe disease. 34. PTS: 1 REF: Page 302 MSC: Toxigenicity is the ability to produce soluble toxins or endotoxins, factors that greatly influence the pathogen's degree of virulence. 35. PTS: 1 REF: Page 302 MSC: Pathogenicity is the ability of an agent to produce disease. Chapter 11: Stress and Disease MULTIPLE CHOICE 1. Exhaustion occurs if stress continues when which stage of the general adaptation syndrome is not successful? a. Flight or fight c. Adaptation b. Alarm d. Arousal Exhaustion occurs if stress continues and adaptation is not successful, ultimately causing impairment of the immune response, heart failure, and kidney failure, leading to death. The other stages occur before the adaptation stage. PTS: 1 REF: Page 339 2. Which organ is stimulated during the alarm phase of the general adaptation syndrome (GAS)? a. Adrenal cortex c. Anterior pituitary b. Hypothalamus d. Limbic system The alarm phase of the GAS begins when a stressor triggers the actions of the hypothalamus and the sympathetic nervous system (SNS) (see Figure 11-1). The other organs are not stimulated by the alarm phase of GAS. PTS: 1 REF: Page 339 3. During an anticipatory response to stress, the reaction from the limbic system is stimulated by the: a. Retronucleus of the anterior pituitary b. Anterior nucleus of the hippocampus c. Paraventricular nucleus of the hypothalamus d. Prefrontal nucleus of the amygdala The paraventricular nucleus (PVN) of the hypothalamus must be stimulated to cause the limbic system to be stimulated. The other options are not involved in the stimulation of the limbic system. PTS: 1 REF: Page 341 4. Which hormone prompts increased anxiety, vigilance, and arousal during a stress response? a. Norepinephrine b. Epinephrine c. Cortisol d. Adrenocorticotropic hormone (ACTH) Only the release of norepinephrine promotes arousal, increased vigilance, increased anxiety, and other protective emotional responses. PTS: 1 REF: Page 343 5. Perceived stress elicits an emotional, anticipatory response that begins where? a. Prefrontal cortex c. Limbic system b. Anterior pituitary d. Hypothalamus Perceived stressors elicit an anticipatory response that begins in the limbic system of the brain, the only option responsible for emotions and cognition. PTS: 1 REF: Page 343 6. During a stress response, the helper T (Th) 1 response is suppress by which hormone? a. ACTH c. Prolactin b. Cortisol d. Growth hormone Stress can activate an excessive immune response and, through cortisol and catecholamines, suppress the Th1 response, causing a Th2 shift. This response is not active by any of the other options. PTS: 1 REF: Page 349 7. What is the effect that low-serum albumin has on the central stress response? a. Impaired circulation of epinephrine and norepinephrine b. Impaired wound healing c. Lessened circulation of cortisol d. Diminished oncotic pressure Low-serum albumin impairs circulation of both epinephrine and norepinephrine since both bind to plasma protein albumin. The other options do not accurately describe the effect of low-serum albumin. PTS: 1 REF: Page 345 8. Stress-age syndrome directly results in depressed function of which system? a. Respiratory c. Digestive b. Endocrine d. Immune Of the available options, immunodepression is the only characteristic change observed in stress-age syndrome. PTS: 1 REF: Page 358 9. Stress-induced sympathetic stimulation of the adrenal medulla causes the secretion of: a. Epinephrine and aldosterone c. Epinephrine and norepinephrine b. Norepinephrine and cortisol d. Acetylcholine and cortisol The sympathetic nervous system is aroused during the stress response and causes the medulla of the adrenal gland to release catecholamines (80% epinephrine and 20% norepinephrine) into the bloodstream. The stress-induced efforts on the adrenal medulla do not include any of the other options. PTS: 1 REF: Pages 344-345 10. Stress-induced norepinephrine results in: a. Pupil constriction c. Increased sweat gland secretions b. Peripheral vasoconstriction d. Decreased blood pressure During stress, norepinephrine raises blood pressure by constricting peripheral vessels; it dilates the pupils of the eye, causes piloerection, and increases sweat gland action in the armpits and palms. PTS: 1 REF: Page 345 11. Released stress-induced cortisol results in the stimulation of gluconeogenesis by affecting which structure? a. Adrenal cortex c. Liver b. Pancreas d. Anterior pituitary One of the primary effects of cortisol is the stimulation of gluconeogenesis through stimulation of the adrenal cortex. The other options do not produce a stimulation of gluconeogenesis when exposed to cortisol. PTS: 1 REF: Page 346 12. What is the effect of increased secretions of epinephrine, glucagon, and growth hormone? a. Hyperglycemia c. Bronchodilation b. Hypertension d. Pupil dilation Cortisol enhances the elevation of blood glucose promoted by other hormones, such as epinephrine, glucagon, and growth hormone. This effect is not true of the other options. PTS: 1 REF: Page 346 13. Which hormone increases the formation of glucose from amino acids and free fatty acids? a. Epinephrine c. Cortisol b. Norepinephrine d. Growth hormone One of the primary effects of cortisol is the stimulation of gluconeogenesis or the formation of glucose from noncarbohydrate sources, such as amino or free fatty acids in the liver. Neither reaction is a result of the effects of any of the other options. PTS: 1 REF: Page 346 14. What effect do androgens have on lymphocytes? a. Suppression of B-cell responses and enhancement of T-cell responses b. Suppression of T-cell responses and enhancement of B-cell responses c. Suppression of B- and T-cell responses d. Enhancement of B- and T-cell responses Androgens suppress T- and B-cell responses. The other options do not occur in response to androgens. PTS: 1 REF: Page 353 15. Which gland regulates the immune response and mediates the apparent effects of circadian rhythms on immunity? a. Anterior pituitary c. Basal ganglia b. Adrenal d. Pineal Of the options available, only the pineal gland regulates the immune response and mediates the apparent effects of circadian rhythm on immunity. PTS: 1 REF: Page 354 16. Which cytokines initiate the production of corticotropin-releasing hormone (CRH)? a. IL–1 and IL-6 c. IFN and IL-12 b. IL-2 and TNF- d. TNF-ß and IL-4 Although a number of stress factors initiate the production of CRH, of the options available, only high levels of IL-1 and IL-6 initiate such a response. PTS: 1 REF: Pages 348-349 17. The release of which cytokines is triggered by bacterial or viral infections, cancer, and tissue injury that, in turn, initiate a stress response? a. IL-1 and IL-2 b. IL-12, TNF-, and colony-stimulating factor c. IFN, TNF-ß, and IL-6 d. IL-4 and IL-24 Of the options offered, only the release of immune inflammatory mediators IL-6, TNF-ß, and IFN is triggered by bacterial or viral infections, cancer, and tissue injury that, in turn, initiates a stress response through the hypothalamic-pituitary-adrenal (HPA) pathway. PTS: 1 REF: Page 349 18. The action of which hormone helps explain increases in affective anxiety and eating disorders, mood cycles, and vulnerability to autoimmune and inflammatory diseases in women as a result of stimulation of the CRH gene promoter and central norepinephrine system? a. Progesterone c. Estrogen b. Cortisol d. Prolactin Of the options provided, only estrogen directly stimulates the CRH gene promoter and the central noradrenergic (norepinephrine) system, which may help explain adult women’s slight hypercortisolism, increases in affective anxiety and eating disorders, mood cycles, and vulnerability to autoimmune and inflammatory disease, all of which follow estradiol fluctuations. PTS: 1 REF: Page 350 19. What effect does estrogen have on lymphocytes? a. Depression of B-cell functions and enhancement of T-cell functions b. Depression of T-cell functions and enhancement of B-cell functions c. Depression of B- and T-cell functions d. Enhancement of B- and T-cell functions Estrogens generally are associated with only a depression of T-cell–dependent immune functions and an enhancement of B-cell functions. PTS: 1 REF: Page 353 20. Which statement is true concerning the differences between stress-induced hormonal alterations of men and women? a. After injury, women produce more proinflammatory cytokines than men, a profile that is associated with poor outcomes. b. Androgens appear to induce a greater degree of immune cell apoptosis after injury, creating greater immunosuppression in injured men than in injured women. c. Psychologic stress associated with some types of competition decreases both testosterone and cortisol, especially in athletes older than 45 years of age. d. After stressful stimuli, estrogen is increased in women, but testosterone is decreased in men. Androgens appear to induce a greater degree of immune cell apoptosis after injury, a mechanism that may elicit a greater immunosuppression in injured men versus injured women. The other options are not true statements concerning the differences between how the genders are affected by stress-induced hormones. PTS: 1 REF: Page 353 21. Diagnostic blood work on individuals who perceive themselves to be in a chronic stress state will likely demonstrate: a. Decreased Th lymphocytes c. Decreased Tc cells b. Increased erythrocytes d. Increased platelets Illustrating the influence of chronic stress appraisal on the physiologic processes, a meta-analysis of the relationships between stressors and immunity found that a higher perception of stress was associated with reduced T cytotoxic (Tc)-cell cytotoxicity, although not with levels of circulating Th or Tc lymphocytes. Research has substantiates the other options. PTS: 1 REF: Page 355 22. What are the signs that a patient is in the adaptive stage of the general adaptation syndrome? a. He or she begins to experience elevated heart and respiratory rates. b. He or she finds it difficult to concentrate on a solution for the stress. c. The patient perceives his or her only options are to run away or fight back. d. The patient has exceeded his or her ability to cope with the current situation. Fight-or-flight behaviors are characteristic of the more advanced adaptive stage, whereas the remaining options are noted in the initial alarm stage. PTS: 1 REF: Page 339 23. The most influential factor in whether a person will experience a stress reaction is his or her: a. General state of physical health c. Intellectual abilities b. Spiritual belief system d. Ability to cope A person does not have a stress reaction unless the stress exceeds his or her coping abilities. The other options do not have the same degree of influence, as does a person’s ability to cope. PTS: 1 REF: Page 339 24. A reduction is an individual’s number of natural killer (NK) cells appears to correlate with an increased risk for the development of: a. Depression b. Type 1 diabetes c. Obsessive compulsive disorder (OCD) d. Gastroesophageal reflux disorder (GERD) A meta-analysis of studies shows a relationship between depression and the reduction in lymphocyte proliferation and natural killer cell activity. Currently, no research supports the other options. PTS: 1 REF: Page 355 25. A nurse is providing care to a terminally ill adult who has been with his life partner for over 56 years. Research supports the nurse’s assessment of the life partner for signs of: a. Suicidal ideations c. Severe stress reaction b. Cardiac dysrhythmia d. Anorexia induced weight loss The results of a Harvard study showed evidence that a spouse’s illness or death can increase a partner’s mortality by causing severe stress and removing a primary source of emotional, psychologic, practical, and financial support. Although the other options may exist, research does not currently support them as having the stated correlation. PTS: 1 REF: Page 357 | What's New box MULTIPLE RESPONSE 26. The effect epinephrine has on the immune system during the stress response is to increase which cells? (Select all that apply.) a. NK cells b. Immunoglobulins c. Cytokines d. T cells e. Th cells , D The injection of epinephrine into healthy human beings is associated with a transient increase of the number of lymphocytes (e.g., T cells, natural killer (NK) cells) in the peripheral blood. This association is not true of the other options. PTS: 1 REF: Page 346 27. Which immune cells are suppressed by the corticotropin-releasing hormone (CRH)? (Select all that apply.) a. Monocyte-macrophage cells b. Cytokines c. Tc cells d. Th cells e. B cells , D Direct suppressive effects of CRH have been reported on two immune cell types possessing CRH receptors—the monocyte and macrophage and CD4 (T helper) lymphocytes. CRH does not suppress the remaining options. PTS: 1 REF: Pages 353-354 28. The increased production of proinflammatory cytokines is associated with which considerations? (Select all that apply.) a. Chronic respiratory dysfunction b. Elevated anxiety levels c. Immune disorders d. Age and gender e. Dementia , C Increased levels of proinflammatory cytokines has been shown to have a possible link between stress and immune function. The other options are not as directly linked to cytokine levels. PTS: 1 REF: Page 353 29. Which statements are true regarding lymphocytes? (Select all that apply.) a. Lymphocytes are involved in the production of the human growth hormone. b. Elevated catecholamine levels influence lymphocytes. c. Lymphocytes are synthesized in the anterior pituitary gland. d. Lymphocytes have receptors for the hormone prolactin. e. Lymphocytes produce endorphins in large amounts. , B, D GH is synthesized from the anterior pituitary gland and is produced by lymphocytes and mononuclear phagocytic cells. Several classes of lymphocytes have receptors for prolactin, suggesting a direct effect of prolactin on immune function. Although the effects of acute elevation of catecholamines on the alteration of lymphocyte function are real, they are short lived, lasting only approximately 2 hours. The other statements regarding lymphocytes are not true. PTS: 1 REF: Page 350 | Page 352 MATCHING Match the hormone with its effects during a stress response. Hormones may be used more than once. A. Epinephrine B. Norepinephrine C. Cortisol 30. Constricts peripheral vessels to increase blood pressure. 31. Increases cardiac output by increasing heart rate and myocardial contractility. 32. Increases gastric secretions. 30. PTS: 1 REF: Page 345 MSC: Norepinephrine regulates blood pressure by constricting smooth muscle in all blood vessels. 31. PTS: 1 REF: Page 345 MSC: Epinephrine enhances myocardial contractility (inotropic effect), increases the heart rate (chronotropic effect), and increases venous return to the heart, all of which increase cardiac output and blood pressure. 32. PTS: 1 REF: Page 347 MSC: Cortisol promotes gastric secretion in the gastrointestinal tract. Chapter 4: Genes and Genetic Diseases MULTIPLE CHOICE 1. Inserting bone marrow cells into an individual who produces abnormal erythrocytes is an example of what type of therapy? a. Somatic cell c. Genetic engineering b. Germ cell d. Recombinant DNA Gene therapy can be applied in two ways. The less controversial approach is somatic cell therapy, which consists of inserting normal genes into the cells of an individual who has a genetic disease. In this approach, a particular tissue, such as bone marrow cells that produce abnormal erythrocytes, is treated. The correct option is the only one that accurately identifies the therapy described in the question. PTS: 1 REF: Page 137 | Box 4-2 2. DNA replication requires the enzyme DNA polymerase to: a. Travel along the single DNA strand, adding the correct nucleotide to the new strand b. Move along the double strand of DNA to unwind the nucleotides of the double helix c. Hold the double strand apart while the correct nucleotides are added to the strand d. Transport the double strand of DNA from the nucleus to the cytoplasm for protein formation The DNA polymerase enzyme travels along the single DNA strand, adding the correct nucleotides to the free end of the new strand (see Figure 4-2, B). The correct option is the only one that accurately describes the process involved in DNA replication using DNA polymerase. PTS: 1 REF: Page 137 3. Transcription is best defined as a process by which: a. DNA polymerase binds to the promoter site on ribonucleic acid (RNA). b. RNA directs the synthesis of polypeptides for protein synthesis. c. RNA is synthesized from a DNA template. d. A base pair substitution results in a mutation of the amino acid sequence. Transcription is the process by which RNA is synthesized from a DNA template. The correct option is the only one that accurately defines the term transcription. PTS: 1 REF: Page 141 4. The purpose of a staining technique of chromosomes such as Giemsa is to: a. Permit the mitotic process to be followed and monitored for variations. b. Allow for the numbering of chromosomes and the identification of variations. c. Identify new somatic cells formed through mitosis and cytokinesis. d. Distinguish the sex chromosomes from the homologous chromosomes. One of the most commonly used stains is Giemsa stain. By using banding techniques, chromosomes can be unambiguously numbered, and individual variation in chromosome composition can be studied. Missing or duplicated portions of chromosomes, which often result in serious diseases, also can be readily identified. The correct option is the only one that accurately describes the purpose of the Giemsa staining technique. PTS: 1 REF: Pages 142-143 5. An amniocentesis indicates a neural tube defect when an increase in which protein is evident? a. Chorionic c. Amniotic b. Alpha fetoprotein d. Embryonic Other disorders can be detected with this procedure. These include most neural tube defects, which cause an elevation of alpha fetoprotein in the amniotic fluid, and hundreds of diseases caused by mutations of single genes. The correct option is the only one that accurately identifies the protein responsible for a neural tube defect. PTS: 1 REF: Page 136 | Box 4-1 6. An amniocentesis is recommended for pregnant women who: a. Have a history of chronic illness b. Have a family history of genetic disorders c. Have experienced in vitro fertilization d. Had a late menarche Amniocentesis is recommended only for pregnancies known to have an elevated risk for a genetic disease or in women older than 30 to 35 years of age. The correct option is the only one that accurately describes a criterion for ordering an amniocentesis. PTS: 1 REF: Page 136 | Box 4-1 7. The most clinically useful technique for prenatal diagnosis of chromosomal abnormalities at 3 months’ (12 weeks’) gestation is: a. Gene mapping c. Amniocentesis b. Linkage analysis d. Chorionic villus sampling Chorionic villus sampling consists of extracting a small amount of villous tissue directly from the chorion. This procedure can be performed at 10 weeks’ gestation and does not require in vitro culturing of cells; sufficient numbers are directly available in the extracted tissue. Thus the procedure allows prenatal diagnosis at approximately 3 months’ gestation rather than at nearly 5 months’ gestation. The correct option is the only one that accurately describes the most useful technique for prenatal diagnosis of chromosomal abnormalities. PTS: 1 REF: Page 136 | Box 4-1 8. The term for an error in which homologous chromosomes fail to separate during meiosis or mitosis is: a. Aneuploidy c. Polyploidy b. Nondisjunction d. Translocation Aneuploidy is usually the result of nondisjunction, an error in which homologous chromosomes or sister chromatids fail to separate normally during meiosis or mitosis. The correct option is the only one that is used to describe an error in chromosomal separation during reproduction. PTS: 1 REF: Page 145 9. Which clinical manifestations would be expected for a child who has complete trisomy of the twenty-first chromosome? a. Widely spaced nipples, reduced carrying angle at the elbow, and sparse body hair b. An IQ of 25 to 70, low nasal bridge, protruding tongue, and flat, low-set ears c. High-pitched voice, tall stature, gynecomastia, and an IQ of 60 to 90 d. Circumoral cyanosis, edema of the feet, short stature, and mental slowness Individuals with this disease are mentally retarded, with IQs usually ranging from 25 to 70. The facial appearance is distinctive and exhibits a low nasal bridge, epicanthal folds (which produce a superficially Asian appearance), protruding tongue, and flat, low-set ears. The correct option is the only one that accurately describes the clinical manifestations of the complete trisomy of the twenty-first chromosome. PTS: 1 REF: Pages 146-147 10. What is the most common cause of Down syndrome? a. Paternal nondisjunction c. Maternal nondisjunction b. Maternal translocations d. Paternal translocation Nondisjunction during the formation of one of the parent’s gametes or during early embryonic development occurs in approximately 97% of infants born with Down syndrome. In approximately 90% to 95% of infants, the nondisjunction occurs in the formation of the mother’s egg cell. The correct option is the only one that accurately describes the most common cause of Down syndrome. PTS: 1 REF: Page 147 11. What syndrome, characterized by an absent homologous X chromosome with only a single X chromosome, exhibits features that include a short stature, widely spaced nipples, and webbed neck? a. Down c. Turner b. Cri du chat d. Klinefelter In Turner syndrome, a sex chromosome is missing, and the person’s total chromosome count is 45. Characteristic signs include short stature, female genitalia, webbed neck, shieldlike chest with underdeveloped breasts and widely spaced nipples, and imperfectly developed ovaries. The correct option is the only one that accurately describes the clinical manifestations described in the question. PTS: 1 REF: Page 147 12. A person with 47, XXY karyotype has the genetic disorder resulting in which syndrome? a. Turner c. Down b. Klinefelter d. Fragile X A disorder in the chromosome (47, XXY karyotype) results in a disorder known as Klinefelter syndrome. The correct option is the only one that accurately describes a genetic disorder that exhibits the described genetic configuration. PTS: 1 REF: Page 148 13. What is the chromosomal variation that causes Klinefelter syndrome? a. Nondisjunction of the X chromosome in the father b. Translocation of the X chromosome in the mother c. Nondisjunction of X chromosome in the mother d. Translocation of the Y chromosome in the father Nondisjunction of the X chromosomes in the mother causes Klinefelter syndrome in the majority of infants, and the frequency of the disorder rises with maternal age. The correct option is the only one that accurately describes the chromosomal variation characteristic of Klinefelter syndrome. PTS: 1 REF: Page 148 14. What is the second most commonly recognized genetic cause of mental retardation? a. Down syndrome c. Klinefelter syndrome b. Fragile X syndrome d. Turner syndrome The fragile X syndrome is the second most common genetic cause of mental retardation (after Down syndrome). The correct option is not observed with enough frequency to be recognized as the second most common cause of mental retardation. PTS: 1 REF: Page 151 15. What is the blood type of a person who is heterozygous, having A and B alleles as codominant? a. A c. O b. B d. AB When the heterozygote is distinguishable from both homozygotes, the locus is said to exhibit codominance. An example is the ABO blood group, in which heterozygotes having the A and B alleles express both of them as A and B antigens on their red cells (forming blood group AB). The correct option is the only one that accurately describes codominance. PTS: 1 REF: Page 151 16. A couple has two children diagnosed with an autosomal dominant genetic disease. What is the probability that the next child will have the same genetic disease? a. One sixth c. One third b. One fourth d. One half Affected heterozygous individuals transmit the trait to approximately one half of their children; however, because gamete transmission is subject to chance fluctuations, it is possible that all or none of the children of an affected parent may have the trait. Nevertheless, when large numbers of matings of this type are studied, the proportion of affected children closely approach one half. PTS: 1 REF: Pages 152-153 17. When a child inherits a disease that is autosomal recessive, it is inherited from: a. Father c. Both parents b. Mother d. Grandparent In most cases of recessive disease, both parents of affected individuals are heterozygous carriers. PTS: 1 REF: Pages 155-156 18. People diagnosed with neurofibromatosis have varying degrees of the condition because of the genetic principle of: a. Penetrance c. Dominance b. Expressivity d. Recessiveness Expressivity is the extent of variation in phenotype associated with a particular genotype. If expressivity of a disease is variable, then the penetrance may be complete but the severity of the disease can vary greatly. A well-known example of variable expressivity in an autosomal dominant disease is type 1 neurofibromatosis. The correct option is the only one that accurately describes the presence of varying degrees of symptomatologic characteristics. PTS: 1 REF: Page 154 19. Which genetic disease has been linked to a mutation of the tumor-suppressor gene? a. Hemochromatosis c. Familial breast cancer b. Retinoblastoma d. Hemophilia A The gene responsible for retinoblastoma has been mapped to the long arm of chromosome 13, and its DNA sequence has been extensively studied. This gene is known as a tumor-suppressor gene; the normal function of its protein product is to regulate the cell cycle so that cells do not grow uncontrollably. The correct option is the only one that accurately identifies a disease resulting from a mutation of the tumor-suppressor gene. PTS: 1 REF: Page 154 20. Cystic fibrosis is caused by what type of gene? a. X-linked dominant c. Autosomal dominant b. X-linked recessive d. Autosomal recessive The most common lethal autosomal recessive disease in white children, cystic fibrosis, occurs in approximately 1 in 2500 births. The correct option is the only one that accurately identifies the gene type responsible for cystic fibrosis. PTS: 1 REF: Page 154 21. Which is an important criterion for discerning autosomal recessive inheritance? a. Consanguinity is sometimes present. b. Females are affected more than males. c. The disease is observed in both the parents, as well as in the siblings. d. On average, one half of the offspring of the carrier will be affected. Consanguinity is often an important characteristic of pedigrees for recessive diseases; relatives share a certain proportion of genes received from a common ancestor. The correct option is the only one that accurately identifies a required factor in autosomal recessive genetic inheritance. PTS: 1 REF: Page 155 22. Consanguinity refers to the mating of persons: a. Who are unrelated b. When one has an autosomal dominant disorder c. Having common family relations d. When one has a chromosomal abnormality Consanguinity refers to the mating of two related individuals, and the offspring of such matings are said to be inbred. The correct option is the only one that accurately identifies consanguinity as it relates to human mating. PTS: 1 REF: Page 155 23. Males, having only one X chromosome (as is expected), are said to be: a. Homozygous c. Hemizygous b. Heterozygous d. Ambizygous Males, having only one X chromosome, are said to be hemizygous for genes on this chromosome. The correct option is the only one that accurately identifies the term for males having only one X chromosome. PTS: 1 REF: Page 156 24. Males are more often affected by which type of genetic disease? a. Sex-linked dominant c. Sex-linked b. Sex-influenced d. Sex-linked recessive Males are more frequently affected by X-linked recessive diseases, with the difference becoming more pronounced as the disease becomes rarer. The correct option is the only one that is a characteristic of a male-dominate disease. PTS: 1 REF: Page 156 25. An X-linked recessive disease can skip generations because: a. Females are hemizygous for the X chromosome. b. The disease can be transmitted through female carriers. c. Mothers cannot pass X-linked genes to their sons. d. These diseases need only one copy of the gene in females. Skipped generations are often observed in X-linked recessive disease pedigrees because the gene can be transmitted through female carriers. Males are hemizygous for genes on the X chromosome. Fathers cannot pass X-linked genes to their sons. X-linked recessive diseases are observed significantly more often in males than in females, because males need only one copy of the gene to express the disease. PTS: 1 REF: Page 157 26. The presence of a zygote having one chromosome with the normal complement of genes and one with a missing gene is characteristic of which genetic disorder? a. Cri du chat c. Klinefelter syndrome b. Down syndrome d. Turner syndrome This description is only accurate for Cri du chat syndrome. PTS: 1 REF: Pages 148-149 27. A child with which genetic disorder has a characteristic cry? a. Down syndrome c. Turner syndrome b. Klinefelter syndrome d. Cri du chat Cri du chat, which literally means “cry of the cat,” describes the characteristic cry of the affected child. The correct option is the only one with the characteristic cry. PTS: 1 REF: Page 148 28. Which statement is true regarding X-linked recessive conditions? a. Such diseases use males as phenotypical carriers. b. These conditions are passed from affected father to all of his female children. c. 25% of an affected individual’s grandsons will be affected. d. Cystic fibrosis is an example of such a condition. X-linked recessive conditions are passed from an affected father to all of his daughters, who, as phenotypically normal carriers, transmit it to approximately one half of their sons, who are affected. Cystic fibrosis is an autosomal dominant disease. PTS: 1 REF: Page 158 29. DNA formation occurs in which of the cell’s structures? a. Nucleus c. Organelle b. Cytoplasm d. Membrane DNA is formed and replicated only in the cell nucleus. PTS: 1 REF: Pages 135-141 30. What is the risk for the recurrence of autosomal dominant diseases? a. 10% c. 50% b. 30% d. 70% The recurrence risk for autosomal dominant diseases is usually 50%. PTS: 1 REF: Pages 152-153 31. An individual’s genetic makeup is referred to as his or her: a. Phenotype c. Heterozygous locus b. Genotype d. Homozygous locus An individual’s genotype is his or her genetic makeup. The correct option is the only one that accurately defines a person’s genetic makeup. PTS: 1 REF: Page 151 MULTIPLE RESPONSE 32. Which disorders have similar modes of inheritance? (Select all that apply.) a. Cri du chat syndrome b. Duchenne muscular dystrophy c. Polycystic kidney disease d. Down syndrome e. Becker muscular dystrophy , E Becker muscular dystrophy and Duchenne muscular dystrophy are the only options that are X-linked recessive disorders. PTS: 1 REF: Page 158 33. The key to accurate DNA replication depends on which complementary base pairs? (Select all that apply.) a. Adenine with thymine b. Adenine with guanine c. Guanine with cytosine d. Cytosine with thymine e. Guanine with thymine , C The consistent pairing of adenine with thymine and of guanine with cytosine, known as complementary base pairing, is the key to accurate DNA replication. The correct options are the only ones that accurately identify complementary base pairs. PTS: 1 REF: Page 137 34. Chromosomal abnormalities are the leading known cause of: (Select all that apply.) a. Mental illness b. Mental retardation c. Fetal miscarriage d. Cardiovascular disease e. Respiratory disorders , C Chromosome abnormalities are the leading known cause of mental retardation and miscarriage. The correct options are the only ones that accurately identify disorders that related to chromosomal abnormalities. PTS: 1 REF: Page 143 35. Examples of prenatal diagnostic studies include: (Select all that apply.) a. Chorionic villus sampling (CVS) b. Amniocentesis c. Carrier screening d. Preimplantation genetic diagnosis (PGD) e. Drug-sensitivity testing , B, D Prenatal testing is conducted before or during the pregnancy but not once labor begins. Such diagnostic studies include amniocenteses, CVS, and PGD. The correct options are the only ones that are exclusively used during the prenatal period. PTS: 1 REF: Page 136 | Box 4-1 36. The advantage derived from human genome sequencing on genetic disorders focuses on: (Select all that apply.) a. Identification of the mutated gene b. Reversal of the mutation c. Diagnosis of the existing disorder d. Appropriate treatment e. Prevention of the disorder , C, D The complete human genome sequence will facilitate gene identification, diagnosis, and disease treatment. The reversal of a mutation is not possible. The mapping has no effect on preventing a genetic disorder. PTS: 1 REF: Page 160 MATCHING Match the genetic terms with the corresponding diseases. Terms may be used more than once. A. Autosomal dominant B. Autosomal recessive C. X-linked dominant D. X-linked recessive 37. Cystic fibrosis 38. Duchenne muscular dystrophy 39. Sickle cell disease 40. Huntington disease 37. PTS: 1 REF: Page 154 MSC: Is an important example of an autosomal recessive disease is cystic fibrosis. 38. PTS: 1 REF: Page 158 MSC: Is an important example of an X-linked recessive disease is Duchenne muscular dystrophy. 39. PTS: 1 REF: Page 136 | Box 4-1 MSC: Is an important example of an autosomal recessive disease is sickle cell disease. 40. PTS: 1 REF: Page 154 MSC: Another well-known autosomal dominant disease is Huntington disease. Chapter 5: Genes, Environment-Lifestyle, and Common Diseases MULTIPLE CHOICE 1. The data reporting that sickle cell disease affects approximately 1 in 600 American blacks is an example of which concept? a. Incidence c. Ratio b. Prevalence d. Risk Prevalence rate is the proportion of the population affected by a disease at a specific point in time. Thus both the incidence rate and the length of the survival period in affected individuals determine prevalence. The incidence rate is the number of new cases of a disease reported during a specific period (typically 1 year), divided by the number of individuals in the population. A numerical expression representing a part of a larger whole or proportion is considered a ratio. Any factor that increases the chance of disease or injury is considered a risk. PTS: 1 REF: Page 165 2. The ratio of the disease among the exposed population to the disease rate in an unexposed population is referred to as what type of risk? a. Attributable c. Causal b. Contingency d. Relative A common measure of the effect of a specific risk factor is the relative risk. Assuming a factor is the cause of a disease, attributable risk is the amount of risk that is due to that factor. A future event or circumstance that is possible but cannot be predicted with certainty is a contingency risk. The probability of the outcome is termed a causal risk factor. PTS: 1 REF: Page 165 3. Empirical risks for most multifactorial diseases are based on: a. Chromosomal testing c. Liability thresholds b. Direct observation d. Relative risks For most multifactorial diseases, empirical risks (i.e., risks based on direct observation of data) have been derived. The other options are not the basis for determining the empirical risk of most multifactorial diseases. PTS: 1 REF: Page 167 4. What is the cause of familial hypercholesterolemia (FH)? a. Diet high in saturated fats b. Increased production of cholesterol by the liver c. Reduction in the number of low-density lipoprotein (LDL) receptors on cell surfaces d. Abnormal function of lipoprotein receptors circulating in the blood A reduction in the number of functional LDL receptors on cell surfaces causes FH. Lacking the normal number of LDL receptors, cellular cholesterol uptake is reduced and circulating cholesterol levels increase (see Box 5-3). The other options are not the basis for developing familial FH. PTS: 1 REF: Page 173 | Box 5-3 5. Which risk factor for hypertension is influenced by genetic factors and lifestyle? a. Sodium intake c. Psychosocial stress b. Physical inactivity d. Obesity The most important environmental risk factors for hypertension are increased sodium intake, decreased physical activity, psychosocial stress, and obesity. However, obesity is, itself, influenced by genes and the environment. PTS: 1 REF: Page 172 6. What percentage of all cases of breast cancer are identified as an autosomal dominant form? a. 5 c. 15 b. 10 d. 20 An autosomal dominant form of breast cancer accounts for approximately 5% of breast cancer cases in the United States. PTS: 1 REF: Page 172 | Page 174 7. When a woman has one first-degree relative with breast cancer, her risk of developing breast cancer is how many times greater? a. 2 c. 6 b. 3 d. 10 If a woman has one affected first-degree relative, her risk of developing breast cancer doubles. PTS: 1 REF: Page 172 8. Adoption studies have shown that the offspring of an alcoholic parent when raised by nonalcoholic parents have what amount of an increased risk of developing alcoholism? a. Twofold c. Fourfold b. Threefold d. Tenfold Adoption studies have shown that the offspring of an alcoholic parent, even when raised by nonalcoholic parents, have a fourfold increased risk of developing the disorder. PTS: 1 REF: Page 179 9. Studies have identified several genes that play a role in the prevention of obesity by affecting what? a. Regulation of appetite c. Absorption of fat b. Metabolizing of fat d. Altering the sense of satiety Clinical trials using recombinant leptin have demonstrated moderate weight loss in a subset of obese individuals. In addition, leptin participates in important interactions with other components of appetite control, such as neuropeptide Y and –melanocyte-stimulating hormone and its receptor, the melanocortin-4 receptor (MC4R). Currently, no research supports the other options as being genetically regulated. PTS: 1 REF: Page 178 10. The BRCA1 and BRCA2 mutations increase the risk of which cancer in women? a. Ovarian c. Uterine b. Lung d. Pancreatic BRCA1 mutations increase the risk of ovarian cancer among women (20% to 50% lifetime risk), and BRCA2 mutations also confer an increased risk of ovarian cancer (10% to 20% lifetime prevalence). BRCA1 and BRCA2 mutations are not currently believed to be linked with risks of lung, uterine, or pancreatic cancers. PTS: 1 REF: Page 174 11. Blood pressure variations are associated with: a. ß1-adrenergic receptors to increase heart rate b. The release of an antidiuretic hormone (ADH) that increases water reabsorption c. The renin-angiotensin system’s effect on vasoconstriction d. Serum bradykinin, causing vasodilation Significant research is now focused on specific components that may influence blood pressure variation, such as the renin-angiotensin system (involved in sodium reabsorption and vasoconstriction). The other options are not related to hypertension. PTS: 1 REF: Page 172 12. The two most important risk factors for type 2 diabetes are: a. Autoantibodies and human leukocyte antigen associations b. Autoantibodies and obesity c. Obesity and positive family history d. HLA associations and positive family history The two most important risk factors for type 2 diabetes are positive family history and obesity. The other options are not believed to be important risk factors for this form of diabetes. PTS: 1 REF: Pages 177-178 13. A major characteristic of type 1 diabetes mellitus is that there is: a. Partial insulin secretion c. Insulin resistance b. An autoimmune cause factor d. Obesity as a common risk factor A strong association between type 1 diabetes and the presence of several human leukocyte antigen (HLA) class II alleles indicate that type 1 diabetes mellitus is an autoimmune disease. The remaining options are associated with type 2 diabetes. PTS: 1 REF: Page 174 14. Obesity acts as an important risk factor for type 2 diabetes mellitus by: a. Reducing the amount of insulin the pancreas produces b. Increasing the resistance to insulin by cells c. Obstructing the outflow of insulin from the pancreas d. Stimulating the liver to increase glucose production People with type 2 diabetes mellitus suffer from insulin resistance (i.e., their cells have difficulty using insulin). The other options are not associated with the effect of obesity regarding insulin production. PTS: 1 REF: Pages 177-178 15. Traits caused by the combined effects of multiple genes are referred to by which term? a. Polygenic c. Modifiable b. Multifocal d. Involuntary Traits in which variation is thought to be caused by the combined effects of multiple genes are polygenic, meaning many genes. Multifocal means relating to or arising from many points. Modifiable refers to the changeability of something. Involuntary suggests being out of the control of someone or something. PTS: 1 REF: Page 165 16. Regarding type 2 diabetes, obesity is considered to be what type of risk? a. Genetic c. Relative b. Empirical d. Modifiable Obesity is a modifiable risk factor for many diseases including heart disease, stroke, hypertension, and type 2 diabetes. The other terms do not apply. PTS: 1 REF: Pages 177-178 17. Which disease form is identified on the basis of empirical risk observation? a. Polygenic c. Monozygotic b. Multifactorial d. Genetic For most multifactorial diseases, empirical risks (i.e., risks based on direct observation of data) have been derived. Traits in which variation is thought to be caused by the combined effects of multiple genes are polygenic. Monozygotic is a term that refers to identical twins. Genetic refers to issues related to genes and their influence on the body. PTS: 1 REF: Page 167 18. The number of persons living with a specific disease at a specific point in time is referred to by which term? a. Relativity c. Prevalence b. Survivability d. Incidence The prevalence rate is the proportion of the population affected by a disease at a specific point in time. Thus both the incidence rate and the length of the survival period in affected individuals determine prevalence. The description in the question does not relate to any of the other options. PTS: 1 REF: Page 164 19. Which type of cancer is said to aggregate among families? a. Breast c. Skin b. Lung d. Brain Breast cancer appears to aggregate strongly in families. The other cancers are not believed to be familial in nature. PTS: 1 REF: Page 172 20. Which dietary lifestyle choice has been associated with a decreased risk for developing colon cancer? a. Increased consumption of dairy produces b. Increased consumption of foods containing vitamin C c. Decreased consumption of foods high in fat d. Decreased consumption of artificial food coloring A low-fat, high-fiber diet is thought to decrease the risk of colon cancer. PTS: 1 REF: Page 165 21. It is currently believed that the risk for developing Alzheimer disease: a. Is not directly related to genetic predisposition. b. Is higher among men than it is among women. c. Occurs less among Hispanics than in Asians. d. Doubles among those with an affected first-degree relative. The risk of developing Alzheimer disease doubles in individuals who have an affected first-degree relative. The other statements are not true. PTS: 1 REF: Page 178 22. The number of new cases of a disease reported during a specific period divided by the number of individuals in the population is defined as which characteristic of a disease? a. Prevalence rate c. Relative risk b. Incidence rate d. Frequency The incidence rate is the number of new cases of a disease reported during a specific period (typically 1 year) divided by the number of individuals in the population. The description provided in the question does not describe any of the other options. PTS: 1 REF: Page 164 MULTIPLE RESPONSE 23. Cancers that cluster strongly in families include: (Select all that apply.) a. Breast b. Colon c. Ovarian d. Lung e. Brain , B, C Although breast, ovarian, and colon cancers have shown a strong familial tendency, lung and brain cancers have not. PTS: 1 REF: Page 172 24. Which genes are responsible for an autosomal dominant form of breast cancer? (Select all that apply.) a. LCAT b. CHK1 c. CHK2 d. BRCA1 e. BRCA2 , E Women who inherit a mutation in BRCA1 or BRCA2 experience a 50% to 80% lifetime risk of developing breast cancer. The other options do not carry this risk. PTS: 1 REF: Page 174 25. Lifestyle modifications that affect health-related risk factors include: (Select all that apply.) a. Diet b. Exercise c. Education d. Finances e. Stress reduction , B, E Lifestyle modification (e.g., diet, exercise, stress reduction) can often reduce health risks significantly. Education and finances have not been shown to have an effect on health in a way that involves lifestyle modifications. PTS: 1 REF: Page 180 26. What factors are typically considered when assessing an individual’s risk for developing such common diseases as hypertension? (Select all that apply.) a. Age b. Diet c. Exercise habits d. Family history e. Spiritual beliefs , B, C, D Many factors influence the risk of acquiring a common disease, such as cancer, diabetes, or hypertension. These factors can include age, gender, diet, exercise, and family history of the disease. Current research does not support a connection between spiritual beliefs and the development of hypertension. PTS: 1 REF: Page 165 27. Examples of multifactorial diseases associated with adults include: (Select all that apply.) a. Breast cancer b. Coronary heart disease c. Emphysema d. Diabetes mellitus e. Schizophrenia , B, D, E Multifactorial diseases in adults include coronary heart disease, hypertension, breast cancer, colon cancer, diabetes mellitus, obesity, Alzheimer disease, alcoholism, schizophrenia, and bipolar affective disorder. Emphysema is not considered multifactorial. PTS: 1 REF: Pages 171-180 MATCHING Match the terms with the corresponding descriptions. A. Incidence rate effect of multiple genes B. Gene-environment interaction C. Prevalence rate at which some diseases occur D. Obesity E. Polygenic F. Empirical risk G. Relative risk 28. Traits caused by the combined effects of multiple genes 29. Number of persons living with the disease 30. Yields an increased risk for some diseases 31. Modifiable risk factor for many diseases 32. Number of new cases (persons) detected with the disease 33. Measure of the effect of a specific risk factor 34. Risks based on direct observation of data 28. ANS: E PTS: 1 REF: Page 165 MSC: Traits in which variation is thought to be caused by the combined effects of multiple genes are polygenic (i.e., many genes). 29. PTS: 1 REF: Page 164 MSC: The prevalence rate is the proportion of the population affected by a disease at a specific point in time. Thus both the incidence rate and the length of the survival period in affected individuals determine prevalence. 30. PTS: 1 REF: Page 171 MSC: In some cases, a genetic predisposition may interact with an environmental factor to increase the risk of disease to a significantly higher level than would either factor acting alone. A good example of a gene-environment interaction is given by 1-antitrypsin deficiency, a genetic condition that causes pulmonary emphysema and is greatly exacerbated by cigarette smoking (see Box 5-2). 31. PTS: 1 REF: Pages 177-178 MSC: Obesity is a modifiable risk factor for many diseases including heart disease, stroke, hypertension, and type 2 diabetes. 32. PTS: 1 REF: Page 164 MSC: The incidence rate is the number of new cases of a disease reported during a specific period (typically 1 year), divided by the number of individuals in the population. 33. ANS: G PTS: 1 REF: Page 165 MSC: A common measure of the effect of a specific risk factor is the relative risk. 34. ANS: F PTS: 1 REF: Page 167 MSC: For most multifactorial diseases, empirical risks (i.e., risks based on direct observation of data) have been derived. Chapter 6: Epigenetics and Disease MULTIPLE CHOICE 1. What genetic process is likely responsible for the occurrence of asthma in only one of a pair of identical twins? a. Epigenetic modifications c. Transgenerational inheritance b. Genomic imprinting d. Methylation Epigenetic modifications can cause individuals with the same deoxyribonucleic acid (DNA) sequences (such as identical twins) to have different disease profiles. The correct option is the only one that accurately identifies the genetic process likely responsible for the occurrence of asthma in only one of a pair of twins. PTS: 1 REF: Page 183 | Page 185 2. Prader-Willi syndrome causes a chromosomal defect that is: a. Initiated by postnatal exposure to a virus b. Inherited from the father c. Related to maternal alcohol abuse d. Transferred from mother to child Prader-Willi syndrome can be caused by a 4 Mb deletion of chromosome 15q when inherited from the father. The other options do not accurately identify the reason for the chromosomal damage that causes Prader-Willi syndrome. PTS: 1 REF: Pages 187-188 3. A malfunction in DNA methylation can lead to: a. Hypothyroidism c. Cancer b. Blindness d. Diabetes mellitus Aberrant methylation can lead to silencing of tumor-suppressor genes in the development of cancer. No research supports a connection between hypothyroidism, blindness, or diabetes mellitus to a malfunctioning of DNA methylation. PTS: 1 REF: Page 183 | Page 186 4. Which statement is true regarding the embryonic development of stem cells? a. They are already differentiated. b. They are referred to as housekeeping genes. c. They already demonstrate DNA sequencing. d. They are said to be pluripotent. Early in embryonic development, all cells of the embryo have the potential to become any type of cell in the fetus or adult. These embryonic stem cells are said to be pluripotent. The remaining options are not true statements regarding embryonic stem cell development. PTS: 1 REF: Page 184 5. When microRNA (miRNA) are methylated their messenger RNA (mRNA) targets are over-expressed, the resulting effect on existing cancer would be: a. Cell death c. Remission b. Metastasis d. Relapse When miRNA genes are methylated, their mRNA targets are over-expressed, and this over-expression has been associated with metastasis. of the described effect on mRNA targets on existing cancer does not result in any of the other options. PTS: 1 REF: Page 186 6. The difference between DNA sequence mutations and epigenetic modifications is: a. DNA sequence mutations can be directly altered. b. Leukemia is a result of only DNA sequence mutation. c. Epigenetic modifications can be reversed. d. No known drug therapies are available for epigenetic modifications. Unlike DNA sequence mutations, which cannot be directly altered, epigenetic modifications can be reversed. The remaining options are not true statements regarding the difference between DNA sequence mutations and epigenetic modifications. PTS: 1 REF: Page 186 7. Which term refers to the silenced gene of a gene pair? a. Activated c. Mutated b. Altered d. Imprinted Gene silencing, a process during which genes are predictably silenced, depending on which parent transmits them, is known as imprinting; the transcriptionally silenced genes are then said to be imprinted. The remaining options do not accurately identify this process. PTS: 1 REF: Page 187 8. The shape of the face of a child diagnosed with Russell-Silver syndrome is likely to be: a. Round c. Triangular b. Square d. Elongated Growth retardation, proportionate short stature, leg-length discrepancy, and a small, triangular-shaped face characterizes Russell-Silver syndrome. The other face shapes are not characteristic of Russell- Silver syndrome. PTS: 1 REF: Page 188 9. Genes responsible for the maintenance of all cells are referred to as: a. Universal c. Housekeeping b. Managerial d. Executive A small percentage of genes, termed housekeeping genes, are necessary for the function and maintenance of all cells. The remaining options do not accurately refer to these cells. PTS: 1 REF: Page 184 10. What is the belief regarding twins who adopt dramatically different lifestyles? a. They may experience very different aging processes. b. They will retain very similar methylation patterns. c. They will experience identical phenotypes throughout their lifespans. d. They may never demonstrate similar DNA sequences of their somatic cells. Twins with significant lifestyle differences (e.g., smoking versus nonsmoking), accumulate large numbers of differences in their methylation patterns. The twins, despite having identical DNA sequences, become more and more different as a result of epigenetic changes, which in turn affect the expression of genes. These results, along with findings generated in animal studies, suggest that changes in epigenetic patterns may be an important part of the aging process. They will not experience identical phenotypes throughout their lifespans. PTS: 1 REF: Page 185 11. Hypomethylation and the resulting effect on oncogenes result in a(an): a. Decrease in the activity of the oncogene, thus suppressing cancer development b. Deactivation of MLH1 to halt DNA repair c. Increase in tumor progression from benign to malignant d. Over-expression of microRNA, resulting in tumorigenesis Tumor cells typically exhibit hypomethylation (decreased methylation), which can increase the activity of oncogenes. Hypomethylation increases as tumors progress from benign neoplasms to malignancy. Only the correct option accurately describes hypomethylation and its resulting effects. PTS: 1 REF: Page 186 12. When a chromosome lacking 4 Mb is inherited from the mother, the child is at risk for developing which syndrome? a. Prader-Willi c. Beckwith-Wiedemann b. Angelman d. Russell-Silver This anomaly illustrates the inheritance pattern of Angelman syndrome, which can be caused by a 4 Mb deletion of chromosome 15q when inherited from the mother. The anomaly is not the cause of any of the other options. PTS: 1 REF: Page 187 MULTIPLE RESPONSE 13. A child’s diagnosis of Beckwith-Wiedemann syndrome is supported by the presence of: (Select all that apply.) a. An omphalocele b. Neonatal hypoglycemia c. Creased earlobes d. Low birth weight e. A large tongue , B, C, E Beckwith-Wiedemann syndrome is usually identifiable at birth because the child exhibits a large size for gestational age, neonatal hypoglycemia, a large tongue, creases on the earlobe, and omphalocele. PTS: 1 REF: Page 188 14. A diagnosis of Angelman syndrome in a child is supported by which assessment findings? (Select all that apply.) a. Small feet and hands b. Profound cognitive dysfunction c. Obesity d. Ataxic gait e. History of seizures , D, E A child diagnosed with Angelman syndrome demonstrates a characteristic posture, bouts of uncontrolled laughter, severe mental retardation, seizures, and an ataxic gait. PTS: 1 REF: Page 187 Chapter 11: Stress and Disease MULTIPLE CHOICE 1. Exhaustion occurs if stress continues when which stage of the general adaptation syndrome is not successful? a. Flight or fight c. Adaptation b. Alarm d. Arousal Exhaustion occurs if stress continues and adaptation is not successful, ultimately causing impairment of the immune response, heart failure, and kidney failure, leading to death. The other stages occur before the adaptation stage. PTS: 1 REF: Page 339 2. Which organ is stimulated during the alarm phase of the general adaptation syndrome (GAS)? a. Adrenal cortex c. Anterior pituitary b. Hypothalamus d. Limbic system The alarm phase of the GAS begins when a stressor triggers the actions of the hypothalamus and the sympathetic nervous system (SNS) (see Figure 11-1). The other organs are not stimulated by the alarm phase of GAS. PTS: 1 REF: Page 339 3. During an anticipatory response to stress, the reaction from the limbic system is stimulated by the: a. Retronucleus of the anterior pituitary b. Anterior nucleus of the hippocampus c. Paraventricular nucleus of the hypothalamus d. Prefrontal nucleus of the amygdala The paraventricular nucleus (PVN) of the hypothalamus must be stimulated to cause the limbic system to be stimulated. The other options are not involved in the stimulation of the limbic system. PTS: 1 REF: Page 341 4. Which hormone prompts increased anxiety, vigilance, and arousal during a stress response? a. Norepinephrine b. Epinephrine c. Cortisol d. Adrenocorticotropic hormone (ACTH) Only the release of norepinephrine promotes arousal, increased vigilance, increased anxiety, and other protective emotional responses. PTS: 1 REF: Page 343 5. Perceived stress elicits an emotional, anticipatory response that begins where? a. Prefrontal cortex c. Limbic system b. Anterior pituitary d. Hypothalamus Perceived stressors elicit an anticipatory response that begins in the limbic system of the brain, the only option responsible for emotions and cognition. PTS: 1 REF: Page 343 6. During a stress response, the helper T (Th) 1 response is suppress by which hormone? a. ACTH c. Prolactin b. Cortisol d. Growth hormone Stress can activate an excessive immune response and, through cortisol and catecholamines, suppress the Th1 response, causing a Th2 shift. This response is not active by any of the other options. PTS: 1 REF: Page 349 7. What is the effect that low-serum albumin has on the central stress response? a. Impaired circulation of epinephrine and norepinephrine b. Impaired wound healing c. Lessened circulation of cortisol d. Diminished oncotic pressure Low-serum albumin impairs circulation of both epinephrine and norepinephrine since both bind to plasma protein albumin. The other options do not accurately describe the effect of low-serum albumin. PTS: 1 REF: Page 345 8. Stress-age syndrome directly results in depressed function of which system? a. Respiratory c. Digestive b. Endocrine d. Immune Of the available options, immunodepression is the only characteristic change observed in stress-age syndrome. PTS: 1 REF: Page 358 9. Stress-induced sympathetic stimulation of the adrenal medulla causes the secretion of: a. Epinephrine and aldosterone c. Epinephrine and norepinephrine b. Norepinephrine and cortisol d. Acetylcholine and cortisol The sympathetic nervous system is aroused during the stress response and causes the medulla of the adrenal gland to release catecholamines (80% epinephrine and 20% norepinephrine) into the bloodstream. The stress-induced efforts on the adrenal medulla do not include any of the other options. PTS: 1 REF: Pages 344-345 10. Stress-induced norepinephrine results in: a. Pupil constriction c. Increased sweat gland secretions b. Peripheral vasoconstriction d. Decreased blood pressure During stress, norepinephrine raises blood pressure by constricting peripheral vessels; it dilates the pupils of the eye, causes piloerection, and increases sweat gland action in the armpits and palms. PTS: 1 REF: Page 345 11. Released stress-induced cortisol results in the stimulation of gluconeogenesis by affecting which structure? a. Adrenal cortex c. Liver b. Pancreas d. Anterior pituitary One of the primary effects of cortisol is the stimulation of gluconeogenesis through stimulation of the adrenal cortex. The other options do not produce a stimulation of gluconeogenesis when exposed to cortisol. PTS: 1 REF: Page 346 12. What is the effect of increased secretions of epinephrine, glucagon, and growth hormone? a. Hyperglycemia c. Bronchodilation b. Hypertension d. Pupil dilation Cortisol enhances the elevation of blood glucose promoted by other hormones, such as epinephrine, glucagon, and growth hormone. This effect is not true of the other options. PTS: 1 REF: Page 346 13. Which hormone increases the formation of glucose from amino acids and free fatty acids? a. Epinephrine c. Cortisol b. Norepinephrine d. Growth hormone One of the primary effects of cortisol is the stimulation of gluconeogenesis or the formation of glucose from noncarbohydrate sources, such as amino or free fatty acids in the liver. Neither reaction is a result of the effects of any of the other options. PTS: 1 REF: Page 346 14. What effect do androgens have on lymphocytes? a. Suppression of B-cell responses and enhancement of T-cell responses b. Suppression of T-cell responses and enhancement of B-cell responses c. Suppression of B- and T-cell responses d. Enhancement of B- and T-cell responses Androgens suppress T- and B-cell responses. The other options do not occur in response to androgens. PTS: 1 REF: Page 353 15. Which gland regulates the immune response and mediates the apparent effects of circadian rhythms on immunity? a. Anterior pituitary c. Basal ganglia b. Adrenal d. Pineal Of the options available, only the pineal gland regulates the immune response and mediates the apparent effects of circadian rhythm on immunity. PTS: 1 REF: Page 354 16. Which cytokines initiate the production of corticotropin-releasing hormone (CRH)? a. IL–1 and IL-6 c. IFN and IL-12 b. IL-2 and TNF- d. TNF-ß and IL-4 Although a number of stress factors initiate the production of CRH, of the options available, only high levels of IL-1 and IL-6 initiate such a response. PTS: 1 REF: Pages 348-349 17. The release of which cytokines is triggered by bacterial or viral infections, cancer, and tissue injury that, in turn, initiate a stress response? a. IL-1 and IL-2 b. IL-12, TNF-, and colony-stimulating factor c. IFN, TNF-ß, and IL-6 d. IL-4 and IL-24 Of the options offered, only the release of immune inflammatory mediators IL-6, TNF-ß, and IFN is triggered by bacterial or viral infections, cancer, and tissue injury that, in turn, initiates a stress response through the hypothalamic-pituitary-adrenal (HPA) pathway. PTS: 1 REF: Page 349 18. The action of which hormone helps explain increases in affective anxiety and eating disorders, mood cycles, and vulnerability to autoimmune and inflammatory diseases in women as a result of stimulation of the CRH gene promoter and central norepinephrine system? a. Progesterone c. Estrogen b. Cortisol d. Prolactin Of the options provided, only estrogen directly stimulates the CRH gene promoter and the central noradrenergic (norepinephrine) system, which may help explain adult women’s slight hypercortisolism, increases in affective anxiety and eating disorders, mood cycles, and vulnerability to autoimmune and inflammatory disease, all of which follow estradiol fluctuations. PTS: 1 REF: Page 350 19. What effect does estrogen have on lymphocytes? a. Depression of B-cell functions and enhancement of T-cell functions b. Depression of T-cell functions and enhancement of B-cell functions c. Depression of B- and T-cell functions d. Enhancement of B- and T-cell functions Estrogens generally are associated with only a depression of T-cell–dependent immune functions and an enhancement of B-cell functions. PTS: 1 REF: Page 353 20. Which statement is true concerning the differences between stress-induced hormonal alterations of men and women? a. After injury, women produce more proinflammatory cytokines than men, a profile that is associated with poor outcomes. b. Androgens appear to induce a greater degree of immune cell apoptosis after injury, creating greater immunosuppression in injured men than in injured women. c. Psychologic stress associated with some types of competition decreases both testosterone and cortisol, especially in athletes older than 45 years of age. d. After stressful stimuli, estrogen is increased in women, but testosterone is decreased in men. Androgens appear to induce a greater degree of immune cell apoptosis after injury, a mechanism that may elicit a greater immunosuppression in injured men versus injured women. The other options are not true statements concerning the differences between how the genders are affected by stress-induced hormones. PTS: 1 REF: Page 353 21. Diagnostic blood work on individuals who perceive themselves to be in a chronic stress state will likely demonstrate: a. Decreased Th lymphocytes c. Decreased Tc cells b. Increased erythrocytes d. Increased platelets Illustrating the influence of chronic stress appraisal on the physiologic processes, a meta-analysis of the relationships between stressors and immunity found that a higher perception of stress was associated with reduced T cytotoxic (Tc)-cell cytotoxicity, although not with levels of circulating Th or Tc lymphocytes. Research has substantiates the other options. PTS: 1 REF: Page 355 22. What are the signs that a patient is in the adaptive stage of the general adaptation syndrome? a. He or she begins to experience elevated heart and respiratory rates. b. He or she finds it difficult to concentrate on a solution for the stress. c. The patient perceives his or her only options are to run away or fight back. d. The patient has exceeded his or her ability to cope with the current situation. Fight-or-flight behaviors are characteristic of the more advanced adaptive stage, whereas the remaining options are noted in the initial alarm stage. PTS: 1 REF: Page 339 23. The most influential factor in whether a person will experience a stress reaction is his or her: a. General state of physical health c. Intellectual abilities b. Spiritual belief system d. Ability to cope A person does not have a stress reaction unless the stress exceeds his or her coping abilities. The other options do not have the same degree of influence, as does a person’s ability to cope. PTS: 1 REF: Page 339 24. A reduction is an individual’s number of natural killer (NK) cells appears to correlate with an increased risk for the development of: a. Depression b. Type 1 diabetes c. Obsessive compulsive disorder (OCD) d. Gastroesophageal reflux disorder (GERD) A meta-analysis of studies shows a relationship between depression and the reduction in lymphocyte proliferation and natural killer cell activity. Currently, no research supports the other options. PTS: 1 REF: Page 355 25. A nurse is providing care to a terminally ill adult who has been with his life partner for over 56 years. Research supports the nurse’s assessment of the life partner for signs of: a. Suicidal ideations c. Severe stress reaction b. Cardiac dysrhythmia d. Anorexia induced weight loss The results of a Harvard study showed evidence that a spouse’s illness or death can increase a partner’s mortality by causing severe stress and removing a primary source of emotional, psychologic, practical, and financial support. Although the other options may exist, research does not currently support them as having the stated correlation. PTS: 1 REF: Page 357 | What's New box MULTIPLE RESPONSE 26. The effect epinephrine has on the immune system during the stress response is to increase which cells? (Select all that apply.) a. NK cells b. Immunoglobulins c. Cytokines d. T cells e. Th cells , D The injection of epinephrine into healthy human beings is associated with a transient increase of the number of lymphocytes (e.g., T cells, natural killer (NK) cells) in the peripheral blood. This association is not true of the other options. PTS: 1 REF: Page 346 27. Which immune cells are suppressed by the corticotropin-releasing hormone (CRH)? (Select all that apply.) a. Monocyte-macrophage cells b. Cytokines c. Tc cells d. Th cells e. B cells , D Direct suppressive effects of CRH have been reported on two immune cell types possessing CRH receptors—the monocyte and macrophage and CD4 (T helper) lymphocytes. CRH does not suppress the remaining options. PTS: 1 REF: Pages 353-354 28. The increased production of proinflammatory cytokines is associated with which considerations? (Select all that apply.) a. Chronic respiratory dysfunction b. Elevated anxiety levels c. Immune disorders d. Age and gender e. Dementia , C Increased levels of proinflammatory cytokines has been shown to have a possible link between stress and immune function. The other options are not as directly linked to cytokine levels. PTS: 1 REF: Page 353 29. Which statements are true regarding lymphocytes? (Select all that apply.) a. Lymphocytes are involved in the production of the human growth hormone. b. Elevated catecholamine levels influence lymphocytes. c. Lymphocytes are synthesized in the anterior pituitary gland. d. Lymphocytes have receptors for the hormone prolactin. e. Lymphocytes produce endorphins in large amounts. , B, D GH is synthesized from the anterior pituitary gland and is produced by lymphocytes and mononuclear phagocytic cells. Several classes of lymphocytes have receptors for prolactin, suggesting a direct effect of prolactin on immune function. Although the effects of acute elevation of catecholamines on the alteration of lymphocyte function are real, they are short lived, lasting only approximately 2 hours. The other statements regarding lymphocytes are not true. PTS: 1 REF: Page 350 | Page 352 MATCHING Match the hormone with its effects during a stress response. Hormones may be used more than once. A. Epinephrine B. Norepinephrine C. Cortisol 30. Constricts peripheral vessels to increase blood pressure. 31. Increases cardiac output by increasing heart rate and myocardial contractility. 32. Increases gastric secretions. 30. PTS: 1 REF: Page 345 MSC: Norepinephrine regulates blood pressure by constricting smooth muscle in all blood vessels. 31. PTS: 1 REF: Page 345 MSC: Epinephrine enhances myocardial contractility (inotropic effect), increases the heart rate (chronotropic effect), and increases venous return to the heart, all of which increase cardiac output and blood pressure. 32. PTS: 1 REF: Page 347 MSC: Cortisol promotes gastric secretion in the gastrointestinal tract. Chapter 12: Cancer Biology MULTIPLE CHOICE 1. Which cancer originates from connective tissue? a. Osteogenic sarcoma c. Multiple myeloma b. Basal cell carcinoma d. Adenocarcinoma Cancers arising from connective tissue usually have the suffix -sarcoma. The remaining options are not cancers that originate in the connective tissue and, in addition, are lacking the common suffix. PTS: 1 REF: Page 364 2. Carcinoma refers to abnormal cell proliferation originating from which tissue origin? a. Blood vessels c. Connective tissue b. Epithelial cells d. Glandular tissue Only cancers arising from epithelial cells are called carcinomas. PTS: 1 REF: Page 364 3. Carcinoma in situ is characterized by which changes? a. Cells have broken through the local basement membrane. b. Cells have invaded immediate surrounding tissue. c. Cells remain localized in the glandular or squamous cells. d. Cellular and tissue alterations indicate dysplasia. Carcinoma in situ (CIS) refers to preinvasive epithelial malignant tumors of glandular or squamous cell origin. These early stage cancers are localized to the epithelium and have not broken through the local basement membrane or invaded the surrounding tissue. Dysplasia refers to changes in mature cell structure. PTS: 1 REF: Page 364 4. Which term is used to describe a muscle cell showing a reduced ability to form new muscle while appearing highly disorganized? a. Dysplasia c. Myoplasia b. Hyperplasia d. Anaplasia Anaplasia is defined as the loss of cellular differentiation, irregularities of the size and shape of the nucleus, and the loss of normal tissue structure. In clinical specimens, anaplasia is recognized by a loss of organization and a significant increase in nuclear size with evidence of ongoing proliferation. The remaining options refer to specific changes in the cell. PTS: 1 REF: Pages 368-369 5. What are tumor cell markers? a. Hormones, enzymes, antigens, and antibodies that are produced by cancer cells b. Receptor sites on tumor cells that can be identified and marked c. Cytokines that are produced against cancer cells d. Identification marks that are used in administering radiation therapy Tumor (biologic) markers are substances produced by both benign and malignant cells that are found either in or on the tumor cells or in the blood, spinal fluid, or urine. Tumor markers may include hormones, enzymes, genes, antigens, and antibodies. The other options do not accurately describe examples of tumor markers and their function. PTS: 1 REF: Pages 365-366 6. The function of the tumor cell marker is to: a. Provide a definitive diagnosis of cancer. b. Treat certain types of cancer. c. Predict where cancers will develop. d. Screen individuals at high risk for cancer. Screening and identifying individuals at high risk for cancer are ways tumor markers can be used. These markers are not used to definitively diagnosis or treat cancer and are not useful in predicting specific sites of cancer development. PTS: 1 REF: Page 366 7. Which statement supports the hypothesis that intestinal polyps are benign neoplasms and the first stage in the development of colon cancer? a. Cancer cells accumulate slower than noncancer cells. b. An accumulation of mutations in specific genes is required for the development of cancer. c. Tumor invasion and metastasis progress more slowly in the gastrointestinal tract. d. Apoptosis is triggered by diverse stimuli, including excessive growth. Multiple genetic mutations are required for the evolution of full-blown cancer. The remaining options do not address the progression of benign to metastatic tumors. PTS: 1 REF: Pages 372-373 8. Autocrine stimulation is the ability of cancer cells to: a. Stimulate angiogenesis to create their own blood supply. b. Encourage secretions that turn off normal growth inhibitors. c. Secrete growth factors that stimulate their own growth. d. Divert nutrients away from normal tissue for their own use. Cancer cells must have mutations that enable them to proliferate in the absence of external growth signals. To achieve this, some cancers acquire the ability to secrete growth factors that stimulate their own growth, a process known as autocrine stimulation. The remaining options do not describe autocrine stimulation. PTS: 1 REF: Page 380 9. Apoptosis is a(an): a. Normal mechanism for cells to self-destruct when growth is excessive b. Antigrowth signal activated by the tumor-suppressor gene Rb c. Mutation of cell growth stimulated by the TP53 gene d. Transformation of cells from dysplasia to anaplasia Normal cells have a mechanism that causes them to self-destruct when growth is excessive and cell cycle checkpoints have been ignored. Diverse stimuli, including normal development and excessive growth, trigger this self-destruct mechanism, called apoptosis. The remaining options do not describe apoptosis. PTS: 1 REF: Page 381 10. Many cancers create a mutation of ras. ras is a(an): a. Tumor-suppressor gene b. Growth-promoting gene c. Intracellular-signaling protein that regulates cell growth d. Cell surface receptor that allows signaling to the nucleus concerning cell growth Up to one-third of all cancers have an activating mutation in the gene for an intracellular signaling protein called ras. This mutant ras stimulates cell growth even when growth factors are missing. The remaining options do not describe ras. PTS: 1 REF: Page 380 11. Oncogenes are genes that are capable of: a. Undergoing mutation that directs the synthesis of proteins to accelerate the rate of tissue proliferation b. Directing synthesis of proteins to regulate growth and to provide necessary replacement of tissue c. Encoding proteins that negatively regulate the synthesis of proteins to slow or halt the replacement of tissue d. Undergoing mutation that directs malignant tissue toward blood vessels and lymph nodes for metastasis Oncogenes are mutant genes that, before mutation, direct synthesis of proteins that positively regulate (accelerate) proliferation. The remaining options do not describe oncogenes. PTS: 1 REF: Page 374 12. Burkitt lymphomas designate a chromosome that has a piece of chromosome 8 fused to a piece of chromosome 14. This is an example of which mutation of normal genes to oncogenes? a. Point mutation c. Gene amplification b. Chromosome translocation d. Chromosome fusion Chromosome translocations, in which a piece of one chromosome is translocated to another chromosome, can activate oncogenes. One of the best examples is the t(8;14) translocation found in many Burkitt lymphomas; t(8;14) designates a chromosome that has a piece of chromosome 8 fused to a piece of chromosome 14. The remaining options are not best depicted by a Burkitt lymphoma. PTS: 1 REF: Pages 375-376 13. In childhood neuroblastoma, the N-myc oncogene undergoes which type of mutation of normal gene to oncogene? a. Point mutation c. Gene amplification b. Chromosome fusion d. Chromosome translocation Amplifications are the result of the duplication of a small piece of a chromosome over and over again; consequently, instead of the normal two copies of a gene, tens or even hundreds of copies are present (see Chapter 4). The N-myc oncogene is amplified in 25% of childhood neuroblastoma. PTS: 1 REF: Page 376 14. What aberrant change causes the abnormal growth in retinoblastoma? a. Proto-oncogenes are changed to oncogenes. b. The tumor-suppressor gene is turned off. c. Genetic amplification causes the growth. d. Chromosomes 9 and 21 are fused. One of the first discovered tumor-suppressor genes, the retinoblastoma (Rb) gene, normally strongly inhibits the cell division cycle. When it is inactivated, the cell division cycle can proceed unchecked. The Rb gene is mutated in childhood retinoblastoma. The remaining options do not describe the abnormal growth in retinoblastoma. PTS: 1 REF: Page 376 15. Two “hits” are required to inactivate tumor-suppressor genes because: a. Each allele must be altered, and each person has two copies, or alleles, of each gene, one from each parent. b. The first hit stops tissue growth, and the second hit is needed to cause abnormal tissue growth. c. Tumor-suppressor genes are larger than proto-oncogenes, requiring two hits to effect carcinogenesis. d. The first hit is insufficient to cause enough damage to cause a mutation. A single genetic event can activate an oncogene, acting in a dominant manner in the cell. However, each person has two copies, or alleles, of each gene, one from each parent. Therefore two hits are required to inactivate the two alleles of a tumor-suppressor gene, allowing the process to become active. The remaining options do not describe the reason two hits are required. PTS: 1 REF: Page 376 | Page 378 16. The ras gene converts from a proto-oncogene to an oncogene by: a. Designating a chromosome that has a piece of one chromosome fused to a piece of another chromosome b. Duplicating a small piece of a chromosome, repeatedly making numerous copies c. Altering one or more nucleotide base pairs d. Promoting proliferation of growth signals by impairing tumor-suppressor genes A point mutation is the alteration of one or a few nucleotide base pairs. This type of mutation can have profound effects on the activity of proteins. A point mutation in the ras gene converts it from a regulated proto-oncogene to an unregulated oncogene, an accelerator of cellular proliferation. The remaining options do not describe point mutation as it affects the conversion of a ras gene. PTS: 1 REF: Page 375 17. How do cancer cells use the enzyme telomerase? a. To repair the telomeres to restore somatic cell growth b. As an intracellular signaling chemical to stimulate cell division c. To switch off the telomerase to enable cells to divide indefinitely d. To switch on the telomerase to enable cells to divide indefinitely Cancer cells, when they reach a critical age, somehow activate telomerase to restore and maintain their telomeres and thereby make it possible for cells to divide over and over again. The remaining options do not describe how cancer cells use telomerase. PTS: 1 REF: Page 382 18. What are characteristics of benign tumors? a. Benign tumors invade local tissues. b. Benign tumors spread through the lymph nodes. c. Benign tumors cause systemic symptoms. d. Benign tumors include the suffix -oma. Benign tumors are usually encapsulated and well-differentiated. They retain some normal tissue structure and do not invade the capsules surrounding them or spread to regional lymph nodes or distant locations. Benign tumors are generally named according to the tissues from which they arise and include the suffix -oma. Benign tumors do not cause systemic symptoms. PTS: 1 REF: Page 364 19. Which terms represent the correct nomenclature for benign and malignant tumors of adipose tissue, respectively? a. Liposarcoma, lipoma c. Adisarcoma, adipoma b. Lipoma, liposarcoma d. Adipoma, adisarcoma In general, cancers are named according to the cell type from which they originate (e.g., lip for cancers that originate in adipose or fat tissue), whereas benign tumors use the suffix -oma. Cancers arising from connective tissue usually have the suffix sarcoma. PTS: 1 REF: Page 364 | Page 367 | Table 12-2 20. What is the major virus involved in the development of cervical cancer? a. Herpes simplex virus type 6 c. Human papillomavirus b. Herpes simplex virus type 2 d. Human immunodeficiency virus Infection with specific subtypes of human papillomavirus (HPV) cause virtually all cervical cancers. The remaining options are not known to be associated with cervical cancer. PTS: 1 REF: Pages 382-383 21. The Papanicolaou (Pap) test is used to screen for which cancer? a. Ovarian c. Cervical b. Uterine d. Vaginal The Pap test, an examination of cervical epithelial scrapings, readily detects early oncogenic human papillomavirus (HPV)infection. The Pap test is not used for screening the other cancer sites listed. PTS: 1 REF: Page 382 22. What is the skin-related health risk induced by some types of chemotherapy? a. Infection c. Pain b. Ultraviolet damage d. Erythema Decreased renewal rates of the epidermal layers in the skin may lead to skin breakdown and dryness, altering the normal barrier protection against infection. Radiation therapy may cause skin erythema (redness). Pain and ultraviolet damage is not related to chemotherapies. PTS: 1 REF: Page 396 | Box 12-2 23. Which cancers are all associated with chronic inflammation? a. Brain, muscle, and endocrine b. Colon, thyroid gland, and urinary bladder c. Bone, blood cells, and liver d. Eye, tracheal, and kidney Some organs appear to be more susceptible to the oncogenic effects of chronic inflammation; for example, the GI tract, prostate, thyroid gland, pancreas, urinary bladder, pleura, and skin. One large study found a 66% increase in the risk of lung cancer among women with chronic asthma, an inflammatory disease of the airways. At present, no research supports a link between the remaining options and chronic inflammation. PTS: 1 REF: Pages 383-384 24. Chronic inflammation causes cancer by: a. Increasing vasodilation and permeability that alter cellular response to DNA damage b. Liberating lysosomal enzymes when cells are damaged that initiates mutations c. Releasing compounds such as reactive oxygen species that promote mutations d. Increasing the abundance of leukotrienes that are associated with some cancers Inflammatory cells release compounds, such as reactive oxygen species (ROS) and other reactive molecules, that can promote mutations and block the cellular response to DNA damage. At present, no research supports the other options as factors related to inflammation causing cancer. PTS: 1 REF: Page 384 25. Inherited mutations that predispose to cancer are almost invariably what kind of gene? a. Proto-oncogenes c. Tumor-suppressor genes b. Oncogenes d. Growth-promoting genes Inherited mutations that predispose to cancer are almost invariably in tumor-suppressor genes. At present, no research supports the other options as factors related to how inherited mutations cause cancer. PTS: 1 REF: Page 379 26. What is the consequence for cells when the functioning TP53 gene is lost as a result of mutation? a. Cells undergo apoptosis. c. Cells receive less oxygen. b. Cells escape apoptosis. d. Cells adhere more readily. The most common mutations conferring resistance to apoptosis occur in the TP53 gene. The remaining options do not accurately describe the effect when the functioning TP53 gene is lost as a result of mutation. PTS: 1 REF: Page 381 27. Which gastrointestinal tract condition can be an outcome of both chemotherapy and radiation therapy? a. Increased cell turnover c. Stomatitis b. Constipation d. Bloody stool Chemotherapy and radiation therapy may cause a decreased cell turnover, thereby leading to oral ulcers (stomatitis), malabsorption, and diarrhea. None of the other options accurately describe related conditions resulting from chemotherapy and/or radiation therapies. PTS: 1 REF: Page 396 | Box 12-2 28. What is the role of vascular endothelial growth factor (VEGF) and basic fibroblast growth factor (bFGF) in cell metastasis? a. To stimulate growth of nearby tumor cells b. To develop new blood vessels to feed cancer cells c. To prevent cancer cells from escaping apoptosis d. To act as a chemical gradient to guide cells to blood vessels By recruiting new vascular endothelial cells and initiating the proliferation of existing blood vessel cells, the angiogenic factors, such as VEGF and growth factor bFGF, allow small cancers to become large cancers. None of the other options accurately describe the role of the various stated factors on cell metastasis. PTS: 1 REF: Page 381 29. It has been determined that a tumor is in stage 2. What is the meaning of this finding? a. Cancer is confined to the organ of origin. b. Cancer has spread to regional structures. c. Cancer is locally invasive. d. Cancer has spread to distant sites Cancer confined to the organ of origin is stage 1; cancer that is locally invasive is stage 2; cancer that has spread to regional structures, such as the lymph nodes, is stage 3; and cancer that has spread to distant sites, such as a liver cancer spreading to the lung or a prostate cancer spreading to bone, is stage 4. PTS: 1 REF: Pages 393-394 | Figure 12-25 30. Which statement is true regarding pain and cancer? a. Pain is primarily a result of pressure caused by the tumor. b. Pain indicates the metastasis of a cancer. c. Pain is usually the initial symptom of cancer. d. Pain is generally associated with late-stage cancer. Pain is generally associated with the late stages of cancer. Pressure, obstruction, invasion of a structure sensitive to pain, stretching, tissue destruction, and inflammation can cause pain. Pain is not the initial symptom of cancer nor does it indication that the cancer has metastasized. PTS: 1 REF: Page 399 31. Which cancer may be treated with radiation delivered by brachytherapy? a. Lung c. Cervical b. Colon d. Brain Radiation sources, such as small 125I-labeled capsules (also called seeds), can also be temporarily placed into body cavities, a delivery method termed brachytherapy. Brachytherapy is useful in the treatment of cervical, prostate, and head and neck cancers. Brachytherapy is not used in the treatment of the other cancers. PTS: 1 REF: Pages 397-398 32. The survival rate for stage IV Hodgkin disease can be as high as: a. 99% c. 40% b. 70% d. 20% Survival rates for Hodgkin disease is 99% for stage I and 70% for stage IV. PTS: 1 REF: Page 394 | Table 12-10 33. What is the cause of anemia in a patient diagnosed with pancreatic cancer? a. Impaired pancreatic function c. Chronic bleeding b. Malnutrition d. Malabsorption of iron Iron is malabsorbed in individuals with gastric, pancreatic, or upper intestinal cancer. Commonly associated with malignancy, mechanisms of anemia include chronic bleeding (resulting in iron deficiency), severe malnutrition, cytotoxic chemotherapy, and malignancy in blood-forming organs. The pancreas is not involved in the formation of blood components. Chronic bleeding and iron deficiency can accompany colorectal or genitourinary malignancies. PTS: 1 REF: Page 396 | Box 12-2 34. By what process do cancer cells multiply in the absence of external growth signals? a. Proto-oncogene c. Reliance on caretaker genes b. Autocrine stimulation d. Pleomorphology Cancer cells must have mutations that enable them to proliferate in the absence of external growth signals. To achieve this, some cancers acquire the ability to secrete growth factors that stimulate their own growth, a process known as autocrine stimulation. The other options are not involved in the proliferation of cancer cells in the absence of external growth signals. PTS: 1 REF: Page 380 35. What is the role of caretaker genes? a. Maintenance of genomic integrity c. Secretion of growth factors b. Proliferation of cancer cells d. Restoration of normal tissue structure Caretaker genes are responsible for the maintenance of genomic integrity. The other options are not roles assumed by caretaker genes. PTS: 1 REF: Page 379 36. In a normal, nonmutant state, an oncogene is referred to as a: a. Basal cell c. Caretaker gene b. Target cell d. Proto-oncogene In its normal nonmutant state, an oncogene is referred to as a proto-oncogene. The other options are not terms used to identify a nonmutant oncogene. PTS: 1 REF: Page 374 37. Which statement is true regarding pleomorphic cells? a. Pleomorphic cells are similar in size. b. They share a common shape. c. They are a result of anaplasia. d. Pleomorphic cells differentiate uniformly. In contrast to normal cells, which are uniform in size and shape, anaplastic cells are of variable size and shape and abnormally differentiate, making them pleomorphic. PTS: 1 REF: Pages 368-369 38. What is the most commonly reported symptom of cancer treatment? a. Nausea c. Hair loss b. Fatigue d. Weight loss Fatigue is the most frequently reported symptom of cancer and cancer treatment. Although patients report the other options, they are not as frequently experienced as fatigue. PTS: 1 REF: Page 396 | Box 12-2 39. The most common site of metastasis for a patient diagnosed with prostate cancer is which location? a. Bones c. Bladder b. Brain d. Kidney The bone, especially the lumbar spine area, is the most common metastasis site for prostate cancer. PTS: 1 REF: Page 391 | Table 12-8 40. Which statement concerning benign tumors is true? a. The resulting pain is severe. c. Benign tumors are fast growing. b. Benign tumors are not encapsulated. d. The cells are well-differentiated. A benign tumor is well-differentiated with its tissue appearing similar to the tissue from which it arose. The other options are characteristic of a malignant tumor. PTS: 1 REF: Page 364 | Table 12-1 MULTIPLE RESPONSE 41. Normally, which cells are considered immortal (never die)? (Select all that apply.) a. Germ b. Stem c. Blood d. Epithelial e. Muscle , B Usually, germ cells (those that generate sperm and eggs) and stem cells are the only cells in the body that are immortal. Other cells in the body are not immortal and can divide only a limited number of times. The remaining options do not identify the appropriate cells. PTS: 1 REF: Page 382 42. What is the most common route for distant metastasis? (Select all that apply.) a. Seeding b. Blood c. Lymphatic vessels d. Invasion e. Proliferation , C To transition from local to distant metastasis, the cancer cells must also be able to invade local blood and lymphatic vessels. The remaining options are not directly related to distant metastasis. PTS: 1 REF: Page 387 43. What cellular characteristics are affected by anaplasia? (Select all that apply.) a. Size b. Ability to differentiate c. Life expectancy d. Tissue structure e. Shape , B, D, E Anaplasia is defined as the loss of cellular differentiation, irregularities of the size and shape of the nucleus, and loss of normal tissue structure. Life expectancy is not generally included in this term. PTS: 1 REF: Page 364 44. What are the most common causes of nosocomial infections among patients with cancer? (Select all that apply.) a. Indwelling medical devices b. Suppressed immune system c. Visitor-introduced microorganisms d. Poor appetite e. Inadequate wound care , C, E Hospital-acquired (nosocomial) infections increase because of indwelling medical devices, inadequate wound care, and the introduction of microorganisms from visitors and other individuals. A suppressed immune system and a poor appetite are possible causes of infections but they are not nosocomial in nature. PTS: 1 REF: Page 396 | Box 12-2 45. Which statements concerning aging and the occurrence of cancer are true? (Select all that apply.) a. Decline in immunologic functions b. Predisposition to nutritional inadequacies c. Unwillingness to access health care services d. Reluctance to engage in cancer screenings e. Effects of immobility on the immune system , B, E Many common malignancies occur mostly in older age as a result of immunologic functions declining with age. Older persons are predisposed to nutritional inadequacies, and malnutrition impairs immunocompetence. Far-advanced cancer often results in immobility and general debility that worsens with age. No research supports a correlation between aging and a reluctance to seek health care, in general, or cancer screenings, in particular. PTS: 1 REF: Page 397 | Table 12-12 MATCHING Match the phrases with the corresponding terms. A. Is the process of cancer cell growth. B. Is used to kill cancer cells while minimizing damage to normal structures. C. Is guided by molecular analysis in specific diseases. D. Takes advantage of specific vulnerabilities in specific cancer cells. E. Provides a framework to determine treatment. 46. Chemotherapy 47. Radiation 48. Staging 49. Angiogenesis 50. Target agent 46. PTS: 1 REF: Page 396 MSC: All chemotherapeutic agents take advantage of specific vulnerabilities in target cancer cells. 47. PTS: 1 REF: Pages 397-398 MSC: Radiation therapy is used to kill cancer cells while minimizing damage to normal structures. 48. ANS: E PTS: 1 REF: Pages 393-394 MSC: Staging may alter the choice of therapy, with more aggressive therapy being delivered to more invasive disease (advanced staging). 49. PTS: 1 REF: Page 381 MSC: Angiogenesis is the process of growth and proliferation of cancer cells. 50. PTS: 1 REF: Pages 397-398 MSC: The newest highly targeted agents that are used to treat cancer exploit specific vulnerabilities uncovered by molecular analysis in specific diseases. MICS: The newest highly targeted agents that are used to treat cancer exploit specific vulnerabilities uncovered by molecular analysis in specific diseases. Match the organism factor with the cancer it causes. A. HPV B. Human herpesvirus (HHV) 8 C. Hepatis B virus (HBV) D. Helicobacter pylori 51. Cervical cancer 52. Kaposi sarcoma 53. Liver cancer 54. Stomach cancer 51. PTS: 1 REF: Pages 382-383 MSC: HPV infection causes human cervical cancer. 52. PTS: 1 REF: Page 383 MSC: HHV-8 infection causes Kaposi sarcoma. 53. PTS: 1 REF: Page 383 MSC: Chronic hepatitis infection with HBV or hepatis C virus (HCV) is the leading cause of liver cancer. 54. PTS: 1 REF: Page 384 MSC: Chronic H. pylori-associated inflammation causes stomach cancer. Chapter 13: Cancer Epidemiology MULTIPLE CHOICE 1. Which characteristic among women correlates with a high morbidity of cancer of the colon, uterus, and kidney? a. Women older than 45 years of age b. Women who have never had children c. Women who have a high body mass index D. Woman who have smoked for more than 10 years A recent hypothesis states that the observed increased incidence of such cancers as breast, endometrium, colon, liver, kidney, and adenomas of the esophagus may be associated with obesity. No current research supports the remaining options. PTS: 1 REF: Pages 417-418 | Figure 13-11 2. Which substance has been shown to increase the risk of cancer when used in combination with tobacco smoking? a. Alcohol c. Antihistamines b. Steroids d. Antidepressants Alcohol interacts with smoke, increasing the risk of malignant tumors, possibly by acting as a solvent for the carcinogenic chemicals in smoke products. No current research supports the remaining options as having an increased effect on the incidence of cancer when used in combination with tobacco smoking. PTS: 1 REF: Page 420 | Page 422 3. Which cancers pose the highest risk for radiologists? a. Thyroid c. Leukemia b. Breast d. Brain Ionizing radiation exposure places radiologists at risk for the development of leukemia, lymphoma, and skin cancers. PTS: 1 REF: Page 424 | Table 13-8 4. Research has shown a link between cancer and which sexually transmitted disease? a. Syphilis c. Human papillomavirus b. Gonorrhea d. Pelvic inflammatory disease Human papillomavirus (HPV) is the most common sexually transmitted virus in the United States. High-risk, or oncogenic, HPVs can cause cancer. A persistence of infection with high-risk HPV is a prerequisite for the development of cervical intraepithelial neoplasia (CIN) lesions and invasive cervical cancers. No research supports such a link between the remaining options and cancer. PTS: 1 REF: Pages 423-424 5. Research has supported that alcohol consumption has a convincing impact on increasing the risk of which cancer? a. Lung c. Pharynx b. Breast d. Pancreas Research had shown that alcohol consumption has a convincing positive impact on the risk of developing pharynx cancer. No research supports such a connection with the remaining options. PTS: 1 REF: Page 422 | Table 13-5 6. Research supports the premise that exercise has a probable impact on reducing the risk of which cancer? a. Liver c. Stomach b. Endometrial d. Colon The World Cancer Research Fund summarizes the effects as convincing for cancers of the colon and probable for postmenopausal breast cancer and endometrial cancer. The relationship is not supported for the remaining options. PTS: 1 REF: Page 422 | Table 13-6 7. The World Health Organization (WHO) defines grade 1 (overweight) as a BMI of: a. 18.5 to 24.9 c. 30 to 39.9 b. 25 to 29.9 d. 40 to 50.9 A BMI of 25 to 29.9 kg/m2 is considered a grade 1 (overweight) classification. A BMI of 18.5 to 24.9 kg/m2 is considered normal range, whereas 30 to 39.9 kg/m2 is a grade 2 (severe overweight) classification, and a BMI higher than 40 kg/m2 is considered grade 3 (morbidly overweight). PTS: 1 REF: Page 418 | Table 13-4 MULTIPLE RESPONSE 8. Tobacco smoking is associated with which cancers? (Select all that apply) a. Lung b. Skin c. Bladder d. Kidney e. Pancreas , C, D, E Besides lung cancer, tobacco increases the risk for cancers of the mouth, lips, nasal cavity and sinuses, larynx, pharynx, esophagus, stomach, pancreas, kidney, bladder, uterus, cervix, colon and rectum, ovary, and acute leukemia. No current research links tobacco smoking to skin cancer. PTS: 1 REF: Page 413 9. The significant risk factors associated with the development of basal cell carcinoma include: (Select all that apply.) a. Light-colored hair b. Pale eyes c. Fair skin d. Freckles e. History of acne , B, C, D Risk factors associated with a high risk of melanoma include light-colored hair, eyes, and skin; an inability to tan; and a tendency to freckle, sunburn, and develop nevi. No current research supports acne as a risk factor. PTS: 1 REF: Page 431 10. Which environmental factors have been shown to have greatest significant effect on the frequency of genetic mutations in humans? (Select all that apply.) a. Tobacco use b. Radiation exposure c. Obesity d. Severe allergies e. Stressful occupation , B, C The number of environmental factors can alter the frequency and consequences of genetic mutations. The most significant factors include smoking, radiation, obesity, and a few oncogenic viruses. Allergies and stress have not been strongly linked as risk factors for genetic mutations. PTS: 1 REF: Pages 413-433 11. Research data support the relationship between the exposure of ultraviolet light (UVL) and the development of: (Select all that apply.) a. Basal cell carcinoma b. Squamous cell carcinoma c. Hodgkin lymphoma d. Non-Hodgkin lymphoma e. Soft-tissue sarcoma , B Exposure to UVL can cause basal cell carcinoma and squamous cell carcinoma. No current research supports a causative relationship between the remaining options and UVL exposure. PTS: 1 REF: Page 431 12. Alcohol consumption has been proven to be a consistent risk factor in the development of which major cancer? (Select all that apply.) a. Colorectal b. Kidney c. Breast d. Esophageal e. Pancreatic , C, D Research supports a consistent link between alcohol consumption and an increased risk for breast, colorectal, and esophageal cancers; no such data have yet been determined for the remaining options. PTS: 1 REF: Page 420 13. Nutritional counseling regarding cancer stem cell repopulation must stress the importance of including sufficient quantities of what in a daily diet? (Select all that apply.) a. Vitamin A b. Vitamin D c. Protein d. Vitamin C e. Water , B Cancer stem cells use several developmental mechanisms for the self-renewal, and these mechanisms appear to be fundamental to the initiation and recurrence of tumors. Adequate consumption of specific food compounds, including vitamin A and D, genistein, green tea epigallocatechin gallate (EGCG), sulforaphane, theanine, curcumin, and choline may suppress cancer stem renewal. No current research supports such a connection with the other options. PTS: 1 REF: Page 416 14. Which personal lifestyle choices will likely have a positive impact on reducing an individual’s risk for developing cancer? (Select all that apply.) a. Avoiding crowds b. Wearing a hat while golfing c. Eating a strict vegetarian diet d. Having regular health screenings e. Staying indoors during smog alerts , E Avoiding exposure to ultraviolet radiation and air pollutants will help decrease the risk of developing skin and lung cancer. Although avoiding crowds may reduce the risk of contracting infections, no research suggests such a decision will reduce the risk of cancer. Eating a vegetarian diet will reduce the consumption of red meat, but no research suggests that such a diet is a factor in reducing the risk of cancer. Health screening, although important, can detect the possible presence of a specific disease, but it does not prevent diseases. PTS: 1 REF: Page 431 | Page 433 15. Which individual is at risk for developing lung and respiratory tract cancers because of exposure to known cancer-inducing environmental factors? (Select all that apply.) a. A patient whose partner smokes 1 1/2 packs of cigarettes a day b. A 10-year employee of an asbestos fiber manufacturer c. A farmer whose equipment is primarily driven by diesel engines d. An older adult with a family history of lung cancer and chronic obstructive pulmonary disease (COPD) e. A 10-year delivery employee for a paper manufacturing company , B, C, E Numerous environmental factors can cause inflammation and include, for example, inhaling tobacco smoke, asbestos fibers, or fine particles in the air from diesel engine exhaust and industrial sources. Sulfur dioxide is produced by power plants burning oil and coal, copper smelting, and paper mills. These sources are major factors in lung and other respiratory tract cancers. A family history is not considered a risk factor since these cancers are generally affected by environmental rather than genetic factors. PTS: 1 REF: Page 403 | Page 433 16. Which primary prevention strategy has a substantial likelihood of reducing the risk of developing cancer? (Select all that apply.) a. Offering free smoking cessation classes to college students b. Establishing a support group for patients receiving chemotherapy c. Providing a breast cancer screening at a senior center twice a year d. Supplying low sugar, low-fat snacks at an after-school tutoring program e. Funding a support service that offers transportation to cancer treatment centers , C, D Targeted interventions can substantially reduce the incidences of cancers worldwide and should include primary prevention strategies to decrease or eliminate certain lifestyle factors including tobacco avoidance and the cessation of smoking, a reduction in obesity and alcohol consumption, an increase in physical activity, and the implementation of vaccination programs for liver and cervical cancer, as well as effective early detection programs for colorectal, breast, and cervical cancer. Support groups and transportation to cancer centers are secondary strategies. PTS: 1 REF: Page 409 | What's New box 17. When considering topics for a series of community discussions related to cancer prevention strategies, the nurse will include: (Select all that apply.) a. Healthy eating b. Tobacco avoidance c. Importance of early diagnosis d. Impact of obesity on one’s health e. Age-appropriate exercise routines , B, D, E Eliminating smoking, decreasing obesity, participating in appropriate exercises, eliminating infections, and avoiding an unhealthy diet are all essential for preventing cancer. Screening is not a prevention strategy. PTS: 1 REF: Page 412 | What's New box Chapter 14: Cancer in Children MULTIPLE CHOICE 1. What congenital malformation is commonly linked to acute leukemia in children? a. Down syndrome c. Retinoblastoma b. Wilms tumor d. Neuroblastoma Trisomy 21 (Down syndrome) is the most common genetic defect linked to the development of acute leukemia. Children with Down syndrome have a 10- to 20-fold increased risk of developing acute lymphoblastic and myelogenous leukemia and a higher risk for developing acute megakaryocytic leukemia. No current research supports a link between the other stated congenital malformations and acute leukemia. PTS: 1 REF: Page 444 2. When are childhood cancers most often diagnosed? a. During infancy c. After diagnosis of a chronic illness b. At peak times of physical growth d. After an acute illness Childhood cancers are most often diagnosed during peak times of physical growth. No current research supports the other options. PTS: 1 REF: Page 443 3. Prenatal exposure to diethylstilbestrol (DES) can result in which type of cancer? a. Breast cancer c. Vaginal cancer b. Leukemia d. Lymphoma DES has been identified as a transplacental chemical carcinogen; a small percentage of the daughters of women who took DES during pregnancy developed adenocarcinomas of the vagina. No current research supports a link between the drug and other cancers. PTS: 1 REF: Page 445 4. Currently, what percentage of children with cancer can be cured? a. 40% c. 60% b. 50% d. 80% Currently, more than 80% of children diagnosed with cancer are cured. PTS: 1 REF: Page 445 5. Most childhood cancers arise from the: a. Epithelium c. Embryologic ectodermal layer b. Mesodermal germ layer d. Viscera Most childhood cancers originate from the mesodermal germ layer that gives rise to connective tissue, bone, cartilage, muscle, blood, blood vessels, gonads, kidney, and the lymphatic system. The other options are not common sites from which cancers originate. PTS: 1 REF: Page 443 6. Which form of cancer is linked to congenital malformation syndromes? a. Wilms tumor c. Osteosarcoma b. Retinoblastoma d. Rhabdomyosarcoma Wilms tumors are linked with other genetically linked childhood cancers. It is the only form among the available options that is associated with congenital malformation syndromes. PTS: 1 REF: Page 443 | Table 14-1 7. Research data support a carcinogenic relationship in children resulting from exposure to which virus? a. Herpes simplex virus c. Varicella zoster virus b. Influenza d. Epstein-Barr virus The strongest association between viruses and the development of cancer in children has been the Epstein-Barr virus (EBV), Burkitt lymphoma, nasopharyngeal carcinoma, and Hodgkin disease. No current research supports a link between the remaining options and childhood cancer. PTS: 1 REF: Page 445 8. A child diagnosed with acquired immunodeficiency syndrome (AIDS) has an increased risk of developing: a. Non-Hodgkin lymphoma c. Epstein-Barr b. Retinoblastoma d. Leukemia Children with AIDS have an increased risk of developing non-Hodgkin lymphoma and Kaposi sarcoma. Leukemia is linked to retinoblastoma. Epstein-Barr has been linked to the development of some cancers, whereas no known link has been found between AIDS and the development of leukemia. PTS: 1 REF: Page 445 9. Which intervention has the greatest affect on a child’s mortality rate when diagnosed with cancer? a. Age at the time of diagnosis b. Participation in clinical trials c. Proximity to a major cancer treatment center d. Parental involvement in the treatment planning Mortality rates have significantly declined in the past 40 years largely as a result of advances in treatment and participation in clinical trials. Although important, the other options are not believed to have an affect on the decline of the mortality rate of childhood cancer. PTS: 1 REF: Page 445 10. Which statement is likely true regarding children being treated for cancer with radiation therapy? a. They will most likely have a successful remission of tumor growth. b. They seldom require follow-up maintenance treatments. c. They are prone to experience severe developmental delays. d. They are at increased risk for developing adult cancers. Although the need exists for long-term studies, research has shown a correlation between radiation-induced malignancies from radiotherapy (as in cancer treatment) or radiation exposure from diagnostic imaging; both have shown to increase the risk of developing cancer during adulthood. The other options are not necessarily true regarding the outcomes of radiation therapy for childhood cancers. PTS: 1 REF: Page 445 11. How should the nurse reply when a parent questions why a computed tomographic (CT) scan of the head was not ordered for their 5-year-old child after a minor fall? a. Physicians are cautious about ordering CT scan on children younger than 10 years of age. b. CT scans are seldom conclusive when used to diagnosis head injuries in young children. c. The child’s symptoms will determine whether a CT scan is necessary and worth the expense. d. Research suggests that repeated CT scans can increase the risk of developing brain cancer. Findings from a recent study of 176,587 children suggest that those who have two or three CT scans of the head before the age of 22 years are three times more likely to develop brain cancer as those in the general population, and the risk of developing leukemia is three times as great in those who received five to ten CT scans. The other options do not represent the logic behind not ordering a CT scan in relationship to minor head trauma. PTS: 1 REF: Page 445 | What's New box MULTIPLE RESPONSE 12. Childhood exposure to which risk factors increases the susceptibility for developing cancers? (Select all that apply.) a. Low birth weight b. Chemotherapy c. Ionizing radiation d. Cigarette smoke e. Hodgkin disease , C, E Childhood exposure to ionizing radiation, drugs, or existing cancer-causing viruses has been associated with the risk of developing cancer. Although unhealthy, no current research identifies cigarette smoke or low birth weight as risk factors for childhood cancer. PTS: 1 REF: Page 445 | Table 14-3 13. Which statement is true concerning the difference between adult and childhood cancers? (Select all that apply.) a. Numerous differences exist between these two categories of cancer. b. Childhood cancers are far less likely to be associated with genetic mutations. c. Environmental risks are strongly associated with childhood cancers. d. Exposure to pesticides is a minor risk for the development of adult cancers. e. Maternal exposure to carcinogenic substances presents little risk to the fetus. , B Overall, cancers in children are very different than adult cancers and are associated with far fewer genetic mutations. Research does not support the other options. PTS: 1 REF: Page 442 | Page 444 | What's New box 14. Most childhood cancers originate from the mesodermal germ layer that ultimately produces which of the following? (Select all that apply.) a. Connective tissue b. Digestive system c. Muscles d. Kidneys e. Blood , C, D, E Most childhood cancers originate from the mesodermal germ layer, which gives rise to connective tissue, bone cartilage, muscle, blood, blood vessels, gonads, kidney, and the lymphatic system. Cancers of the digestive tract do not originate in the mesodermal germ layer. PTS: 1 REF: Page 443 15. Which statements are true regarding cancers that develop in children? (Select all that apply.) a. Childhood cancers tend to be fast growing. b. Childhood cancers are diagnosed during growth spurts. c. Childhood cancer generally metastasizes by the time of diagnosis. d. Childhood cancer is typically at low risk for aggressive metastasis. e. Childhood cancers respond well to standardized treatment modalities. , B, C Childhood cancers are most often diagnosed during peak times of physical growth. In general, they are extremely fast growing, with 80% having distant spread (metastases) at diagnosis. The other options are not true regarding childhood cancers. PTS: 1 REF: Page 443 16. The nurse is preparing a discussion on cancer and its occurrence among college-aged students. Which cancers will the nurse include in the discussion? (Select all that apply.) a. Colorectal b. Testicular c. Thyroid d. Breast e. Lung , B, C, D The most common cancers among the adolescent and young adult population (15 to 39 years of age) are Hodgkin lymphoma, leukemia, germ-cell tumors (particularly testicular), central nervous system tumors, non-Hodgkin lymphoma, thyroid cancer, melanoma, sarcomas, and breast, cervical, liver, thyroid and colorectal cancers. Lung cancer generally develops after chronic inhalation of nicotine-containing products. PTS: 1 REF: Page 442 Chapter 31: Structure and Function of the Cardiovascular and Lymphatic Systems MULTIPLE CHOICE 1. Which statement does not accurately describe the pericardium? a. The pericardium is a double-walled membranous sac that encloses the heart. b. It is made up of connective tissue and a surface layer of squamous cells. c. The pericardium protects the heart against infection and inflammation from the lungs and pleural space. d. It contains pain and mechanoreceptors that can elicit reflex changes in blood pressure and heart rate. The pericardium is made up of a surface layer of mesothelium over a thin layer of connective tissue. The remaining options accurately describe the pericardium. PTS: 1 REF: Page 1085 2. Which cardiac chamber has the thinnest wall and why? a. The right and left atria; they are low-pressure chambers that serve as storage units and conduits for blood. b. The right and left atria; they are not directly involved in the preload, contractility, or afterload of the heart. c. The left ventricle; the mean pressure of blood coming into this ventricle is from the lung, which has a low pressure. d. The right ventricle; it pumps blood into the pulmonary capillaries, which have a lower pressure compared with the systemic circulation. The two atria have the thinnest walls because they are low-pressure chambers that serve as storage units and conduits for blood that is emptied into the ventricles. This selection is the only option that correctly identifies which heart chambers have the thinnest walls and why that helps cardiac function. PTS: 1 REF: Page 1086 3. Which chamber of the heart endures the highest pressures? a. Right atrium c. Left ventricle b. Left atrium d. Right ventricle Pressure is greatest in the systemic circulation, driven by the left ventricle. PTS: 1 REF: Page 1086 4. What is the process that ensures mitral and tricuspid valve closure after the ventricles are filled with blood? a. Chordae tendineae relax, which allows the valves to close. b. Increased pressure in the ventricles pushes the valves to close. c. Trabeculae carneae contract, which pulls the valves closed. d. Reduced pressure in the atria creates a negative pressure that pulls the valves closed. During ventricular relaxation, the two atrioventricular valves open and blood flows from the higher pressure atria to the relaxed ventricles. With increasing ventricular pressure, these valves close and prevent backflow into the atria as the ventricles contract. This selection is the only option that correctly identifies the process that ensures closing of the mitral and tricuspid valves. PTS: 1 REF: Page 1088 5. Regarding the heart’s valves, what is a function of the papillary muscles? a. The papillary muscles close the semilunar valve. b. These muscles prevent backward expulsion of the atrioventricular valve. c. They close the atrioventricular valve. d. The papillary muscles open the semilunar valve. The papillary muscles are extensions of the myocardium that pull the cusps together and downward at the onset of ventricular contraction, thus preventing their backward expulsion into the atria. This selection is the only option that correctly describes the function of the papillary muscles. PTS: 1 REF: Pages 1087-1088 6. During the cardiac cycle, why do the aortic and pulmonic valves close after the ventricles relax? a. Papillary muscles relax, which allows the valves to close. b. Chordae tendineae contract, which pulls the valves closed. c. Reduced pressure in the ventricles creates a negative pressure, which pulls the valves closed. d. Blood fills the cusps of the valves and causes the edges to merge, closing the valves. When the ventricles relax, blood fills the cusps and causes their free edges to meet in the middle of the vessel, closing the valve and preventing any backflow. This selection is the only option that accurately explains why the aortic and pulmonic valves close after the ventricles contract. PTS: 1 REF: Page 1088 7. Oxygenated blood flows through which vessel? a. Superior vena cava c. Pulmonary artery b. Pulmonary veins d. Coronary veins Only the four pulmonary veins, two from the right lung and two from the left lung, carry oxygenated blood from the lungs to the left side of the heart. PTS: 1 REF: Page 1088 8. The significance of the atrial kick is that it affects the contraction of the: a. Right atria, which is necessary to open the tricuspid valve. b. Right atria, which is necessary to increase the blood volume from the vena cava. c. Left atria, which increases the blood volume into the ventricle. d. Left atria, that is necessary to open the mitral valve. Left atrial contraction, the atrial kick, provides a significant increase of blood to the left ventricle. PTS: 1 REF: Page 1088 9. Occlusion of the left anterior descending artery during a myocardial infarction would interrupt blood supply to which structures? a. Left and right ventricles and much of the interventricular septum b. Left atrium and the lateral wall of the left ventricle c. Upper right ventricle, right marginal branch, and right ventricle to the apex d. Posterior interventricular sulcus and the smaller branches of both ventricles The left anterior descending artery (LAD), also called the anterior interventricular artery, delivers blood to portions of the left and right ventricles and much of the interventricular septum. This selection is the only option affected by the occlusion described. PTS: 1 REF: Page 1090 10. Occlusion of the circumflex artery during a myocardial infarction would interrupt blood supply to which area? a. Left and right ventricles and much of the interventricular septum b. Posterior interventricular sulcus and the smaller branches of both ventricles c. Upper right ventricle, right marginal branch, and right ventricle to the apex d. Left atrium and the lateral wall of the left ventricle The circumflex artery supplies blood to the left atrium and the lateral wall of the left ventricle. The circumflex artery often branches to the posterior surfaces of the left atrium and left ventricle. This selection is the only option affected by the occlusion described. PTS: 1 REF: Pages 1090-1091 11. The coronary ostia are located in the: a. Left ventricle c. Coronary sinus b. Aortic valve d. Aorta Coronary arteries receive blood through openings in the aorta, called the coronary ostia. PTS: 1 REF: Page 1090 12. The coronary sinus empties into which cardiac structure? a. Right atrium c. Superior vena cava b. Left atrium d. Aorta The cardiac veins empty only into the right atrium through another ostium, the opening of a large vein called the coronary sinus. PTS: 1 REF: Page 1090 13. What is the ratio of coronary capillaries to cardiac muscle cells? a. 1:1 (one capillary per one muscle cell) b. 1:2 (one capillary per two muscle cells) c. 1:4 (one capillary per four muscle cells) d. 1:10 (one capillary per ten muscle cells) The heart has an extensive capillary network, with approximately 3300 capillaries per square millimeter (ca/mm2) or approximately one capillary per one muscle cell (muscle fiber). PTS: 1 REF: Page 1092 14. During the cardiac cycle, which structure directly delivers action potential to the ventricular myocardium? a. Sinoatrial (SA) node c. Purkinje fibers b. Atrioventricular (AV) node d. Bundle branches Each cardiac action potential travels from the SA node to the AV node to the bundle of His (AV bundle), through the bundle branches, and finally to the Purkinje fibers and the ventricular myocardium, where the impulse is stopped. The refractory period of cells that have just been polarized prevents the impulse from reversing its path. The refractory period ensures that diastole (relaxation) will occur, thereby completing the cardiac cycle. This selection is the only option that accurately describes the structure that delivers the action potential directly to the myocardium. PTS: 1 REF: Page 1124 15. What causes depolarization of a cardiac muscle cell to occur? a. Decrease in the permeability of the cell membrane to potassium b. Rapid movement of sodium into the cell c. Decrease in the movement of sodium out of the cell d. Rapid movement of calcium out of the cell Phase 0 consists of depolarization, which lasts 1 to 2 milliseconds (ms) and represents rapid sodium entry into the cell. This selection is the only option that accurately describes the cause of cardiac muscle cell depolarization. PTS: 1 REF: Page 1094 16. Which event occurs during phase 1 of the normal myocardial cell depolarization and repolarization? a. Repolarization when potassium moves out of the cells b. Repolarization when sodium rapidly enters into the cells c. Early repolarization when sodium slowly enters the cells d. Early repolarization when calcium slowly enters the cells Phase 1 is early repolarization and the only time during which calcium slowly enters the cell. PTS: 1 REF: Page 1094 17. Which phase of the normal myocardial cell depolarization and repolarization correlates with diastole? a. Phase 1 c. Phase 3 b. Phase 2 d. Phase 4 Potassium is moved out of the cell during phase 3, with a return to resting membrane potential only in phase 4. The time between action potentials corresponds to diastole. PTS: 1 REF: Page 1094 18. In the normal electrocardiogram, what does the PR interval represent? a. Atrial depolarization b. Ventricular depolarization c. Atrial activation to onset of ventricular activity d. Electrical systole of the ventricles The PR interval is a measure of time from the onset of atrial activation to the onset of ventricular activation; it normally ranges from 0.12 to 0.20 second. The PR interval represents the time necessary to travel from the sinus node through the atrium, the atrioventricular (AV) node, and the His–Purkinje system to activate ventricular myocardial cells. This selection is the only option that accurately describes the PR interval. PTS: 1 REF: Page 1095 19. The cardiac electrical impulse normally begins spontaneously in the sinoatrial (SA) node because it: a. Has a superior location in the right atrium. b. Is the only area of the heart capable of spontaneous depolarization. c. Has rich sympathetic innervation via the vagus nerve. d. Depolarizes more rapidly than other automatic cells of the heart. The electrical impulse normally begins in the SA node because its cells depolarize more rapidly than other automatic cells. This selection is the only option that accurately explains why cardiac electrical impulses normally begin spontaneously in the SA node. PTS: 1 REF: Page 1095 20. What period follows depolarization of the myocardium and represents a period during which no new cardiac potential can be propagated? a. Refractory c. Threshold b. Hyperpolarization d. Sinoatrial (SA) During the refractory period, no new cardiac action potential can be initiated by a stimulus. This selection is the only option that accurately identifies the period described in the question. PTS: 1 REF: Page 1095 21. Which complex (wave) represents the sum of all ventricular muscle cell depolarizations? a. PRS c. QT interval b. QRS d. P Only the QRS complex represents the sum of all ventricular muscle cell depolarizations. PTS: 1 REF: Page 1095 22. What can shorten the conduction time of action potential through the atrioventricular (AV) node? a. Parasympathetic nervous system c. Vagal stimulation b. Catecholamines d. Sinoatrial node (SA) Catecholamines speed the heart rate, shorten the conduction time through the AV node, and increase the rhythmicity of the AV pacemaker fibers. This selection is the only option that can perform that function. PTS: 1 REF: Page 1096 23. If the sinoatrial (SA) node fails, then at what rate (depolarizations per minute) can the atrioventricular (AV) node depolarize? a. 60 to 70 c. 30 to 40 b. 40 to 60 d. 10 to 20 If the SA node is damaged, then the AV node will become the heart’s pacemaker at a rate of approximately 40 to 60 spontaneous depolarizations per minute. PTS: 1 REF: Page 1095 24. What is the effect of epinephrine on 3 receptors on the heart? a. Decreases coronary blood flow. b. Supplements the effects of both 1 and 2 receptors. c. Increases the strength of myocardial contraction. d. Prevents overstimulation of the heart by the sympathetic nervous system. 3 receptors are found in the myocardium and coronary vessels. In the heart, stimulation of these receptors opposes the effects of 1- and 2-receptor stimulation and negative inotropic effect. Thus 3 receptors may provide a safety mechanism that decreases myocardial contractility to prevent overstimulation of the heart by the sympathetic nervous system. This selection is the only option that accurately describes the effect of epinephrine on 2 receptors on the heart. PTS: 1 REF: Page 1097 25. Where in the heart are the receptors for neurotransmitters located? a. Semilunar and atrioventricular (AV) valves b. Endocardium and sinoatrial (SA) node c. Myocardium and coronary vessels d. Epicardium and AV node Sympathetic neural stimulation of the myocardium and coronary vessels depends on the presence of adrenergic receptors, which specifically bind with neurotransmitters of the sympathetic nervous system. The 1 receptors are found mostly in the heart, specifically the conduction system (AV and SA nodes, Purkinje fibers) and the atrial and ventricular myocardium, whereas the 2 receptors are found in the heart and also on vascular smooth muscle. 3 receptors are also found in the myocardium and coronary vessels. This selection is the only option that accurately identifies the location of the receptors for neurotransmitters. PTS: 1 REF: Page 1097 26. What enables electrical impulses to travel in a continuous cell-to-cell fashion in myocardial cells? a. Sarcolemma sclerotic plaques c. Trabeculae carneae b. Intercalated disks d. Bachmann bundles Only intercalated disks, thickened portions of the sarcolemma, enable electrical impulses to spread quickly in a continuous cell-to-cell (syncytial) fashion. PTS: 1 REF: Page 1097 27. Within a physiologic range, what does an increase in left ventricular end-diastolic volume (preload) result in? a. Increase in force of contraction c. Increase in afterload b. Decrease in refractory time d. Decrease in repolarization This concept is expressed in the Frank-Starling law; the cardiac muscle, like other muscles, increases its strength of contraction when it is stretched. This selection is the only option that accurately describes the result of an increase in preload. PTS: 1 REF: Pages 1101-1102 28. As stated in the Frank-Starling law, a direct relationship exists between the of the blood in the heart at the end of diastole and the of contraction during the next systole. a. Pressure; force c. Viscosity; force b. Volume; strength d. Viscosity; strength As stated in the Frank-Starling law, the volume of blood in the heart at the end of diastole (the length of its muscle fibers) is directly related to the force (strength) of contraction during the next systole. This selection is the only option that accurately describes the relationship associated with the Frank-Starling law. PTS: 1 REF: Pages 1101-1102 29. Pressure in the left ventricle must exceed pressure in which structure before the left ventricle can eject blood? a. Superior vena cava c. Inferior vena cava b. Aorta d. Pulmonary veins Pressure in the ventricle must exceed aortic pressure before blood can be pumped out during systole. The aorta is the only structure in which pressure must be less than the amount of blood in the left ventricle for ejection to occur. PTS: 1 REF: Page 1103 30. Continuous increases in left ventricular filing pressures result in which disorder? a. Mitral regurgitation c. Pulmonary edema b. Mitral stenosis d. Jugular vein distention Pressure changes are important because increased left ventricular filling pressures back up into the pulmonary circulation, where they force plasma out through vessel walls, causing fluid to accumulate in lung tissues (pulmonary edema). This selection is the only option that accurately identifies the disorder described in the question. PTS: 1 REF: Page 1103 31. When the volume of blood in the ventricle at the end of diastole increases, the force of the myocardial contraction during the next systole will also increase, which is an example of which law or theory about the heart? a. Laplace’s law c. Cross-bridge theory b. Poiseuille law d. Frank-Starling law This concept is expressed only in the Frank-Starling law; the cardiac muscle, like other muscles, increases its strength of contraction when it is stretched. PTS: 1 REF: Page 1101 32. The resting heart rate in a healthy person is primarily under the control of which nervous system? a. Sympathetic c. Somatic b. Parasympathetic d. Spinal The resting heart rate in healthy individuals is primarily under the control of parasympathetic stimulation. This selection is the only option that accurately identifies the nervous system responsible for the healthy resting heart. PTS: 1 REF: Page 1106 33. The Bainbridge reflex is thought to be initiated by sensory neurons in which cardiac location? a. Atria c. Sinoatrial (SA) node b. Aorta d. Ventricles The Bainbridge reflex causes changes in the heart rate after intravenous infusions of blood or other fluid. The changes in heart rate are thought to be caused by a reflex mediated by volume receptors found only in the atria that are innervated by the vagus nerve. PTS: 1 REF: Page 1106 34. After the baroreceptor reflex is stimulated, the resulting impulse is transmitted from the carotid artery by which sequence of events? a. Vagus nerve to the medulla to increase parasympathetic activity and to decrease sympathetic activity b. Glossopharyngeal cranial nerve through the vagus nerve to the medulla to increase sympathetic activity and to decrease parasympathetic activity c. Glossopharyngeal cranial nerve through the vagus nerve to the medulla to increase parasympathetic activity and to decrease sympathetic activity d. Glossopharyngeal cranial nerve through the vagus nerve to the hypothalamus to increase parasympathetic activity and to decrease sympathetic activity Neural impulses are transmitted over the glossopharyngeal nerve (ninth cranial nerve) from the carotid artery and through the vagus nerve from the aorta to the cardiovascular control centers in the medulla. These centers initiate an increase in parasympathetic activity and a decrease in sympathetic activity, causing blood vessels to dilate and the heart rate to decrease. This selection is the only option that accurately describes the correct sequence of events asked for in the question. PTS: 1 REF: Page 1106 35. Reflex control of total cardiac output and total peripheral resistance is controlled by what mechanism? a. Parasympathetic stimulation of the heart, arterioles, and veins b. Sympathetic stimulation of the heart, arterioles, and veins c. Autonomic control of the heart only d. Somatic control of the heart, arterioles, and veins Reflex control of total cardiac output and peripheral resistance includes (1) sympathetic stimulation of the heart, arterioles, and veins; and (2) parasympathetic stimulation of the heart only. Neither autonomic nor somatic controls are involved in this process. PTS: 1 REF: Pages 1114-1115 36. What is the most important negative inotropic agent? a. Norepinephrine c. Acetylcholine b. Epinephrine d. Dopamine Chemicals affecting contractility are called inotropic agents. The most important negative inotropic agent is acetylcholine released from the vagus nerve. The most important positive inotropic agents produced by the body are norepinephrine released from the sympathetic nerves that supply the heart and epinephrine released by the adrenal cortex. Other positive inotropes include thyroid hormone and dopamine. Many medications have positive or negative inotropic properties that can have profound effects on cardiac function. This selection is the only option that accurately identifies the regulation that is involved in the described process. PTS: 1 REF: Page 1103 37. The right lymphatic duct drains into which structure? a. Right subclavian artery c. Right subclavian vein b. Right atrium d. Superior vena cava The right lymphatic duct drains lymph only into the right subclavian vein. PTS: 1 REF: Page 1118 38. Where is the major cardiovascular center in the central nervous system? a. Frontal lobe c. Brainstem b. Thalamus d. Hypothalamus The major cardiovascular control center is in the brainstem in the medulla with secondary areas in the hypothalamus, the cerebral cortex, the thalamus, and the complex networks of exciting or inhibiting interneurons (connecting neurons) throughout the brain. This selection is the only option that accurately identifies the cardiovascular control center. PTS: 1 REF: Page 1104 39. What is an expected change in the cardiovascular system that occurs with aging? a. Arterial stiffening b. Decreased left ventricular wall tension c. Decreased aortic wall thickness d. Arteriosclerosis Arterial stiffening occurs with aging even in the absence of clinical hypertension. Aging is not responsible for the other conditions. PTS: 1 REF: Page 1123 40. What is the major determinant of the resistance that blood encounters as it flows through the systemic circulation? a. Volume of blood in the systemic circulation b. Muscle layer of the metarterioles c. Muscle layer of the arterioles d. Force of ventricular contraction Of the options available, only the thick, smooth muscle layer of the arterioles is a major determinant of the resistance blood encounters as it flows through the systemic circulation. PTS: 1 REF: Page 1108 41. Which function of the cardiovascular system is often affected by ischemia? a. Cardiac output (CO) c. Heart rate (HR) b. Stroke volume (SV) d. Cardiac index (CI) Common causes of an abnormal heart rate include ischemia, electrolyte imbalance, and drug toxicity. The other options are related to vascular resistance changes. PTS: 1 REF: Page 1120 | Table 31-4 42. What physical sign is the result of turbulent blood flow through a vessel? a. Increased blood pressure during periods of stress b. Bounding pulse felt on palpation c. Cyanosis observed on excretion d. Murmur heard on auscultation Where flow is obstructed, the vessel turns or blood flows over rough surfaces. The flow becomes turbulent with whorls or eddy currents that produce noise, causing a murmur to be heard on auscultation, such as occurs during blood pressure measurement with a sphygomanometer. This selection is the only option that accurately identifies the physical sign of turbulent vascular blood flow. PTS: 1 REF: Page 1113 43. What is the major effect of a calcium channel blocker such as verapamil on cardiac contractions? a. Increases the rate of cardiac contractions. b. Decreases the strength of cardiac contractions. c. Stabilizes the rhythm of cardiac contractions. d. Stabilizes the vasodilation during cardiac contractions. The L-type, or long-lasting, channels are the predominant type of calcium channels and are the channels blocked by calcium channel–blocking drugs (verapamil, nifedipine, diltiazem). The major effect of these medications is to decrease the strength of cardiac contraction. This selection is the only option that accurately identifies the effect of a calcium channel blocker on the cardiac contractions. PTS: 1 REF: Page 1099 44. An early diastole peak caused by filling of the atrium from peripheral veins is identified by which intracardiac pressure? a. A wave c. C wave b. V wave d. X descent The V wave is an early diastolic peak caused by the filling of the atrium from the peripheral veins. This event is not identified by any of the other options. PTS: 1 REF: Page 1089 45. Which intracardiac pressure is generated by the atrial contraction? a. A wave c. Y descent b. C wave d. X descent Atrial pressure curves are made up of only the A wave, which is generated by atrial contraction. PTS: 1 REF: Page 1089 46. Which intracardiac pressure is produced because of the descent of the tricuspid valve ring and by the ejection of blood from both ventricles? a. V wave c. Y descent b. C wave d. X descent The X descent follows an A wave and is produced because of the descent of the tricuspid valve ring and by the ejection of blood from both ventricles. PTS: 1 REF: Page 1089 MULTIPLE RESPONSE 47. Which statements are true concerning the method in which substances pass between capillaries and the interstitial fluid? (Select all that apply.) a. Substances pass through junctions between endothelial cells. b. Substances pass through pores or oval windows (fenestrations). c. Substances pass between vesicles by active transport across the endothelial cell membrane. d. Substances pass across the endothelial cell membrane by osmosis. e. Substances pass through endothelial cell membranes by diffusion. , B, C, E Substances pass between the capillary lumen and the interstitial fluid in several ways: (1) through junctions between endothelial cells, (2) through fenestrations in endothelial cells, (3) in vesicles moved by active transport across the endothelial cell membrane, or (4) by diffusion through the endothelial cell membrane. PTS: 1 REF: Page 1108 MATCHING Match the description with the corresponding terms. A. Relationship among blood flow, pressure, and resistance B. Increased heart rate from increased volume C. Relationship of wall tension, intraventricular pressure, internal radius, and wall thickness D. Cycles of attachment, movement, and dissociation of thin filaments during the attachments of actin to myosin E. Length-tension relationship of cardiac muscle 48. Poiseuille law 49. Cross-bridge theory 50. Frank-Starling law 51. Laplace’s law 52. Bainbridge reflex 48. PTS: 1 REF: Pages 1111-1112 MSC: Poiseuille law for resistance to fluid flow through a tube takes into account the length of the tube, the viscosity of the fluid, and the radius of the tube's lumen. 49. PTS: 1 REF: Page 1099 MSC: With the attachment of actin to myosin at the cross-bridge, the myosin head molecule undergoes a position change, exerting traction on the rest of the myosin bridge, causing the thin filaments to slide past the thick filaments. During contraction, each cross-bridge undergoes cycles of attachment, movement, and dissociation from the thin filaments. 50. ANS: E PTS: 1 REF: Page 1101 MSC: The Frank-Starling law states that the cardiac muscle, like other muscles, increases its strength of contraction when it is stretched. 51. PTS: 1 REF: Pages 1102-1103 MSC: In Laplace's law, wall tension is directly related to the product of intraventricular pressure and internal radius and inversely to the wall thickness. 52. PTS: 1 REF: Page 1106 MSC: The Bainbridge reflex causes changes in the heart rate after intravenous infusions of blood or other fluid. Chapter 32: Alterations of Cardiovascular Function MULTIPLE CHOICE 1. What is the initiating event that leads to the development of atherosclerosis? a. Release of the inflammatory cytokines b. Macrophages adhere to vessel walls. c. Injury to the endothelial cells that line the artery walls d. Release of the platelet-deprived growth factor Atherosclerosis begins with an injury to the endothelial cells that line the arterial walls. Possible causes of endothelial injury include the common risk factors for atherosclerosis, such as smoking, hypertension, diabetes, increased levels of low-density lipoprotein (LDL), decreased levels of high-density lipoprotein (HDL), and autoimmunity. The remaining options occur only after the endothelial cells are injured. PTS: 1 REF: Page 1145 2. What is the effect of oxidized low-density lipoproteins (LDLs) in atherosclerosis? a. LDLs cause smooth muscle proliferation. b. LDLs cause regression of atherosclerotic plaques. c. LDLs increase levels of inflammatory cytokines. d. LDLs direct macrophages to the site in the endothelium. Oxidized LDLs are toxic to endothelial cells, cause smooth muscle proliferation, and activate further immune and inflammatory responses. This selection is the only option that accurately identifies the effects of LDLs. PTS: 1 REF: Page 1145 3. Which inflammatory cytokines are released when endothelial cells are injured? a. Granulocyte-macrophage colony-stimulating factor (GM-CSF) b. Interferon-beta (IFN-), interleukin 6 (IL-6), and granulocyte colony-stimulating factor (G-CSF) c. Tumor necrosis factor–alpha (TNF-), interferon-gamma (IFN-), and interleukin 1 (IL-1) d. Interferon-alpha (IFN-), interleukin-12 (IL-12), and macrophage colony-stimulating factor (M-CSF) Numerous inflammatory cytokines are released, including TNF-, IFN-, IL-1, toxic oxygen radicals, and heat shock proteins. This selection is the only option that accurately identifies which inflammatory cytokines are associated with endothelial cell injury. PTS: 1 REF: Page 1145 4. When endothelia cells are injured, what alteration contributes to atherosclerosis? a. The release of toxic oxygen radicals that oxidize low-density lipoproteins (LDLs). b. Cells are unable to make the normal amount of vasodilating cytokines. c. Cells produce an increased amount of antithrombotic cytokines. d. Cells develop a hypersensitivity to homocysteine and lipids. Injured endothelial cells become inflamed and cannot make normal amounts of antithrombotic and vasodilating cytokines. This selection is the only option that accurately identifies the factor that contributes to atherosclerosis. PTS: 1 REF: Page 1145 5. Which factor is responsible for the hypertrophy of the myocardium associated with hypertension? a. Increased norepinephrine c. Angiotensin II b. Adducin d. Insulin resistance Of the available options, only angiotensin II is responsible for the hypertrophy of the myocardium and much of the renal damage associated with hypertension. PTS: 1 REF: Pages 1132-1138 6. What pathologic change occurs to the kidney’s glomeruli as a result of hypertension? a. Compression of the renal tubules b. Ischemia of the tubule c. Increased pressure from within the tubule d. Obstruction of the renal tubule In the kidney, vasoconstriction and resultant decreased renal perfusion cause tubular ischemia and preglomerular arteriopathy. This selection is the only option that accurately identifies the pathologic change to the kidney that occurs as a result of hypertension. PTS: 1 REF: Pages 1134-1136 7. What effect does atherosclerosis have on the development of an aneurysm? a. Atherosclerosis causes ischemia of the intima. b. It increases nitric oxide. c. Atherosclerosis erodes the vessel wall. d. It obstructs the vessel. Atherosclerosis is a common cause of aneurysms because plaque formation erodes the vessel wall. This selection is the only option that accurately identifies the effect that atherosclerosis has on aneurysm development. PTS: 1 REF: Pages 1141-1142 8. Regarding the endothelium, what is the difference between healthy vessel walls and those that promote clot formation? a. Inflammation and roughening of the endothelium of the artery are present. b. Hypertrophy and vasoconstriction of the endothelium of the artery are present. c. Excessive clot formation and lipid accumulation in the endothelium of the artery are present. d. Evidence of age-related changes that weaken the endothelium of the artery are present. Invasion of the tunica intima by an infectious agent also roughens the normally smooth lining of the artery, causing platelets to adhere readily. This selection is the only option that accurately describes the mechanism that supports abnormal clot formation. PTS: 1 REF: Pages 1142-1143 9. What is the usual source of pulmonary emboli? a. Deep venous thrombosis c. Valvular disease b. Endocarditis d. Left heart failure Pulmonary emboli originate in the venous circulation (mostly from the deep veins of the legs) or in the right heart. This selection is the only option that accurately identifies the usual source of pulmonary emboli. PTS: 1 REF: Page 1143 10. Which factor can trigger an immune response in the bloodstream that may result in an embolus? a. Amniotic fluid c. Bacteria b. Fat d. Air Of the options available, only amniotic fluid displaces blood, thereby reducing oxygen, nutrients, and waste exchange; however, it also introduces antigens, cells, and protein aggregates that trigger inflammation, coagulation, and the immune response in the bloodstream. PTS: 1 REF: Pages 1143-1144 11. Which statement best describes thromboangiitis obliterans (Buerger disease)? a. Inflammatory disorder of small- and medium-size arteries in the feet and sometimes in the hands b. Vasospastic disorder of the small arteries and arterioles of the fingers and, less commonly, of the toes c. Autoimmune disorder of the large arteries and veins of the upper and lower extremities d. Neoplastic disorder of the lining of the arteries and veins of the upper extremities Buerger disease is an inflammatory disease of the peripheral arteries. Inflammation, thrombus formation, and vasospasm can eventually occlude and obliterate portions of small- and medium-size arteries. The digital, tibial, and plantar arteries of the feet and the digital, palmar, and ulnar arteries of the hands are typically affected. This selection is the only option that accurately describes Buerger disease. PTS: 1 REF: Page 1144 12. Which statement best describes Raynaud disease? a. Inflammatory disorder of small- and medium-size arteries in the feet and sometimes in the hands b. Neoplastic disorder of the lining of the arteries and veins of the upper extremities c. Vasospastic disorder of the small arteries and arterioles of the fingers and, less commonly, of the toes d. Autoimmune disorder of the large arteries and veins of the upper and lower extremities Attacks of vasospasm in the small arteries and arterioles of the fingers and, less commonly, of the toes characterize Raynaud phenomenon and Raynaud disease and is the only option that accurately describes this disease. PTS: 1 REF: Page 1144 13. What change in a vein supports the development of varicose veins? a. Increase in osmotic pressure c. Damage to the venous endothelium b. Damage to the valves in veins d. Increase in hydrostatic pressure If a valve is damaged, permitting backflow, then a section of the vein is subjected to the pressure exerted by a larger volume of blood under the influence of gravity. The vein swells as it becomes engorged, and the surrounding tissue becomes edematous because increased hydrostatic pressure pushes plasma through the stretched vessel wall. This selection is the only option that accurately describes the development of varicose veins. PTS: 1 REF: Pages 1129-1130 14. Superior vena cava syndrome is a result of a progressive increase of which process? a. Inflammation c. Distention b. Occlusion d. Sclerosis Superior vena cava syndrome (SVCS) is a progressive occlusion of the superior vena cava (SVC) that leads to venous distention in the upper extremities and head. The remaining options are not associated with this disorder. PTS: 1 REF: Page 1131 15. What term is used to identify when a cell is temporarily deprived of blood supply? a. Infarction c. Necrosis b. Ischemia d. Inflammation Coronary artery disease (CAD) can diminish the myocardial blood supply until deprivation impairs myocardial metabolism enough to cause ischemia, a local state in which the cells are temporarily deprived of blood supply. This term is the only option that is used to identify a temporarily deprived blood supply. PTS: 1 REF: Page 1148 16. The risk of developing coronary artery disease is increased up to threefold by which factor? a. Diabetes mellitus c. Obesity b. Hypertension d. High alcohol consumption Hypertension is the only factor responsible for a twofold-to-threefold increased risk of atherosclerotic cardiovascular disease. PTS: 1 REF: Page 1151 17. Which risk factor is associated with coronary artery disease (CAD) because of its relationship with the alteration of hepatic lipoprotein? a. Diabetes mellitus c. Obesity b. Hypertension d. High alcohol consumption Of the available options, only diabetes mellitus is associated with CAD because of the resulting alteration of hepatic lipoprotein synthesis; it increases triglyceride levels and is involved in low-density lipoprotein oxidation. PTS: 1 REF: Pages 1148-1151 18. Nicotine increases atherosclerosis by the release of which neurotransmitter? a. Histamine c. Angiotensin II b. Nitric oxide d. Epinephrine Nicotine stimulates the release of catecholamines (e.g., epinephrine, norepinephrine), which increases the heart rate and causes peripheral vascular constriction. As a result, blood pressure increases, as do both cardiac workload and oxygen demand. None of the other options are associated with this mechanism. PTS: 1 REF: Page 1151 19. Which substance is manufactured by the liver and primarily contains cholesterol and protein? a. Very low–density lipoproteins (VLDLs) b. Low-density lipoproteins (LDLs) c. High-density lipoproteins (HDLs) d. Triglycerides A series of chemical reactions in the liver results in the production of several lipoproteins that vary in density and function. These include VLDLs, primarily triglycerides and protein; LDLs, mostly cholesterol and protein; and HDLs, mainly phospholipids and protein. LDLs are the only lipoproteins that are manufactured by the liver and primarily contain cholesterol and protein. PTS: 1 REF: Page 1149 20. Which elevated value may be protective of the development of atherosclerosis? a. Very low–density lipoproteins (VLDLs) b. Low-density lipoproteins (LDLs) c. High-density lipoproteins (HDLs d. Triglycerides Low levels of HDL cholesterol are also a strong indicator of coronary risk, whereas high levels of HDLs may be more protective for the development of atherosclerosis than low levels of LDLs. Neither VLDLs nor elevated triglycerides are associated with a protective mechanism. PTS: 1 REF: Pages 1149-1151 21. Which laboratory test is an indirect measure of atherosclerotic plaque? a. Homocysteine b. Low-density lipoprotein (LDL) c. Erythrocyte sedimentation rate (ESR) d. C-reactive protein (CRP) Highly sensitive CRP (hs-CRP) is an acute phase reactant or protein mostly synthesized in the liver and, of the available options, is an indirect measure of atherosclerotic plaque-related inflammation. PTS: 1 REF: Page 1152 22. Cardiac cells can withstand ischemic conditions and still return to a viable state for how many minutes? a. 10 c. 20 b. 15 d. 25 Cardiac cells remain viable for approximately 20 minutes under ischemic conditions. If blood flow is restored, then aerobic metabolism resumes, contractility is restored, and cellular repair begins. If the coronary artery occlusion persists beyond 20 minutes, then myocardial infarction (MI) occurs. PTS: 1 REF: Page 1153 23. Which form of angina occurs most often during sleep as a result of vasospasms of one or more coronary arteries? a. Unstable c. Silent b. Stable d. Prinzmetal Of the options available, only Prinzmetal angina (also called variant angina) is chest pain attributable to transient ischemia of the myocardium that occurs unpredictably and almost exclusively at rest. PTS: 1 REF: Page 1154 24. When is the scar tissue that is formed after a myocardial infarction (MI) most vulnerable to injury? a. Between 5 and 9 days c. Between 15 and 20 days b. Between 10 and 14 days d. Between 20 and 30 days During the recovery period (10 to 14 days after infarction), individuals feel more capable of increasing activities and thus may stress the newly formed scar tissue. After 6 weeks, the necrotic area is completely replaced by scar tissue, which is strong but unable to contract and relax like healthy myocardial tissue. PTS: 1 REF: Page 1160 25. An individual who is demonstrating elevated levels of troponin, creatine kinase–isoenzyme MB (CK-MB), and lactic dehydrogenase (LDH) is exhibiting indicators associated with which condition? a. Myocardial ischemia c. Myocardial infarction (MI) b. Hypertension d. Coronary artery disease (CAD) Cardiac troponins (troponin I and troponin T) are the most specific indicators of MI. Other biomarkers released by myocardial cells include CK-MB and LDH, but they are not associated with the other options. PTS: 1 REF: Pages 1160-1161 26. What is the expected electrocardiogram (ECG) pattern when a thrombus in a coronary artery permanently lodges in the vessel and the infarction extends through the myocardium from the endocardium to the epicardium? a. Prolonged QT interval b. ST elevation myocardial infarction (STEMI) c. ST depression myocardial infarction (STDMI) d. Non-ST elevation myocardial infarction (non-STEMI) Individuals with this pattern on an ECG usually have significant elevations in the ST segments and are categorized as having STEMI. The other options are not associated with the described pathologic condition. PTS: 1 REF: Pages 1157-1158 27. How does angiotensin II increase the workload of the heart after a myocardial infarction (MI)? a. By increasing the peripheral vasoconstriction b. By causing dysrhythmias as a result of hyperkalemia c. By reducing the contractility of the myocardium d. By stimulating the sympathetic nervous system Angiotensin II is released during myocardial ischemia and contributes to the pathogenesis of a myocardial infarction (MI) in several ways. First, it results in the systemic effects of peripheral vasoconstriction and fluid retention. These homeostatic responses are counterproductive in that they increase myocardial work and thus exacerbate the effects of the loss of myocyte contractility. Angiotensin II is also locally released, where it is a growth factor for vascular smooth muscle cells, myocytes, and cardiac fibroblasts; promotes catecholamine release; and causes coronary artery spasm. This selection is the only option that accurately describes how angiotensin II increases workload after a MI. PTS: 1 REF: Page 1159 28. The pulsus paradoxus that occurs as a result of pericardial effusion is caused by a dysfunction in which mechanism? a. Diastolic filling pressures of the right ventricle and reduction of blood volume in both ventricles b. Blood ejected from the right atrium and reduction of blood volume in the right ventricle c. Blood ejected from the left atrium and reduction of blood volume in the left ventricle d. Diastolic filling pressures of the left ventricle and reduction of blood volume in all four heart chambers. Pulsus paradoxus means that the arterial blood pressure during expiration exceeds arterial pressure during inspiration by more than 10 mm Hg. This clinical finding reflects impairment of diastolic filling of the left ventricle plus a reduction of blood volume within all four cardiac chambers. This selection is the only option that accurately describes the mechanism. PTS: 1 REF: Page 1164 29. A patient reports sudden onset of severe chest pain that radiates to the back and worsens with respiratory movement and when lying down. These clinical manifestations describe: a. Myocardial infarction (MI) c. Restrictive pericarditis b. Pericardial effusion d. Acute pericarditis Most individuals with acute pericarditis describe several days of fever, myalgias, and malaise, followed by the sudden onset of severe chest pain that worsens with respiratory movements and with lying down. Although the pain may radiate to the back, it is generally felt in the anterior chest and may be initially confused with the pain of an acute MI. Individuals with acute pericarditis also may report dysphagia, restlessness, irritability, anxiety, and weakness. This selection is the only option with these symptoms. PTS: 1 REF: Page 1163 30. Ventricular dilation and grossly impaired systolic function, leading to dilated heart failure, characterize which form of cardiomyopathy? a. Congestive c. Septal b. Hypertrophic d. Dystrophic Only dilated cardiomyopathy (congestive cardiomyopathy) is characterized by ventricular dilation and grossly impaired systolic function, leading to dilated heart failure. PTS: 1 REF: Page 1165 31. A disproportionate thickening of the interventricular septum is the hallmark of which form of cardiomyopathy? a. Dystrophic c. Restrictive b. Hypertrophic d. Dilated Only hypertrophic cardiomyopathy is characterized by a thickening of the septal wall, which may cause outflow obstruction to the left ventricle outflow tract. PTS: 1 REF: Page 1166 32. Amyloidosis, hemochromatosis, or glycogen storage disease usually causes which form of cardiomyopathy? a. Infiltrative c. Septal b. Restrictive d. Hypertrophic Restrictive cardiomyopathy may occur idiopathically or as a cardiac manifestation of systemic diseases, such as scleroderma, amyloidosis, sarcoidosis, lymphoma, and hemochromatosis, or a number of inherited storage diseases. This characterization is not true of the other forms of cardiomyopathy. PTS: 1 REF: Page 1167 33. Which condition is a cause of acquired aortic regurgitation? a. Congenital malformation c. Rheumatic fever b. Cardiac failure d. Coronary artery disease (CAD) Rheumatic heart disease, bacterial endocarditis, syphilis, hypertension, connective tissue disorders (e.g., Marfan syndrome, ankylosing spondylitis), appetite suppressing medications, trauma, or atherosclerosis can cause acquired aortic regurgitation. This selection is the only available option that is known to cause acquired aortic regurgitation. PTS: 1 REF: Page 1169 34. Which predominantly female valvular disorder is thought to have an autosomal dominant inheritance pattern, as well as being associated with connective tissue disease? a. Mitral valve prolapse c. Tricuspid valve prolapse b. Tricuspid stenosis d. Aortic insufficiency Mitral valve prolapse tends to be most prevalent in young women. Studies suggest an autosomal dominant and X-linked inheritance pattern. Because mitral valve prolapse often is associated with other inherited connective tissue disorders (e.g., Marfan syndrome, Ehlers-Danlos syndrome, osteogenesis imperfecta), it is thought to result from a genetic or environmental disruption of valvular development during the fifth or sixth week of gestation. This provided history is not associated with any of the other options. PTS: 1 REF: Page 1170 35. Which disorder causes a transitory truncal rash that is nonpruritic and pink with erythematous macules that may fade in the center, making them appear as a ringworm? a. Fat emboli b. Rheumatic fever c. Bacterial endocarditis d. Myocarditis of acquired immunodeficiency syndrome Erythema marginatum is a distinctive truncal rash that often accompanies acute rheumatic fever. It consists of nonpruritic, pink erythematous macules that never occur on the face or hands. This presentation is not associated with any of the other options. PTS: 1 REF: Page 1172 36. What is the most common cause of infective endocarditis? a. Virus c. Bacterium b. Fungus d. Rickettsiae Infective endocarditis is a general term used to describe infection and inflammation of the endocardium—especially the cardiac valves. Bacteria are the most common cause of infective endocarditis, especially streptococci, staphylococci, or enterococci. PTS: 1 REF: Page 1173 37. What is the most common cardiac disorder associated with acquired immunodeficiency syndrome (AIDS) a. Cardiomyopathy c. Left heart failure b. Myocarditis d. Heart block Pericardial effusion and left heart failure are the most common complications of human immunodeficiency virus (HIV) infection. Other conditions include cardiomyopathy, myocarditis, tuberculous pericarditis, infective and nonbacterial endocarditis, heart block, pulmonary hypertension, and nonantiretroviral drug-related cardiotoxicity. PTS: 1 REF: Page 1175 38. A patient is diagnosed with pulmonary disease and elevated pulmonary vascular resistance. Which form of heart failure may result from pulmonary disease and elevated pulmonary vascular resistance? a. Right heart failure c. Low-output failure b. Left heart failure d. High-output failure Right heart failure is defined as the inability of the right ventricle to provide adequate blood flow into the pulmonary circulation at a normal central venous pressure. This condition is often a result of pulmonary disease and the resulting elevated pulmonary vascular resistance. PTS: 1 REF: Page 1181 39. What cardiac pathologic condition contributes to ventricular remodeling? a. Left ventricular hypertrophy c. Myocardial ischemia b. Right ventricular failure d. Contractile dysfunction Of the options available, myocardial ischemia contributes to inflammatory, immune, and neurohumoral changes that mediate a process called ventricular remodeling. PTS: 1 REF: Page 1175 40. In systolic heart failure, what effect does the renin-angiotensin-aldosterone system (RAAS) have on stroke volume? a. Increases preload and decreases afterload. b. Increases preload and increases afterload. c. Decreases preload and increases afterload. d. Decreases preload and decreases afterload. Activation of the RAAS not only causes an increase in preload and afterload, but it also causes direct toxicity to the myocardium. This selection is the only option that accurately identifies the effect that the RAAS has on stroke volume in this situation. PTS: 1 REF: Page 1175 | Page 1177 41. What is the cause of the dyspnea resulting from a thoracic aneurysm? a. Pressure on surrounding organs c. Formation of atherosclerotic lesions b. Poor oxygenation d. Impaired blood flow Clinical manifestations depend on the location of the aneurysm. Pressure of a thoracic aneurysm on surrounding organs cause symptoms of dysphagia (difficulty in swallowing) and dyspnea (breathlessness). This selection is the only option that accurately describes the cause of dyspnea resulting from a thoracic aneurysm. PTS: 1 REF: Page 1142 42. Which statement is true concerning the cells’ ability to synthesize cholesterol? a. Cell production of cholesterol is affected by the aging process. b. Cells produce cholesterol only when dietary fat intake is low. c. Most body cells are capable of producing cholesterol. d. Most cholesterol produced by the cells is converted to the low-density form. Although cholesterol can easily be obtained from dietary fat intake, most body cells can also manufacture cholesterol. This selection is the only option that accurately describes the cellular role in cholesterol synthesis. PTS: 1 REF: Page 1149 43. What is the trigger for angina pectoris? a. Atherosclerotic lesions c. Myocardial necrosis b. Hyperlipidemia d. Myocardial ischemia Angina pectoris is chest pain caused by myocardial ischemia. None of the other options are considered triggers for angina pectoris. PTS: 1 REF: Page 1154 44. Individuals being effectively managed for type 2 diabetes mellitus often experience a healthy decline in blood pressure as a result of what intervention? a. Managed carbohydrate intake b. Appropriate exercise c. Insulin-sensitivity medication therapy d. Introduction of minimal doses of insulin Many people with type 2 diabetes mellitus, who are treated with drugs that increase insulin sensitivity, experience a decline in their blood pressure without taking antihypertensive drugs. Although the other medications may be included in the management plan, the other options are not associated with a decrease in hypertension. PTS: 1 REF: Page 1136 MULTIPLE RESPONSE 45. Which statements are true regarding fatty streaks? (Select all that apply.) a. Fatty streaks progressively damage vessel walls. b. Fatty streaks are capable of producing toxic oxygen radials. c. When present, inflammatory changes occur to the vessel walls. d. Oxidized low-density lipoproteins (LDLs) are involved in their formation. e. Fatty streaks are formed by killer T cells filled with oxidized LDLs. , B, C, D The oxidized LDLs penetrate the intima of the arterial wall and are engulfed by macrophages. Macrophages filled with oxidized LDLs are called foam. Once these lipid-laden foam cells accumulate in significant amounts, they form a lesion called a fatty streak. Once formed, fatty streaks produce more toxic oxygen radicals and cause immunologic and inflammatory changes, resulting in progressive damage to the vessel wall. PTS: 1 REF: Page 1145 | Page 1147 46. What factors contribute to the development of orthostatic hypotension? (Select all that apply.) a. Altered body chemistry b. Drug action of certain antihypertensive agents c. Prolonged immobility d. Effects of aging on postural reflexes e. Any condition that produces volume overload , B, C, D Orthostatic hypotension may be acute or chronic. Acute orthostatic hypotension (temporary type) may result from (1) altered body chemistry, (2) drug action (e.g., antihypertensives, antidepressants), (3) prolonged immobility caused by illness, (4) starvation, (5) physical exhaustion, (6) any condition that produces volume depletion (e.g., massive diuresis, potassium or sodium depletion), and (7) venous pooling (e.g., pregnancy, extensive varicosities of the lower extremities). Older adults are susceptible to this type of orthostatic hypotension, in which postural reflexes are slowed as part of the aging process. PTS: 1 REF: Page 1140 47. Which assessment findings are clinical manifestations of aortic stenosis? (Select all that apply.) a. Jugular vein distention b. Bounding pulses c. Hypotension d. Angina e. Syncope , E The classic manifestations of aortic stenosis are angina, syncope, and heart failure. None of the other options are associated with aortic stenosis. PTS: 1 REF: Pages 1168-1169 48. Which risk factors are associated with infective endocarditis? (Select all that apply.) a. Rheumatic fever b. Intravenous drug use c. Long-term indwelling catheterization d. Aortic regurgitation e. Heart valve disease , C, E Risk factors for infective endocarditis include acquired valvular heart disease, intravenous drug abuse, long-term indwelling catheterization (e.g., for pressure monitoring, hyperalimentation, or hemodialysis), and recent cardiac surgery. Neither rheumatic fever nor aortic regurgitation is considered a risk factor for infective endocarditis. PTS: 1 REF: Page 1173 | Box 32-3 MATCHING Match the descriptions with the corresponding terms. A. Impairs flow from left atrium to left ventricle B. Impairs flow from the left ventricle C. Backflow into left atrium D. Backflow into right atrium E. Backflow into left ventricle 49. Aortic stenosis 50. Aortic regurgitation 51. Mitral stenosis 52. Tricuspid regurgitation 53. Mitral regurgitation 49. PTS: 1 REF: Page 1168 MSC: Outflow obstruction increases pressure within the left ventricle as it tries to eject blood through the narrowed opening. Left ventricular hypertrophy develops to compensate for the increased workload. 50. ANS: E PTS: 1 REF: Pages 1169-1170 MSC: During systole, blood is ejected from the left ventricle into the aorta. If the aortic semilunar valve fails to close completely, then some of the ejected blood flows back into the left ventricle during diastole. 51. PTS: 1 REF: Page 1169 MSC: Mitral stenosis impairs the flow of blood from the left atrium to the left ventricle. 52. PTS: 1 REF: Page 1170 MSC: Tricuspid regurgitation is more common than tricuspid stenosis and is usually associated with cardiac failure and dilation of the right ventricle, secondary to pulmonary hypertension. 53. PTS: 1 REF: Page 1170 MSC: Mitral regurgitation permits the backflow of blood from the left ventricle into the left atrium during ventricular systole, giving rise to a loud pansystolic (throughout systole) murmur heard best at the apex that radiates into the back and axillae. Chapter 33: Alterations of Cardiovascular Function in Children MULTIPLE CHOICE 1. Most cardiovascular developments occur between which weeks of gestation? a. Fourth and seventh weeks c. Twelfth and fourteenth weeks b. Eighth and tenth weeks d. Fifteenth and seventeenth weeks Cardiogenesis begins at approximately 3 weeks’ gestation; however, most cardiovascular development occurs between 4 and 7 weeks’ gestation. PTS: 1 REF: Page 1194 2. The function of the foramen ovale in a fetus allows what to occur? a. Right-to-left blood shunting c. Blood flow from the umbilical cord b. Left-to-right blood shunting d. Blood flow to the lungs The nonfused septum secundum and ostium secundum result in the formation of a flapped orifice known as the foramen ovale, which allows the right-to-left shunting necessary for fetal circulation. The foramen ovale is not involved in the blood flow described by the other options. PTS: 1 REF: Pages 1195-1196 3. At birth, which statement is true? a. Systemic resistance and pulmonary resistance fall. b. Gas exchange shifts from the placenta to the lung. c. Systemic resistance falls and pulmonary resistance rises. d. Systemic resistance and pulmonary resistance rise. From the available options, the only change that takes place in the circulation at birth is the shift of gas exchange from the placenta to the lungs. PTS: 1 REF: Page 1197 4. When does systemic vascular resistance in infants begin to increase? a. One month before birth b. During the beginning stage of labor c. One hour after birth d. Once the placenta is removed from circulation The low-resistance placenta is removed from circulation, which causes an immediate increase in systemic vascular resistance to approximately twice of that before birth. PTS: 1 REF: Page 1197 5. Which event triggers congenital heart defects that cause acyanotic congestive heart failure? a. Right-to-left shunts c. Obstructive lesions b. Left-to-right shunts d. Mixed lesions Congenital heart defects that cause acyanotic congestive heart failure usually involve left-to-right shunts (see Table 33-4). Acyanotic congestive heart failure does not involve any of the other options. PTS: 1 REF: Pages 1201-1202 | Table 33-4 6. Older children with an unrepaired cardiac septal defect experience cyanosis because of which factor? a. Right-to-left shunts c. Obstructive lesions b. Left-to-right shunts d. Mixed lesions Older children who have an unrepaired septal defect with a left-to-right shunt may become cyanotic because of pulmonary vascular changes secondary to increased pulmonary blood flow. None of the other options accurately describe the process that results in cyanosis. PTS: 1 REF: Page 1202 7. Which congenital heart defects occur in trisomy 13, trisomy 18, and Down syndrome? a. Coarctation of the aorta (COA) and pulmonary stenosis (PS) b. Tetralogy of Fallot and persistent truncus arteriosus c. Atrial septal defect (ASD) and dextrocardia d. Ventricular septal defect (VSD) and patent ductus arteriosus (PDA) Congenital heart defects that are related to dysfunction of trisomy 13, trisomy 18, and Down syndrome include VSD and PDA (see Table 33-2). The other defects are not associated with dysfunction of trisomy 13, trisomy 18, and Down syndrome. PTS: 1 REF: Page 1200 | Table 33-2 8. An infant has a continuous machine-type murmur best heard at the left upper sternal border throughout systole and diastole, as well as a bounding pulse and a thrill on palpation. These clinical findings are consistent with which congenital heart defect? a. Atrial septal defect (ASD) c. Patent ductus arteriosus (PDA) b. Ventricular septal defect (VSD) d. Atrioventricular canal (AVC) defect If pulmonary vascular resistance has fallen, then infants with PDA will characteristically have a continuous machine-type murmur best heard at the left upper sternal border throughout systole and diastole. If the PDA is significant, then the infant also will have bounding pulses, an active precordium, a thrill on palpation, and signs and symptoms of pulmonary overcirculation. The presentations of the other congenital heart defects are not consistent with the described the symptoms. PTS: 1 REF: Pages 1203-1204 9. An infant has a crescendo-decrescendo systolic ejection murmur located between the second and third intercostal spaces along the left sternal border. A wide fixed splitting of the second heart sound is also found. These clinical findings are consistent with which congenital heart defect? a. Atrial septal defect (ASD) c. Patent ductus arteriosus (PDA) b. Ventricular septal defect (VSD) d. Atrioventricular canal (AVC) defect Because most children with ASD are asymptomatic, diagnosis is usually made during a routine physical examination by the auscultation of a crescendo-decrescendo systolic ejection murmur that reflects increased blood flow through the pulmonary valve. The location of the murmur is between the second and third intercostal spaces along the left sternal border. A wide fixed splitting of the second heart sound is also characteristic of ASD, reflecting volume overload to the right ventricle and causing prolonged ejection time and a delay of pulmonic valve closure. The presentations of other congenital heart defects are not consistent with the described symptoms. PTS: 1 REF: Pages 1204-1205 10. An infant has a loud, harsh, holosystolic murmur and systolic thrill that can be detected at the left lower sternal border that radiates to the neck. These clinical findings are consistent with which congenital heart defect? a. Atrial septal defect (ASD) c. Patent ductus arteriosus (PDA) b. Ventricular septal defect (VSD) d. Atrioventricular canal (AVC) defect On physical examination, a loud, harsh, holosystolic murmur and systolic thrill can be detected at the left lower sternal border. The intensity of the murmur reflects the pressure gradient across the VSD. An apical diastolic rumble may be present with a moderate-to-large defect, reflecting increased flow across the mitral valve. The presentations of the other congenital heart defects are not consistent with the described symptoms. PTS: 1 REF: Page 1205 11. Where can coarctation of the aorta (COA) be located? a. Exclusively on the aortic arch b. Proximal to the brachiocephalic artery c. Between the origin of the aortic arch and the bifurcation of the aorta in the lower abdomen d. Between the origin of the aortic arch and the origin of the first intercostal artery COA can occur anywhere between the origin of the aortic arch and the bifurcation of the aorta in the lower abdomen. The other options do not accurately describe the location of a COA. PTS: 1 REF: Page 1210 12. Classic manifestations of a systolic ejection murmur heard at the left interscapular area, cool mottled skin on the lower extremities but hypertension noted in the upper extremities, and decreased or absent femoral pulse are indicative of an older child with which congenital defect? a. Tetralogy of Fallot c. Ventricular septum defect (SD) b. Aortic stenosis d. Coarctation of the aorta (OA) Clinical manifestations of coarctation of the aorta include hypertension noted in the upper extremities with decreased or absent pulses in the lower extremities. Children may also have cool mottled skin and occasionally experience leg cramps during exercise. A systolic ejection murmur, heard best at the left interscapular area, is also considered a classic clinical manifestation of this disorder. The other options are not initially associated with these symptoms. PTS: 1 REF: Pages 1210-1212 13. What is the initial manifestation of aortic coarctation observed in a neonate? a. Congestive heart failure (CHF) c. Pulmonary hypertension b. Cor pulmonale d. Cerebral hypertension Initially, the newborn usually exhibits symptoms of CHF. The other options are not initially associated with aortic coarctation. PTS: 1 REF: Page 1212 14. Which compensatory mechanism is spontaneously used by children diagnosed with tetralogy of Fallot to relieve hypoxic spells? a. Lying on their left side c. Squatting b. Performing the Valsalva maneuver d. Hyperventilating Squatting is a spontaneous compensatory mechanism used by older children to alleviate hypoxic spells. Squatting and its variants increase systemic resistance while decreasing venous return to the heart from the inferior vena cava. The other options would not result in these changes. PTS: 1 REF: Page 1209 15. An infant diagnosed with a small patent ductus arteriosus (PDA) would likely exhibit which symptom? a. Intermittent murmur c. Need for surgical repair b. Lack of symptoms d. Triad of congenital defects Infants with a small PDA usually remain asymptomatic; the other options are incorrect. PTS: 1 REF: Pages 1203-1204 16. What is the most common cause of chronic sustained hypertension observed only in a newborn? a. Renal parenchymal disease c. Renal artery stenosis b. Primary hypertension d. Congenital renal malformation Congenital renal malformation is a cause of chronic sustained hypertension in a newborn. Although renal artery stenosis is observed in newborns, it is also observed in older children. Renal parenchymal disease and primary hypertension are commonly observed in older children diagnosed with chronic sustained hypertension. PTS: 1 REF: Page 1220 | Table 33-8 17. Which condition is consistent with the cardiac defect of transposition of the great vessels? a. The aorta arises from the right ventricle. b. The pulmonary trunk arises from the right ventricle. c. The right ventricle pumps blood to the lungs. d. An intermittent murmur is present. Transposition of the great arteries refers to a condition in which the aorta arises from the right ventricle and the pulmonary artery arises from the left ventricle. A transposition of the great vessels is not associated with any of the other options. PTS: 1 REF: Pages 1214-1215 18. Which scenario describes total anomalous pulmonary venous return? a. The foramen ovale closes after birth. b. Pulmonary venous return is to the right atrium. c. Pulmonary venous return is to the left atrium. d. The left atrium receives oxygenated blood. Total anomalous pulmonary venous return occurs when the pulmonary veins abnormally connect to the right side of the heart either directly or through one or more systemic veins that drain into the right atrium. None of the other options accurately describe the presentation of a total anomalous pulmonary venous return. PTS: 1 REF: Page 1216 19. Which heart defect produces a systolic ejection murmur at the right upper sternal border that transmits to the neck and left lower sternal border? a. Coarctation of the aorta c. Aortic stenosis b. Pulmonic stenosis d. Hypoplastic left heart syndrome Blood flow through the stenotic area of the aorta produces a systolic ejection murmur at the right upper sternal border that transmits to the neck and left lower sternal border. None of the other options produce the described assessment findings. PTS: 1 REF: Page 1212 20. Which heart defect produces a systolic ejection click at the upper left sternal border with a thrill palpated at the upper left sternal border? a. Coarctation of the aorta (COA) c. Aortic stenosis b. Pulmonary stenosis (PS) d. Hypoplastic left heart syndrome PS results in a systolic ejection murmur at the left upper sternal border, reflecting an obstruction to flow through the narrowed pulmonary valve. A variable systolic ejection click is present in some children, as well as valvular stenosis at the upper left sternal border. PS also produces a thrill that may be palpated at the upper left sternal border. None of the other options produce the described assessment findings. PTS: 1 REF: Page 1213 21. Which heart defect results in a single vessel arising from both ventricles, providing blood to both the pulmonary and systemic circulations? a. Coarctation of the aorta b. Tetralogy of Fallot c. Total anomalous pulmonary connection d. Truncus arteriosus Truncus arteriosus is the failure of the large embryonic artery, the truncus arteriosus, to divide into the pulmonary artery and the aorta, which results in a single vessel arising from both ventricles, providing blood flow to the pulmonary and systemic circulations. None of the other options produce the described structural malformation. PTS: 1 REF: Page 1217 22. What is the suggested mean blood pressure for an 8- to 9-year-old child? a. 104/55 mm Hg c. 112/62 mm Hg b. 106/58 mm Hg d. 121/70 mm Hg The suggested mean blood pressure for an 8- to 9- year-old child is 106/58 mm Hg. For a child of 6 to 7 years old, 104/55 mm Hg is appropriate; for a 12- to 13-year-old child, 112/62 mm Hg is appropriate, and for a 16- to 18-year-old young man, 121/70 mm Hg is appropriate. PTS: 1 REF: Page 1220 | Table 33-6 MULTIPLE RESPONSE 23. What congenital heart defects are associated with intrauterine exposure to rubella? (Select all that apply.) a. Pulmonary stenosis (PS) b. Cardiomegaly c. Patent ductus arteriosus (PDA) d. Coarctation of aorta (COA) e. Ventricular septal defect (VSD) , C, D PS, PDA, and COA are congenital heart defects associated with intrauterine exposure to rubella. Cardiomegaly and VSD are associated with maternal diabetes. PTS: 1 REF: Page 1199 | Table 33-1 24. Which symptoms meet the diagnostic criteria for Kawasaki disease in a child? (Select all that apply.) a. Fever for 5 days or longer b. “Strawberry tongue” c. Peripheral edema d. Inguinal lymphadenopathy e. Bilateral conjunctival infection , B, C, E The child must exhibit five of the following six criteria: (1) fever for 5 days or longer, (2) bilateral conjunctival infection without exudation, (3) changes in oral mucus such as strawberry tongue, (4) a polymorphous rash, (5) cervical lymphadenopathy, and (6) changes in the extremities such as peripheral edema. PTS: 1 REF: Page 1218 | Box 33-3 25. Which statements related to the ambulatory blood pressure monitoring (ABPM) system with children are true? (Select all that apply.) a. ABPM monitors blood pressure for a 24-hour period. b. ABPM assists in identifying children with white coat hypertension. c. ABPM is effective in identifying children at risk for target organ damage d. ABPM assists in identifying children who demonstrate masked hypertension. e. ABPM is effective in determining blood pressure load or hypertension for at least 48 hours. , B, C, D ABPM records blood pressure over a 24-hour period to help identify those children with white coat hypertension and masked hypertension. ABPM is useful in documenting the blood pressure load, which is the total amount of time the blood pressure is elevated above normal limits during a 24-hour period. By measuring blood pressure load, the ABPM may be able to identify those children who are at greatest risk for target organ damage. PTS: 1 REF: Page 1221 | What's New box MATCHING Match the phrases with the corresponding terms. A. Causes atrial separation B. Gap between the septum primum and the septum secundum C. Conal portion of the ventricular septum D. Abnormal communication between the atria E. Allows right-to-left shunting 26. Atrial septal defect 27. Foramen ovale 28. Septum secundum 29. Ostium primum 30. Bulbus cordis 26. PTS: 1 REF: Page 1196 MSC: An atrial septal defect is an abnormal communication between the atria. 27. ANS: E PTS: 1 REF: Pages 1195-1196 MSC: The nonfused septum secundum and ostium secundum result in the formation of a flapped orifice known as the foramen ovale, which allows the right-to-left shunting necessary for fetal circulation. 28. PTS: 1 REF: Pages 1195-1196 MSC: The septum secundum is also a fenestrated, membranelike structure located anteriorly that grows toward the endocardial cushions. During fetal development, this structure does not completely fuse with the endocardial cushions, which results in atrial separation. 29. PTS: 1 REF: Page 1195 MSC: The septum primum forms along the posterior wall of the common atrium and grows downward toward the septum secundum. The gap between the two structures, known as the ostium primum, normally closes by extensions from the endocardial cushions. 30. PTS: 1 REF: Page 1196 MSC: The conal portion of the ventricular septum that separates the aorta from the pulmonary artery forms from the bulbus cordis. Chapter 34: Structure and Function of the Pulmonary System MULTIPLE CHOICE 1. What pulmonary defense mechanism propels a mucous blanket that entraps particles moving toward the oropharynx? a. Nasal turbinates c. Cilia b. Alveolar macrophages d. Irritant receptors on the nares The submucosal glands of the bronchial lining produce mucus, contributing to the mucous blanket that covers the bronchial epithelium. The ciliated epithelial cells rhythmically beat this mucous blanket toward the trachea and pharynx, where it can be swallowed or expectorated by coughing. This selection is the only option that accurately identifies the pulmonary defense mechanism described. PTS: 1 REF: Page 1229 2. Which term is used to identify the movement of gas and air into and out of the lungs? a. Perfusion c. Respiration b. Ventilation d. Diffusion Of the options available, ventilation is the only term used to identify the mechanical movement of gas or air into and out of the lungs. PTS: 1 REF: Page 1232 3. When an individual aspirates food particles, where would the nurse expect to hear decreased or absent breath sounds? a. Left lung c. Trachea b. Right lung d. Carina The right mainstem bronchus extends from the trachea more vertically than the left main bronchus; therefore aspirated fluids or foreign particles tend to enter the right lung rather than the left or any of the other locations listed. PTS: 1 REF: Page 1228 4. Aspiration is most likely to occur in the right mainstem bronchus because it: a. Extends vertically from the trachea. b. Is narrower than the left mainstem bronchus. c. Comes into contact with food and drink first. d. Is located at the site where the bronchi bifurcate. The right mainstem bronchus extends from the trachea more vertically than the left mainstem bronchus; therefore aspirated fluids or foreign particles tend to enter the right lung rather than the left. The size of both mainstems is equal. The trachea comes into contact with food and drink first, and the carina is the site where the bronchi bifurcate. PTS: 1 REF: Page 1228 5. Air passage among alveoli is collateral and evenly distributed because of the function of which structures? a. Type I alveolar cells c. Acinus pores b. Pores of Kohn d. Alveolar pores Tiny passages called pores of Kohn permit some air to pass through the septa from alveolus to alveolus, promoting collateral ventilation and even distribution of air among the alveoli. This selection is the only option that accurately describes the function that allows air passage among alveoli. PTS: 1 REF: Page 1229 6. Where in the lung does gas exchange occur? a. Trachea c. Alveolocapillary membrane b. Segmental bronchi d. Main bronchus Gas exchange occurs only across the alveolocapillary membrane. PTS: 1 REF: Page 1230 7. Surfactant produced by type II alveolar cells facilitates alveolar distention and ventilation by which mechanism? a. Decreasing thoracic compliance b. Attracting water to the alveolar surface c. Decreasing surface tension in the alveoli d. Increasing surface tension in the alveoli Surfactant, a lipoprotein produced by type II alveolar cells, has a detergent-like effect that separates the liquid molecules, thereby decreasing alveolar surface tension. This selection is the only option that accurately describes the mechanism that allows surfactant to facilitate alveolar distention and ventilation. PTS: 1 REF: Pages 1235-1236 8. Which part of the brainstem provides basic automatic rhythm of respiration by sending efferent impulses to the diaphragm and intercostal muscles? a. Dorsal respiratory group (DRG) c. Pneumotaxic center b. Ventral respiratory group d. Apneustic center The basic automatic rhythm of respiration is set by the DRG, a cluster of inspiratory nerve cells located in the medulla that sends efferent impulses to the diaphragm and inspiratory intercostal muscles. This selection is the only option that accurately identifies the appropriate brainstem location. PTS: 1 REF: Page 1233 9. Which structures secrete surfactant? a. Type I alveolar cells c. Alveolar macrophages b. Type II alveolar cells d. Stretch receptors Two major types of epithelial cells appear in the alveolus. Type I alveolar cells provide structure, and type II alveolar cells secrete surfactant, a lipoprotein that coats the inner surface of the alveolus and facilitates its expansion during inspiration, lowers alveolar surface tension at end-expiration, and thereby prevents lung collapse. Neither alveolar macrophages nor stretch receptors secrete surfactant. PTS: 1 REF: Page 1229 10. Which structure is not associated with any lymphatic vessels? a. Trachea c. Acinus b. Bronchi d. Terminal bronchioles No lymphatic structures are located in the acinus. The other options are associated with lymphatic vessels. PTS: 1 REF: Page 1230 11. Which describes the pressure in the pleural space? a. Atmospheric c. Above atmospheric b. Below atmospheric d. Variable Pressure in the pleural space is usually negative or subatmospheric (4 to 10 mm Hg). This selection is the only option that accurately describes pleural space pressure. PTS: 1 REF: Page 1231 12. The adequacy of a person’s alveolar ventilation is assessed best by monitoring which mechanism? a. Ventilatory rate c. Respiratory effort b. Ventilatory pattern d. Arterial blood gas Observation of the ventilatory rate, pattern, or effort cannot determine the adequacy of alveolar ventilation. If a health care professional needs to determine the adequacy of ventilation, then an arterial blood gas analysis must be performed to measure partial pressure of arterial carbon dioxide (PaCO2). PTS: 1 REF: Page 1232 13. Which normal physiologic change occurs in the aging pulmonary system? a. Decreased flow resistance c. Stiffening of the chest wall b. Fewer alveoli d. Improved elastic recoil Normal alterations include (1) loss of elastic recoil, (2) stiffening of the chest wall, (3) alterations in gas exchange, and (4) increases in flow resistance (see Figure 34-18). The number of alveoli is not affected by age. PTS: 1 REF: Page 1244 14. How is most of the oxygen in the blood transported? a. Dissolved in plasma c. In the form of carbon dioxide (CO2) b. Bound to hemoglobin d. Bound to protein Oxygen is transported in the blood in two forms. A small amount dissolves in plasma, and the remainder binds to hemoglobin molecules. The other options are not involved in this process. PTS: 1 REF: Page 1240 15. Stretch receptors and peripheral chemoreceptors send afferent impulses regarding ventilation to which location in the brain? a. Pneumotaxic center in the pons b. Apneustic center in the pons c. Dorsal respiratory group (DRG) in the medulla oblongata d. Ventral respiratory group (VRG) in the medulla oblongata The respiratory center is made up of several groups of neurons located bilaterally in the brainstem: the DRG, the VRG, the pneumotaxic center, and the apneustic center. Of the options available, only the DRG group in the medulla oblongata receives afferent impulses in the situation described. PTS: 1 REF: Page 1234 16. Which substances cause airway epithelium to constrict? a. Epinephrine and acetylcholine c. Bradykinin and thromboxane A b. Histamine and prostaglandin d. Leukotrienes and prostacyclin Constriction occurs if the irritant receptors in the airway epithelium are stimulated by irritants in inspired air, by endogenous substances (e.g., histamine, serotonin, prostaglandins), by many drugs, and by humoral substances. Of the options available, only histamine and prostaglandin cause constriction. PTS: 1 REF: Page 1234 17. If a patient develops acidosis, the nurse would expect the oxyhemoglobin dissociation curve to react in which manner? a. Shift to the right, causing more oxygen (O2) to be released to the cells b. Shift to the left, allowing less O2 to be released to the cells c. Show no change, allowing the O2 concentration to remain stable d. Show dramatic fluctuation, allowing the O2 concentration to increase A shift to the right depicts hemoglobin’s decreased affinity for O2 or an increase in the ease with which oxyhemoglobin dissociates and O2 moves into the cells. The oxyhemoglobin dissociation curve is shifted to the right by acidosis (low pH) and hypercapnia (increased partial pressure of arterial carbon dioxide [PaCO2]). This selection is the only option that accurately identifies what will happen to the oxyhemoglobin dissociation curve if acidosis occurs. PTS: 1 REF: Pages 1241-1243 18. How is most carbon dioxide (CO2) in the blood transported? a. Attached to oxygen c. Combined with albumin b. In the form of bicarbonate d. Dissolved in the plasma Approximately 60% of the CO2 in venous blood and 90% of the CO2 in arterial blood are carried in the form of bicarbonate. PTS: 1 REF: Page 1243 19. The sternocleidomastoid and scalene muscles are referred to as which group? a. Diaphragmatic muscles c. Intercostal muscles b. Muscles of expiration d. Muscles of inspiration The accessory muscles of inspiration are the sternocleidomastoid and scalene muscles. These muscles are not associated with the other options. PTS: 1 REF: Page 1235 20. An increase in surface tension caused by decreased surfactant production results in which alteration? a. Decrease in alveolar macrophage production b. Increase in lung compliance c. Decrease in alveoli collapse d. Increase in alveoli fluid collection The decrease in surface tension caused by surfactant is also responsible for keeping the alveoli free of fluid. In the absence of surfactant, the surface tension tends to attract fluid into the alveoli. If surfactant production is disrupted or surfactant is not produced in adequate quantities, then the alveolar surface tension increases, causing alveolar collapse, decreased lung expansion, increased work of breathing, and severe gas-exchange abnormalities. The decrease in surface tension caused by surfactant is also responsible for keeping the alveoli free of fluid. The remaining options are not associated with decreased surfactant production. PTS: 1 REF: Pages 1235-1236 21. Decreased lung compliance means that the lungs are demonstrating which characteristic? a. Difficult deflation c. Stiffness b. Easy inflation d. Inability to diffuse oxygen A decrease in compliance indicates that the lungs or chest wall is abnormally stiff or difficult to inflate. This selection is the only option that accurately identifies the meaning of decreased compliance. PTS: 1 REF: Page 1236 22. The lung is innervated by the parasympathetic nervous system via which nerve? a. Vagus c. Brachial b. Phrenic d. Pectoral Fibers of the parasympathetic division of the autonomic nervous system (ANS) travel only in the vagus nerve to the lung. PTS: 1 REF: Page 1234 23. What event is characteristic of the function in Zone 1 of the lung? a. Blood flow through the pulmonary capillary bed increases in regular increments. b. Alveolar pressure is greater than venous pressure but not greater than arterial pressure. c. The capillary bed collapses, and normal blood flow ceases. d. Blood flows through Zone 1, but it is impeded to a certain extent by alveolar pressure. Alveolar pressure exceeds pulmonary arterial and venous pressures in Zone 1. The capillary bed collapses, and normal blood flow ceases. Zone II is the portion where alveolar pressure is greater than venous pressure but not greater than arterial pressure. Blood flows through zone II, but it is impeded to a certain extent by alveolar pressure. Zone II is normally above the level of the left atrium. In zone III, arterial and venous pressures are greater than alveolar pressure and blood flow is not affected by alveolar pressure. Zone III is in the base of the lung. Blood flow through the pulmonary capillary bed increases in regular increments from the apex to the base. PTS: 1 REF: Pages 1239-1240 24. Hypoventilation that results in the retention of carbon dioxide will stimulate which receptors in an attempt to maintain a normal homeostatic state? a. Irritant receptors c. Peripheral chemoreceptors b. Central chemoreceptors d. Stretch receptors Central chemoreceptors indirectly monitor arterial blood by sensing changes in the pH of cerebrospinal fluid (CSF). The central chemoreceptors are sensitive to very small changes in the pH of CSF (equivalent to a 1 to 2 mm Hg change in partial pressure of carbon dioxide [PCO2]) and are able to maintain a normal partial pressure of arterial carbon dioxide (PaCO2) under many different conditions, including strenuous exercise. This selection is the only option that accurately identifies the receptors that are associated with the retention of carbon dioxide. PTS: 1 REF: Page 1234 25. What is the most important cause of pulmonary artery constriction? a. Low alveolar partial pressure of arterial oxygen (PaO2) b. Hyperventilation c. Respiratory alkalosis d. Epinephrine The most important cause of pulmonary artery constriction is a low alveolar PaO2. PTS: 1 REF: Page 1230 26. Where does the tracheal bifurcation occur? a. Larynx c. Carina b. Bronchi d. Nasopharynx The trachea, which is supported by U-shaped cartilage, connects the larynx to the bronchi, the conducting airways of the lungs. The trachea divides into the two main airways, or bronchi, at the carina (see Figure 34-1). The division occurs only at the carina. PTS: 1 REF: Page 1228 27. How low must the partial pressure of arterial oxygen (PaO2) drop before the peripheral chemoreceptors influence ventilation? a. Below 100 mm Hg c. Below 70 mm Hg b. Below 80 mm Hg d. Below 60 mm Hg The PaO2 must drop well below normal (to approximately 60 mm Hg) before the peripheral chemoreceptors have much influence on ventilation. PTS: 1 REF: Page 1234 28. Which receptors are located in the smooth muscles of airways? a. Central chemoreceptors c. Peripheral chemoreceptors b. Stretch receptors d. J-receptors Of the options available, only the stretch receptors are located in the smooth muscles of airways. PTS: 1 REF: Page 1234 29. Which receptors are located near the respiratory center? a. Peripheral chemoreceptors c. Central chemoreceptors b. Stretch receptors d. J-receptors Of the options available, only the central chemoreceptors are located near the respiratory center. PTS: 1 REF: Page 1234 30. Which receptors are located in the aortic bodies, aortic arch, and carotid bodies? a. Central chemoreceptors c. J-receptors b. Stretch receptors d. Peripheral chemoreceptors Of the options available, only the peripheral chemoreceptors are located in the aortic bodies, aortic arch, and carotid bodies at the bifurcation of the carotids, near the baroreceptors. PTS: 1 REF: Page 1234 31. What is the purpose of the spirometry measurement? a. To evaluate the cause of hypoxia b. To measure the volume and flow rate during forced expiration c. To measures the gas diffusion rate at the alveolocapillary membrane d. To determine pH and oxygen and carbon dioxide concentrations Spirometry measures volume and flow rate during forced expiration. The alveolar-arterial oxygen gradient is used to evaluate the cause of hypoxia. Diffusing capacity is a measure of the gas diffusion rate at the alveolocapillary membrane. Arterial blood gas analysis can be used to determine pH and oxygen and carbon dioxide concentrations. PTS: 1 REF: Page 1243 MULTIPLE RESPONSE 32. Which structures belong to the upper conduction airway? (Select all that apply.) a. Oropharynx b. Larynx c. Nasopharynx d. Trachea e. Bronchi , C The conducting airways are the portion of the pulmonary system that provides a passage for the movement of air into and out of the gas-exchange portions of the lung. The nasopharynx, oropharynx, and related structures are often called the upper airway. The remaining options are not considered to be included in the upper conduction airway. PTS: 1 REF: Pages 1225-1226 33. Regarding the respiratory process referred to as remodeling, which statements are true? (Select all that apply.) a. Remodeling involves the vascular walls. b. Scarring and thickening occurs during this respiratory process. c. Remodeling results in a permanent change. d. Pulmonary artery hypotension results. e. Remodeling increases blood flow resistance. , B, C, E Remodeling is a process by which the vascular wall becomes scarred and thickened, thus resulting in permanent decreases in luminal diameter, increased resistance to blood flow, and permanent pulmonary artery hypertension. PTS: 1 REF: Page 1232 | What's New box 34. What are the effects of aging on the pulmonary system? a. Decreased chest wall compliance b. Decreased lung recoil c. Reduced ventilatory reserve d. Decreased partial pressure of arterial oxygen (PaO2) e. Reduced respiratory rate , B, C, D Aging affects the mechanical aspects of ventilation by decreasing chest wall compliance and elastic recoil of the lungs. Changes in these elastic properties reduce ventilatory reserve. Aging causes the PaO2 to decrease but does not affect the partial pressure of arterial carbon dioxide (PaCO2) or respiratory rate. PTS: 1 REF: Pages 1244-1245 MATCHING Match the receptor with its function. A. Irritant receptors B. Stretch receptors C. J-receptors D. Peripheral chemoreceptors E. Central chemoreceptors 35. Initiates rapid, shallow breathing 36. Monitors pH, partial pressure of carbon dioxide (PaCO2), and partial pressure of oxygen (PaO2) in arterial blood 37. Initiates cough reflex 38. Senses pH of cerebrospinal fluid 39. Hering-Breuer expiratory reflex 35. PTS: 1 REF: Page 1234 MSC: J-receptors are sensitive to increased pulmonary capillary pressure, which stimulates them to initiate rapid, shallow breathing; hypotension; and bradycardia. 36. PTS: 1 REF: Page 1234 MSC: Although the peripheral chemoreceptors are sensitive to changes in PaCO2 and pH, they are primarily sensitive to oxygen levels in arterial blood (PaO2) and are responsible for all of the increase in ventilation that occurs in response to arterial hypoxemia. 37. PTS: 1 REF: Page 1234 MSC: Irritant receptors are sensitive to noxious aerosols (vapors), gases, and particulate matter (e.g., inhaled dusts), which cause them to initiate the cough reflex. 38. ANS: E PTS: 1 REF: Page 1234 MSC: Central chemoreceptors monitor arterial blood indirectly by sensing changes in the pH of cerebrospinal fluid (CSF). 39. PTS: 1 REF: Page 1234 MSC: Stretch receptors decrease ventilatory rate and volume when stimulated, an occurrence sometimes referred to as the Hering-Breuer expiratory reflex. Chapter 35: Alterations of Pulmonary Function MULTIPLE CHOICE 1. Besides dyspnea, what is the most common characteristic associated with pulmonary disease? a. Chest pain c. Cough b. Digit clubbing d. Hemoptysis Pulmonary disease is associated with many signs and symptoms, and their specific characteristics often help in identifying the underlying disorder. The most common characteristics are dyspnea and cough. Others include abnormal sputum, hemoptysis, altered breathing patterns, hypoventilation and hyperventilation, cyanosis, clubbing of the digits, and chest pain. PTS: 1 REF: Page 1248 2. Sitting up in a forward-leaning position generally relieves which breathing disorder? a. Hyperpnea c. Apnea b. Orthopnea d. Dyspnea on exertion Of the options available, only orthopnea is generally relieved by sitting up in a forward-leaning posture or supporting the upper body on several pillows. PTS: 1 REF: Page 1249 3. Kussmaul respirations as a respiratory pattern may be associated with which characteristic(s)? a. Alternating periods of deep and shallow breathing b. Pulmonary fibrosis c. Chronic obstructive pulmonary disease d. Slightly increased ventilatory rate, large tidal volumes, and no expiratory pause Kussmaul respirations are characterized by a slightly increased ventilatory rate, very large tidal volume, and no expiratory pause. Kussmaul respirations are not associated with any of the other options. PTS: 1 REF: Page 1249 4. Respirations that are characterized by alternating periods of deep and shallow breathing are a result of which respiratory mechanism? a. Decreased blood flow to the medulla oblongata b. Increased partial pressure of arterial carbon dioxide (PaCO2), decreased pH, and decreased partial pressure of arterial oxygen (PaO2) c. Stimulation of stretch or J-receptors d. Fatigue of the intercostal muscles and diaphragm Alternating periods of deep and shallow breathing are characteristic of Cheyne-Stokes respirations and are the result of any condition that slows the blood flow to the brainstem, which in turn slows impulses that send information to the respiratory centers of the brainstem. None of the remaining options are responsible for the described breathing pattern. PTS: 1 REF: Page 1250 5. With a total hemoglobin of 9 g/dl, how many grams per deciliter of hemoglobin must become desaturated for cyanosis to occur? a. 3 c. 7 b. 5 d. 9 Cyanosis generally develops when 5 g/dl of hemoglobin is desaturated, regardless of hemoglobin concentration. PTS: 1 REF: Page 1250 6. Which statement is true regarding ventilation? a. Hypoventilation causes hypocapnia. b. Hyperventilation causes hypercapnia. c. Hyperventilation causes hypocapnia. d. Hyperventilation results in an increased partial pressure of arterial carbon dioxide (PaCO2). Hyperventilation is alveolar ventilation that exceeds metabolic demands. The lungs remove carbon dioxide at a faster rate than produced by cellular metabolism, resulting in decreased PaCO2 or hypocapnia. None of the remaining options are accurate statements. PTS: 1 REF: Page 1250 7. What term is used to describe the selective bulbous enlargement of the distal segment of a digit that is commonly associated with diseases that interfere with oxygenation of the blood? a. Edema c. Angling b. Clubbing d. Osteoarthropathy Clubbing is the selective bulbous enlargement of the end (distal segment) of a digit (finger or toe) (see Figure 35-1) and is commonly associated with diseases that interfere with oxygenation, such as bronchiectasis, cystic fibrosis, pulmonary fibrosis, lung abscess, and congenital heart disease. None of the remaining options are terms used to identify the condition described. PTS: 1 REF: Page 1250 8. Pulmonary edema and pulmonary fibrosis cause hypoxemia by which mechanism? a. Creating alveolar dead space b. Decreasing the oxygen in inspired gas c. Creating a right-to-left shunt d. Impairing alveolocapillary membrane diffusion Diffusion of oxygen through the alveolocapillary membrane is impaired if the alveolocapillary membrane is thickened or if the surface area available for diffusion is decreased. Abnormal thickness, as occurs with edema (tissue swelling) and fibrosis (formation of fibrous lesions), increases the time required for diffusion across the alveolocapillary membrane. None of the remaining options accurately describes the mechanism that triggers hypoxemia as a result of pulmonary edema or pulmonary fibrosis. PTS: 1 REF: Pages 1251-1252 9. High altitudes may produce hypoxemia through which mechanism? a. Shunting c. Decreased inspired oxygen b. Hypoventilation d. Diffusion abnormalities The presence of adequate oxygen content of the inspired air is the first factor. Oxygen content is lessened at high altitudes. At high altitudes none of the remaining options would be the cause of hypoxemia. PTS: 1 REF: Page 1251 10. Which condition is capable of producing alveolar dead space? a. Pulmonary edema c. Atelectasis b. Pulmonary emboli d. Pneumonia A pulmonary embolus that impairs blood flow to a segment of the lung results in an area where alveoli are ventilated but not perfused, which causes alveolar dead space. Alveolar dead space is not the result of any of the remaining options. PTS: 1 REF: Page 1252 11. What is the most common cause of pulmonary edema? a. Right-sided heart failure c. Mitral valve prolapse b. Left-sided heart failure d. Aortic stenosis The most common cause of pulmonary edema is heart disease. When the left ventricle fails, filling pressures on the left side of the heart increase and cause a concomitant increase in pulmonary capillary hydrostatic pressure. The remaining options are not common triggers for pulmonary edema. PTS: 1 REF: Page 1260 12. Pulmonary edema usually begins to develop at a pulmonary capillary wedge pressure or left atrial pressure of how many millimeters of mercury (mm Hg)? a. 10 c. 30 b. 20 d. 40 Pulmonary edema usually begins to develop at a pulmonary capillary wedge pressure or left atrial pressure of 20 mm Hg. PTS: 1 REF: Page 1260 13. The collapse of lung tissue caused by the lack of collateral ventilation through the pores of Kohn is referred to as what type of atelectasis? a. Compression c. Absorption b. Perfusion d. Hypoventilation Absorption atelectasis is a result of the gradual absorption of air from obstructed or hypoventilated alveoli or from inhalation of concentrated oxygen or anesthetic agents. The other forms of atelectasis are not a result of the described mechanism. PTS: 1 REF: Pages 1256-1257 | Figure 35-5 14. In what form of bronchiectasis do both constrictions and dilations deform the bronchi? a. Varicose c. Cylindric b. Symmetric d. Saccular Varicose bronchiectasis exists when both constrictions and dilations deform the bronchi. None of the other options involve both constriction and dilation, resulting in bronchi deformity. PTS: 1 REF: Pages 1256-1258 15. Which pleural abnormality involves a site of pleural rupture that acts as a one-way valve, permitting air to enter on inspiration but preventing its escape by closing during expiration? a. Spontaneous pneumothorax c. Open pneumothorax b. Tension pneumothorax d. Secondary pneumothorax In tension pneumothorax, the site of pleural rupture acts as a one-way valve, permitting air to enter on inspiration but preventing its escape by closing up during expiration. As more and more air enters the pleural space, air pressure in the pneumothorax begins to exceed barometric pressure. None of the other options result from the pathologic condition described. PTS: 1 REF: Page 1254 16. In which type of pleural effusion does the fluid become watery and diffuse out of the capillaries as a result of increased blood pressure or decreased capillary oncotic pressure? a. Exudative c. Transudative b. Purulent d. Large In transudative pleural effusion, the fluid, or transudate, is watery and diffuses out of the capillaries as a result of disorders that increase intravascular hydrostatic pressure or decrease capillary oncotic pressure. The described mechanism is not associated with the other forms of pleural effusion. PTS: 1 REF: Page 1254 17. Which condition involves an abnormally enlarged gas-exchange system and the destruction of the lung’s alveolar walls? a. Transudative effusion c. Exudative effusion b. Emphysema d. Abscess Emphysema is abnormal permanent enlargement of gas-exchange airways (acini) accompanied by the destruction of alveolar walls without obvious fibrosis. The described mechanism is not associated with the other options. PTS: 1 REF: Page 1268 18. Which term is used to identify a circumscribed area of suppuration and destruction of lung parenchyma? a. Consolidation c. Empyema b. Cavitation d. Abscess An abscess is a circumscribed area of suppuration and destruction of lung parenchyma. The described pathologic abnormality is not associated with the other options. PTS: 1 REF: Page 1274 19. Which condition is not a cause of chest wall restriction? a. Pneumothorax c. Gross obesity b. Severe kyphoscoliosis d. Neuromuscular disease Unlike the other options that result in chest wall restriction, a pneumothorax is the presence of air or gas in the pleural space caused by a rupture in the visceral pleura (which surrounds the lungs) or the parietal pleura and chest wall. PTS: 1 REF: Page 1254 20. What causes pneumoconiosis? a. Pneumococci bacteria c. Exposure to asbestos b. Inhalation of inorganic dust particles d. Inhalation of cigarette smoke Pneumoconiosis represents any change in the lung caused by the inhalation of inorganic dust particles, which usually occurs in the workplace. Pneumoconiosis is not a result of any of the other options. PTS: 1 REF: Page 1259 21. Which condition is a fulminant form of respiratory failure characterized by acute lung inflammation and diffuse alveolocapillary injury? a. Acute respiratory distress syndrome (ARDS) b. Pneumonia c. Pulmonary emboli d. Acute pulmonary edema ARDS is a fulminant form of respiratory failure characterized by acute lung inflammation and diffuse alveolocapillary injury. The described pathologic characteristics are not associated with the other options. PTS: 1 REF: Page 1261 22. Which structure(s) in acute respiratory distress syndrome (ARDS) release inflammatory mediators such as proteolytic enzymes, oxygen-free radicals, prostaglandins, leukotrienes, and platelet-activating factor? a. Complement cascade c. Macrophages b. Mast cells d. Neutrophils The role of neutrophils is central to the development of ARDS. Activated neutrophils release a battery of inflammatory mediators, among them proteolytic enzymes, oxygen-free radicals (superoxide radicals, hydrogen peroxide, hydroxyl radicals), arachidonic acid metabolites (prostaglandins, thromboxanes, leukotrienes), and platelet-activating factor. These mediators cause extensive damage to the alveolocapillary membrane and greatly increase capillary membrane permeability. The described responses are not associated with the other options. PTS: 1 REF: Page 1261 23. Pulmonary edema in acute respiratory distress syndrome (ARDS) is the result of an increase in: a. Levels of serum sodium and water c. Capillary hydrostatic pressure b. Capillary permeability d. Oncotic pressure Increased capillary permeability, a hallmark of ARDS, allows fluids, proteins, and blood cells to leak from the capillary bed into the pulmonary interstitium and alveoli. The resulting pulmonary edema and hemorrhage severely reduce lung compliance and impair alveolar ventilation. The other options would not trigger ARDS-associated pulmonary edema. PTS: 1 REF: Page 1261 24. In acute respiratory distress syndrome (ARDS), alveoli and respiratory bronchioles fill with fluid as a result of which mechanism? a. Compression on the pores of Kohn, thus preventing collateral ventilation b. Increased capillary permeability, which causes alveoli and respiratory bronchioles to fill with fluid c. Inactivation of surfactant and the impairment of type II alveolar cells d. Increased capillary hydrostatic pressure that forces fluid into the alveoli and respiratory bronchioles Lung inflammation and injury damage the alveolar epithelium and the vascular endothelium. Surfactant is inactivated, and its production by type II alveolar cells is impaired as alveoli and respiratory bronchioles fill with fluid or collapse. The other options would not trigger the described response. PTS: 1 REF: Page 1261 25. Which type of pulmonary disease requires more force to expire a volume of air? a. Restrictive c. Acute b. Obstructive d. Communicable Obstructive pulmonary disease is characterized by airway obstruction that is worse with expiration. Either more force (i.e., the use of accessory muscles of expiration) or more time is required to expire a given volume of air. The other options are not associated with a need for an increase of force to expire air. PTS: 1 REF: Page 1263 26. Which immunoglobulin (Ig) may contribute to the pathophysiologic characteristics of asthma? a. IgA c. IgG b. IgE d. IgM Asthma is a familial disorder, and more than 100 genes have been identified that may play a role in the susceptibility of and the pathogenetic mechanisms that cause asthma, including those that influence the production of interleukin (IL)–4, IL-5, and IL-13; IgE; eosinophils; mast cells; adrenergic receptors; and leukotrienes. The pathophysiologic characteristics of asthma are not associated with the other immunoglobulins. PTS: 1 REF: Pages 1263-1264 27. Which statement about the late asthmatic response is true? a. Norepinephrine causes bronchial smooth muscle contraction and mucus secretion. b. The release of toxic neuropeptides contributes to increased bronchial hyperresponsiveness. c. The release of epinephrine causes bronchial smooth muscle contraction and increases capillary permeability. d. Immunoglobulin G initiates the complement cascade and causes smooth muscle contraction and increased capillary permeability. The late asthmatic response begins 4 to 8 hours after the early response when the release of toxic neuropeptides contributes to increased bronchial hyperresponsiveness. This selection is the only option associated with the late asthmatic response. PTS: 1 REF: Page 1264 28. Clinical manifestations of inspiratory and expiratory wheezing, dyspnea, nonproductive cough, and tachypnea are indicative of which condition? a. Chronic bronchitis c. Pneumonia b. Emphysema d. Asthma At the beginning of an attack, the individual experiences chest constriction, expiratory wheezing, dyspnea, nonproductive coughing, prolonged expiration, tachycardia, and tachypnea. Severe attacks involve the use of accessory muscles of respiration, and wheezing is heard during both inspiration and expiration. The presentations of none of the other options are consistent with the described symptoms. PTS: 1 REF: Page 1264 | Page 1266 29. The most successful treatment for chronic asthma begins with which action? a. Avoidance of the causative agent b. Administration of broad-spectrum antibiotics c. Administration of drugs that reduce bronchospasm d. Administration of drugs that decrease airway inflammation Chronic management of asthma begins with the avoidance of allergens and other triggers. The effectiveness of the other options is reliant on the avoidance of triggers. PTS: 1 REF: Page 1266 30. Which factor contributes to the production of mucus associated with chronic bronchitis? a. Airway injury c. Increased Goblet cell size b. Pulmonary infection d. Bronchospasms Continual bronchial inflammation causes bronchial edema and increases the size and number of mucous glands and goblet cells in the airway epithelium. Thick, tenacious mucus is produced and cannot be cleared because of impaired ciliary function (see Figure 35-13). The lung’s defense mechanisms are therefore compromised, increasing a susceptibility to pulmonary infection, which contributes to airway injury. Frequent infectious exacerbations are complicated by bronchospasm with dyspnea and productive cough. PTS: 1 REF: Page 1267 31. Clinical manifestations of decreased exercise tolerance, wheezing, shortness of breath, and productive cough are indicative of which respiratory disorder? a. Chronic bronchitis c. Pneumonia b. Emphysema d. Asthma The symptoms that lead individuals with chronic bronchitis to seek medical care include decreased exercise tolerance, wheezing, and shortness of breath. Individuals usually have a productive cough (“smoker’s cough”). The described symptoms are not associated with any of the other options. PTS: 1 REF: Page 1267 32. Clinical manifestations that include unexplained weight loss, dyspnea on exertion, use of accessory muscles, and tachypnea with prolonged expiration are indicative of which respiratory disorder? a. Chronic bronchitis c. Pneumonia b. Emphysema d. Asthma Individuals with emphysema usually have dyspnea on exertion that later progresses to significant dyspnea, even at rest (see Table 35-3). Little coughing and very little sputum are produced. The individual is often thin, has tachypnea with prolonged expiration, and must use accessory muscles for ventilation. The anteroposterior diameter of the chest is increased (barrel chest), and the chest has a hyperresonant sound with percussion. The described symptoms are not associated with any of the other options. PTS: 1 REF: Page 1270 33. Which of the following is the most common route of lower respiratory tract infection? a. Aspiration of oropharyngeal secretions b. Inhalation of microorganisms c. Microorganisms spread to the lung via blood d. Poor mucous membrane protection Aspiration of oropharyngeal secretions is the most common route of lower respiratory tract infection; thus the nasopharynx and oropharynx constitute the first line of defense for most infectious agents. The other options are not common routes of lower respiratory tract infections. PTS: 1 REF: Page 1271 34. What is the initial step in the management of emphysema? a. Inhaled anticholinergic agents c. Cessation of smoking b. Beta agonists d. Surgical reduction of lung volume Chronic management of emphysema begins with smoking cessation. Pharmacologic management includes inhaled anticholinergic agents, and beta agonists should be prescribed. Pulmonary rehabilitation, improved nutrition, and breathing techniques all can improve symptoms. Oxygen therapy is indicated in chronic hypoxemia but must be administered with care. In selected patients, lung volume reduction surgery or transplantation can be considered. PTS: 1 REF: Pages 1270-1271 35. In tuberculosis, the body walls off the bacilli in a tubercle by stimulating which action? a. Macrophages that release tumor necrosis factor–alpha (TNF- b. Phagocytosis by neutrophils and eosinophils c. Formation of immunoglobulin G to initiate the complement cascade d. Apoptotic infected macrophages that activate cytotoxic T cells In defense, macrophages and lymphocytes release interferon, which inhibits the replication of the microorganism and stimulates more macrophages to attack the bacterium. Apoptotic infected macrophages can also activate cytotoxic T cells (cluster of differentiation [CD] 8). Tuberculosis does not trigger the mechanisms described by the other options. PTS: 1 REF: Pages 1273-1274 36. The progression of chronic bronchitis is best halted by which intervention? a. Regular use of bronchodilators b. Smoking cessation c. Postural chest drainage techniques d. Identification of early signs of infection By the time an individual seeks medical care for symptoms, considerable airway damage is present. If the individual stops smoking, then disease progression can be halted. If smoking is stopped before symptoms occur, then the risk of chronic bronchitis decreases considerably and eventually reaches that of nonsmokers. The other interventions, although appropriate, are not directed toward halting the progression of the disease process. PTS: 1 REF: Page 1267 37. Clinical manifestations of inspiratory crackles, increased tactile fremitus, egophony, and whispered pectoriloquy are indicative of which respiratory condition? a. Chronic bronchitis c. Pneumonia b. Emphysema d. Asthma Physical examination may reveal signs of pulmonary consolidation, such as inspiratory crackles, increased tactile fremitus, egophony, and whispered pectoriloquy, which support a diagnosis of pneumonia. The presentations of the other options are not consistent with the described symptoms. PTS: 1 REF: Page 1273 38. Pulmonary artery hypertension (PAH) results from which alteration? a. Narrowed pulmonary capillaries c. Destruction of alveoli b. Narrowed bronchi and bronchioles d. Ischemia of the myocardium PAH is characterized by endothelial dysfunction with an overproduction of vasoconstrictors (e.g., thromboxane, endothelin) and decreased production of vasodilators (e.g., nitric oxide, prostacyclin), resulting in narrowed pulmonary capillaries. None of the remaining options result in pulmonary hypertension. PTS: 1 REF: Page 1277 39. Squamous cell carcinoma of the lung is best described as a tumor that causes which alterations? a. Abscesses and ectopic hormone production b. Airway obstruction and atelectasis c. Pleural effusion and shortness of breath d. Chest wall pain and early metastasis Typically, the tumors are centrally located near the hila and project into bronchi. Because of this central location, nonproductive cough or hemoptysis is common. Pneumonia and atelectasis are often associated with squamous cell carcinoma. Chest pain is a late symptom associated with large tumors. These tumors can remain fairly well localized and tend not to metastasize until late in the course of the disease. Squamous cell carcinomas are not associated with any of the other options. PTS: 1 REF: Pages 1280-1281 | Table 35-4 40. What medical term is used to identify the accumulation of air in the pleural space? a. Flail chest c. Pleural effusion b. Pneumothorax d. Exudate effusion Pneumothorax is the presence of air or gas in the pleural space caused by a rupture in the visceral pleura (which surrounds the lungs) or the parietal pleura and chest wall. The condition is not identified by any of the other options. PTS: 1 REF: Page 1254 41. What medical term is used to identify the presence of pus in the pleural space? a. Plural effusion c. Empyema b. Asthma d. Pneumonia Empyema is the presence of pus in the pleural space. This condition is not identified by any of the other options. PTS: 1 REF: Page 1255 42. Fluid in the pleural space characterizes which condition? a. Pleural effusion c. Bronchiectasis b. Atelectasis d. Ischemia Pleural effusion is the presence of fluid in the pleural space. This condition is not identified by any of the other options. PTS: 1 REF: Page 1254 43. Which statement is true regarding hypoxemia? a. Hypoxemia results in the increased oxygenation of arterial blood. b. Respiratory alterations cause hypoxemia. c. Hypoxemia results in the decreased oxygenation of tissue cells. d. Various system changes cause hypoxemia. Hypoxemia, or reduced oxygenation of arterial blood (PaO2), is caused by respiratory alterations, whereas hypoxia, or reduced oxygenation of cells in tissues, may be caused by alterations of other systems as well. PTS: 1 REF: Page 1251 44. Which medication classification is generally included in the treatment of silicosis? a. Corticosteroids c. Bronchodilators b. Antibiotics d. Expectorants No specific treatment exists for silicosis, although corticosteroids may produce some improvement in the early, more acute stages. The other options are not generally prescribed. PTS: 1 REF: Page 1259 45. What medical term is used for a condition that results from pulmonary hypertension, creating chronic pressure overload in the right ventricle? a. Hypoxemia c. Bronchiectasis b. Hypoxia d. Cor pulmonale Cor pulmonale develops as pulmonary hypertension and creates chronic pressure overload in the right ventricle similar to that created in the left ventricle by systemic hypertension. None of the other options identify the condition. PTS: 1 REF: Page 1278 MULTIPLE RESPONSE 46. What are the causes of dyspnea? (Select all that apply.) a. Decreased pH, increased partial pressure of arterial carbon dioxide (PaCO2) and decreased partial pressure of arterial oxygen (PaO2) b. Decreased blood flow to the medulla oblongata c. Stimulation of stretch or J-receptors d. Presence of anxiety e. Presence of pain , C, D Dyspnea can be triggered by decreased pH, increased PaCO2, and decreased PaO2. Stimulation of either stretch or J-receptors is also known as a cause of dyspnea. Dyspnea may be the result of pulmonary disease or many other conditions, such as pain, heart disease, trauma, and anxiety. No data are available to support the role of decreased blood flow to the medulla oblongata as being a cause of dyspnea. PTS: 1 REF: Pages 1248-1249 47. Which inflammatory mediators are produced in asthma? (Select all that apply.) a. Histamine b. Bradykinin c. Leukotrienes d. Prostaglandins e. Neutrophil proteases , B, C, D A large number of inflammatory mediators, such as histamine, prostaglandins, and leukotrienes, are produced by asthma. Neutrophil proteases are not produced in relationship to asthma. PTS: 1 REF: Page 1265 | Figure 35-12 48. Which clinical manifestation is associated with pulmonary hypertension? (Select all that apply.) a. Systemic blood pressure greater than 130/90 mm Hg b. Rhonchi bilaterally c. Dyspnea on exertion d. Peripheral edema e. Jugular venous distention , D, E Symptoms of fatigue, chest discomfort, tachypnea, and dyspnea on exertion, palpitations, and cough are common. Examination may reveal peripheral edema, jugular venous distention, a precordial heave, and accentuation of the pulmonary compartment of the second heart sound. Neither rhonchi nor a systemic blood pressure of 130/90 mm Hg are associated with pulmonary hypertension. PTS: 1 REF: Pages 1277-1278 49. Which statements are true regarding exudative effusion? (Select all that apply.) a. Exudative effusion contains high concentrations of white blood cells. b. Exudative effusion produces a very thick exudate. c. Exudative effusion may occur in response to an inflammatory process. d. The presence of a malignant cancer can trigger exudative effusion. e. Exudative effusion is a result of increased capillary permeability. , C, D, E Exudative effusion is less watery and contains high concentrations of white blood cells and plasma proteins. Exudative effusion occurs in response to inflammation, infection, or malignancy and involves inflammatory processes that increase capillary permeability. PTS: 1 REF: Page 1254 50. Which characteristics are symptomatic of a flail chest? (Select all that apply.) a. Involves the fracture of several consecutive ribs. b. Involves multiple fractures to individual ribs. c. Can involve the fracture of the sternum. d. Is generally a result of the inflammatory process. e. Is more common among the older adult population. , B, C A flail chest results from the fracture of several consecutive ribs in more than one place or the fracture of the sternum and several consecutive ribs. Age and inflammation are not generally considered factors in this disorder. PTS: 1 REF: Page 1253 51. Which statements regarding Mycobacterium tuberculosis are true regarding the bacilli’s ability to go into dormancy? (Select all that apply.) a. Neutrophils and macrophages all play a role in its dormancy. b. Mycobacterium tuberculosis is capable of dormancy but for only a short period. c. The immune system is the controlling factor regarding its length of dormancy. d. The bacilli are sealed off in tubercles to allow for dormancy. e. An attack by lymphocytes brings the bacilli out of their dormant state. , C, D Neutrophils, lymphocytes, and macrophages seal off the colonies of bacilli, forming a granulomatous lesion called a tubercle. Once the bacilli are isolated in tubercles and immunity develops, tuberculosis may remain dormant for life. If the immune system is impaired, however, or if live bacilli escape into the bronchi, active disease occurs and may spread through the blood and lymphatic system to other organs. This microorganism can remain dormant for extended periods. Reverting from dormancy is not related to a lymphocyte attack. PTS: 1 REF: Pages 1273-1274 MATCHING Match the descriptions with the corresponding terms. A. Passage of fluid and solid particles into the lung B. Inflammatory obstruction of small airways C. Excessive amount of connective tissue in the lung D. Lung tissue collapse E. Abnormal dilation of the bronchi 52. Pulmonary fibrosis 53. Atelectasis 54. Bronchiectasis 55. Aspiration 56. Bronchiolitis 52. PTS: 1 REF: Page 1258 MSC: Pulmonary fibrosis is an excessive amount of fibrous or connective tissue in the lung. 53. ANS: MSC: D PTS: 1 REF: Page 1256 Atelectasis is the collapse of lung tissue. 54. ANS: MSC: E PTS: 1 REF: Page 1256 Bronchiectasis is persistent abnormal dilation of the bronchi. 55. ANS: MSC: A PTS: 1 REF: Page 1255 Aspiration is the passage of fluid and solid particles into the lung. 56. ANS: MSC: B PTS: 1 REF: Page 1258 Bronchiolitis is inflammation of the small airways or bronchioles. Chapter 36: Alterations of Pulmonary Function in Children MULTIPLE CHOICE 1. How does chest wall compliance in an infant differ from that of an adult? a. An adult’s chest wall compliance is lower than an infant’s. b. An adult’s chest wall compliance is higher than an infant’s. c. An adult’s chest wall compliance is the same as an infant’s. d. An adult’s chest wall compliance is dissimilar to that of an infant’s. Chest wall compliance is higher in infants than it is in adults, particularly in premature infants. PTS: 1 REF: Page 1292 2. Why is nasal congestion a serious threat to young infants? a. Infants are obligatory nose breathers. b. Their noses are small in diameter. c. Infants become dehydrated when mouth breathing. d. Their epiglottis is proportionally greater than the epiglottis of an adult’s. Infants up to 2 to 3 months of age are obligatory nose breathers and are unable to breathe in through their mouths. Nasal congestion is therefore a serious threat to a young infant. This selection is the only option that accurately describes why nasal congestion is a serious threat to young infants. PTS: 1 REF: Page 1290 3. The risk for respiratory distress syndrome (RDS) decreases for premature infants when they are born between how many weeks of gestation? a. 16 and 20 c. 24 and 30 b. 20 and 24 d. 30 and 36 Surfactant is secreted into fetal airways between 30 and 36 weeks. The other options are not true regarding the timeframe when the risk for RDS decreases. PTS: 1 REF: Page 1292 4. Which type of croup is most common? a. Bacterial c. Fungal b. Viral d. Autoimmune In 85% of children with croup, a virus is the cause, most commonly parainfluenza. However, other viruses such as influenza A or respiratory syncytial virus (RSV) also can cause croup. PTS: 1 REF: Pages 1295-1296 5. What is the chief predisposing factor for respiratory distress syndrome (RDS) of the newborn? a. Low birth weight b. Alcohol consumption during pregnancy c. Premature birth d. Smoking during pregnancy RDS of the newborn, also known as hyaline membrane disease (HMD), is a major cause of morbidity and mortality in premature newborns. None of the other options are considered the chief predisposing factors for RDS. PTS: 1 REF: Page 1301 6. What is the primary cause of respiratory distress syndrome (RDS) of the newborn? a. Immature immune system c. Surfactant deficiency b. Small alveoli d. Anemia RDS is primarily caused by surfactant deficiency and secondarily by a deficiency in alveolar surface area for gas exchange. None of the other options are related to the cause of RDS. PTS: 1 REF: Page 1301 7. What is the primary problem resulting from respiratory distress syndrome (RDS) of the newborn? a. Consolidation c. Atelectasis b. Pulmonary edema d. Bronchiolar plugging The primary problem is atelectasis, which causes significant hypoxemia and is difficult for the neonate to overcome because a significant negative inspiratory pressure is required to open the alveoli with each breath. None of the other options are considered a primary problem associated with RDS. PTS: 1 REF: Page 1301 8. Which option shows the correct sequence of events after atelectasis develops in respiratory distress syndrome of the newborn? a. Increased pulmonary vascular resistance, atelectasis, hypoperfusion b. Hypoxic vasoconstriction, right-to-left shunt hypoperfusion c. Respiratory acidosis, hypoxemia, hypercapnia d. Right-to-left shunt, hypoxic vasoconstriction, hypoperfusion Atelectasis results in a decrease in tidal volume, causing alveolar hypoventilation and hypercapnia. Hypoxia and hypercapnia cause pulmonary vasoconstriction, which increases intrapulmonary resistance and shunting. This results in hypoperfusion of the lung and a decrease in effective pulmonary blood flow. This selection is the only option that identifies the correct sequence of events. PTS: 1 REF: Page 1301 9. Which statement about the advances in the treatment of respiratory distress syndrome (RDS) of the newborn is incorrect? a. Administering glucocorticoids to women in preterm labor accelerates the maturation of the fetus’s lungs. b. Administering oxygen to mothers during preterm labor increases their arterial oxygen before the birth of the fetus. c. Treatment includes the instillation of exogenous surfactant down an endotracheal tube of infants weighing less than 1000 g. d. Using continuous positive airway pressure (CPAP) supports the infant’s respiratory function. Administering oxygen to the mother is not a valid treatment of RDS. The other statements provide correct information regarding RDS. PTS: 1 REF: Pages 1301-1303 10. Bronchiolitis tends to occur during the first years of life and is most often caused by what type of infection? a. Respiratory syncytial virus (RSV) c. Adenoviruses b. Influenzavirus d. Rhinovirus The most common associated pathogen is RSV, but bronchiolitis may also be associated with adenovirus, rhinovirus, influenza, parainfluenza virus (PIV), and Mycoplasma pneumoniae. PTS: 1 REF: Page 1305 11. Which immunoglobulin (Ig) is present in childhood asthma? a. IgM c. IgE b. IgG d. IgA Included in the long list of asthma-associated genes are those that code for increased levels of immune and inflammatory mediators (e.g., interleukin [IL]–4, IgE, leukotrienes), nitric oxide, and transmembrane proteins in the endoplasmic reticulum. None of the other options are associated with childhood asthma. PTS: 1 REF: Pages 1308-1309 12. Which T-lymphocyte phenotype is the key determinant of childhood asthma? a. Cluster of differentiation (CD) 4 T-helper Th1 lymphocytes b. CD4 T-helper Th2 lymphocytes c. CD8 cytotoxic T lymphocytes d. Memory T lymphocytes Asthma develops because the Th2 response (in which CD4 T-helper cells produce specific cytokines, such as interleukin [IL]–4, IL-5, and IL-13) promotes an atopic and allergic response in the airways. This selection is the only option that accurately identifies the appropriate T-lymphocyte phenotype. PTS: 1 REF: Page 1309 13. Which cytokines activated in childhood asthma produce an allergic response? a. Interleukin (IL)–1, IL-2, and interferon-alpha (IFN-) b. IL-8, IL-12, and tumor necrosis factor-alpha (TNF-) c. IL-4, IL-10, and colony-stimulating factor (CSF) d. IL-4, IL-5, and IL-13 Related to asthma, IL-4 and IL-13 are particularly important for B-cell switching to favor immunoglobulin E (IgE) production, and IL-5 is crucial for local differentiation and enhanced survival of eosinophils within the airways. This selection is the only option that accurately describes how cytokines produce a childhood asthmatic response. PTS: 1 REF: Page 1309 14. Which statement accurately describes childhood asthma? a. An obstructive airway disease characterized by reversible airflow obstruction, bronchial hyperreactivity, and inflammation b. A pulmonary disease characterized by severe hypoxemia, decreased pulmonary compliance, and diffuse densities on chest x-ray imaging c. A pulmonary disorder involving an abnormal expression of a protein, producing viscous mucus that lines the airways, pancreas, sweat ducts, and vas deferens d. An obstructive airway disease characterized by atelectasis and increased pulmonary resistance as a result of a surfactant deficiency Asthma is an obstructive airway disease characterized by reversible airflow obstruction, bronchial hyperreactivity, and inflammation. This selection is the only option that accurately describes childhood asthma. PTS: 1 REF: Page 1308 15. Which criterion is used to confirm a diagnosis of asthma in an 8-year-old child? a. Parental history of asthma b. Serum testing that confirms increased immunoglobulin E (IgE) and eosinophil levels c. Reduced expiratory flow rates confirmed by spirometry testing d. Improvement on a trial of asthma medication Confirmation of the diagnosis of asthma relies on pulmonary function testing using spirometry, which can be accomplished only after the child is 5 to 6 years of age. Reduced expiratory flow rates that are reversible in response to an inhaled bronchodilator would be characteristic abnormalities. For younger children, an empiric trial of asthma medications is commonly initiated. The remaining options are major historical and physical factors that contribute but do not confirm the diagnosis of asthma in children. PTS: 1 REF: Pages 1309-1310 16. Which statement best describes acute respiratory distress syndrome (ARDS)? a. An obstructive airway disease characterized by reversible airflow obstruction, bronchial hyperreactivity, and inflammation b. A pulmonary disease characterized by severe hypoxemia, decreased pulmonary compliance, and the presence of bilateral infiltrates on chest x-ray imaging c. A respiratory disorder involving an abnormal expression of a protein producing viscous mucus that lines the airways, pancreas, sweat ducts, and vas deferens d. A pulmonary disorder characterized by atelectasis and increased pulmonary resistance as a result of a surfactant deficiency ARDS is a condition that can result from either a direct or indirect pulmonary insult. It is defined as respiratory failure of acute onset characterized by severe hypoxemia that is refractory to treatment with supplemental oxygen, bilateral infiltrates on chest x-ray imaging, and no evidence of heart failure, as well as decreased pulmonary compliance. This selection is the only option that accurately describes ARDS. PTS: 1 REF: Page 1310 17. When considering the signs and symptoms of acute respiratory distress syndrome (ARDS), the absence of which condition is considered characteristic? a. Progressive respiratory distress c. Decreased pulmonary compliance b. Bilateral infiltrates d. Heart failure ARDS is characterized by progressive respiratory distress, severe hypoxemia refractory to treatment with supplemental oxygen, decreased pulmonary compliance, bilateral infiltrates on chest x-ray imaging, and no evidence of heart failure. PTS: 1 REF: Page 1310 18. Examination of the throat in a child demonstrating signs and symptoms of acute epiglottitis may contribute to which life-threatening complication? a. Retropharyngeal abscess c. Rupturing of the tonsils b. Laryngospasms d. Gagging induced aspiration Examination of the throat may trigger laryngospasm and cause respiratory collapse. Death may occur in a few hours. This selection is the only option that accurately identifies the life-threatening complication that can result from an examination of the throat of a child who demonstrates the signs and symptoms of acute epiglottitis. PTS: 1 REF: Page 1297 19. Which statement best describes cystic fibrosis? a. Obstructive airway disease characterized by reversible airflow obstruction, bronchial hyperreactivity, and inflammation b. Respiratory disease characterized by severe hypoxemia, decreased pulmonary compliance, and diffuse densities on chest x-ray imaging c. Pulmonary disorder involving an abnormal expression of a protein-producing viscous mucus that obstructs the airways, pancreas, sweat ducts, and vas deferens d. Pulmonary disorder characterized by atelectasis and increased pulmonary resistance as a result of a surfactant deficiency Cystic fibrosis is best described as a pulmonary disorder involving an abnormal expression of a protein-producing viscous mucus that obstructs the airways, pancreas, sweat ducts, and vas deferens. This selection is the only option that accurately describes cystic fibrosis. PTS: 1 REF: Pages 1310-1311 20. Cystic fibrosis is caused by which process? a. Autosomal recessive inheritance c. Infection b. Autosomal dominant inheritance d. Malignancy Cystic fibrosis is an autosomal recessive inherited disorder that is associated with defective epithelial ion transport. None of the other options cause cystic fibrosis. PTS: 1 REF: Page 1310 21. What are the abnormalities in cytokines found in children with cystic fibrosis (CF)? a. Deficit of interleukin (IL)–1 and an excess of IL-4, IL-12, and interferon-alpha (IFN- b. Deficit of IL-6 and an excess of IL-2, IL-8, and granulocyte colony-stimulating factor (G-CSF) c. Deficit of IL-10 and an excess of IL-1, IL-8, and TNF- d. Deficit of IL-3 and an excess of IL-14, IL-24, and colony-stimulating factor (CSF) Abnormal cytokine profiles have been documented in CF airway fluids, including deficient IL-10 and excessive IL-1, IL-8, and TNF-, all changes conducive to promoting inflammation. This selection is the only option that accurately identifies the abnormalities in cytokines observed in children with CF. PTS: 1 REF: Pages 1311-1312 22. Between which months of age does sudden infant death syndrome (SIDS) most often occur? a. 0 and 1 c. 5 and 6 b. 2 and 4 d. 6 and 7 The incidence of SIDS is low during the first month of life but sharply increases in the second month of life, peaking at 2 to 4 months and is unusual after 6 months of age. PTS: 1 REF: Page 1313 23. Where in the respiratory tract do the majority of foreign objects aspirated by children finally lodge? a. Trachea c. Bronchus b. Left lung d. Bronchioles Approximately 75% of aspirated foreign bodies lodge in a bronchus. The other options are not locations where children aspirate the majority of foreign objects. PTS: 1 REF: Page 1298 24. What is the most common predisposing factor to obstructive sleep apnea in children? a. Chronic respiratory infections c. Obligatory mouth breathing b. Adenotonsillar hypertrophy d. Paradoxic breathing In otherwise healthy children, the most common predisposing factor is adenotonsillar hypertrophy, which causes physical impingement on the nasopharyngeal airway. The other options are not associated with obstructive sleep apnea in children. PTS: 1 REF: Page 1300 MULTIPLE RESPONSE 25. Which statement is true regarding alveoli? (Select all that apply.) a. The number of functioning alveoli is determined by birth. b. The alveoli begin to increase in size starting at 8 years of age. c. The complexity of the alveoli increases into adulthood. d. These structures produce surfactant. e. Capillaries are the origin of alveoli. , C, E Capillaries grow into the distal respiratory units that keep subdividing (alveolarization) to maximize the surface area for gas exchange. The number of alveoli continues to increase during the first 5 to 8 years of life, after which the alveoli increase in size and complexity. Surfactant is a lipid-protein mix that is produced by type II alveolar cells. PTS: 1 REF: Page 1291 26. Children diagnosed with chronic asthma are likely to exhibit which symptoms? (Select all that apply.) a. Nasal flaring b. Musical expiratory wheezing c. Clubbing of fingers and toes d. Substernal retractions e. Diaphoresis , B, D, E On physical examination, expiratory wheezing that is often described as high pitched and musical is exhibited, along with prolongation of the expiratory phase of the respiratory cycle. Hyperinflation is sometimes visible. The respiratory rate is elevated, as is the heart rate. Nasal flaring and accessory muscle use are evident, with retractions in the substernal, subcostal, intercostal, suprasternal, or sternocleidomastoid areas. Infants may appear to be “head bobbing” because of sternocleidomastoid muscle use. Pulsus paradoxus may also be present. The child may appear anxious or diaphoretic, which are important signs of respiratory compromise. Clubbing of fingers and toes is not typically associated with asthma. PTS: 1 REF: Page 1309 27. Which symptom is not a clinical manifestation of croup? a. Rhinorrhea b. Sore throat c. Low-grade fever d. Barking cough e. Coarse rhonchi ANS: E Typically, a prodrome of rhinorrhea, sore throat, and low-grade fever is exhibited for a few days with croup. The child then develops the characteristic harsh (seal-like) barking cough, hoarse voice, and inspiratory stridor. Rhonchi are associated with lower respiratory diseases. PTS: 1 REF: Page 1295 28. What are the clinical manifestations of bacterial pneumonia in children? (Select all that apply.) a. Fever with chills b. Productive cough c. Dyspnea d. Respiratory alkalosis e. Malaise , B, C, E The clinical presentation of bacterial pneumonia, particularly pneumococcal, may include a preceding viral illness, followed by fever with chills and rigors, shortness of breath, and an increasingly productive cough. Auscultation usually reveals such abnormalities as crackles or decreased breath sounds. Other less specific findings may include malaise, emesis, abdominal pain, and chest pain. Respiratory alkalosis is not usually associated with bacterial pneumonia in children. PTS: 1 REF: Page 1306 MATCHING Match the sound of stridor with the location of the problem. A. Sonorous snoring B. Muffled voice C. High-pitched inspiratory sound, voice change, hoarse D. Expiratory stridor or monophonic wheeze E. Inspiratory wheezes 29. Tracheal problems 30. Laryngeal problems 31. Upper trachea obstruction 32. Nasopharyngeal obstruction, such as adenotonsillar hypertrophy. 33. Supralaryngeal obstructions 29. PTS: 1 REF: Page 1293 MSC: Expiratory stridors or monophonic wheezes suggest tracheal problems. 30. PTS: 1 REF: Page 1294 MSC: Abnormalities of voice or cry (weak or hoarse) suggest problems at the larynx. 31. ANS: E PTS: 1 REF: Page 1298 MSC: Foreign bodies lodged in the upper trachea typically produce inspiratory stridor. 32. PTS: 1 REF: Page 1293 MSC: Sonorous snoring is associated with nasopharyngeal obstruction, such as adenotonsillar hypertrophy. 33. PTS: 1 REF: Page 1294 MSC: Muffling of the voice, especially in an acute condition, suggests supralaryngeal obstruction, such as epiglottitis or retropharyngeal abscess. Chapter 27: Structure and Function of the Hematologic System MULTIPLE CHOICE 1. What is the most abundant class of plasma protein? a. Globulin c. Clotting factors b. Albumin d. Complement proteins Albumin (approximately 60% of total plasma protein at a concentration of about 4 g/dl) is the most abundant plasma protein. PTS: 1 REF: Page 946 2. What is the effect of low plasma albumin? a. Clotting factors decrease, thus increasing the chance of prolonged bleeding. b. Fewer immunoglobulins are synthesized, thus impairing the immune function. c. Less iron is stored, thus increasing the incidence of iron deficiency anemia. d. Osmotic pressure decreases, thus water moves from the capillaries to the interstitium. In the case of decreased production (e.g., cirrhosis, other diffuse liver diseases, protein malnutrition) or excessive loss of albumin (e.g., certain kidney diseases, extensive burns), the reduced oncotic pressure leads to excessive movement of fluid and solutes into the tissues and decreased blood volume. The other options are not accurate descriptions of the effect of low plasma albumin. PTS: 1 REF: Page 946 3. What is the life span of an erythrocyte (in days)? a. 20 to 30 c. 100 to 120 b. 60 to 90 d. 200 to 240 Because it cannot undergo mitotic division, the erythrocyte has a limited life span of approximately 120 days. PTS: 1 REF: Pages 947-948 4. Which statement concerning erythrocytes is true? a. Erythrocytes contain a nucleus, mitochondria, and ribosomes. b. Erythrocytes synthesize proteins. c. Erythrocytes have the ability to change shape to squeeze through microcirculation. d. Erythrocyte colony-stimulating factor (E-CSF) stimulates erythrocytes. Reversible deformity enables the erythrocyte to assume a more compact torpedo-like shape, squeeze through the microcirculation, and return to normal. The other options are not accurate statements about erythrocytes. PTS: 1 REF: Page 948 5. Granulocytes that contain granules of vasoactive amines, such as histamine, are called: a. Neutrophils c. Monocytes b. Eosinophils d. Basophils Basophils contain cytoplasmic granules that hold an abundant mixture of biochemical mediators, including histamine, chemotactic factors, proteolytic enzymes, and an anticoagulant (heparin) (see Figure 27-3, C). This is not an accurate description of any of the other options. PTS: 1 REF: Page 949 6. Which of the following are formed elements of the blood that are not cells but are disk-shaped cytoplasmic fragments essential for blood clotting? a. Monocytes c. Macrophages b. Platelets d. Erythrocytes Platelets (thrombocytes) are not true cells but are disk-shaped cytoplasmic fragments that are essential for blood coagulation and control of bleeding. This description is not accurate for any of the other options. PTS: 1 REF: Pages 950-951 7. Blood cells that differentiate into macrophages are known as: a. Monocytes c. Eosinophils b. Neutrophils d. Basophils Only monocytes migrate into a variety of tissues and fully mature into tissue macrophages and myeloid dendritic cells (see Table 27-3). PTS: 1 REF: Page 950 8. Without prior exposure to an antigen, which cells are able to destroy some types of tumor cells and some virus-infected cells? a. Lymphocytes c. Megakaryocytes b. Plasma cells d. Natural killer (NK) cells NK cells, which resemble large granular lymphocytes, kill some types of tumor cells (in vitro) and some virus-infected cells without being induced by previous exposure to these antigens. This capability is not true of the other options. PTS: 1 REF: Page 950 9. What is the life span of platelets (in days)? a. 10 c. 90 b. 30 d. 120 A platelet circulates for approximately 10 days and ages. Macrophages of the mononuclear phagocyte system, mostly in the spleen, remove platelets. PTS: 1 REF: Page 951 10. Fetal hematopoiesis occurs in which structure? a. Gut c. Bone marrow b. Spleen d. Thymus The spleen is the largest of the secondary lymphoid organs and the site of fetal hematopoiesis. PTS: 1 REF: Page 951 11. What is the consequence of a splenectomy? a. The level of iron in circulation increases. b. Antibody production increases to improve immune function. c. The number of defective cells in circulation increases. d. The number of clotting factors increases. Splenic absence from any cause (e.g., atrophy, traumatic injury, removal because of disease) has several secondary effects on the body, among them an increase in morphologically defective blood cells in the circulation, confirming the spleen’s role in removing old or damaged cells. This description of the consequence of a splenectomy is not accurate for the other options. PTS: 1 REF: Pages 951-952 12. During an infection, why do lymph nodes enlarge and become tender? a. B lymphocytes proliferate. b. The nodes are inflamed. c. The nodes fill with purulent exudate. d. The nodes are not properly functioning. The B lymphocyte proliferation in response to significant antigen (e.g., during infection) may result in lymph node enlargement and tenderness (reactive lymph node). This description is not accurate for the other options. PTS: 1 REF: Page 954 13. Which blood cells are the chief phagocytes involved in the early inflammation process? a. Neutrophils c. Eosinophils b. Monocytes d. Erythrocytes Neutrophils are the chief phagocytes of early inflammation. PTS: 1 REF: Page 949 14. Which blood cells are biconcave in shape and have the capacity to be reversibly deformed? a. Neutrophils c. Eosinophils b. Monocytes d. Erythrocytes The erythrocyte’s size and shape are ideally suited to its function as a gas carrier. A red blood cell (RBC) is a small disk with two unique properties: (1) a biconcave shape and (2) the capacity to be reversibly deformed. These are characteristics not observed in any of the other options. PTS: 1 REF: Page 948 15. Which hemoglobin is made from oxidized ferric iron (Fe3+) and lacks the ability to bind oxygen? a. Deoxyhemoglobin c. Methemoglobin b. Oxyhemoglobin d. Glycosylated hemoglobin Without reactivation by methemoglobin reductase, the Fe3+-containing hemoglobin (methemoglobin) cannot bind oxygen. This capability is not true of the other types of hemoglobin mentioned. PTS: 1 REF: Page 961 16. The absence of parietal cells would prevent the absorption of an essential nutrient necessary to prevent which type of anemia? a. Iron deficiency c. Folic acid deficiency anemia b. Pernicious anemia d. Aplastic anemia Dietary vitamin B12 is a large molecule that requires a protein secreted by parietal cells into the stomach (intrinsic factor [IF]) to transport across the ileum. Defects in IF production lead to decreased B12 absorption and pernicious anemia. The other options are not the result of this process. PTS: 1 REF: Page 962 17. Which nutrients are necessary for the synthesis of DNA and the maturation of erythrocytes? a. Protein and niacin c. Cobalamin (vitamin B12) and folate b. Iron and vitamin B6 (pyridoxine) d. Pantothenic acid and vitamin C Cobalamin and folate are necessary for the synthesis of DNA and for the maturation of erythrocytes. The remaining options are not necessary for these processes to occur. PTS: 1 REF: Page 962 | Table 27-6 18. Which nutrients are necessary for hemoglobin synthesis? a. Protein and niacin c. Cobalamin (vitamin B12) and folate b. Iron and vitamin B6 (pyridoxine) d. Pantothenic acid and vitamin C Iron and B6 (pyridoxine) are necessary for hemoglobin synthesis (see Table 27-6). The remaining options are not necessary for hemoglobin synthesis. PTS: 1 REF: Page 962 | Table 27-6 19. Recycling of iron from erythrocytes is made possible by which of the following? a. Transferrin c. Apoferritin b. Hemosiderin d. Erythropoietin Transferrin is recycled (transferrin cycle) in the following manner: (1) the transferrin-iron complex binds to a transferring receptor on the erythroblast’s plasma membrane; (2) the complex moves into the cell by receptor-mediated endocytosis; (3) iron is released (dissociated) from transferrin; and (4) the dissociated transferrin is returned to the bloodstream for reuse. The other options do not present an accurate description of the recycling of erythrocytic iron. PTS: 1 REF: Pages 963-964 20. By which structure are mature erythrocytes removed from the bloodstream? a. Liver c. Thymus b. Lymph nodes d. Spleen After approximately 100 to 120 days in the circulation, old erythrocytes are removed by tissue macrophages, primarily in the spleen. PTS: 1 REF: Page 962 21. Which substance is used to correct the chronic anemia associated with chronic renal failure? a. Iron c. Cobalamin (vitamin B12) b. Erythropoietin d. Folate One of the most significant advances in the study of hematopoietic growth factors has been the development of erythropoietin for individuals with chronic renal failure. The other options are not associated with the treatment of chronic anemia. PTS: 1 REF: Pages 960-961 22. What is the role of thromboxane A (TXA2) in the secretion stage of hemostasis? a. Stimulates the synthesis of serotonin. b. Promotes vasodilation. c. Stimulates platelet aggregation. d. Promotes formation of cyclooxygenase. Platelet aggregation is primarily stimulated by TXA2 and adenosine diphosphate (ADP), which induce functional fibrinogen receptors on the platelet. The other options do not present an accurate description of the role of thromboxane A. PTS: 1 REF: Page 969 23. Which of the following is the role of nitric oxide (NO) in hemostasis? a. Stimulates the release of fibrinogen to maintain the platelet plug. b. Stimulates the release of clotting factors V and VII. c. Causes vasoconstriction and stimulates platelet aggregation. d. Controls platelet activation through cyclic adenosine monophosphate (cAMP)–mediated signaling. Endothelial cell NO synthase produces NO, which controls platelet activation through cAMP-mediated signaling. The other options do not present an accurate description of the role of NO in hemostasis. PTS: 1 REF: Page 966 | Figure 27-18 24. The drug heparin acts in hemostasis by which processes? a. Inhibiting thrombin and antithrombin III (AT-III) b. Preventing the conversion of prothrombin to thrombin c. Shortening the fibrin strands to retract the blood clot d. Degrading the fibrin within blood clots Clinically administered heparin or heparin sulfate (on the surface of endothelial cells) binds to AT-III and induces a conformational change that greatly enhances its activity. Under normal conditions, the presence of endothelial cell heparin sulfate and available AT-III in the circulation cooperate to protect the vessels from the effects of spontaneously activated thrombin. The other options do not accurately describe the role heparin plays in hemostasis. PTS: 1 REF: Page 970 25. What is plasmin’s role in the clotting process? a. Stimulates platelet aggregation. b. Inhibits platelet adhesion and aggregation. c. Prevents the conversion of prothrombin to degrade the fibrin within blood clots. d. Degrades the fibrin within blood clots. Plasmin (also called fibrinase or fibrinolysin) is a serine protease that degrades fibrin polymers in clots. It is not capable of the functions described in the remaining options. PTS: 1 REF: Pages 971-972 26. What does polycythemia at birth indicate? a. Hypoxia in utero c. Congenitally absent spleen b. Dysfunctional bone marrow d. Dehydration in utero The hypoxic intrauterine environment stimulates erythropoietin production in the fetus and accelerates fetal erythropoiesis, producing polycythemia (excessive proliferation of erythrocyte precursors) of the newborn. The other options are not related to polycythemia. PTS: 1 REF: Page 975 27. Where are Kupffer cells located? a. Kidneys c. Pancreas b. Liver d. Spleen The liver macrophages are the only location for Kupffer cells. PTS: 1 REF: Page 950 | Page 962 | Table 27-3 28. Where are Langerhans cells found? a. Skin c. Kidney b. Intestinal lining d. Thyroid Of the available options, only the skin is the location for Langerhans cells. PTS: 1 REF: Page 950 | Table 27-3 29. What is the role of collagen in the clotting process? a. Initiates the clotting cascade. c. Stimulates fibrin. b. Activates platelets. d. Deactivates fibrinogen. In the clotting process, collagen provides a particularly strong stimulus to activate platelets. Collagen does not bring about any of the other options. PTS: 1 REF: Page 969 30. Which form of iron (Fe) can be used in the formation of normal hemoglobin? a. Fe+ c. Fe3+ b. Fe2+ d. Fe4+ It is crucial that the iron be correctly charged; only reduced ferrous iron (Fe2+) can bind oxygen in the lungs and release it in the tissues. PTS: 1 REF: Page 961 31. Where are alveolar macrophages found? a. Skin c. Gastrointestinal tract b. Breasts d. Lungs The lung is the only location for alveolar macrophages. PTS: 1 REF: Page 950 | Table 27-3 32. What changes to the hematologic system is related to age? a. Platelet adhesiveness decreases. b. Lymphocyte function decreases. c. Cellular immunity increases. d. Erythrocyte reproduction accelerates. Blood composition changes little with age. A delay in erythrocyte replenishment may occur after bleeding, presumably because of iron deficiency. Lymphocyte function appears to decrease with age. Particularly affected is a decrease in cellular immunity. Platelet adhesiveness probably increases with age. PTS: 1 REF: Page 975 33. What is the function of erythrocytes? a. Tissue oxygenation c. Infection control b. Hemostasis d. Allergy response Erythrocytes are solely responsible for tissue oxygenation. PTS: 1 REF: Pages 947-948 MULTIPLE RESPONSE 34. Which characteristics allow erythrocytes to function as gas carriers? (Select all that apply.) a. Permanent shape b. Compactness c. Reversible deformability d. Presence of hyperactive mitochondria e. Biconcavity , E A red blood cell (RBC) is a small disk with two unique properties: (1) a biconcave shape and (2) the capacity to be reversibly deformed. The other options are not relevant to the function of gas transport. PTS: 1 REF: Page 948 35. Which statements about plasma proteins are correct? (Select all that apply.) a. Provide clotting factors. b. Transport triglycerides. c. Synthesize complement proteins. d. Create hydrostatic pressure. e. Transport cholesterol. , B, C, E Plasma proteins do not create hydrostatic pressure. The other options are all accurate statements regarding plasma proteins. PTS: 1 REF: Pages 945-947 36. What are the primary anticoagulant mechanisms? (Select all that apply.) a. Antithrombin III b. Tissue factor pathway inhibitor c. Hematopoiesis d. Protein C e. Phagocytosis , B, D The major regulatory factors that control hemostasis reside where the greatest probability of clotting would occur—on the endothelial cell surface. The primary anticoagulant mechanisms include thrombin inhibitors (e.g., antithrombin III), tissue factor inhibitors (e.g., tissue factor pathway inhibitor), and mechanisms for degrading activated clotting factors (e.g., protein C). Hematopoiesis and phagocytosis are processes that are not related to anticoagulation. PTS: 1 REF: Page 970 37. Which statements are true regarding the role of the endothelium in clot formation? (Select all that apply.) a. The surface of the endothelium produces plasma protease inhibitors. b. Plasma protease inhibitors assist in preventing clot formation. c. Thrombomodulin is a protein that is converted on the surface of endothelial cells. d. Protein A binds to thrombomodulin. e. Activated protein C enhances the adhesion ability of neutrophils. , B, C The surface of the endothelium produces plasma protease inhibitors to resist clot formation. Thrombomodulin is a membrane thrombin-binding protein matter and is converted to activated protein C (see Figure 27-18) on the surface of endothelial cells. Protein C in the circulation binds to thrombomodulin. Activated protein C inhibits the adhesion of neutrophils to the endothelium. PTS: 1 REF: Pages 970-971 38. Which statements characterize albumin? (Select all that apply.) a. Retains sodium to maintain water balance. b. Provides colloid osmotic pressure. c. Is synthesized in the liver. d. Is a carrier for drugs that have low water solubility. e. Is a small molecule , C, D Albumin is a plasma protein produced by the liver. It serves as a carrier molecule for the normal components of blood, as well as for drugs that have low solubility in water (e.g., free fatty acids, lipid-soluble hormones, thyroid hormones, bile salts). Albumin molecules are large and do not diffuse freely through the vascular endothelium, thus they maintain the critical colloidal osmotic pressure (or oncotic pressure) that regulates the passage of water and solutes into the surrounding tissues (see Chapters 1 and 3). PTS: 1 REF: Page 946 MATCHING Match the descriptions with the corresponding terms. A. Clotting B. Red blood cell development C. Red blood cell destruction D. Platelet formation E. Blood cell production 39. Endomitosis 40. Hemostasis 41. Hematopoiesis 42. Erythropoiesis 43. Phagocytosis 39. PTS: 1 REF: Page 965 MSC: During thrombopoiesis, the megakaryocyte progenitor is programmed to undergo an endomitotic cell cycle called endomitosis, during which DNA replication of platelets occurs. 40. PTS: 1 REF: Page 965 MSC: Hemostasis is defined as arrest of bleeding. 41. ANS: E PTS: 1 REF: Page 954 MSC: Blood cell production (hematopoiesis) is ongoing, occurring in the liver and spleen of the fetus and only in bone marrow (medullary hematopoiesis) after birth. 42. PTS: 1 REF: Page 959 MSC: It was not until the 1850s that the bone marrow was identified as the site of erythropoiesis, or the development of red blood cells. 43. PTS: 1 REF: Page 950 MSC: Monocytes and macrophages are active phagocytes that participate in the immune and inflammatory responses. They also ingest dead or defective host cells, particularly blood cells. Chapter 28: Alterations of Erythrocyte Function MULTIPLE CHOICE 1. What term is used to describe the capacity of some erythrocytes to vary in size, especially in relationship to some anemias? a. Poikilocytosis c. Anisocytosis b. Isocytosis d. Microcytosis Additional descriptors of erythrocytes associated with some anemias include anisocytosis (assuming various sizes) or poikilocytosis (assuming various shapes) (see Figure 28-1). The remaining terms are not associated with this condition. PTS: 1 REF: Page 982 2. What is the fundamental physiologic manifestation of anemia? a. Hypotension c. Hypoxia b. Hyperesthesia d. Ischemia The fundamental physiologic manifestation of anemia is a reduced oxygen-carrying capacity of the blood, resulting in tissue hypoxia. PTS: 1 REF: Page 982 | Page 985 3. The paresthesia that occurs in vitamin B12 deficiency anemia is a result of which of the following? a. Reduction in acetylcholine receptors in the postsynaptic nerves b. Myelin degeneration in the spinal cord c. Destruction of myelin in peripheral nerves d. Altered function of neurons in the parietal lobe Effects on the nervous system can occur if a vitamin B12 deficiency causes the anemia. Myelin degeneration may occur with the resultant loss of fibers in the spinal cord, producing paresthesia (numbness), gait disturbances, extreme weakness, spasticity, and reflex abnormalities. This selection is the only option that accurately describes the cause of paresthesia in such anemias. PTS: 1 REF: Page 985 | Page 987 4. Which of the following describes how the body compensates for anemia? a. Increasing rate and depth of breathing b. Decreasing capillary vasoconstriction c. Hemoglobin holding more firmly onto oxygen d. Kidneys releasing more erythropoietin Tissue hypoxia creates additional demands and compensatory actions on the pulmonary and hematologic systems. The rate and depth of breathing increase in an attempt to increase the availability of oxygen. This selection is the only option that accurately describes the compensation mechanism in such anemias. PTS: 1 REF: Page 985 5. Which of the following is classified as a megaloblastic anemia? a. Iron deficiency c. Sideroblastic b. Pernicious d. Hemolytic Pernicious anemia is the most common type of megaloblastic anemia. The remaining options are not classified as megaloblastic anemias. PTS: 1 REF: Pages 987-988 6. Deficiencies in folate and vitamin B12 alter the synthesis of which of the following? a. RNA c. DNA b. Cell membrane d. Mitochondria Deficiencies in folate and vitamin B12 result in defective erythrocyte precursor DNA synthesis. These deficiencies are not associated with alterations of the other options. PTS: 1 REF: Page 987 7. The underlying disorder of which anemia is a result of the defective secretion of the intrinsic factor, which is essential for the absorption of vitamin B12? a. Microcytic c. Hypochromic b. Pernicious d. Hemolytic Vitamin B12 deficiency causes pernicious anemia, the most common type of megaloblastic anemia. PTS: 1 REF: Pages 987-988 8. After a person has a subtotal gastrectomy for chronic gastritis, which type of anemia will result? a. Iron deficiency c. Folic acid b. Aplastic d. Pernicious From the options available, only pernicious anemia is caused by vitamin B12 deficiency, which is often associated with the end-stage type A chronic atrophic gastritis. PTS: 1 REF: Page 988 9. What causes the atrophy of gastric mucosal cells that result in pernicious anemia? a. Erythrocyte destruction c. Vitamin B12 malabsorption b. Folic acid malabsorption d. Poor nutritional intake Deficiency in intrinsic factor (IF) secretion may be congenital or may result from adult onset gastric mucosal atrophy and the destruction of parietal cells. In older adults, virtually all vitamin B12-deficiency anemia is caused by a failure of IF-related absorption. This selection is the only option that accurately identifies the cause of gastric mucosal cell atrophy. PTS: 1 REF: Page 988 10. Which statement best describes a Schilling test? a. Administration of radioactive cobalamin and the measurement of its excretion in the urine to test for vitamin B12 deficiency b. Measurement of antigen-antibody immune complexes in the blood to test for hemolytic anemia c. Measurement of serum ferritin and total iron-binding capacity in the blood to test for iron deficiency anemia d. Administration of folate and measurement in 2 hours of its level in a blood sample to test for folic acid deficiency anemia. The Schilling test indirectly evaluates vitamin B12 absorption by administering radioactive B12 and measuring excretion in the urine. This selection is the only option that accurately describes a Schilling test. PTS: 1 REF: Page 988 11. What is the treatment of choice for pernicious anemia (PA)? a. Cyanocobalamin by oral intake b. Vitamin B12 by injection c. Ferrous fumarate by Z-track injection d. Folate by oral intake Replacement of vitamin B12 (cobalamin) is the treatment of choice for PA. Initial injections of vitamin B12 are administered weekly until the deficiency is corrected, followed by monthly injections for the remainder of the individual’s life. The other options are not treatments for PA. PTS: 1 REF: Page 988 12. Which condition resulting from untreated pernicious anemia (PA) is fatal? a. Brain hypoxia c. Heart failure b. Liver hypoxia d. Renal failure Of the options available, untreated PA is fatal, usually because of heart failure. PTS: 1 REF: Page 989 13. How is the effectiveness of vitamin B12 therapy measured? a. Reticulocyte count c. Hemoglobin b. Serum transferring d. Serum vitamin B12 The effectiveness of cobalamin replacement therapy is determined by a rising reticulocyte count. The other options are not used as indicators of the effectiveness of vitamin B12 therapy PTS: 1 REF: Pages 988-989 14. Which statement about folic acid is false? a. Folic acid absorption is dependent on the enzyme folacin. b. Folic acid is stored in the liver. c. Folic acid is essential for RNA and DNA synthesis within erythrocytes. d. Folic acid is absorbed in the upper small intestine. Folic acid absorption is not dependent on the enzyme folacin. The other options are true statements regarding folic acid. PTS: 1 REF: Page 989 15. Which anemia produces small, pale erythrocytes? a. Folic acid c. Iron deficiency b. Hemolytic d. Pernicious The microcytic-hypochromic anemias, which include iron deficiency anemia (IDA), are characterized by erythrocytes that are abnormally small and contain abnormally reduced amounts of hemoglobin. This description is not true of the other options. PTS: 1 REF: Pages 989-990 16. Which type of anemia is characterized by fatigue, weakness, and dyspnea, as well as conjunctiva of the eyes and brittle, concave nails? a. Pernicious c. Aplastic b. Iron deficiency d. Hemolytic Early symptoms of iron deficiency anemia (IDA) include fatigue, weakness, and shortness of breath. Pale earlobes, palms, and conjunctivae (see Figure 28-4) are also common signs. Progressive IDA causes more severe alterations, with structural and functional changes apparent in epithelial tissue (see Figure 28-4). The nails become brittle, thin, coarsely ridged, and spoon-shaped or concave (koilonychia) as a result of impaired capillary circulation. The tongue becomes red, sore, and painful. These symptoms are not associated with the other options. PTS: 1 REF: Pages 990-991 17. What is the most common cause of iron deficiency anemia (IDA)? a. Decreased dietary intake c. Vitamin deficiency b. Chronic blood loss d. Autoimmune disease The most common cause of IDA in well-developed countries is pregnancy and chronic blood loss. PTS: 1 REF: Page 990 18. Continued therapy of pernicious anemia (PA) generally lasts how long? a. 6 to 8 weeks c. Until the iron level is normal b. 8 to 12 months d. The rest of one’s life Because PA cannot be cured, maintenance therapy is a life-long endeavor. PTS: 1 REF: Pages 988-989 19. Sideroblastic anemia can occasionally result from an autosomal recessive transmission inherited from which relative? a. Mother c. Grandfather b. Father d. Grandmother An occasional autosomal recessive transmission occurs only with mitochondrial mutations from the mother. PTS: 1 REF: Page 992 20. Clinical manifestations of mild-to-moderate splenomegaly and hepatomegaly, bronze-colored skin, and cardiac dysrhythmias are indicative of which anemia? a. Iron deficiency c. Sideroblastic b. Pernicious d. Aplastic Of the options available, only sideroblastic anemia exhibits mild-to-moderate enlargement of the spleen (splenomegaly) and liver (hepatomegaly), as well as abnormal skin pigmentation (bronze colored). Heart rhythm disturbances, along with congestive heart failure, are major life-threatening complications related to cardiac iron overload. PTS: 1 REF: Page 992 21. Considering sideroblastic anemia, what would be the expected effect on the plasma iron levels? a. Plasma iron levels would be high. b. Levels would be low. c. Levels would be normal. d. Levels would be only minimally affected. Plasma iron levels would be high (see Table 28-3). PTS: 1 REF: Page 986 | Table 28-3 22. In aplastic anemia (AA), pancytopenia develops as a result of which of the following? a. Suppression of erythropoietin to produce adequate amounts of erythrocytes b. Suppression of the bone marrow to produce adequate amounts of erythrocytes, leukocytes, and thrombocytes c. Lack of DNA to form sufficient quantities of erythrocytes, leukocytes, and thrombocytes d. Lack of stem cells to form sufficient quantities of leukocytes AA is a critical condition characterized by pancytopenia, which is a reduction or absence of all three blood cell types, resulting from the failure or suppression of bone marrow to produce adequate amounts of blood cells. This selection is the only option that accurately identifies the cause of pancytopenia. PTS: 1 REF: Page 993 23. What is the most common pathophysiologic process that triggers aplastic anemia (AA)? a. Autoimmune disease against hematopoiesis by activated cytotoxic T (Tc) cells b. Malignancy of the bone marrow in which unregulated proliferation of erythrocytes crowd out other blood cells c. Autoimmune disease against hematopoiesis by activated immunoglobulins d. Inherited genetic disorder with recessive X-linked transmission Most cases of AA result from an autoimmune disease directed against hematopoietic stem cells. Tc cells appear to be the main culprits. None of the remaining options is considered a common trigger of AA. PTS: 1 REF: Page 995 24. An allogenic bone marrow transplantation remains the preferred method for treating which anemia? a. Polycythemia vera c. Sideroblastic b. Aplastic d. Anemia of chronic disease (ACD) Of the options available, bone marrow and, most recently, peripheral blood stem cell transplantation from a histocompatible sibling are the preferred treatments for the underlying bone marrow failure observed in aplastic anemias. PTS: 1 REF: Page 995 25. Which statement is true regarding warm autoimmune hemolytic anemia? a. Warm autoimmune hemolytic anemia occurs primarily in men. b. It is self-limiting and rarely produces hemolysis. c. Erythrocytes are bound to macrophages and sequestered in the spleen. d. Immunoglobulin M coats erythrocytes and binds them to receptors on monocytes. The immunoglobulin G–coated red blood cells bind to the Fc receptors on monocytes and splenic macrophages and are removed by phagocytosis. The other options are not true when considering this type of anemia. PTS: 1 REF: Page 998 26. When considering hemolytic anemia, which statement is true regarding the occurrence of jaundice? a. Erythrocytes are destroyed in the spleen. b. Heme destruction exceeds the liver’s ability to conjugate and excrete bilirubin. c. The patient has elevations in aspartate transaminase (AST) and alanine transaminase (ALT). d. The erythrocytes are coated with an immunoglobulin. Jaundice (icterus) is present when heme destruction exceeds the liver’s ability to conjugate and excrete bilirubin. This selection is the only option that accurately describes the process that affects the occurrence of hemolytic anemia–related jaundice. PTS: 1 REF: Page 1000 27. Erythrocyte life span of less than 120 days, ineffective bone marrow response to erythropoietin, and altered iron metabolism describe the pathophysiologic characteristics of which type of anemia? a. Aplastic c. Anemia of chronic disease b. Sideroblastic d. Iron deficiency Anemia of chronic disease results from a combination of (1) decreased erythrocyte life span, (2) suppressed production of erythropoietin, (3) ineffective bone marrow erythroid progenitor response to erythropoietin, and (4) altered iron metabolism and iron sequestration in macrophages. This result is not true of the other options. PTS: 1 REF: Page 1001 28. What is the primary cause of the symptoms of polycythemia vera? a. Decreased erythrocyte count c. Increased blood viscosity b. Destruction of erythrocytes d. Neurologic involvement As polycythemia vera progresses, many of the symptoms are related to the increased blood cellularity and viscosity. No other option is the primary cause of the symptoms of polycythemia vera. PTS: 1 REF: Page 1003 29. Treatment for polycythemia vera involves which of the following? a. Therapeutic phlebotomy and radioactive phosphorus b. Restoration of blood volume by plasma expanders c. Administration of cyanocobalamin d. Blood transfusions In low-risk individuals, the recommended therapy is phlebotomy and low-dose aspirin, whereas radioactive phosphorus has been used to suppress erythropoiesis. The other options are not considered in the treatment of polycythemia vera. PTS: 1 REF: Pages 1003-1004 30. Considering iron replacement therapy prescribed for iron deficiency anemia, who is likely to require long-term daily maintenance dosage? a. A woman who has not yet experienced menopause b. A teenager who is involved in strenuous athletics c. A middle-aged man who smokes two packs of cigarettes a day d. An older person demonstrating signs of dementia Menstruating women may need daily oral iron replacement therapy (325 mg/day) until menopause as a result of their menstrual blood loss. None of the other options are a chronic source of blood loss. PTS: 1 REF: Page 991 31. Which statement is true regarding the physical manifestations of vitamin B12 deficiency anemia? a. Vitamin B12 deficiency anemia seldom results in neurologic symptoms. b. The chances of a cure are good with appropriate treatment. c. The condition is reversible in 75% of the cases. d. Symptoms are a result of demyelination. The neurologic manifestations characteristic of vitamin B12 deficiency anemia result from nerve demyelination that may produce neuronal death. These complications pose a serious threat because they are not reversible, even with appropriate treatment. PTS: 1 REF: Page 988 MULTIPLE RESPONSE 32. A 2000 ml blood loss will produce which assessment finding? (Select all that apply.) a. Air hunger b. Normal blood pressure in the supine position c. Rapid thready pulse d. Cold clammy skin e. lactic acidosis , C, D With a 2000 ml loss of blood, central venous pressure, cardiac output, and arterial blood pressure are below normal, even when at rest and in the supine position. The person commonly has air hunger; a rapid, thready pulse; and cold, clammy skin. With a 1500 ml loss of blood, supine blood pressure and pulse can still be normal. Lactic acidosis is observed with a blood loss of 2500 ml or more. PTS: 1 REF: Page 996 | Table 28-5 33. Which medications are associated with an intermediate increase in a person’s risk for developing aplastic anemia? (Select all that apply.) a. Penicillin b. Chloramphenicol (Chloromycetin) c. Phenytoin (Dilantin) d. Trimethoprim-sulfamethoxazole (Bactrim) e. Thiazides , C, D Chloramphenicol (Chloromycetin), phenytoin (Dilantin), and trimethoprim-sulfamethoxazole (Bactrim) are associated with an intermediate increase in the risk of developing aplastic anemia. The other options are not associated with a rare increase in risk. PTS: 1 REF: Page 994 | Table 28-4 34. Which conditions are generally included in the symptoms of pernicious anemia (PA)? (Select all that apply.) a. Weakness b. Weight gain c. Low hemoglobin d. Paresthesias e. Low hematocrit , C, D, E When the hemoglobin and hematocrit levels in the blood have significantly decreased, the individual experiences the classic symptoms of PA—weakness, fatigue, paresthesias of the feet and fingers, difficulty in walking, loss of appetite, abdominal pains, and weight loss. PTS: 1 REF: Page 986 | Page 988 | Table 28-3 35. What are the clinical manifestations of folate deficiency anemia? (Select all that apply.) a. Constipation b. Flatulence c. Dysphagia d. Stomatitis e. Cheilosis , C, D, E Specific symptoms of folate deficiency anemia include severe cheilosis (scales and fissures of the lips and corners of the mouth), stomatitis (inflammation of the mouth), and painful ulcerations of the buccal mucosa and tongue. Gastrointestinal symptoms may be present and include dysphagia (difficulty swallowing), flatulence, and watery diarrhea. PTS: 1 REF: Page 989 36. Which diseases are commonly associated with anemia of chronic disease? (Select all that apply.) a. Rheumatoid arthritis b. Acquired immunodeficiency syndrome (AIDS) c. Polycythemia vera d. Systemic lupus erythematosus e. Chronic hepatitis , B, D, E AIDS, rheumatoid arthritis, systemic lupus erythematosus, malaria, acute and chronic hepatitis, and chronic renal failure are commonly associated with anemias of chronic disease. Polycythemia vera is not associated with this form of anemia. PTS: 1 REF: Page 1001 MATCHING Match the phrases with the corresponding terms. Options may be used more than once. A. Normocytic-normochromic anemia B. Microcytic-hypochromic anemia C. Macrocytic-normochromic anemia 37. Pernicious anemia 38. Sideroblastic anemia 39. Aplastic anemia 37. PTS: 1 REF: Pages 987-988 MSC: Pernicious anemia, a form of macrocytic-normochromic anemia, is caused by vitamin B12 deficiency. 38. PTS: 1 REF: Page 989 MSC: The microcytic-hypochromic anemias include sideroblastic anemia. 39. PTS: 1 REF: Page 993 MSC: Normocytic-normochromic anemias, including aplastic anemia, are characterized by erythrocytes that are relatively normal in size but with hemoglobin content that is insufficient in number. Chapter 29: Alterations of Leukocyte, Lymphoid, and Hemostatic Function MULTIPLE CHOICE 1. What change is observed in leukocytes during an allergic disorder (type I hypersensitivity) often caused by asthma, hay fever, and drug reactions? a. Neutrophilia c. Eosinophilia b. Basophilia d. Monocytosis Eosinophilia is an absolute increase (more than 450/µL) in the total numbers of circulating eosinophils. Allergic disorders (type I hypersensitivity) associated with asthma, hay fever, and drug reactions, as well as parasitic infections (particularly with metazoal parasites), are often cited as causes. This change is not identified by any of other options. PTS: 1 REF: Pages 1010-1011 2. In infectious mononucleosis (IM), what does the Monospot test detect? a. Immunoglobulin E (IgE) c. Immunoglobulin G (IgG) b. Immunoglobulin M (IgM) d. Immunoglobulin A (IgA) Heterophile antibodies are a heterogeneous group of IgM antibodies that are agglutinins against nonhuman red blood cells (e.g., sheep, horse) and are detected by qualitative (monospot) or quantitative (heterophile antibody) test methods. This statement is not true of the other options. PTS: 1 REF: Pages 1012-1013 3. Which description is consistent with acute lymphocytic leukemia (ALL)? a. ALL is a progressive neoplasm defined by the presence of greater than 30% lymphoblasts in the bone marrow or blood. b. Leukocytosis and a predominance of blast cells characterize the bone marrow and peripheral blood. As the immature blasts increase, they replace normal myelocytic cells, megakaryocytes, and erythrocytes. c. B cells fail to mature into plasma cells that synthesize immunoglobulins. d. The translocation of genetic material from genes 9 and 22 create an abnormal, fused gene identified as BCR-ABL. ALL is a progressive neoplasm defined by the presence of greater than 30% lymphoblasts in the bone marrow or blood. None of the other options provide an accurate description of ALL. PTS: 1 REF: Page 1016 4. Which description is consistent with chronic myelogenous leukemia (CML)? a. Defects exist in the ras oncogene, TP53 tumor-suppressor gene, and INK4A, the gene encoding a cell-cycle regulatory protein. b. Leukocytosis and a predominance of blast cells characterize the bone marrow and peripheral blood. As the immature blasts increase, they replace normal myelocytic cells, megakaryocytes, and erythrocytes. c. B cells fail to mature into plasma cells that synthesize immunoglobulins. d. The translocation of genetic material from genes 9 and 22 creates an abnormal, fused protein identified as BCR-ABL1. The Philadelphia chromosome is present in more than 95% of those with CML, and the presence of the BCR-ABL1 protein is responsible for the initiation of CML. The other options do not accurately describe CML. PTS: 1 REF: Pages 1014-1015 | Page 1021 5. Which description is consistent with chronic lymphocytic leukemia (CLL)? a. Defects exist in the ras oncogene, TP53 tumor-suppressor gene, and INK4A, the gene encoding a cell-cycle regulatory protein. b. Leukocytosis and a predominance of blast cells characterize the bone marrow and peripheral blood. As the immature blasts increase, they replace normal myelocytic cells, megakaryocytes, and erythrocytes. c. B cells fail to mature into plasma cells that synthesize immunoglobulins. d. The translocation of genetic material from genes 9 and 22 creates an abnormal, fused protein identified as BCR-ABL. CLL is derived from transformation of a partially mature B cell that has not yet encountered antigen. The other options do not accurately describe CLL. PTS: 1 REF: Pages 1020-1021 6. Which electrolyte imbalance accompanies multiple myeloma (MM)? a. Hyperkalemia c. Hyperphosphatemia b. Hypercalcemia d. Hypernatremia Elevated levels of calcium in the blood (hypercalcemia) characterize the common presentation of MM. The other options do not accompany MM. PTS: 1 REF: Page 1032 7. Reed-Sternberg (RS) cells represent malignant transformation and proliferation of which of the following? a. Interleukin (IL)–1, IL-2, IL-5, and IL-6 b. Tumor necrosis factor–beta c. B cells d. T cells Although the molecular events that cause malignant transformation remain controversial, RS cells are apparently from B-cell lineage. The other options are not relevant to this process. PTS: 1 REF: Pages 1024-1025 8. Local signs and symptoms of Hodgkin disease–related lymphadenopathy are a result of which of the following? a. Pressure and ischemia c. Inflammation and ischemia b. Pressure and obstruction d. Inflammation and pressure Local symptoms caused by pressure and obstruction of the lymph nodes are the result of lymphadenopathy. The other options do not contribute to the lymphadenopathy associated with Hodgkin disease. PTS: 1 REF: Page 1025 9. Which virus is associated with Burkitt lymphoma in African children? a. Cytomegalovirus c. Human papillomavirus b. Adenovirus d. Epstein-Barr virus Epstein-Barr virus, found in nasopharyngeal secretions, is associated with Burkitt lymphoma in African children. The other options are not associated with this malignancy. PTS: 1 REF: Page 1029 10. Which term is used to describe a red-purple discoloration caused by diffuse hemorrhage into the skin tissue? a. Petechiae c. Ecchymosis b. Hematoma d. Purpura Diffuse hemorrhage into skin tissues that is visible through the skin causes a red-purple discoloration identified as a purpura. None of the other options are used to identify this symptom. PTS: 1 REF: Page 1037 11. Which statement best describes heparin-induced thrombocytopenia (HIT)? a. Immunoglobulin G immune–mediated adverse drug reaction that reduces circulating platelets b. Hematologic reaction to heparin in which the bone marrow is unable to produce sufficient platelets to meet the body’s needs c. Immunoglobulin E–mediated allergic drug reaction that reduces circulating platelets d. Cell-mediated drug reaction in which macrophages process the heparin and platelet complexes that are then destroyed by activated cytotoxic T cells. Heparin is a common cause of drug-induced thrombocytopenia. HIT is an immune-mediated, adverse drug reaction caused by immunoglobulin G antibodies that leads to increased platelet consumption and a decrease in platelet counts. None of the other options accurately describe HIT. PTS: 1 REF: Page 1038 12. Immune thrombocytopenia (ITP) is a(n) condition in adults and a(n) condition in children. a. Acute; acute c. Acute; chronic b. Chronic; chronic d. Chronic; acute ITP may be acute or chronic. The acute form is frequently observed in children. Chronic ITP is more commonly observed in adults, with the highest prevalence in women between 20 and 40 years of age. PTS: 1 REF: Pages 1038-1039 13. Vitamin is required for normal clotting factor synthesis by the . a. K; kidneys c. K; liver b. D; kidneys d. D; liver Vitamin K, a fat-soluble vitamin, is necessary for the synthesis and regulation of prothrombin, procoagulant factors (VII, IX, X), and anticoagulant regulators (proteins C and S) in the liver. PTS: 1 REF: Pages 1042-1043 14. What is the most common cause of vitamin K deficiency? a. Administration of warfarin (Coumadin) b. Total parenteral nutrition with antibiotic therapy c. An immunoglobulin G–mediated autoimmune disorder d. Liver failure The most common cause of vitamin K deficiency is parenteral nutrition in combination with broad-spectrum antibiotics that destroy normal gut flora. None of the other options are commonly associated with vitamin K deficiency. PTS: 1 REF: Page 1043 15. Which disorder is described as an unregulated release of thrombin with subsequent fibrin formation and accelerated fibrinolysis? a. Disseminated intravascular coagulation (DIC) b. Immune thrombocytopenic purpura (ITP) c. Heparin-induced thrombocytopenia (HIT) d. Essential thrombocythemia (ET) DIC is an acquired clinical syndrome characterized by widespread activation of coagulation resulting in the formation of fibrin clots in medium and small vessels throughout the body. This description does not accurately identify any of the other options. PTS: 1 REF: Page 1043 16. In disseminated intravascular coagulation (DIC), what activates the coagulation cascade? a. Cytokines, such as platelet-activating factor (PAF), and tumor necrosis factor- alpha (TNF-) b. Thromboxane A, causing platelets to aggregate and consume clotting factors c. Tissue factor (TF) located in the endothelial layer of blood vessels and subcutaneous tissue d. Endotoxins from gram-negative and gram-positive bacteria circulating in the bloodstream Direct tissue damage (ischemia and necrosis, surgical manipulation, crushing injury) causes the endothelium to release TF. The common pathway for DIC appears to be excessive and widespread exposure of TF. The other options are not responsible for the activation of the coagulation cascade. PTS: 1 REF: Pages 1043-1044 17. Which proinflammatory cytokines are responsible for the development and maintenance of disseminated intravascular coagulation (DIC)? a. Granulocyte colony-stimulating factor (G-CSF); interleukin (IL)–2, IL-4, and IL-10; and tumor necrosis factor-gamma (IFN-) b. Granulocyte-macrophage colony-stimulating factor (GM-CSF); and IL-3, IL-5, IL-9, and IFN- c. Macrophage colony-stimulating factor (M-CSF); IL-7, IL-11, and IL-14; and PAF d. Tumor necrosis factor-alpha (TNF-); IL-1, IL-6, and IL-8; and platelet-activating factor (PAF) Endotoxin, in particular, triggers the release of multiple cytokines that play a significant role in the development and maintenance of DIC. Proinflammatory cytokines—TNF-; IL-1, IL-6, IL-8; PAF—are responsible for the clinical signs and symptoms associated with the sepsis associated with DIC. None of the other options perform this function. PTS: 1 REF: Page 1044 18. In disseminated intravascular coagulation (DIC), what are the indications of microvascular thrombosis? a. Reduced amplitude in peripheral pulses b. Symmetric cyanosis of fingers and toes c. Numbness and tingling in fingers and toes d. Bilateral pallor and edema of fingers and toes Several organ systems are susceptible to microvascular thrombosis that affects their function. Indicators of multisystem failure include changes in the level of consciousness, behavior, and mentation; confusion; seizure activity; oliguria; hematuria; hypoxia; hypotension; hemoptysis; chest pain; and tachycardia. Symmetric cyanosis of the fingers and toes (i.e., “blue finger/toe syndrome”) and, in some instances, of the nose and breasts may be present. The other options are not recognized indicators of microvascular thrombosis. PTS: 1 REF: Page 1046 19. What is the most reliable and specific test for diagnosing disseminated intravascular coagulation (DIC)? a. Prothrombin time (PT) b. Activated partial thromboplastin time (aPTT) c. Fibrin degradation products (FDP) d. D-dimer D-dimer testing measures a specific DIC-related product. This statement is not true of the other options. PTS: 1 REF: Page 1047 20. What term is used to identify thrombi that occlude arterioles and capillaries and are made up of platelets with minimal fibrin and erythrocytes? a. Essential (primary) thrombocythemia (ET) b. Acute idiopathic thrombotic thrombocytopenic purpura c. Thrombotic thrombocytopenic purpura (TTP) d. Immune thrombocytopenic purpura (ITP) Of the available options, only TTP is characterized by thrombotic microangiopathy in which platelets aggregate and cause occlusion of arterioles and capillaries in the microcirculation. PTS: 1 REF: Page 1040 21. Which of the following is characterized by what is referred to as pathognomonic pentad of symptoms? a. Acute idiopathic thrombotic thrombocytopenic purpura b. Essential (primary) thrombocythemia (ET) c. Immune thrombocytopenic purpura (ITP) d. Thrombotic thrombocytopenic purpura (TTP) Acute idiopathic thrombotic thrombocytopenic purpura is characterized by a pathognomonic pentad of symptoms that includes extreme thrombocytopenia (fewer than 20,000 platelets/mm3), intravascular hemolytic anemia, ischemic signs and symptoms most often involving the central nervous system (approximately 65% exhibit memory disturbances, behavioral irregularities, headaches, or coma), kidney failure (affecting approximately 65% of individuals), and fever (present in approximately 33% of individuals The other options do not demonstrate these symptoms. PTS: 1 REF: Page 1040 22. Which statement relates to immune thrombocytopenic purpura (ITP)? a. ITP is formed in conditions of low flow and is made up of mostly red cells with larger amounts of fibrin and few platelets. b. An alteration of multipotent stem cells, resulting in an excess production of platelets, causes ITP. c. Mononuclear phagocytes in the spleen remove antibody-coated platelets from circulation. d. Arterial clots are made up of mostly platelet aggregates held together by fibrin strands. ITP involves the antigen usually forming immune complexes with circulating antibodies, and it is thought that the immune complexes bind to Fc receptors on platelets, leading to their destruction in the spleen. None of the other options are accurately related to ITP. PTS: 1 REF: Pages 1038-1039 23. When the demand for mature neutrophils exceeds the supply, immature neutrophils are released indicating: a. A shift to the right c. Leukocytosis b. A shift to the left d. Leukemia When the demand for circulating mature neutrophils exceeds the supply, the marrow begins to release immature neutrophils (and other leukocytes) into the blood. Premature release of the immature white cells is responsible for the phenomenon known as a shift to the left or leukemoid reaction. None of the remaining options would be used to identify the process described. PTS: 1 REF: Page 1009 24. Hodgkin disease is characterized by the presence of which of the following? a. Philadelphia chromosome c. Microvascular thrombi b. Virchow triad d. Reed-Sternberg (RS) cells Hodgkin disease is characterized by its progression from one group of lymph nodes to another, the development of systemic symptoms, and the presence of RS cells (see Figure 29-8), but not the involvement of the Philadelphia chromosome. Virchow triad is a symptom related to thrombus formation. Disseminated intravascular coagulation is associated with microvascular thrombi. PTS: 1 REF: Page 1024 MULTIPLE RESPONSE 25. Which classic clinical manifestations are symptoms of IM? (Select all that apply.) a. Lymph node enlargement b. Hepatitis c. Pharyngitis d. Edema in the area of the eyes e. Fever , C, E At the time of diagnosis, the individual usually has the classic triad of symptoms: fever, pharyngitis, and lymphadenopathy of the cervical lymph nodes. The triad does not include hepatitis or orbital edema. PTS: 1 REF: Page 1012 26. Early detection of acute leukemia would include which of the following symptoms? (Select all that apply.) a. Dizziness b. Paresthesia c. Anorexia d. Bruising e. Bone pain , D, E Signs and symptoms related to bone marrow depression include fatigue caused by anemia, bleeding resulting from thrombocytopenia (reduced numbers of circulating platelets), and fever caused by infection. Anorexia can occur in all varieties of acute leukemia and is associated with weight loss. Pain in the bones and joints is thought to result from leukemia infiltration with secondary stretching of the periosteum. The other options are not generally associated with acute leukemia. PTS: 1 REF: Page 1017 | Page 1019 27. What are the most significant risk factors for the development of thrombus formation as referred to by the Virchow triad? (Select all that apply.) a. Endothelial injury to blood vessels b. Turbulent arterial blood flow c. Rapid coagulation of the blood d. Stagnant venous blood flow e. History of obesity , B, C, D The risk for developing spontaneous thrombi is related to several factors, referred to as the Virchow triad: (1) injury to the blood vessel endothelium, (2) abnormalities of blood flow, and (3) hypercoagulability of the blood. Obesity is not associated with the triad. PTS: 1 REF: Pages 1048-1049 28. Which statements are true regarding leukemias? (Select all that apply.) a. A single progenitor cell undergoes a malignant change. b. Leukemia is a result of uncontrolled cellular proliferation. c. Bone marrow becomes overcrowded. d. Leukocytes are under produced. e. Hematopoietic cell production is decreased. , B, C, E In the leukemias, a single progenitor cell undergoes malignant transformation. The common feature of all forms of leukemia is an uncontrolled proliferation of malignant leukocytes, causing an overcrowding of bone marrow and decreased production and function of normal hematopoietic cells. PTS: 1 REF: Page 1015 29. The two major forms of leukemia, acute and chronic, are classified by which criteria? (Select all that apply.) a. Predominant cell type b. Rate of progression c. Age of individual when cells differentiation occurs d. Stage of cell development when malignancy first occurs e. Serum level of leukocytes , B The current classification of leukemia is based on (1) the predominant cell of origin (either myeloid or lymphoid) and (2) the rate of progression, which usually reflects the degree at which cell differentiation was arrested when the cell became malignant (acute or chronic) (see Figure 29-2). The remaining options are inaccurate statements regarding the classification criteria. PTS: 1 REF: Page 1013 30. What are the clinical manifestations of advanced non-African Burkitt lymphoma? (Select all that apply.) a. Abdominal swelling b. Night sweats c. Fever d. Weight gain e. Dementia , B, C In non-African Burkitt lymphoma, the most common presentation is abdominal swelling. More advanced disease may exhibit night sweats, fever, and weight loss. Dementia is not associated with this disease. PTS: 1 REF: Page 1029 MATCHING Match the causes or diagnostic tests with the hematologic disorders. A. Epstein-Barr virus B. Bence Jones protein C. Diagnosed by the Reed-Sternberg cell D. Diagnosed by the Philadelphia chromosome 31. Infectious mononucleosis 32. Chronic myelogenous leukemia 33. Multiple myeloma 34. Hodgkin lymphoma 31. PTS: 1 REF: Page 1011 MSC: The most common etiologic agent is Epstein-Barr virus, a ubiquitous, lymphotrophic, gamma-group herpesvirus. 32. PTS: 1 REF: Pages 1014-1015 MSC: The Philadelphia chromosome is present in more than 95% of patients with chronic myelogenous leukemia. 33. PTS: 1 REF: Page 1032 MSC: The myeloma may produce free immunoglobulin light chain (Bence Jones protein) that is present in the blood and urine in approximately 80% of patients. 34. PTS: 1 REF: Page 1024 MSC: Hodgkin lymphoma is characterized by its progression from one group of lymph nodes to another, the development of systemic symptoms, and the presence of Reed-Sternberg cells Chapter 30: Alterations of Hematologic Function in Children MULTIPLE CHOICE 1. What is the cause of polycythemia in the fetus? a. Fetal hemoglobin has a greater affinity for oxygen as a result of diphosphoglycerate (DPG). b. The fetus has a different hemoglobin structure of two - and two -chains rather than two -and two -chains. c. Increased erythropoiesis occurs in response to the hypoxic intrauterine environment. d. The lungs of the fetus are undeveloped and unable to diffuse oxygen adequately to the pulmonary capillaries. The hypoxic intrauterine environment stimulates erythropoietin production in the fetus. This selection is the only option that accurately explains why the fetus develops polycythemia. PTS: 1 REF: Page 1057 2. Why does fetal hemoglobin have a greater affinity for oxygen than adult hemoglobin? a. The fetus does not have its own oxygen supply and must rely on oxygen from the maternal vascular system. b. The fetus has two -chains on the hemoglobin, rather than two -chains as in the adult. c. Fetal hemoglobin interacts less readily with diphosphoglycerate (DPG), which inhibits hemoglobin-oxygen binding. d. Fetal hemoglobin production occurs in the vessels and liver rather than in the bone marrow as in the adult. Fetal hemoglobin has greater affinity for oxygen than does adult hemoglobin because it interacts less readily with the enzyme, DPG, which inhibits hemoglobin-oxygen binding. This selection is the only option that accurately explains why fetal hemoglobin has a greater affinity for oxygen than adult hemoglobin. PTS: 1 REF: Page 1056 3. Which blood cell type is elevated at birth but decreases to adult levels during the first year of life? a. Monocytes c. Neutrophils b. Platelets d. Lymphocytes Only monocyte counts are high in the first year of life and then decrease to adult levels. PTS: 1 REF: Page 1057 4. In a full-term infant, the normal erythrocyte life span is days, whereas the adult erythrocyte life span is _ days. a. 30 to 50; 80 c. 90 to 110; 140 b. 60 to 80; 120 d. 120 to 130; 150 In full-term infants, normal erythrocyte life span is 60 to 80 days; in premature infants it may be as short as 20 to 30 days; and in children and adolescents, it is 120 days, the same as that in adults. PTS: 1 REF: Page 1057 5. What is the most common cause of insufficient erythropoiesis in children? a. Folic acid deficiency c. Hemoglobin abnormality b. Iron deficiency d. Erythrocyte abnormality Similar to the anemias of adulthood, ineffective erythropoiesis or premature destruction of erythrocytes causes the anemias of childhood. The most common cause of insufficient erythropoiesis is iron deficiency. The other options may be causes but are not common ones. PTS: 1 REF: Page 1058 6. How does hemolytic disease of the newborn (HDN) cause acquired congenital hemolytic anemia? a. HDN develops when hypoxia or dehydration causes the erythrocytes to change shapes, which are then recognized as foreign and removed from circulation. b. HDN is an alloimmune disease in which the mother’s immune system produces antibodies against fetal erythrocytes, which are recognized as foreign and removed from circulation. c. HDN develops when the polycythemia present in fetal life continues after birth, causing the excessive number of erythrocytes to be removed from circulation. d. HDN is an autoimmune disease in which the fetus’s immune system produces antibodies against fetal erythrocytes, which are recognized as foreign and removed from circulation. HDN is an alloimmune disease in which maternal blood and fetal blood are antigenically incompatible, causing the mother’s immune system to produce antibodies against fetal erythrocytes. Fetal erythrocytes that have been attacked by (or bound to) maternal antibodies are recognized as foreign or defective by the fetal mononuclear phagocyte system and are removed from the circulation by phagocytosis, usually in the fetal spleen. This selection is the only option that accurately explains how HDN causes acquired congenital hemolytic anemia. PTS: 1 REF: Page 1058 7. Erythroblastosis fetalis is defined as an: a. Allergic disease in which maternal blood and fetal blood are antigenically incompatible b. Alloimmune disease in which maternal blood and fetal blood are antigenically incompatible c. Autoimmune disease in immature nucleated cells that are released into the bloodstream d. Autosomal dominant hereditary disease Erythroblastosis fetalis, also known as hemolytic disease of the newborn (HDN), is an alloimmune disease in which maternal blood and fetal blood are antigenically incompatible, causing the mother’s immune system to produce antibodies against fetal erythrocytes. This selection is the only option that accurately defines erythroblastosis fetalis. PTS: 1 REF: Page 1058 8. An infant’s hemoglobin must fall below g/dl before signs of pallor, tachycardia, and systolic murmurs occur. a. 11 c. 7 b. 9 d. 5 When the hemoglobin falls below 5 g/dl, pallor, tachycardia, and systolic murmurs may occur. PTS: 1 REF: Page 1059 9. Which vitamin improves the absorption of oral iron taken to treat iron deficiency anemia in children? a. A c. C b. B d. E Of the available options, only additional vitamin C may be used to promote iron absorption. PTS: 1 REF: Page 1059 10. Hemolytic disease of the newborn (HDN) can occur if the mother: a. Is Rh-positive and the fetus is Rh-negative b. Is Rh-negative and the fetus is Rh-positive c. Has type A blood and the fetus has type O d. Has type AB blood and the fetus has type B HDN can occur only if antigens on fetal erythrocytes differ from antigens on maternal erythrocytes. Maternal-fetal incompatibility exists only if the mother and fetus differ in ABO blood type or if the fetus is Rh-positive and the mother is Rh-negative. This erythrocyte incompatibility does not exist in any of the other options. PTS: 1 REF: Page 1059 11. When diagnosed with hemolytic disease of the newborn (HDN), why does the newborn develop hyperbilirubinemia after birth but not in utero? a. Excretion of unconjugated bilirubin through the placenta into the mother’s circulation is no longer possible. b. Hemoglobin does not break down into bilirubin in the intrauterine environment. c. The liver of the fetus is too immature to conjugate bilirubin from a lipid-soluble to water-soluble form. d. The destruction of erythrocytes producing bilirubin is greater after birth. Hyperbilirubinemia occurs in the neonate after birth because excretion of lipid-soluble unconjugated bilirubin through the placenta is no longer possible. This selection is the only option that accurately explains why HDN causes hyperbilirubinemia after birth but not in utero. PTS: 1 REF: Page 1060 12. Fetuses who do not survive anemia in utero are usually stillborn with gross edema of the entire body. Which term is used to identify this condition? a. Spherocytosis c. Erythroblastosis fetalis b. Icterus gravis neonatorum d. Hydrops fetalis Fetuses that do not survive anemia in utero are usually stillborn with gross edema in the entire body, a condition called hydrops fetalis. This selection is the only option that accurately identifies the condition with these specific symptoms. PTS: 1 REF: Page 1060 13. What is the name of the disorder in which levels of bilirubin remain excessively high in the newborn and are deposited in the brain? a. Kernicterus c. Jaundice b. Icterus neonatorum d. Icterus gravis neonatorum Without replacement transfusions, during which the child receives Rh-negative erythrocytes, the bilirubin is deposited in the brain, a condition called kernicterus. This selection is the only option that accurately identifies the condition with the provided symptoms. PTS: 1 REF: Page 1060 14. What treatment prevents the development of kernicterus in an infant born with hemolytic disease of the newborn (HDN)? a. Administration of intravenous fluids to dilute the blood b. Replacement transfusion of new Rh-positive blood that is not contaminated with anti-Rh antibodies c. Performance of a splenectomy to prevent the destruction of abnormal erythrocytes d. Replacement transfusion of Rh-negative erythrocytes If antigenic incompatibility of the mother’s erythrocytes is not discovered in time to administer Rh immunoglobulin and the child is born with HDN, then the treatment consists of exchange transfusions in which the neonate’s blood is replaced with new Rh-positive blood that is not contaminated with anti-Rh antibodies. This treatment is administered during the first 24 hours of extrauterine life to prevent kernicterus. This selection is the only option that accurately prevents kernicterus. PTS: 1 REF: Page 1061 15. Glucose 6-phosphate dehydrogenase (G6PD) deficiency is what type of inherited disorder? a. X-linked dominant c. Autosomal dominant b. X-linked recessive d. Autosomal recessive G6PD deficiency is an inherited, X-linked recessive disorder. This selection is the only option that accurately identifies the mode of inheritance for the stated disorder. PTS: 1 REF: Page 1062 16. Sickle cell disease is classified as a(an): a. Inherited X-linked recessive disorder b. Inherited autosomal recessive disorder c. Disorder initiated by hypoxemia and acidosis d. Disorder that is diagnosed equally in men and women Sickling is an occasional, intermittent phenomenon that can be triggered or sustained by one or more of the following stressors: decreased oxygen tension (PO2) of the blood (hypoxemia), increased hydrogen ion concentration in the blood (decreased pH), increased plasma osmolality, decreased plasma volume, and low temperature (see Figure 30-7). This selection is the only option that accurately identifies the classification of the stated disorders. PTS: 1 REF: Page 1062 | Page 1065 17. Hemoglobin S (HbS) is formed in sickle cell disease as a result of which process? a. Deficiency in glucose 6-phosphate dehydrogenase (G6PD) that changes hemoglobin A (HbA) to HbS. b. Genetic mutation in which two amino acids (histidine and leucine) are missing. c. Genetic mutation in which one amino acid (valine) is replaced by another (glutamic acid). d. Autoimmune response in which one amino acid (proline) is detected as an antigen by abnormal immunoglobulin G (IgG). HbS is formed by a genetic mutation in which one amino acid (valine) replaces another (glutamic acid) (see Figure 30-5, A). This selection is the only option that accurately identifies how HbS is formed in sickle cell disease. PTS: 1 REF: Page 1063 18. Sickle cell disease (SCD) is what type of inherited disorder? a. Autosomal dominant c. X-linked dominant b. Autosomal recessive d. X-linked recessive SCD is an inherited autosomal recessive disorder that is expressed as sickle cell anemia, sickle cell–thalassemia disease, or sickle cell–hemoglobin C (HbC) disease, depending on mode of inheritance. This selection is the only option that accurately identifies the mode of inheritance for the stated disorder. PTS: 1 REF: Page 1063 19. What is the reason most children diagnosed with sickle cell anemia are not candidates for either bone marrow or stem cell transplants? a. Well-matched stem cell donors are difficult to find. b. The child is usually too weak to survive the procedure. c. The child’s immune system will not appropriately respond to the antirejection medications. d. Although effective for adults, neither procedure has been proven effective for children. Bone marrow or stem cell transplants can cure sickle cell anemia. However, the only reason these procedures are not currently an option for most children is because well-matched stem cell donors are often difficult to find. PTS: 1 REF: Page 1063 20. Which manifestations of vasoocclusive crisis are associated with sickle cell disease (SCD) in infants? a. Atelectasis and pneumonia b. Edema of the hands and feet c. Stasis ulcers of the hands, ankles, and feet d. Splenomegaly and hepatomegaly Symmetric, painful swelling of the hands and feet (hand-foot syndrome) caused by infarction in the small vessels of the extremities is often the initial manifestation of SCD in infants. This selection is the only option that accurately identifies the manifestations of a vasoocclusive crisis associated with SCD in infancy. PTS: 1 REF: Page 1066 21. What is the chance with each pregnancy that a child born to two parents with the sickle trait will have sickle cell disease (SCD)? a. 20% c. 33% b. 25% d. 50% A 25% chance exists with each pregnancy that a child born to two parents with sickle cell trait will have SCD. Genetic counseling enables people with SCD or with the sickle cell trait to make informed decisions about transmitting this genetic disorder to their offspring. PTS: 1 REF: Pages 1068-1069 22. Which type of anemia occurs as a result of thalassemia? a. Microcytic, hypochromic c. Macrocytic, hyperchromic b. Microcytic, normochromic d. Macrocytic, normochromic The only option that identifies the anemic manifestation of thalassemia is microcytic-hypochromic hemolytic anemia. PTS: 1 REF: Page 1069 23. What is the fundamental defect that results in beta-thalassemia major? a. The spleen prematurely destroys the precipitate-carrying cells. b. A severe uncoupling of - and -chain synthesis occurs. c. All four beta-forming genes are defective. d. Hemoglobin H (HbH) develops when three genes are defective. The fundamental defect in beta-thalassemia is the uncoupling of - and -chain synthesis. -Chain production is moderately depressed in the heterozygous form—beta-thalassemia minor, and severely depressed in the homozygous form—beta-thalassemia major (also called Cooley anemia). The defects described in the other options are not related to beta-thalassemia major. PTS: 1 REF: Page 1069 24. The alpha- and beta-thalassemias are considered what types of inherited disorder? a. Autosomal recessive c. X-linked recessive b. Autosomal dominant d. X-linked dominant The alpha- and beta-thalassemias are inherited autosomal recessive disorders. This selection is the only option that correctly identifies how these disorders are inherited. PTS: 1 REF: Page 1069 25. Hemophilia B is caused by a deficiency of which clotting factor? a. V c. IX b. VIII d. X Only factor IX deficiency causes hemophilia B (Christmas disease). PTS: 1 REF: Page 1071 26. Hemophilia A is considered to be what type of inherited disorder? a. Autosomal dominant c. X-linked recessive b. Autosomal recessive d. X-linked dominant Hemophilia A (classic hemophilia) is inherited as an X-linked recessive disorder. This selection is the only option that correctly identifies how this disorder is inherited. PTS: 1 REF: Page 1071 27. Which disease is an autosomal dominant inherited hemorrhagic disease? a. Hemophilia A c. Christmas disease b. von Willebrand disease d. Hemophilia B The bleeding disorder, von Willebrand disease, results only from an inherited autosomal dominant trait. PTS: 1 REF: Page 1071 28. Idiopathic thrombocytopenic purpura (ITP) is an autoimmune process involving antibodies attacking which type of cells? a. Neutrophils c. Platelets b. Eosinophils d. Basophils ITP is a disorder of platelet consumption. This selection is the only option that correctly identifies which type of cell is attacked by ITP. PTS: 1 REF: Page 1073 29. Which disorder results in decreased erythrocytes and platelets with changes in leukocytes and has clinical manifestations of pallor, fatigue, petechiae, purpura, bleeding, and fever? a. Idiopathic thrombocytopenic purpura (ITP) b. Acute lymphocytic leukemia (ALL) c. Non-Hodgkin lymphoma (NHL) d. Iron deficiency anemia (IDA) Pallor, fatigue, petechiae, purpura, bleeding, and fever are generally present with the most common symptoms reflecting the consequence of bone marrow failure, which results in decreased red blood cells and platelets and changes in white blood cells. This selection is the only option that correctly identifies the disease with the symptoms described. PTS: 1 REF: Page 1076 30. When does fetal erythrocyte production shift from the liver to the bone marrow? a. Fourth month of gestation c. Eighth month of gestation b. Fifth month of gestation d. At birth By the fifth month of gestation, hematopoiesis begins to occur in the bone marrow and increases rapidly until hematopoietic (red) marrow fills the entire bone marrow space. By the time of delivery, the bone marrow is the only significant site of hematopoiesis. PTS: 1 REF: Pages 1055-1056 31. Which disease is caused by clotting factor VIII deficiency and is an autosomal dominant trait? a. Hemophilia A c. Hemophilia C b. Hemophilia B d. von Willebrand disease Of the options available, only von Willebrand disease is caused by factor VIII deficiency and is the result of an inherited autosomal dominant trait. PTS: 1 REF: Page 1071 32. Which type of hemophilia affects only men? a. Hemophilia A c. Hemophilia C b. Hemophilia B d. von Willebrand disease Of the options available, only hemophilia A (classic hemophilia) is caused by factor VIII deficiency and is inherited as an X-linked recessive disorder that affects men and is transmitted by women. PTS: 1 REF: Page 1071 33. Which hemophilia occurs equally in both men and women? a. Hemophilia A c. Hemophilia C b. Hemophilia B d. von Willebrand disease Of the options available, only hemophilia C (factor XI deficiency) occurs as an autosomal recessive disease while occurring equally in men and women. PTS: 1 REF: Page 1071 34. During childhood, when is dietary iron deficiency commonly diagnosed? a. Between 2 months and 1 year c. Between 12 months and 3 years b. Between 6 months and 2 years d. Between 18 months and 4 years Iron deficiency anemia is the most common blood disorder of infancy and childhood, with the highest incidence occurring between 6 months and 2 years of age. PTS: 1 REF: Page 1059 35. What is the significance of hyperdiploidy when diagnosing and treating leukemia? a. Hyperdiploidy indicates a good prognosis. b. Hyperdiploidy indicates a poor prognosis. c. Hyperdiploidy indicates poor response to a specific treatment. d. Hyperdiploidy indicates the achievement of remission. Hyperdiploidy (increased number of chromosome copies) is associated with a good prognosis. The other options are incorrect regarding the significance of hyperdiploidy. PTS: 1 REF: Page 1075 MULTIPLE RESPONSE 36. What are the common triggers for sickle cell crisis? (Select all that apply.) a. Fever b. Infection c. Dehydration d. Alkalosis e. Exposure to the cold , B, C, E Crises can be prevented by avoiding fever, infection, acidosis, dehydration, constricting clothes, and exposure to cold. PTS: 1 REF: Page 1068 MATCHING Match each sickle cell crisis with its description. Terms may be used more than once. A. Vasoocclusive crisis B. Aplastic crisis C. Sequestration crisis D. Hyperhemolytic crisis 37. Large amounts of blood become acutely pooled in the liver and spleen. 38. Blood flow is impaired by tangled masses of rigid, sickled cells. 39. Crisis occurs in association with certain drugs or infection. 40. Compensatory erythropoiesis is compromised, thus limiting the number of erythrocytes that are replaced. 37. PTS: 1 REF: Page 1066 MSC: In sequestration crisis, large amounts of blood become acutely pooled in the liver and spleen. 38. PTS: 1 REF: Page 1066 MSC: Vasoocclusive crisis (thrombotic crisis) begins with sickling in the microcirculation. As blood flow is obstructed by tangled masses of rigid, sickled cells, vasospasm occurs and a log jam effect brings all blood flow through the vessel to a halt. 39. PTS: 1 REF: Page 1066 40. PTS: 1 REF: Page 1066 MSC: Aplastic crisis, a transient cessation in red blood cell production that results in acute anemia, occurs as a result of a viral infection. The virus causes a temporary shutdown of red blood cell production in the bone marrow, or reticulocytosis. [Show More]

Last updated: 1 year ago

Preview 1 out of 289 pages

Reviews( 1 )

user-profile-pic


by arslanbang · 2 years ago

Clear and Informative work

Recommended For You

 *NURSING> EXAM > Brunner And Suddarths Textbook Of Medical Surgical Nursing 14th Edition: TEST BANK. Chapter 1-74. (All)

preview
Brunner And Suddarths Textbook Of Medical Surgical Nursing 14th Edition: TEST BANK. Chapter 1-74.

Brunner And Suddarths Textbook Of Medical Surgical Nursing 14th Edition: TEST BANK. Chapter 1-74. Q&A with Feedback. 2100 pages. Table of Contents Table of Contents 1 Chapter 01: Health Care D elivery...

By Kerrie jones , Uploaded: Apr 14, 2022

$12.5

 *NURSING> EXAM > Wongs Nursing Care of Infants and Children 10th Edition by Hockenberry TEST BANK. Chapter 1 - 36 Questions And Answers 275 Pages (All)

preview
Wongs Nursing Care of Infants and Children 10th Edition by Hockenberry TEST BANK. Chapter 1 - 36 Questions And Answers 275 Pages

Wongs Nursing Care of Infants and Children 10th Edition by Hockenberry TEST BANK. Chapter 1 - 36 Questions And Answers 275 Pages Chapter 1: Fetal Lung Development Test Bank Page : 4 Chapter 2: Fetal G...

By Kerrie jones , Uploaded: Apr 12, 2022

$13

 *NURSING> EXAM > Seidels Guide to Physical Examination 8th Edition by Ball. TEST BANK. Chapter 1-27 Questions And Answers in 656 Pages.__Graded A+ (All)

preview
Seidels Guide to Physical Examination 8th Edition by Ball. TEST BANK. Chapter 1-27 Questions And Answers in 656 Pages.__Graded A+

Seidels Guide to Physical Examination 8th Edition by Ball. TEST BANK. Chapter 1-27 Questions And Answers in 656 Pages.__Graded A Chapter 01: The History and Interviewing Process Page : 15 Chapter 02:...

By Kerrie jones , Uploaded: Apr 12, 2022

$14

 Business> EXAM > CURRENT Medical Diagnosis and Treatment 2020 Test Bank. Chapter 1. Disease Prevention & Health Promotion-converted (All)

preview
CURRENT Medical Diagnosis and Treatment 2020 Test Bank. Chapter 1. Disease Prevention & Health Promotion-converted

CURRENT Medical Diagnosis and Treatment 2020 Test Bank. Chapter 1. Disease Prevention & Health Promotion-converted CURRENT Medical Diagnosis and Treatment 2020 Test Bank. Chapter 1. Disease Prevention...

By QUIZBANK , Uploaded: Feb 17, 2022

$11

 *NURSING> EXAM > Wongs Nursing Care of Infants and Children 10th Edition by Hockenberry TEST BANK. Chapter 1 - 36 Questions And Answers 275 Pages All Answers Are Correct. (All)

preview
Wongs Nursing Care of Infants and Children 10th Edition by Hockenberry TEST BANK. Chapter 1 - 36 Questions And Answers 275 Pages All Answers Are Correct.

***HERE IS A LIST OF THE CHAPTERS*** Chapter 1: Fetal Lung Development Test Bank Page : 4 Chapter 2: Fetal Gas Exchange and Circulation Test Bank 1 Page : 7 Chapter 3: Antenatal Assessm ent and High R...

By FallonCarrington , Uploaded: Jul 22, 2021

$16

 *NURSING> EXAM > Seidels Guide to Physical Examination 8th Edition by Ball. TEST BANK. Chapter 1-27 Questions And Answers in 656 Pages. Download To Score An A. (All)

preview
Seidels Guide to Physical Examination 8th Edition by Ball. TEST BANK. Chapter 1-27 Questions And Answers in 656 Pages. Download To Score An A.

***HERE IS A LIST OF THE CHAPTERS*** Chapter 01: The History and Interviewing Process Page : 15 Chapter 02: Cultural Competency Page : 24 Chapter 03: Examination Techniques and Equipment Page : 38 Cha...

By FallonCarrington , Uploaded: Jul 22, 2021

$17.5

 *NURSING> EXAM > Bates’ Guide to Physical Examination and History Taking, 12th Edition Test Bank. Chapter 11 (All)

preview
Bates’ Guide to Physical Examination and History Taking, 12th Edition Test Bank. Chapter 11

Bates’ Guide to Physical Examination and History Taking, 12th Edition Chapter 11: The Abdomen Multiple Choice 1. A 52-year-old secretary comes to your office, complaining about acci...

By Best Grades , Uploaded: Apr 28, 2021

$5

 *NURSING> EXAM > Bates’ Guide to Physical Examination and History Taking, 12th Edition Test Bank. Chapter 11 (All)

preview
Bates’ Guide to Physical Examination and History Taking, 12th Edition Test Bank. Chapter 11

Bates’ Guide to Physical Examination and History Taking, 12th Edition Chapter 11: The Abdomen Multiple Choice 1. A 52-year-old secretary comes to your office, complaining about acci...

By Quiz Merchant , Uploaded: Apr 27, 2021

$6

 *NURSING> EXAM > 2024 CNA – Chapter 30 | Questions & Answers (Solved) With 100% Verified Answers (All)

preview
2024 CNA – Chapter 30 | Questions & Answers (Solved) With 100% Verified Answers

1. Hemoptysis Answer: blood sputum 2. Ketone Body Answer: a ketone or acetone 3. Melena Answer: a black, tarry stool 4. Glycosuria Answer: or glucosuria, sugar in the urine 5. Septum Answer:...

By ExamNavigator , Uploaded: Apr 11, 2024

$8

 *NURSING> EXAM > Wongs Nursing Care of Infants and Children 10th Edition by Hockenberry TEST BANK. Chapter 1 - 36 Questions And Answers 275 Pages (All)

preview
Wongs Nursing Care of Infants and Children 10th Edition by Hockenberry TEST BANK. Chapter 1 - 36 Questions And Answers 275 Pages

Wongs Nursing Care of Infants and Children 10th Edition by Hockenberry TEST BANK. Chapter 1 - 36 Questions And Answers 275 Pages Chapter 1: Fetal Lung Development Test Bank Page : 4 Chapter 2: Fetal G...

By brendan , Uploaded: Aug 30, 2022

$11

$22.00

Add to cart

Instant download

Can't find what you want? Try our AI powered Search

OR

GET ASSIGNMENT HELP
100
2

Document information


Connected school, study & course



About the document


Uploaded On

Apr 14, 2021

Number of pages

289

Written in

Seller


seller-icon
gradesblaze

Member since 3 years

21 Documents Sold


Additional information

This document has been written for:

Uploaded

Apr 14, 2021

Downloads

 2

Views

 100

Document Keyword Tags

THE BEST STUDY GUIDES

Avoid resits and achieve higher grades with the best study guides, textbook notes, and class notes written by your fellow students

custom preview

Avoid examination resits

Your fellow students know the appropriate material to use to deliver high quality content. With this great service and assistance from fellow students, you can become well prepared and avoid having to resits exams.

custom preview

Get the best grades

Your fellow student knows the best materials to research on and use. This guarantee you the best grades in your examination. Your fellow students use high quality materials, textbooks and notes to ensure high quality

custom preview

Earn from your notes

Get paid by selling your notes and study materials to other students. Earn alot of cash and help other students in study by providing them with appropriate and high quality study materials.


$22.00

WHAT STUDENTS SAY ABOUT US


What is Browsegrades

In Browsegrades, a student can earn by offering help to other student. Students can help other students with materials by upploading their notes and earn money.

We are here to help

We're available through e-mail, Twitter, Facebook, and live chat.
 FAQ
 Questions? Leave a message!

Follow us on
 Twitter

Copyright © Browsegrades · High quality services·